222
TUYỂN CHỌN CÁC BÀI TOÁN TRONG KÌ THI CHỌN ĐỘI TUYỂN CỦA CÁC TỈNH, THÀNH PHỐ NĂM HỌC 2016 – 2017 −1 Blog TOÁN HỌC CHO MỌI NGƯỜI https://thcmn.wordpress.com/ https://www.facebook.com/thcmn/ [email protected]

t 2017 - WordPress.com...Bài 5. (B›c Ninh) Cho a, b c¨0 thäa mãn đi•u ki»n ¯ ˘9. Tìm giá trà lîn nh t cıabi”uthøc T ˘ ab 3a¯4b¯5c ¯ bc 3b¯4c¯5a ¯ ca 3c¯4a¯5b

  • Upload
    others

  • View
    1

  • Download
    0

Embed Size (px)

Citation preview

Page 1: t 2017 - WordPress.com...Bài 5. (B›c Ninh) Cho a, b c¨0 thäa mãn đi•u ki»n ¯ ˘9. Tìm giá trà lîn nh t cıabi”uthøc T ˘ ab 3a¯4b¯5c ¯ bc 3b¯4c¯5a ¯ ca 3c¯4a¯5b

TUYỂN CHỌN CÁC BÀI TOÁN TRONG KÌ THI CHỌN ĐỘI TUYỂN

CỦA CÁC TỈNH, THÀNH PHỐNĂM HỌC 2016 – 2017

−1 Blog TOÁN HỌC CHO MỌI NGƯỜI

https://thcmn.wordpress.com/ https://www.facebook.com/thcmn/ [email protected]

Page 2: t 2017 - WordPress.com...Bài 5. (B›c Ninh) Cho a, b c¨0 thäa mãn đi•u ki»n ¯ ˘9. Tìm giá trà lîn nh t cıabi”uthøc T ˘ ab 3a¯4b¯5c ¯ bc 3b¯4c¯5a ¯ ca 3c¯4a¯5b
Page 3: t 2017 - WordPress.com...Bài 5. (B›c Ninh) Cho a, b c¨0 thäa mãn đi•u ki»n ¯ ˘9. Tìm giá trà lîn nh t cıabi”uthøc T ˘ ab 3a¯4b¯5c ¯ bc 3b¯4c¯5a ¯ ca 3c¯4a¯5b

TRẦN MINH NGỌC – LƯƠNG VĂN KHẢI

VÕ THÀNH ĐẠT – HOÀNG ĐÌNH HIẾU – LÊ THÀNH LONG – ĐẶNG NHÌ – NGUYỄN DUY TÙNG

NGUYỄN TRƯỜNG HẢI – ĐỖ TRẦN NGUYÊN HUY – PHẠM THỊ HỒNG NHUNG – PHẠM QUỐC THẮNG

TUYỂN CHỌN CÁC BÀI TOÁN TRONG KÌ THI CHỌN ĐỘI TUYỂN

CỦA CÁC TỈNH, THÀNH PHỐ NĂM HỌC 2016 – 2017

Tháng 12 năm 2016

Page 4: t 2017 - WordPress.com...Bài 5. (B›c Ninh) Cho a, b c¨0 thäa mãn đi•u ki»n ¯ ˘9. Tìm giá trà lîn nh t cıabi”uthøc T ˘ ab 3a¯4b¯5c ¯ bc 3b¯4c¯5a ¯ ca 3c¯4a¯5b
Page 5: t 2017 - WordPress.com...Bài 5. (B›c Ninh) Cho a, b c¨0 thäa mãn đi•u ki»n ¯ ˘9. Tìm giá trà lîn nh t cıabi”uthøc T ˘ ab 3a¯4b¯5c ¯ bc 3b¯4c¯5a ¯ ca 3c¯4a¯5b

LỜI NÓI ĐẦU

Ban biên tập

"Đi nhiều người, bạn sẽ đi rất xa."

Với mục đích giúp quý thầy cô và các bạn học sinh có một tài liệu chấtlượng để chuẩn bị cho kì thi Học sinh giỏi Quốc gia môn Toán (VMO), tập thểcác quản trị viên blog Toán học cho mọi người đã cùng nhau biên soạn cuốnsách "Tuyển chọn theo chuyên đề các bài toán trong kì thi chọn đội tuyển VMOcủa các tỉnh, thành phố".

Trong cuốn sách này, các bài toán được liệt kê trước, sau đó là phần lờigiải, đáp số. Trong một số bài toán, chúng tôi có đưa ra nhiều hơn một cáchtiếp cận, nhưng cũng có những bài toán mà chúng tôi thấy chỉ cần hướng dẫnsơ lược lời giải, qua đó giúp bạn đọc chủ động trong quá trình đọc tài liệu. Nhiềubài giải của chúng tôi trong đây chưa phải là cách làm hay nhất, tốt nhất chocác bài toán tương ứng, và chúng tôi rất mong nhận được sự đánh giá, đóng gópcủa bạn đọc để những lần biên soạn sau, chất lượng cuốn tuyển tập này đượcnâng lên.

Các phần của cuốn sách và người biên soạn cụ thể như sau:

• Bất đẳng thức: Võ Thành Đạt (Sinh viên khoa Toán - Tin học Đại học Khoa họcTự nhiên Tp. HCM).

• Đa thức, Phương trình và Hệ phương trình: Đỗ Trần Nguyên Huy (Học sinhtrường Phổ thông Năng khiếu - ĐHQG Tp. HCM) và Phạm Quốc Thắng (Họcsinh trường THPT chuyên Long An).

• Hình học: Trần Minh Ngọc (Học viên Cao học Đại học Sư phạm Tp. HCM),Lương Văn Khải và Nguyễn Duy Tùng (Sinh viên khoa Toán - Tin học trườngĐại học Khoa học tự nhiên Tp. HCM).

• Số học: Phạm Thị Hồng Nhung (Học sinh trường THPT chuyên Lê Quý Đôn,tỉnh Bà Rịa - Vũng Tàu).

• Tổ hợp: Hoàng Đình Hiếu (Sinh viên khoa Công nghệ thông tin trường Đạihọc Khoa học Tự nhiên Tp. HCM) và Đặng Nhì (Sinh viên khoa Toán - Tin họctrường Đại học Khoa học Tự nhiên Tp. HCM).

• Giải tích: Nguyễn Trường Hải (Học sinh trường THPT chuyên Trần Hưng Đạo,Bình Thuận).

• Phương trình hàm: Lê Thành Long (Sinh viên khoa Điện - Điện tử trường Đạihọc Bách khoa Tp. HCM).

5

Page 6: t 2017 - WordPress.com...Bài 5. (B›c Ninh) Cho a, b c¨0 thäa mãn đi•u ki»n ¯ ˘9. Tìm giá trà lîn nh t cıabi”uthøc T ˘ ab 3a¯4b¯5c ¯ bc 3b¯4c¯5a ¯ ca 3c¯4a¯5b

Chúng tôi xin chân thành cảm ơn TS Trần Nam Dũng (trường Đại họcKhoa học Tự nhiên Tp. HCM), anh Lê Phúc Lữ (FPT Software, Tp. HCM), bạnĐào Nguyễn Nguyên Trân (Swiss UMEF, Thuỵ Sĩ), bạn Đỗ Thuỳ Anh (THPTchuyên Lam Sơn, Thanh Hoá), bạn Nguyễn Trần Hữu Thịnh (THPT chuyên LýTự Trọng, Cần Thơ), bạn Hoàng Hữu Quốc Huy (THPT chuyên Lê Quý Đôn,Bà Rịa - Vũng Tàu) đã giúp đỡ chúng tôi rất nhiều trong quá trình biên soạncuốn sách này. Cảm ơn các thành viên của các diễn đàn NangKhieuToan.com(nangkhieutoan.com), Diễn đàn Mathscope (forum.mathscope.org), Diễn đànToán học Việt Nam (diendantoanhoc.net), Diễn đàn Art of Problem Solving(artofproblemsolving.com) đã đóng góp các đề bài và lời giải.

Trong quá trình biên soạn, chắc chắn chúng tôi không tránh khỏi nhữngsai sót ở các đề bài và lời giải, rất mong được lắng nghe những nhận xét, góp ývà phê bình thẳng thắn từ các bạn. Mọi thắc mắc và đóng góp xin vui lòng liênhệ fanpage Toán học cho mọi người ở địa chỉ www.facebook.com/thcmn hoặcqua email [email protected].

Cảm ơn tất cả các bạn !

6 http://www.facebook.com/thcmn

Page 7: t 2017 - WordPress.com...Bài 5. (B›c Ninh) Cho a, b c¨0 thäa mãn đi•u ki»n ¯ ˘9. Tìm giá trà lîn nh t cıabi”uthøc T ˘ ab 3a¯4b¯5c ¯ bc 3b¯4c¯5a ¯ ca 3c¯4a¯5b

Mục lục

I CÁC BÀI TOÁN 8

1 Bất đẳng thức 8

2 Đa thức 11

3 Giải tích 13

4 Hình học 19

5 Phương trình và hệ phương trình 28

6 Số học 30

7 Tổ hợp 33

II LỜI GIẢI 39

1 Bất đẳng thức 39

2 Đa thức 60

3 Giải tích 81

4 Hình học 112

5 Phương trình và hệ phương trình 167

6 Số học 179

7 Tổ hợp 201

http://thcmn.wordpress.com 7

Page 8: t 2017 - WordPress.com...Bài 5. (B›c Ninh) Cho a, b c¨0 thäa mãn đi•u ki»n ¯ ˘9. Tìm giá trà lîn nh t cıabi”uthøc T ˘ ab 3a¯4b¯5c ¯ bc 3b¯4c¯5a ¯ ca 3c¯4a¯5b

Phần I

CÁC BÀI TOÁN

1 Bất đẳng thức

Bài 1. (THPT chuyên KHTN - ĐH KHTN, ĐHQG Hà Nội)

1. Cho x, y là các số thực dương sao cho 2x+y và 2y+x khác 2. Tìm giá trị nhỏ nhấtcủa biểu thức

P = (2x2 + y)(4x + y2)

(2x + y −2)2+ (2y2 +x)(4y +x2)

(x +2y −2)2−3(x + y)

2. Cho a,b,c > 0 sao cho a +b + c = 3. Chứng minh rằng

a

b2(ca +1)+ b

c2(ab +1)+ c

a2(bc +1)≥ 9

(1+abc)(ab +bc + ca)

Bài 2. (Trường Phổ thông Năng Khiếu - ĐHQG Tp. HCM) Tìm số nguyên dương k

nhỏ nhất sao cho bất đẳng thức

xk yk zk (x3 + y3 + z3) ≤ 3

đúng với mọi số thực dương x, y, z thỏa mãn điều kiện x + y + z = 3.

Bài 3. ( THPT chuyên Đại học Vinh) Tìm tất cả các số thực k sao cho bất đẳng thứcsau đúng với mọi số thực không âm a,b,c

ab +bc + ca ≤ (a +b + c)2

3+k.max{(a −b)2, (b − c)2, (c −a)2} ≤ a2 +b2 + c2

Bài 4. (Bà Rịa - Vũng Tàu)

1. Cho x, y, z là ba số thực dương thỏa mãn x y z = 1. Chứng minh bất đẳng thức

1

(2x + y + z)2+ 1

(2y + z +x)2+ 1

(2z +x + y)2≤ 3

16

2. Cho x, y, z không âm và thỏa x2 + y2 + z2 = 1. Chứng minh bất đẳng thức

(x2 y + y2z + z2x)

(1p

x2 +1+ 1√

y2 +1+ 1p

z2 +1

)≤ 3

2.

8 http://www.facebook.com/thcmn

Page 9: t 2017 - WordPress.com...Bài 5. (B›c Ninh) Cho a, b c¨0 thäa mãn đi•u ki»n ¯ ˘9. Tìm giá trà lîn nh t cıabi”uthøc T ˘ ab 3a¯4b¯5c ¯ bc 3b¯4c¯5a ¯ ca 3c¯4a¯5b

Bài 5. (Bắc Ninh) Cho a,b,c > 0 thỏa mãn điều kiện a +b + c = 9. Tìm giá trị lớn nhấtcủa biểu thức

T = ab

3a +4b +5c+ bc

3b +4c +5a+ ca

3c +4a +5b− 1p

ab(a +2c)(b +2c)

Bài 6. (Bến Tre) Cho a,b,c là các số thực dương . Tìm giá trị nhỏ nhất của biểu thức:

P = 1344

a +pab + 3pabc

− 2016pa +b + c

Bài 7. (Bình Thuận) Cho các số thực dương x, y, z. Chứng minh rằng

x2

y + z+ y2

z +x+ z2

x + y≥ x + y + z

2≥ y z

y + z+ x y

x + y+ zx

z +x

Bài 8. (Đồng Nai) Cho các số thưc dương a,b,c thỏa mãn abc = 1. Chứng minh rằng :

√a

b + c+

√b

a + c+

√c

a +b≥ 3

p3p

a3 +b3 + c3 +3

Bài 9. (Hà Nam) Cho a,b,c ≥ 0. Tìm giá trị nhỏ nhất của:

P =√

a

b + c+

√b

a + c+

√c

a +b

Bài 10. (Hà Nội) Cho a,b,c > 0 thỏa mãn ab +bc + ca +2abc = 1 Tìm giá trị nhỏ nhấtcủa

P = 1

a+ 1

b+ 1

c−2(a +b + c).

Bài 11. (Hà Tĩnh) Cho các số thực dương a,b,c và thỏa mãn a5 +b5 + c5 = 3. Chứngminh rằng

a6b6 +b6c6 + c6a6 ≤ 3

.

Bài 12. (Hải Phòng) Cho a,b,c ≥ 1

2thỏa mãn a +b + c = 6. chứng minh rằng

ab +bc + ca ≥ 3p

abc +ab +bc + ca −4

http://thcmn.wordpress.com 9

Page 10: t 2017 - WordPress.com...Bài 5. (B›c Ninh) Cho a, b c¨0 thäa mãn đi•u ki»n ¯ ˘9. Tìm giá trà lîn nh t cıabi”uthøc T ˘ ab 3a¯4b¯5c ¯ bc 3b¯4c¯5a ¯ ca 3c¯4a¯5b

Bài 13. (Hòa Bình) Cho a,b,c là các số dương thỏa mãn abc = 1 và x, y, z thuộc R

Chứng minh rằng :

x2(a +b)+ y2(b + c)+ z2(c +a) ≥ 2(x y + y z + zx)

Bài 14. (Khánh Hòa) Cho hai số thực x và y thỏa mãn x2 + x y + y2 ≤ 2. Chứng minhrằng

5x2 +2x y +2y2 ≤ 12

Bài 15. (Lạng Sơn) Cho x, y, z > 0 thỏa mãn x y z = 1. Tìm giá trị lớn nhất của :

P = 1

x2 +2y2 +3+ 1

y2 +2z2 +3+ 1

z2 +2x2 +3

Bài 16. (Nam Định) Cho các số thực dương a,b,c thỏa mãn a+b+c = 3. Chứng minhrằng:

(a +pb)2

pa2 −ab +b2

+ (b +pc)2

pb2 −bc + c2

+ (c +pa)2

pc2 − ca +a2

≤ 12

.

Bài 17. (Ninh Bình) Cho x, y, z > 0 thỏa mãn x + y + z = 3. Chứng minh rằng:

1px +p

y+ 1p

y +pz+ 1p

z +px≥ 4

(1

x +7+ 1

y +7+ 1

z +7

)

Bài 18. (Quảng Bình) Cho a,b,c là độ dài ba cạnh tam giác và a ≥ b ≥ c . Chứng minhrằng √

a(a +b −p

ab)+√

b(a + c −pac)+

√c(c +b −

pbc) ≥ a +b + c

Bài 19. (Quảng Nam) Cho các số thực không âm a,b,c,d . Chứng minh bất đẳng thức:

(a +b + c +d)3 ≤ 4(a3 +b3 + c3 +d 3)+24(abc +bcd + cd a +d ab)

Bài 20. (Quảng Ninh) Cho a,b,c > 0 thỏa mãn (a +b)(b + c)(c +a) = 1. Tìm giá trị nhỏnhất của biểu thức:

P =p

a2 −ab +b2

pab +1

+p

b2 −bc + c2

pbc +1

+p

c2 − ca +a2

pca +1

10 http://www.facebook.com/thcmn

Page 11: t 2017 - WordPress.com...Bài 5. (B›c Ninh) Cho a, b c¨0 thäa mãn đi•u ki»n ¯ ˘9. Tìm giá trà lîn nh t cıabi”uthøc T ˘ ab 3a¯4b¯5c ¯ bc 3b¯4c¯5a ¯ ca 3c¯4a¯5b

Bài 21. (Quảng Ngãi) Cho ba số thực dương a,b,c thỏa a+b+c = 3. Chứng minh rằng

1

a+ 1

b+ 1

c+ a +1

1+b2+ b +1

1+ c2+ c +1

1+a2≥ 6 ≥ 8

a2 +b2 +2+ 8

b2 + c2 +2+ 8

c2 +a2 +2

Bài 22. (Quảng Trị) Cho 3 số không âm x, y, z thỏa x + y + z = 2. Chứng minh rằng

x2 y + y2z + z2x ≤ x3 + y3 + z3 ≤ 1+ 1

2(x4 + y4 + z4)

Bài 23. (Tp. HCM) Cho a,b,c là các số thực thỏa mãn a +b + c = 0. Tìm giá trị nhỏnhất của biểu thức

(a2 +1)2 + (b2 +1)2 + c2 +1)2 +6p

6abc

Bài 24. (Thái Nguyên)

1. Cho a,b,c là các số thực dương. Chứng minh rằng

1

(1+a)3+ 1

(1+a)3+ 1

(1+a)3+ 3

32(ab +bc + ca) ≥ 21

32

2. Cho x, y, z là các số thực dương thỏa mãn x y z ≥ 1 và z ≤ 1. Tìm giá trị nhỏ nhấtcủa biểu thức

F = x

1+ y+ y

1+x+ 4− z3

3(1+x y)

Bài 25. (Thanh Hóa) Cho x, y, z > 0 thỏa x + y + z = 1

x+ 1

y+ 1

z. Chứng minh rằng

1

(2x y + y z + zx)2+ 1

(2y z + zx +x y)2+ 1

(2zx +x y + y z)2≤ 3

16x2 y2z2.

2 Đa thức

Bài 1. (THPT chuyên KHTN - ĐH KHTN, ĐHQG Hà Nội) Tìm tất cả đa thức hệ sốthực thỏa mãn

2

(P (x)−P

(1

x

))2

+3P (x2)P

(1

x2

)= 0.

Bài 2. (Bến Tre) Cho khai triển (1−2x +x3)n = a0 +a1x +a2x2 + ...++a3n x3n . Xác định

hệ số a6 biết rằng a0 + a1

2+ a2

22+ ...+ a3n

23n=

(1

2

)15

.

http://thcmn.wordpress.com 11

Page 12: t 2017 - WordPress.com...Bài 5. (B›c Ninh) Cho a, b c¨0 thäa mãn đi•u ki»n ¯ ˘9. Tìm giá trà lîn nh t cıabi”uthøc T ˘ ab 3a¯4b¯5c ¯ bc 3b¯4c¯5a ¯ ca 3c¯4a¯5b

Bài 3. (Bến Tre) Cho phương trình

x5 − 1

2x4 −5x3 +x2 +4x −1 = 0

Chứng minh rằng phương trình trên có đúng 5 nghiệm phân biệt. Với xi (i = 1,5) là

nghiệm của phương trình trên, tính tổng S biết: S =5∑

i=1

xi +1

2x5i−x4

i −2.

Bài 4. (Bình Dương) Cho dãy các đa thức hệ số thực {Pn(x)},n = 1,2,3, ... thỏa mãnđiều kiện Pn(2cos x) = 2n cos(nx),∀x ∈ R,∀n ∈N∗. Chứng minh rằng với mỗi n ∈N∗ thìPn(x) là đa thức hệ số nguyên bậc n và x ≤ n

√Pn(x),∀x > 2.

Bài 5. (Đà Nẵng) Tìm số nguyên dương n nhỏ nhất sao cho tồn tại đa thức f (x) bậcn có hệ số nguyên thỏa mãn: f (0) = 0, f (1) = 1 và với mọi m ∈N∗, f (m)( f (m)−1) là bộicủa 2017.

Bài 6. (Đà Nẵng) Chứng minh rằng với mọi m ∈N, tồn tại đa thức fm(x) có hệ số hữutỉ thỏa mãn với mọi n ∈N∗ thì: 12m+1 +22m+1 + ...+n2m+1 = fm(n(n +1)).

Bài 7. (Đồng Nai) Cho số tự nhiên n ≥ 2 và n số thực a1, a2, . . . , an sao cho a1 >−1, a2 ≥n −1

2. Giả sử phương trình xn

n +a1xn−1 +a2xn−2 + . . .+an−1x +an = 0 có đúng n nghiệm

thực. Chứng minh rằng tất cả các nghiệm đó nằm trong đoạn [−a1, a1 +2].

Bài 8. (Hà Nam) Cho P,Q,R là 3 đa thức hệ số thực thỏa mãn: P (Q(x))+P (R(x)) = c

∀x ∈R với c = const ∈R. Chứng minh rằng P (x) ≡ const hoặc [Q(x)+R(x)] ≡ const

Bài 9. (Hà Tĩnh) Cho các đa thức P (x),Q(x),R(x) với hệ số thực có bậc tương ứng là3,2,3 thỏa mãn đẳng thức P 2(x)+Q2(x) = R2(x), ∀x ∈R. Hỏi đa thức T (x) = P (x).Q(x).R(x)có ít nhất bao nhiêu nghiệm thực (kể cả nghiệm bội).

Bài 10. (Hải Phòng) Cho dãy đa thức hệ số thực{Pn (x)

}+∞n=0 xác định như sau P0 (x) =

2,P1 (x) = 2x,Pn+1 (x) = 2x.Pn (x)+(1−x2

)Pn−1 (x)∀n ≥ 1.

1. Xác định công thức tổng quát của Pn (x).

2. Tìm tất cả các số tự nhiên n để Pn (x) chia hết cho x2 +3.

Bài 11. (Hòa Bình) Cho đa thức P (x) = x4+ax3+bx2+cx+d và Q(x) = x2+px+q cùngthuộc Q[x] . Biết rằng hai đa thức cùng nhận giá trị âm trên khoảng I có độ dài lớnhơn hai và ngoài khoảng I chúng đều nhận giá trị không âm . Chứng minh rằng tồntại xo ∈R đề P (xo) <Q(xo).

Bài 12. (Tp. HCM) Cho đa thức P (x) = x2016 + a2015x2015 + a2014x2014 + ...+ a1x + a0 có

hệ số thực với P (1)P (2) 6= 0 và 4P /(2)

P (2)> P /(1)

P (1)+2016. Giả sử P (x) có 2016 nghiệm thực,

chứng minh rằng trong số đó, có ít nhất một nghiệm thuộc khoảng (1;2).

Bài 13. (Khánh Hòa) Cho P (x) là đa thức với hệ số nguyên. Chứng minh rằng tồn tạihai đa thức Q(x) và R(x) sao cho

12 http://www.facebook.com/thcmn

Page 13: t 2017 - WordPress.com...Bài 5. (B›c Ninh) Cho a, b c¨0 thäa mãn đi•u ki»n ¯ ˘9. Tìm giá trà lîn nh t cıabi”uthøc T ˘ ab 3a¯4b¯5c ¯ bc 3b¯4c¯5a ¯ ca 3c¯4a¯5b

1. P (x)R(x) là các đa thức của x2.

2. P (x)R(x) là các đa thức của x3.

Bài 14. (Long An) Tìm tất cả đa thức P (x) thỏa mãn:

P (−x).P (3x)+ (P (2x)

)2 = P (x).P (5x),∀x ∈R.

Bài 15. (Nghệ An) Cho m là số nguyên dương thỏa mãn m ≡ 1(mod2017). Chứngminh rằng đa thức P (x) = x2017 −mx +2016 là đa thức bất khả quy trên Z[x].

Bài 16. (Phú Thọ) Tìm tất cả các đa thức P (x) hệ số thực thỏa mãn

(x2 −6x +8)P (x)− (x2 +2x)P (x −2) = 6x2 −12x.

Bài 17. (Quảng Bình) Cho đa thức

f (x) = x2017 +ax2 +bx + c

trong đó a,b,c ∈ Z có ba nghiệm nguyên x1, x2, x3 . Chứng minh rằng biểu thức sau làbội của 2017

(a2017 +b2017 + c2017 +1)(x1 −x2)(x2 −x3)(x3 −x1)

Bài 18. (Quảng Nam) Tìm tất cả các đa thức P (x) với hệ số thực thỏa mãn điều kiện:

P (x2)+P (x).P (x +1) = 0,∀x ∈R

Bài 19. (Vĩnh Phúc) Cho Pi (x) = x2 +bi x + ci (i = 1,2, ...,n) là n đa thức đôi một phânbiệt với hệ số thực sao cho với mọi 1 ≤ i < j ≤ n thì đa thức Qi , j (x) = Pi (x)+P j (x) cónghiệm thực duy nhất. Tìm giá trị lớn nhất có thể của n.

3 Giải tích

Bài 1. (THPT chuyên KHTN, ĐH KHTN, ĐHQG HN) Cho dãy số (xn) thỏa x1 = 3, x2 = 7và:

xn+2 = x2n+1 −x2

n +xn ,n ∈N∗

Đặt dãy:

yn =n∑

k=1

1

xk

Chứng minh (yn) có giới hạn hữu hạn và tìm giới hạn đó.

http://thcmn.wordpress.com 13

Page 14: t 2017 - WordPress.com...Bài 5. (B›c Ninh) Cho a, b c¨0 thäa mãn đi•u ki»n ¯ ˘9. Tìm giá trà lîn nh t cıabi”uthøc T ˘ ab 3a¯4b¯5c ¯ bc 3b¯4c¯5a ¯ ca 3c¯4a¯5b

Bài 2. (Trường Phổ thông Năng khiếu, ĐHQG Tp. HCM) Tìm a để dãy số (un) hội tụ,biết u1 = a và:

un+1 =

2un −1 khi un > 0

−1 khi −1 ≤ un ≤ 0

u2n +4un +2 khi un <−1

,n ∈N∗

Bài 3. (THPT chuyên ĐH Vinh) Tìm tất cả các hàm số f :R→R thỏa mãn:

1. f (1) > 0.

2. f (x y −1)+2 f (x) f (y) = 3x y −1∀x, y ∈R.

Bài 4. (THPT chuyên ĐH Vinh) Cho số thực a ≥ 2 và dãy số(un

)xác đinh bởi:

u1 = a

un+1 = un + lnun +1

2un −3,n ∈N∗

Chứng minh rằng dãy un có giới hạn hữu hạn và tìm giới hạn đó.

Bài 5. (Bà Rịa - Vũng Tàu)

1. Chứng minh rằng không tồn tại hàm số f :R→R thỏa mãn:

f (x −2016 f (y)) = y −2017 f (x) ∀x, y ∈R.

2. Tìm tất cả các hàm số f :R→R thỏa mãn

f (x + y f (x)) = x f (y)+ f (x) ∀x, y ∈R.

Bài 6. (Bà Rịa - Vũng Tàu) Cho dãy số xn xác định bởi:x1 = 1

2

xn+1 =nx2

n

1+ (n +1)xn,n ∈N∗

1. Chứng minh xn ≤ 1

n(n +1),∀n ≥ 1.

2. Với mỗi số nguyên dương n, đặt yn =n∑

k=1

kxk

1+ (k +1)xk. Chứng minh dãy số có giới

hạn hữu hạn và tính giới hạn đó.

Bài 7. (Bình Dương) Tìm tất cả các hàm số f : R→ R thỏa mãn1

3f (x y)+ 1

3f (xz)−

f (x) f (y z) ≥ 1

9.

Bài 8. (Bình Thuận)

14 http://www.facebook.com/thcmn

Page 15: t 2017 - WordPress.com...Bài 5. (B›c Ninh) Cho a, b c¨0 thäa mãn đi•u ki»n ¯ ˘9. Tìm giá trà lîn nh t cıabi”uthøc T ˘ ab 3a¯4b¯5c ¯ bc 3b¯4c¯5a ¯ ca 3c¯4a¯5b

a. Tìm limun với un = 1

2· 3

4· · · 2n +1

2n +2n ∈N.

b. Cho dãy số (un) xác định bởi:u1 = 1

un+1 =√

1+u2n −1

un,n ∈N∗

Tìm công thức tổng quát của (un).

Bài 9. (Đà Nẵng) Cho dãy Fibonacci xác định như sau:{u1 = u2 = 1

un = un−1 +un−2

Chứng minh rằng với mọi số nguyên tố p ≥ 7 thì có đúng 1 trong 2 số up−1,up+1 là bộicủa p.

Bài 10. (Đồng Nai) Tìm tất cả các hàm f :R→R thỏa mãn

f (x2 −2y f (x))+ f (y2) = f 2(x − y),∀x, y ∈R.

Bài 11. (Đồng Nai) Cho dãy số (un) xác định bởi:

u1 ∈

(1;2

)un+1 = 1+un − u2

n

2,∀n ∈N∗

Chứng minh rằng(un

)có giới hạn hữu hạn và tìm giới hạn đó.

Bài 12. (Hà Nam) Cho hai dãy số được xác định bởi:x1 = y1 =

p3

xn+1 = xn +√

1+x2n

yn+1 = yn

1+√

1+ y2n

,∀n ∈N∗

1. Chứng minh rằng xn yn ∈ (2;3) ∀n ≥ 2.

2. Tính limn→+∞ yn.

Bài 13. (Hà Nội) Cho dãy số(un

)có u1 = 1,un = n

n −1un−1 +n với n ∈N và n ≥ 2.

1. Xác định công thức của(un

).

2. Chứng minh u1 +u2 + ...+u2016 < 20163.

http://thcmn.wordpress.com 15

Page 16: t 2017 - WordPress.com...Bài 5. (B›c Ninh) Cho a, b c¨0 thäa mãn đi•u ki»n ¯ ˘9. Tìm giá trà lîn nh t cıabi”uthøc T ˘ ab 3a¯4b¯5c ¯ bc 3b¯4c¯5a ¯ ca 3c¯4a¯5b

Bài 14. (Hà Tĩnh) Với mỗi số nguyên dương n, xét hàm số fn trên R được xác địnhbởi fn(x) = x2n +x2n−1 + ...+x2 +x +1.

1. Chứng minh hàm số fn đạt giá trị nhỏ nhất tại một điểm duy nhất.

2. Gọi giá trị nhỏ nhất của hàm số fn là sn. Chứng minh dãy số (sn) có giới hạn hữuhạn.

Bài 15. (Hải Phòng) Cho dãy số (un) thỏa:u1 = 1

un+1 =u2

n +n

2un,n ∈N∗

Chứng minh rằng dãy

(unp

n

)có giới hạn hữu hạn.

Bài 16. (Hòa Bình) Xác định tất cả các hàm f : R→ R thỏa mãn: f ([x]y) = f (x)[ f (y)]

với [x] là số nguyên lớn nhất không vượt quá x.

Bài 17. (Hòa Bình) Cho (xn) được xác định như sau:

x0 > 0; xn+1 = xn

1+x2n

,n ∈N

Tìm limp

2nxn.

Bài 18. (Hòa Bình) Cho dãy số (xn) xác định bởi:xo = 1

x1 = 41

xn+2 = 3xn +√

8(x2n+1 +x2

n),n ∈N

Chứng minh rằng mọi số hạng của dãy đều là số nguyên.

Bài 19. (Tp. HCM) Cho dãy số un xác định bởi công thức:u1 = 1,u2 = 3

2un+2 = un+1 +2

un +2,n ∈N∗

Chứng minh dãy số un có giới hạn hữu hạn.

Bài 20. (Khánh Hòa) Tìm các hàm số f :R→R thỏa mãn: f (x y)+ f (x−y)+ f (x+y+1) =x y +2x +1 với mọi x, y ∈RBài 21. (Khánh Hòa) Cho dãy số (un) xác định bởi:

u1 = a

un+1 = un

n+ n

un,n ∈N∗

16 http://www.facebook.com/thcmn

Page 17: t 2017 - WordPress.com...Bài 5. (B›c Ninh) Cho a, b c¨0 thäa mãn đi•u ki»n ¯ ˘9. Tìm giá trà lîn nh t cıabi”uthøc T ˘ ab 3a¯4b¯5c ¯ bc 3b¯4c¯5a ¯ ca 3c¯4a¯5b

Chứng minh rằng[

u2n

]= n khi n ≥ 4.

Bài 22. (Lạng Sơn) Cho dãy số (un) xác định bởiu1 =−1

3un+1 +1 = un +1√

u2n +1

∀n ∈N∗.

1. Chứng minh rằng un+1 +1 < 3(un +1)p10

∀n ∈N∗.

2. Chứng minh rằng dãy (un) hội tụ. Tính limn→+∞un.

Bài 23. (Lạng Sơn) Tìm tất cả các hàm số f :R→R đơn điệu trên R thỏa mãn:

f (x3 + f (y)) = f 3(x)+ y∀x, y ∈R

Bài 24. (Lào Cai) Cho dãy số thực (xn) được xác định bởix1 = 5

2

xn+1 =√

x3n −12xn + 20n +21

n +1

.

Chứng minh rằng dãy số (xn) có giới hạn hữu hạn và tìm giới hạn đó .

Bài 25. (Ninh Bình) Cho hàm số f :N∗ →N∗ thỏa mãn các điều kiện sau:

i. f (m) < f (n) ∀m,n ∈N∗;m < n.

ii. f (mn) = f (m) f (n) ∀m,n ∈N∗; (m,n) = 1.

iii. ∃i ∈N∗, i > 1 sao cho f (i ) = i .

1. Chứng minh rằng f (1) = 1 , f (3) = 3.

2. Tìm tất cả các hàm f (n) thỏa mãn yêu cầu đề bài.

Bài 26. (Ninh Bình) Cho dãy số (xn) xác định bởi hệ thức:x1 = 1

xn+1 =√

xn(xn +1)(xn +2)(xn +3)+1,n ∈N∗

Đặt yn =n∑

i=1

1

xi +2. Tính lim yn .

http://thcmn.wordpress.com 17

Page 18: t 2017 - WordPress.com...Bài 5. (B›c Ninh) Cho a, b c¨0 thäa mãn đi•u ki»n ¯ ˘9. Tìm giá trà lîn nh t cıabi”uthøc T ˘ ab 3a¯4b¯5c ¯ bc 3b¯4c¯5a ¯ ca 3c¯4a¯5b

Bài 27. (Phú Thọ) Xét dãy số thực vô hạn x1, x2, · · · , xn thỏa mãn

|xm+n −xm −xn | < 1

m +n

với mọi số nguyên dương m,n. Chứng minh rằng (xn) là cấp số cộng.

Bài 28. (Quảng Bình) Tìm tất cả hàm số f : N∗ → N∗ sao cho ba số a, f (b), f (b+ f (a)−1)luôn là độ dài ba cạnh của một tam giác với mọi a,b ∈ N∗

Bài 29. (Quảng Binh) Cho a là một số thực và dãy số thực (xn) xác định bởi

xn = 2016n +a.3√

n3 +1.

1. Tìm a sao cho dãy số (xn) có giới hạn hữu hạn.

2. Tìm a để dãy số là dãy tăng từ một lúc nào đó.

Bài 30. (Quảng Ninh) Cho a,b là các số thực dương. Xét dãy số un được xác định bởiun = a2n2 +bn, với n ∈N∗. Tính Li m

{pun

}.

Bài 31. (Quảng Trị) Cho dãy số xn xác định bởix1 = 2

xn+1 = 2xn +1

xn +2

Tìm số hạng tổng quát xn và tìm lim xn.

Bài 32. (Thái Bình) Cho dãy số (an) có a1 ∈R và an+1 =∣∣∣an −21−n

∣∣∣ ,∀n ∈N∗. Tìm lim an .

Bài 33. (Thanh Hóa) Tìm tất cả các hàm số f :R→R thỏa mãn:

f ( f (x)+ f (y)) = f (x2)+2x2 f (y)+ ( f (y))2

với mọi x, y ∈R

Bài 34. (Thanh Hóa) Với số thực a 6= −1p2

cho trước ,xét dãy số an cho bởi

a1 = a

an+1 =7√

2a2n +7−14

4an +√

2a2n +7

Xác định a để dãy có giới hạn hữu han.

18 http://www.facebook.com/thcmn

Page 19: t 2017 - WordPress.com...Bài 5. (B›c Ninh) Cho a, b c¨0 thäa mãn đi•u ki»n ¯ ˘9. Tìm giá trà lîn nh t cıabi”uthøc T ˘ ab 3a¯4b¯5c ¯ bc 3b¯4c¯5a ¯ ca 3c¯4a¯5b

4 Hình học

Bài 1. (THPT chuyên KHTN, ĐH KHTN, ĐHQG HN) 4ABC nhọn (AB < AC ) có H ,O

lần lượt là trực tâm và tâm đường tròn ngoại tiếp. Điểm E thuộc cạnh AC sao choOE//BC . OE cắt đường tròn ngoại tiếp 4EBC tại F . Tiếp tuyển tại F của đường trònngoại tiếp 4EBC cắt BC , AH ở P,Q.

1. Chứng minh đường tròn (K ) ngoại tiếp 4BPQ đi qua trung điểm M của AH .

2. PA,PH cắt (K ) ở S,T khác P . Chứng minh hai tiếp tuyển của (K ) tại S,T cắt nhautại một điểm trên ME .

Bài 2. (THPT chuyên KHTN, ĐH KHTN, ĐHQG HN) Tứ giác ABC D nội tiếp (O) saocho ABC D không phải hình thang. Tiếp tuyến tại C ,D của (O) cắt nhau tại T . T A cắtBD tại S, E dối xứng với D qua S. AB cắt đường tròn ngoại tiếp 4EBC tại F . EC cắt T Atại P .

1. Chứng minh rằng PF tiếp xúc đường tròn ngoại tiếp 4EBC .

2. Giả sử PF cắt AC tại Q, H ,K lần lượt là hình chiếu của Q lên F A,FC . M là trungđiểm F A. Chứng minh rằng tiếp tuyến qua A của (O) và đường thẳng qua Qvsong song AO cắt nhau trên đường tròn ngoại tiếp 4M HK .

Bài 3. (THPT chuyên KHTN, ĐH KHTN, ĐHQG HN) 4ABC nhọn nội tiếp (O) có H

là trực tâm. P là một điểm nằm trên trung trực của BC và nằm trong 4ABC . Đườngthẳng qua A song song PH cắt (O) tại E khác A. Đường thẳng qua E song song AH cắt(O) tại F khác E . Gọi Q là điểm đối xứng với P qua O. Đường thẳng qua F song songvới AQ cắt PH tại G.

1. Chứng minh rằng B ,C ,P,G cùng thuộc một đường tròn tâm K .

2. AQ cắt (O) tại R khác A. PQ cắt F R tại L. Chứng minh K L =OP .

Bài 4. (THPT chuyên KHTN, ĐH KHTN, ĐHQG HN) 4ABC nội tiếp (O), ngoại tiếp(I ). Đường tròn qua B ,C tiếp xúc (I ) tại P . AI giao BC tại X . Tiếp tuyến qua X của (I )khác BC , giao tiếp tuyến tại (I ) tại P tại S. AS giao (O) tại T khác A. Chứng minh rằng�AT I = 90o

Bài 5. (Trường Phổ thông Năng khiếu, ĐHQG Tp. HCM) Cho 4ABC nhọn. Đườngtròn (I ) có tâm I thuộc BC và tiếp xúc với các cạnh AB , AC lần lượt tại E ,F .Llấy haiđiểm M , N bên trong tứ giác BC EF sao cho tứ giác EF N M nội tiếp (I ) và các đườngthẳng BC , M N ,EF đồng quy. MF cắt N E tại P , AP cắt BC tại D.

1. Chứng minh A,D,E ,F cùng thuộc một đường tròn.

2. Trên đường thẳng B N ,C M lấy các điểm H ,K sao cho �AC H = �ABK = 90o. Lấy T làtrung điểm HK . Chứng minh T B = TC .

http://thcmn.wordpress.com 19

Page 20: t 2017 - WordPress.com...Bài 5. (B›c Ninh) Cho a, b c¨0 thäa mãn đi•u ki»n ¯ ˘9. Tìm giá trà lîn nh t cıabi”uthøc T ˘ ab 3a¯4b¯5c ¯ bc 3b¯4c¯5a ¯ ca 3c¯4a¯5b

Bài 6. (Trường Phổ thông Năng khiếu, ĐHQG Tp. HCM) 4ABC có �B AC tù, H làchân đường cao kẻ từ A xuống BC . Điểm M thay đổi trên cạnh AB . Dựng N sao cho4B M N ∼4HC A (H , N ) nằm khác phía với AB .

1. C M cắt đường tròn ngoại tiếp 4B M N tại K khác M . Chứng minh N K luôn điqua điểm cố định.

2. N H cắt AC tại P . Dựng Q sao cho 4HPQ ∼ 4H N M (Q, M khác phía với N P .Chứng minh Q thuộc một đường thẳng cố định.

Bài 7. (THPT chuyên Đại học Sư phạm Hà Nội) Cho 4ABC . Đường tròn (I ) nội tiếp4ABC tiếp xúc với BC ,C A, AB tại D,E ,F . Đường thẳng D I cắt đường tròn tâm A bánkính AE tại M , N (N ) nằm giữa M và D). Các đường thẳng AD,EF cắt nhau tại P . cácđường thẳng M A, N P cắt nhau tại Q. Gọi H là giao điểm thứ hai của AD và (I ). Đườngthẳng qua trung điểm của D H ,DE cắt AC tại L. Chứng minh rằng:

1. QH ⊥ AD.

2. DL//EF .

Bài 8. (THPT chuyên Đại học Sư phạm Hà Nội) Hai đường tròn (O1) và (O2) tiếp xúcngoài nhau tại A. BC là một tiếp tuyến chung ngoài của (O1) và (O2), với B ∈ (O1) vàC ∈ (O2). Gọi M là trung điểm BC , P,Q theo thứ tự là điểm đối xứng của B ,C qua O1,O2.MP theo thứ tự cắt BO2,B A tại X ,Y . MQ cắt CO1,C A tại Z ,T . Chứng minh rằng:

1. Các tứ giác B Z T P,C X Y Q nội tiếp.

2. AM , Z T, X Y đồng quy.

Bài 9. (THPT chuyên ĐH Vinh) 4ABC vuông tại A nội tiếp đường tròn (O). Đườngthẳng qua A song song BC cắt (O) tại điểm thứ hai là D. Gọi I là giao điểm của AC vàBD. Đường thẳng qua I song song AB cắt AD tại J . Đường tròn tâm C bán kính C I cắt(O) tại E và F (E thuộc cung B AC .

1. Gọi S là giao điểm của I J với C D. Chứng minh S,E ,F thẳng hàng.

2. Chứng minh E J ⊥ AF .

Bài 10. (THPT chuyên ĐH Vinh) 4ABC có M di chuyển trên cạnh AC . Đường trònngoại tiếp 4AB M cắt cạnh BC tại điểm thứ hai là D. Đường tròn ngoại tiếp 4BC Mcắt cạnh AB tại điểm thứ hai là E .

1. Gọi O là giao điểm của AD và C E . Chứng minh A,E ,O, M cùng thuộc một đườngtròn.

2. Gọi I , J , N lần lượt là giao điểm của các cặp đường thẳng AB và DM , BC và E M ,AJ và C I . Chứng minh đường thẳng M N luôn đi qua điểm cố định.

20 http://www.facebook.com/thcmn

Page 21: t 2017 - WordPress.com...Bài 5. (B›c Ninh) Cho a, b c¨0 thäa mãn đi•u ki»n ¯ ˘9. Tìm giá trà lîn nh t cıabi”uthøc T ˘ ab 3a¯4b¯5c ¯ bc 3b¯4c¯5a ¯ ca 3c¯4a¯5b

Bài 11. (Bà Rịa - Vũng Tàu) Trên nửa đường tròn tâm O đường kính AB lấy hai điểmM , N sao cho AM < AN và M N không song song với AB . Đường tròn ngoại tiếp 4OM Ncắt AB tại điểm D khác O. Đường thẳng AN cắt đường tròn ngoại tiếp 4MBD tại haiđiểm E ,F (E nằm giữa A và N ). Đường thẳng B M cắt đường tròn ngoại tiếp 4N AD tạihai điểm P,Q (P nằm giữa B và M).

1. Chứng minh E ,F,P,Q cùng thuộc một đường tròn.

2. Đường thẳng AP cắt BE tại điểm X , đường thẳng BE cắt AQ tại điểm Y . Chứngminh bốn đường thẳng EP,QF, X Y , AB đồng quy.

Bài 12. (Bắc Ninh) Tứ giác ABC D nội tiếp đường tròn (O). Giả sử AD cắt BC tại N ,AB cắt C D tại M , AC cắt BD tại E . Đường thẳng OE cắt M N tại K . Chứng minh KO làphân giác của �BK D.

Bài 13. (Bến Tre) Cho đường tròn (O1), (O2) tiếp xúc ngoài tại điểm T . Một đườngthẳng cắt đường tròn (O1) tại hai điểm A,B phân biệt và tiếp xúc với (O2) tại X . đườngthẳng X T cắt (O1) Tại S ( S khác T và C là một điểm trên cung TS không chứa A và B .Cho C Y là tiếp tuyến của (O2) tại Y sao cho các đoạn thẳng C Y và ST không cắt nhau.Cho I là giao điển của các đường thẳng X Y và SC . Chứng minh rằng:

1. C ,T,Y và I cùng thuộc một đường tròn.

2. S A = SI

Bài 14. (Bình Dương) 4ABC nội tiếp đường tròn (O). Điểm P nằm trong 4ABC saocho PA,PB ,PC cắt (O) lần lượt tại D,E ,F . Tiếp tuyến tại A của (O) cắt BC tại T . Chứngminh rằng nếu T A = T P thì DE = DF .

Bài 15. (Bình Định) Cho 2 đường tròn (O), (O′) cắt nhau tại A,B . trên tia B A lấy M (M

nằm ngoài (O′)). Từ M Kẻ 2 tiếp tuyến MC , MD của (O′) (C ,D là 2 tiếp điểm). AC , ADlần lượt cắt (O) tại P,Q.

1. Chứng minhD A

DQ= C A

CQ.

2. Chứng minh C đi qua trung điểm PQ.

3. Chứng minh C D đi qua một điểm cố định khi M di chuyển trên tia B A.

Bài 16. (Bình Thuận) Cho tam giác ABC với I là tâm đường tròn nội tiêp và M là mộtđiểm nằm trong giác. Gọi A1,B1,C1 là các điểm đối xứng với điểm M lần lượt qua cácđường thẳng AI ,B I ,C I . Chứng minh rằng các đường thẳng A A1,BB1,CC1 đồng quy.

Bài 17. (Bình Thuận) Cho tứ giác ABC D nội tiếp đường tròn (O). Gọi M , N ,P lần lượtlà giao điểm của AB và C D, AC và BD. Lấy K là trung điểm của đoạn M N . Đoạn PK

cắt (O) tại H , M H cắt (O) tại I khác H , N H cắt (O) tại J khác H . Hãy phân tích−−→PK theo

hai vectơ−−→M I ,

−−→M J .

Bài 18. ((Đà Nẵng) Cho tam giác ABC nhọn nội tiếp (O), H là trực tâm tam giác.Đường thẳng qua A vuông góc OH cắt BC tại D. K .L là tâm (ADB), (ADC ).

http://thcmn.wordpress.com 21

Page 22: t 2017 - WordPress.com...Bài 5. (B›c Ninh) Cho a, b c¨0 thäa mãn đi•u ki»n ¯ ˘9. Tìm giá trà lîn nh t cıabi”uthøc T ˘ ab 3a¯4b¯5c ¯ bc 3b¯4c¯5a ¯ ca 3c¯4a¯5b

1. Chứng minh A,K ,L,O thuộc một đường tròn gọi là (S).

2. AH cắt lại (S) tại E .F đối xứng với E qua BC . Chứng minh H A = HF.

Bài 19. (Đà Nẵng) Cho tứ giác ABC D lồi, P là điểm nằm bên trong tứ giác thỏa�PAD = �C AB .M , N đối xứng C qua AB , AD. (C P M), (C P N ) cắt đoạn AB , AD tại S,T.X ,Ytâm (PSC ), (PTC ).Q là giao của X Y và trung trực AP.

1. Chứng minh AQ là tiếp tuyến đường tròn ngoại tiếp 4AX Y .

2. Tiếp tuyến tại P,C của (PST ), (C ST ) cắt nhau ở G .(PST ), (C ST ) cắt lại AP, AC ởU ,V. Chứng minh tâm (AUV ) thuộc AG.

Bài 20. (Đồng Nai) Cho tam giác ABC nhọn (AB < AC ) có M là trung điểm BC , cácđường cao AD,BE ,C F cắt nhau tại H . Gọi K là trung điểm AH , L là giao điểm EF vàAH , N là giao điểm của đoạn AM và đường tròn ngoại tiếp 4BC H .

1. Chứng minh rằng 5 điểm A,E , N , H ,F cùng thuộc một đường tròn.

2. Chứng minh rằng �H M A = �LN K .

Bài 21. (Đồng Nai) Gọi O, I lần lượt là tâm đường tròn ngoại tiếp và nội tiếp của4ABC không đều. Chứng minh rằng

�AIO ≤ 900 ⇔ 2BC ≤ AB + AC .

Bài 22. (Hà Nội) 4ABC nhọn (AB < AC ) có trung tuyến AM . Đường thẳng AM cắtđường tròn ngoại tiếp 4ABC tại điểm thứ hai D. Đường thẳng AB và C D cắt nhautại E , đường thẳng AC và BD cắt nhau tại F . ĐƯờng tròn ngoại tiếp 4ABF cắt đườngtròn ngoại tiếp 4AC E tại điểm thứ hai P . Gọi (S1) là đường tròn đi qua C và tiếp xúcvới AB tại A, (S2) là đường tròn đi qua B và tiếp xúc AC tại A. (S1) cắt (S2) tại điểm thứhai Q. Chứng minh 4OPQ vuông.

Bài 23. (Hà Tĩnh) 4ABC nhọn không cân nội tiếp đường tròn (O). Gọi H là trực tâm4ABC , A′,B ′,C ′ lần lượt là chân các đường vuông góc kẻ từ A,B ,C xuống BC ,C A, AB .Gọi A1,B1,C1 là các điểm trên (O) sao cho A A1//BC ,BB1//C A,CC1//AB . A′

1,B ′1,C ′

1 là cácđiểm trên (O) sao cho A1 A′

1//A A′, B1B ′1//BB ′,C1C ′

1//CC ′.

1. Chứng minh các đường tròn ngoại tiếp các tam giác O A1 A′1,OB1B ′

1,OC1C ′1 cùng

đi qua một điểm K khác O.

2. Chứng minh OK .OH = abc

p, trong đó a,b,c là độ dài 3 cạnh của 4ABC và p là

chu vi tam giác A1B1C1.

22 http://www.facebook.com/thcmn

Page 23: t 2017 - WordPress.com...Bài 5. (B›c Ninh) Cho a, b c¨0 thäa mãn đi•u ki»n ¯ ˘9. Tìm giá trà lîn nh t cıabi”uthøc T ˘ ab 3a¯4b¯5c ¯ bc 3b¯4c¯5a ¯ ca 3c¯4a¯5b

Bài 24. (Hà Tĩnh) 4ABC nhọn nội tiếp đường tròn (O), E là trung điểm cung BC

không chứa A. Gọi D là giao điểm AE và BC . P,Q lần lượt là hai điểm di động trênđoạn AD sao cho �ABQ = �DBP và Q nằm giữa A,P . Lấy điểm S sao cho QS//AO vàDS ⊥ AO. Gọi M là trung điểm BC , N là điểm đối xứng của M qua AE , R là hình chiếuvuông góc của Q lên BC .

1. Chứng minhM N

MR= 2ME

QE.

2. Chứng minh đường thẳng qua P vuông góc SM đi qua một điểm cố định khiP,Q thay đổi.

Bài 25. (Hải Phòng) 4ABC nhọn (AB < AC ) có AD là phân giác trong đỉnh A (D ∈ BC ).E là điểm trên đoạn BC sao cho BD = C E . Phân giác ngoài đỉnh A cắt đường thẳngqua D và vuông góc với BC tại F . Gọi I là trung điểm DF , đường thẳng E I cắt AD tạiM , đường thẳng EF cắt đường thẳng qua M và vuông góc với BC tại K .

1. Đường thẳng AF cắt đường thẳng BC tại P , K D cắt đường tròn đường kính DFtại L khác D. Chứng minh đường thẳng PL tiếp xúc với đường tròn đường kínhDF .

2. Chứng minh I là tâm đường tròn nội tiếp 4K BC .

Bài 26. (Hải Phòng) 4ABC nhọn (AB < AC ) có trực tâm H , D,E là chân các đườngcao kẻ từ B và C xuống C A, AB . Gọi M là trung điểm BC , F là giao điểm của hai đườngthẳng DE và BC , K là giao điểm thứ hai của AM với đường tròn (C ) ngoại tiếp 4ADE .

1. Chứng minh F, H ,K thẳng hàng.

2. Gọi S là trung điểm MF , T là giao điểm của đường thẳng DE với đường thẳngqua A và song song với BC . Chứng minh đường thẳng ST tiếp xúc với đườngtròn (C ).

3. Chứng minh các đường tròn ngoại tiếp các tam giác O A1 A′1,OB1B ′

1,OC1C ′1 cùng

đi qua một điểm K khác O.

4. Chứng minh OK .OH = abc

p, trong đó a,b,c là độ dài 3 cạnh của 4ABC và p là

chu vi tam giác A1B1C1.

Bài 27. (Hoà Bình) Cho tam giác ABC nhọn nội tiếp đường tròn (O) và P là một điểmnằm trên cung nhỏ BC . Tiếp tuyến tại B ,C cắt nhau tại T . Đường thẳng qua O vàvuông góc với PT cắt C A, AB lần lượt tại E ,F . Hai đường thẳng PE ,PF cắt đường tròn(O) lần lượt tại M , N khác P . Lấy K ,L sao cho �K AC = �K N P = �L AB =�LMP = 90o.

1. Chứng minh rằng �BQF = �K AB với Q là giao của EF với PT .

2. Chứng minh rằng K B và LC vắt nhau tại 1 điểm thuộc (O).

http://thcmn.wordpress.com 23

Page 24: t 2017 - WordPress.com...Bài 5. (B›c Ninh) Cho a, b c¨0 thäa mãn đi•u ki»n ¯ ˘9. Tìm giá trà lîn nh t cıabi”uthøc T ˘ ab 3a¯4b¯5c ¯ bc 3b¯4c¯5a ¯ ca 3c¯4a¯5b

Bài 28. (Hoà Bình) Cho hai đường tròn (O1) và (O2) cắt nhau tại A,B . C D là tiếp tuyếnchung của hai đường tròn (O1) và (O2) với C ∈ (O1) , D ∈ (O) , và B gần C D hơn A.

1. Gọi E là giao điểm của BC và AD, F là giao điểm của DB và AC . Chứng minhrằng EF //C D.

2. Gọi N là giao điểm của AB và EF . Lấy K trên C D sao cho �B AC = �D AK . Chứngminh rằng K E = K F .

Bài 29. (Tp. HCM) Cho 4ABC có tâm đường tròn nội tiếp I . Gọi M , N ,P lần lượt làcác điểm nằm trên các cạnh AB ,BC ,C A sao cho AN − AP = BP −B M = C P −C M . GọiX ,Y , Z lần lượt là tâm đường tròn nội tiếp của 4AN P,4BP M ,4C M N . Chứng minhrằng I là tâm đường tròn nội tiếp của 4X Y Z .

Bài 30. (Tp. HCM) Đường tròn nội tiếp 4ABC lần lượt tiếp xúc với BC ,C A, AB tạiA1,B1,C1.

1. Chứng minh các đường thẳng A A1,BB1,CC1 đồng quy tại một điểm T .

2. Đường tròn ω1 đi qua T tiếp xúc với C A,C B tại B2, A3. Các điểm C2, A2,B3,C3 xácđịnh tương tự. Chứng minh sáu điểm A2,B2,C2, A3,B3,C3 cùng thuộc một đườngtròn.

Bài 31. (Khánh Hoà) Tứ giác ABC D nội tiếp đường tròn (O). Đường thẳng qua C cắtcác tia đối của tia B A,D A lần lượt tại M , N . Chứng minh

4S ABC

S AM N≤

(BD

AC

)2

.

Bài 32. (Khánh Hoà) 4ABC nhọn có AM ,B N ,C P là các trung tuyến. Gọi R và r lần

lượt là bán kính đường tròn ngoại tiếp và đường tròn nội tiếp 4ABC . Chứng minh

AM +B N +C P ≤ 4R + r.

Bài 33. (Lào Cai) Cho tam giác nhọn ABC có đường cao AH , trực tâm K . Đường

thẳng BK cắt đường tròn đường kính (C A) tại D,E (BD < BE). Đường thẳng C K cắt(AB) tại F,G (C F <CG). Và (D HF ) cắt BC tại điểm thứ hai là P .

1. Chứng minh rằng các điểm G , H ,P,E cùng thuộc một đường tròn.

2. Chứng minh rằng các đường thẳng BF,C D,PK đồng quy.

Bài 34. (Lạng Sơn) Cho 4ABC nhọn nội tiếp (O) với I là tâm nội tiếp tam giác. Đườngtròn đi qua C tiếp xúc với AI tại I cắt AC tại E và cắt (O) tại H (E , H 6=C ).

1. Chứng minh E H đi qua trung điểm của AI .

2. Đường tròn đi qua B tiếp xúc với AI tại I cắt AB tại F và cắt (O) tại G (G ,F 6= B).Chứng minh rằng 2 đường tròn (E I F ) và (G I H) tiếp xúc nhau.

24 http://www.facebook.com/thcmn

Page 25: t 2017 - WordPress.com...Bài 5. (B›c Ninh) Cho a, b c¨0 thäa mãn đi•u ki»n ¯ ˘9. Tìm giá trà lîn nh t cıabi”uthøc T ˘ ab 3a¯4b¯5c ¯ bc 3b¯4c¯5a ¯ ca 3c¯4a¯5b

Bài 35. (Lạng Sơn) Cho 4ABC nhọn nội tiếp (O). Các đường cao AD,BE ,C F cắt nhauttai5 H (D ∈ BC ,E ∈ C A,F ∈ AB). Gọi M là trung điểm của BC . 2 đường tròn (DEF ) và(HBC ) cắt nhau tại X và Y

1. Chứng minh AX = AY .

2. Gọi R là trung điểm của X Y . AR cắt H M tại S. Chứng minh tứ giác HDSR nộitiếp.

Bài 36. (Long An) Từ một điểm M tùy ý trong tam giác ABC , các đường thẳngM A, MB , MC lần lượt cắt BC ,C A, AB tại A1,B1,C1. Chứng minh rằng

M A1

A A1+ MB1

BB1+ MC1

CC1= 1.

Bài 37. (Nghệ An) Cho hai đường tròn (O1) và (O2) cắt nhau tại A và B . Một đườngthẳng qua B cắt hai đường tròn (O1), (O2) lần lượt tại C và D. Gọi M là trung điểmC D. Đường thẳng AM cắt (O1) tại điểm thứ hai là K (K và C khác phía so với AB), cắtđường tròn (O2) tại điểm thứ hai P . Đường thẳng (d) qua M vuông góc với O1M cắtđường thẳng AC tại Q và cắt đường thẳng K B tại R.

1. Gọi I , J lần lượt là giao điểm củaAB ,C K với đường thẳng (d). Chứng minh M làtrung điểm I J vàRQ.

2. Chứng minh đường thẳng PQ đi qua một điểm cố định.

Bài 38. (Nghệ An) 4X Y Z đều, các đỉnh X ,Y , Z lần lượt nằm trên các cạnh BC ,C A, AB

của 4ABC nhọn. Chứng minh tâm đường tròn nội tiếp 4ABC nằm trong 4X Y Z .

Bài 39. (Ninh Bình) Cho 4ABC . Đường tròn (O) đi qua A và C cắt các cạnh AB ,BC

tại K , N . Đường tròn (K B N ) cắt đường tròn (ABC ) tại B và M . Tính �B MO.

Bài 40. (Ninh Bình) Cho 4ABC và điểm (O) nằm trog 4ABC Đường thẳng d1 đi quaO song song với BC lần lượt cắt AB , AC tại J ,G. Đường thẳng d2 đi qua O song song vớiC A lần lượt cắt BC ,C A tại F, J . Đường thẳng d3 đi qua O song song với AB lần lượt cắtC A,C B tại H ,E . Dựng các hình bình hành OE A1F,OGB1H ,OIC1 J . Chứng minh rằngcác đường thẳng A A1,BB1,CC1 đồng quy.

Bài 41. (Nam Định) Cho tam giác nhọn ABC không cân nội tiếp đường tròn ω tâmO. Một đường tròn ω, đi qua B ,C cắt các cạnh AB , AC lần lượt ở E ,F (E ,F 6= A). Đườngtròn ngoại tiếp tam giác AEF cắt lại đường tròn ω tại K (A 6= K ). K E ,K F lần lượt cắtlại đường tròn ω tại Q,P (P,Q 6= K ). Gọi T là giao điểm của BQ và C P ; M , N lần lượt làtrung điểm BF,C E .

1. Chứng minh rằng A,O,T thẳng hàng.

2. Chứng minh rằng K A là tiếp tuyến của đường tròn ngoại tiếp tam giác AM N .

http://thcmn.wordpress.com 25

Page 26: t 2017 - WordPress.com...Bài 5. (B›c Ninh) Cho a, b c¨0 thäa mãn đi•u ki»n ¯ ˘9. Tìm giá trà lîn nh t cıabi”uthøc T ˘ ab 3a¯4b¯5c ¯ bc 3b¯4c¯5a ¯ ca 3c¯4a¯5b

Bài 42. (Nam Định) 4ABC nhọn nội tiếp đường tròn (O) ngoại tiếp đường tròn (I ).Gọi P là trung điểm cung BC không chứa A của (O), J là điểm đối xứng với I qua O.Tiếp tuyến tại I của đường tròn ngoại tiếp 4I BC cắt BC tại M , H là hình chiếu củaM trên OI . Gọi D là trung điểm BC và K là giao điểm thứ hai của I D với đường trònngoại tiếp 4OD H .

1. Chứng minh rằng 4JP M vuông ở J .

2. Chứng minh H , A,K thẳng hàng.

Bài 43. (Phú Thọ) Cho tam giác ABC ngoại tiếp (I ). Các cạnh AB , AC tiếp xúc với (I )

tại E ,F . Đường thằng qua B song song với AC cắt EF tại K . C K cắt AB tại G. Chứngminh tam giác AIG vuông.

Bài 44. (Phú Thọ) Cho đường tròn (O) và dây cung AB . Các đường tròn (O1) và (O2)

nằm về một phía đối với đường thẳng AB , Tiếp xúc với nhau tại T đồng thời tiếp xúctrong với đường tròn (O). Tiếp tuyến chung tại T của (O1), (O2) cắt đường tròn (O) tại C(Với C thuộc nửa mặt phẳng bờ AB chứa (O1), (O2)). Chứng minh rằng T là tâm đườngtròn nội tiếp tam giác ABC .

Bài 45. (Quảng Bình) Cho tam giác ABC nhọn không cân. P là một điểm bất kì trêncạnh BC và không trùng với B ,C . Đường tròn ngoại tiếp tam giác ABP cắt AC tại Ykhác A. Tương tự xác định Z . Gọi BY cắt C Z tại K . Gọi T là hình chiếu của A lên BC ,H là trực tâm tam giác ABC , A′ là điểm đối xứng của A qua BC .

1. Chứng minh A′,P,K thẳng hàng.

2. Chứng minh khi P di chuyển trên BC , tâm đường tròn ngoại tiếp tam giác AY Zluôn di chuyển trên một đường thẳng cố định.

Bài 46. (Quảng Nam) Cho đường tròn (O) và dây cung BC khác đường kính. Điểm A

di động trên cung lớn BC sao cho tam giác ABC không cân tại A. Gọi M là giao điểmcủa hai tiếp tuyến với (O) tại B và C , AM cắt (O) tại D khác A. Dựng đường kính DEcủa (O). Các đường thẳng BD,C E cắt nhau tại X , các đường thẳng BE ,C D cắt nhau tạiY .

1. Chứng minh rằng M X = MY .

2. Gọi N là giao điểm của AE và X Y . Chứng minh rằng N nằm trên một đườngthẳng cố định.

Bài 47. (Quảng Ngãi) 4ABC nội tiếp đường tròn (O;R), D là điểm di chuyển trêncung BC không chứa A (D không trùng B ,C ).

1. Gọi E ,F lần lượt là giao điểm của các đường thẳng AD và BC , AB và C D. Gọi P làtrung điểm EF , M là điểm đối xứng với B qua P . Chứng minh đường thẳng DMluôn đi qua một điểm cố định.

2. Gọi H , I ,K lần lượt là chân đường vuông góc kẻ từ D đến các cạnh BC ,C A, AB .

Cho BC = Rp

3. Tìm GTNN của tổngAB

DK+ AC

D I+ BC

D H.

26 http://www.facebook.com/thcmn

Page 27: t 2017 - WordPress.com...Bài 5. (B›c Ninh) Cho a, b c¨0 thäa mãn đi•u ki»n ¯ ˘9. Tìm giá trà lîn nh t cıabi”uthøc T ˘ ab 3a¯4b¯5c ¯ bc 3b¯4c¯5a ¯ ca 3c¯4a¯5b

Bài 48. (Quảng Ninh) Cho 4ABC nhọn ko cân tại A nội tiếp (O) . Gọi M là trungđiểm của BC . 2 tiếp tuyến tại B và C của (O) cắt nhau tại P . Gọi D,E lần lượt là hìnhchiếu vuông góc của B va C lên AP , Q là giao điểm của AP và BC . Đường tròn (CQD)và (BQE) lần lượt cắt (O) tại điểm thứ 2 là K ,L.

1. Chứng minh �MKC =�MLB .

2. Kẻ đường kính AT của (O). Giả sử các đường thẳng T B ,TC , AK , AL đôi 1 cắt nhautại 4 điểm phân biệt. CMR: 4 giao điểm này là 4 đỉnh của 1 tứ giác nội tiếp.

Bài 49. (Quảng Ninh) Cho 4ABC nhọn nội tiếp (O) (AB < AC ), trực tâm H . Lấy điểmT trên (O) sao cho AT ∥ BC . Giả sử AH cắt (O) tại K , T H cắt (O) tại D trên cung nhỏBC . Gọi I là trung điểm của HT .

1. Chứng minh 5 điểm A,L,O,K ,D cùng thuộc 1 đường tròn.

2. Gọi P là giao điểm thứ 2 của AO với (O). Đường thẳng qua H song song với BCcắt PD tại X . Chứng minh rằng X A là tiếp tuyến của (O).

Bài 50. (Quảng Trị) Cho tứ giác ABC D nội tiếp đường tròn (O). Gọi I là giao điểm AC

và BD, H và K lần lượt là trực tâm của 4I AD và 4I BC . M và N lần lượt là trung điểmAB và C D; Pvà Q lần lượt là chân đường vuông góc kẻ từ I đến BC và AD.

1. Chứng minh HK ⊥ M N .

2. Chứng minh M N đi qua trung điểm PQ.

Bài 51. (Quảng Trị) Cho tam giác ABC nội tiếp đường tròn (O). Đường tròn (O′) tiếpxúc với hai cạnh AB , AC theo thứ tự tại P.Q và tiếp xúc trong với (O) tại S. Hai đườngthẳng SP.SQ cắt lại (O) theo thứ tự tại M , N . Gọi E ,D,F theo thứ tự là hình chiếu vuônggóc của S trên các đường thẳng AM , M N , AN .

1. Chứng minh SM .AN = SN .AM .

2. Chứng minh DE = DF .

Bài 52. (Thái Bình) Cho đường tròn (O) có hai đường kính AB và C D. Tiếp tuyến vớiđường tròn (O) tại B cắt AC tại P . Gọi G là giao điểm thứ hai của đường thẳng DP với(O), I là trung điểm AP . Chứng minh rằng:

1. O,P,C , I cùng thuộc một đường tròn.

2. AG ,BC ,OP đồng quy.

Bài 53. (Thái Nguyên) Cho tam giác ABC nhọn, có AC > AB . Gọi D là hình chiếu củaA trên BC và E là hình chiếu của D trên AC . Xét điểm F trên đoạn DE . Chứng minhrằng

AF ⊥ BF ⇐⇒ EF.DC = BD.DE

http://thcmn.wordpress.com 27

Page 28: t 2017 - WordPress.com...Bài 5. (B›c Ninh) Cho a, b c¨0 thäa mãn đi•u ki»n ¯ ˘9. Tìm giá trà lîn nh t cıabi”uthøc T ˘ ab 3a¯4b¯5c ¯ bc 3b¯4c¯5a ¯ ca 3c¯4a¯5b

.

Bài 54. (Thanh Hoá) Cho tam giác ABC không cân tại A.Gọi D,E ,F lần lượt là tiếpđiểm của đường tròn nội tiếp 4ABC với BC ,C A, AB . Đường tròn ngoại tiếp 4ABC cắtđường thẳng EF tại Pvà Q. Gọi M là trung điểm BC , O1 và O2 lần lượt là tâm đườngtròn ngoại tiếp các tam giác I AB và I AC .

1. Chứng minh rằng D,P,Q, M nằm trên một đường tròn.

2. Chứng minh rằng tâm đường tròn ngoại tiếp 4DPQ nằm trên đường thẳngO1O2.

Bài 55. (Vĩnh Phúc) 4ABC nhọn nội tiếp đường tròn (O) có AB < AC . Gọi D,E lầnlượt là trung điểm AB , AC . Các đường tròn ngoại tiếp 4ABE và 4AC D cắt nhau tại Kkhác A. Đường thẳng LB cắt đường tròn ngoại tiếp 4ABE tại điểm thứ hai M , đườngthẳng LC cắt đường tròn ngoại tiếp AC D tại điểm thứ hai N .

1. Chứng minh M ,K ,L thẳng hàng và M N ⊥OL.

2. Chứng minh K là trung điểm M N .

Bài 56. (Vĩnh Phúc) 4ABC nhọn nội tiếp đường tròn (O), P nằm trong 4ABC nằmtrên đường phân giác trong góc �B AC . Gọi K ,L lần lượt là giao điểm khác P của BPvới (APC ), C P với (APB). Gọi E ,F lần lượt là giao điểm của các phân giác trong góc�ABC , �AC B với (O). các đường thẳng AE , AF cắt (APC ), (APB) lần lượt tại M , N khác A.Chứng minh K ,L, M , N cùng thuộc một đường tròn.

5 Phương trình và hệ phương trình

Bài 1. (Bắc Ninh) Giải hệ phương trình:6x(x +1)+ y(y −4x −1) = 2

[(x +1)

√x2 + y −1

]2x +5x = 13− y

6+44l og2

(2+ 131x

3−5x

) (x, y ∈R)

Bài 2. (Bến Tre) Giải hệ phương trình :{x3 − y3 +6y2 +2(x −7y)+12 = 0p

3−x +√y −3 = x2 + y2 −10x −5y +22

(x, y ∈R)

Bài 3. (Hà Nội)

1. Giải phương trình √2x2 −2x +4+

√5x2 +4+x2 −7x +1 = 0

28 http://www.facebook.com/thcmn

Page 29: t 2017 - WordPress.com...Bài 5. (B›c Ninh) Cho a, b c¨0 thäa mãn đi•u ki»n ¯ ˘9. Tìm giá trà lîn nh t cıabi”uthøc T ˘ ab 3a¯4b¯5c ¯ bc 3b¯4c¯5a ¯ ca 3c¯4a¯5b

2. Giải hệ phương trình {√2x + y +√

x +4y = 5√x +4y +2x − y = 3

Bài 4. (Hòa Bình) Giải hệ phương trình{ √y −3x +4+√

y +5x +4 = 4√5y +3−p

7x −2 = 2x −1−4y

Bài 5. (Khánh Hòa) Giải phương trình trên tập số thực

(x −4)p

x −2−1p4−x +x −5

= 2+ (2x −4)p

x −2

x −1

Bài 6. (Khánh Hòa) Giải hệ phương trìnhx +√

x(x2 +3x +3) = 3√

y +4+√y +3−1 =

7p

x +1−√

x2 +18x +18 = 3√

y +4+1

Bài 7. (Lào Cai) Giải hệ phương trình(x +1)√

y2 + y +2+ (y −1)√

x2 +x +1 = x + y

(x2 +x)√

x − y +3 = 2x2 +x + y +1

Bài 8. (Ninh Bình) Giải hệ phương trình:{y3 +4x2 y +3x y2 = x6 +3x5 +4x4√

x2 +3+√3y +1 = 4

Bài 9. (Quảng Trị)

1. Giải phương trình px +3+p

3x +1 = 2p

x +p2x +2

2. Giải hệ phương trình x3 + y = 2(x2 +1)

2y3 + z = 2(2y2 +1)3z3 +x = 2(3z2 +1)

http://thcmn.wordpress.com 29

Page 30: t 2017 - WordPress.com...Bài 5. (B›c Ninh) Cho a, b c¨0 thäa mãn đi•u ki»n ¯ ˘9. Tìm giá trà lîn nh t cıabi”uthøc T ˘ ab 3a¯4b¯5c ¯ bc 3b¯4c¯5a ¯ ca 3c¯4a¯5b

Bài 10. (Thái Nguyên) Giải bất phương trình

2(1−x)√

x2 +2x −1 É x2 −2x −1

Bài 11. (Thái Nguyên) Giải hệ phương trình 2y3 + y +2xp

1−x = 3p

1−x√9−4y2 = 2x2 +6y2 −7

(x, y ∈R)

Bài 12. (Vĩnh Phúc) Giải hệ phương trình2x2 +7x +3+px +1 = y2 +2y −x y +p

y −x

3√

6+x − y +3√

3x + y −5 = y +6(x, y ∈R)

6 Số học

Bài 1. (Trường THPT chuyên KHTN - ĐH KHTN, ĐHQG HN)

1. Giải phương trình nghiệm nguyên dương (a, p,n) trong đó p là một số nguyêntố thỏa mãn :

a2(a2 +1) = 5n(5n+1 −p3).

2. Một số nguyên dương n ≥ 2 được gọi là tốt nếu với mọi 2 ≤ k ≤ n thì n có dạngn = a1 +a2 + ....+ak trong đó (n, ak ) = 1 và các số ai là nguyên dương . Tính tổngtất cả các số tốt nhỏ hơn 2016.

Bài 2. (Trường THPT chuyên KHTN - ĐH KHTN, ĐHQG HN) Tìm tất cả các số nguyêntố p sao cho 2p2 −1 là một lũy thừa của 7 .

Bài 3. (Trường THPT chuyên KHTN - ĐH KHTN, ĐHQG HN) Cho n nguyên dương.Chứng minh rằng tồn tại số nguyên dương m có n chữ số, chỉ gồm các chữ số 1,2,3và chia hết cho S(m)(tổng các chữ số của m) .

Bài 4. (Trường Phổ thông Năng khiếu, ĐHQG Tp.HCM) Cho hàm số f :N∗ →N∗ thỏamãn các điều kiện: f tăng thực sự và f (2n) = 2 f (n) với mọi số n nguyên dương.

1. Giả sử f (a) = 3 và p là số nguyên tố lớn hơn 3. Chứng minh tồn tại n sao chof (n) chia hết cho p.

2. Cho q là một số nguyên tố lẻ. Hãy xây dựng một hàm f thỏa mãn bài toán màf (n) không chia hết cho q với mọi số nguyên dương n.

30 http://www.facebook.com/thcmn

Page 31: t 2017 - WordPress.com...Bài 5. (B›c Ninh) Cho a, b c¨0 thäa mãn đi•u ki»n ¯ ˘9. Tìm giá trà lîn nh t cıabi”uthøc T ˘ ab 3a¯4b¯5c ¯ bc 3b¯4c¯5a ¯ ca 3c¯4a¯5b

Bài 5. (Trường Phổ thông Năng khiếu, ĐHQG Tp.HCM) Với mỗi số nguyên dương n

,tồn tại duy nhất số tự nhiên a thỏa mãn a2 ≤ n < (a +1)2. Đặt ∆n = n −a2.

1. Tìm giá trị nhỏ nhất của ∆n khi n thay đổi và luôn thỏa n = 15m2 với m là sốnguyên dương.

2. Cho p, q là các số nguyên dương và d = 5(4p +3)q2. Chứng minh ∆d ≥ 5.

Bài 6. (THPT chuyên ĐH Vinh) Tìm các số nguyên dương a,b,c,d thỏa mãn

a +2b +3c = 3d !+1,

biết rằng tồn tại các số nguyên tố p, q sao cho a = (p +1)(2p +1) = (q +1)(q −1)2

Bài 7. (Bắc Ninh) Cho dãy số (an) xác định bởi{a1 = 34

an+1 = 4a3n −104a2

n −107an.

với mọi số n nguyên dương. Tìm tất cả các số nguyên tố p thỏa mãn hai điều kiện

p ≡ 3( mod 4) và a2017 +1... p.

Bài 8. (Bến Tre) Cho p là số nguyên tố lẻ và T =p∑

k=0C k

pC kp+k −(2p +1). Chứng minh rằng

T chia hết cho p2 .

Bài 9. (Bình Dương) Tìm tất cả các số nguyên dương n sao cho phương trình sau cónghiệm nguyên dương

(x + y)2 = n(4x y +1).

Bài 10. (Hà Nội) Tìm tất cả các bộ ba số nguyên (x; y ; z) thỏa mãn 12x + y4 = 56z

Bài 11. (Hà Tĩnh) Tìm tất cả các cặp số nguyên dương (x; y) thỏa mãn

x19 −1 = (x −1)(y12 −1)

.

Bài 12. (Hải Dương) Tìm số nguyên dương n nhỏ nhất sao cho 1n +2n + . . .+2016n

không chia hết cho 2017.

Bài 13. (Hải Phòng)

1. Cho p là một số nguyên tố lẻ. Chứng minh rằng nếu or d p2 (2) = p(p − 1) và

vp (2p(p−1) −1) = 2 thì or d pk (2) = pk−1(p −1) và vp (2pk−1(p−1) −1) = k,∀k ≥ 2,trong đó or db(a) là cấp của a theo mod b và vp (a) là số mũ đúng của a theo modb.

http://thcmn.wordpress.com 31

Page 32: t 2017 - WordPress.com...Bài 5. (B›c Ninh) Cho a, b c¨0 thäa mãn đi•u ki»n ¯ ˘9. Tìm giá trà lîn nh t cıabi”uthøc T ˘ ab 3a¯4b¯5c ¯ bc 3b¯4c¯5a ¯ ca 3c¯4a¯5b

2. Có bao nhiêu số nguyên dương n ≤ 10.3310 thỏa mãn đồng thời

2n ≡ 2017 ( mod 311) ; 2n ≡ 2016 ( mod 113)?

Bài 14. (Tp. HCM) Với mỗi số nguyên dương n lớn hơn 1, số1

nđược biểu diễn dưới

dạng thập phân như sau1

n= 0, a1a2 . . .. Tìm tất cả các số n sao cho n = a1 +a2 + . . .

Bài 15. (Khánh Hòa) Cho các số nguyên tố thỏa mãn p1 < p2 < p3 < p4 và p4 −p1 = 8.Giả sử p1 > 5. Chứng minh rằng p1 chia 30 dư 11 .

Bài 16. (Kiên Giang) Chứng minh rằng nếu n và m = 2+2√

12n2 +1 là các số nguyêndương thì m là số chính phương .

Bài 17. (Lào Cai) Tìm tất cả các cặp số nguyên (x; y) thỏa mãn

9x2 +24x +15 = y3.

Bài 18. (Lạng Sơn) Cho đa thức P (x) = 4x3−18x2+27x+m. Chứng minh rằng với mỗi

m ∈Z, ∃n ∈Z sao cho P (n)...107 .

Bài 19. (Long An) Cho số nguyên dương n và d1 < d2 < d3 < d4 là 4 ước nguyên dươngnhỏ nhất của n. Tìm tất cả các số nguyên dương n thỏa mãn n = d 2

1 +d 22 +d 2

3 +d 24 .

Bài 20. (Nam Định) Tìm tất cả các số nguyên dương n có tính chất: nếu a,b là ướccủa n và (a,b) = 1 thì a +b −1 cũng là ước của n.

Bài 21. (Nghệ An) Cho số nguyên dương a không chính phương. Chứng minh số

{p

a}+ {p

a}2 + . . .+ {p

a}n

là số vô tỷ.

Bài 22. (Nghệ An)

1. Tồn tại hay không các số thực phân biệt a,b sao cho a +b ∉Q nhưng an +bn ∈Qvới mỗi số nguyên dương n > 1?

2. Cho a,b là các số thực khác nhau sao cho an−bn là số nguyên với mỗi số nguyêndương n. Hỏi a,b là nguyên, hữu tỷ hay vô tỷ?

Bài 23. (Ninh Bình) Chứng minh rằng với mỗi số nguyên dương m luôn tồn tại vô số

số nguyên dương n thỏa mãn (3.2n +n)...m .

Bài 24. (Phú Thọ) Với mỗi số nguyên dương n, gọi f (n) là số cách chọn các dấu cộng,trừ trong biểu thức En =±1±2±·· ·±n sao cho En = 0. Chứng minh rằng:

1. f (n) = 0 khi n ≡ 1,2( mod 4).

32 http://www.facebook.com/thcmn

Page 33: t 2017 - WordPress.com...Bài 5. (B›c Ninh) Cho a, b c¨0 thäa mãn đi•u ki»n ¯ ˘9. Tìm giá trà lîn nh t cıabi”uthøc T ˘ ab 3a¯4b¯5c ¯ bc 3b¯4c¯5a ¯ ca 3c¯4a¯5b

2. Khi n ≡ 0,3 ( mod 4) ta có

p2n

2≤ f (n) ≤ 2n −2

1+[n

2].

Bài 25. (Quảng Bình) Cho đa thức f (x) = x2017 +ax2 +bx + c, trong đó a,b,c ∈ Z có 3nghiệm nguyên x1, x2, x3. Chứng minh rằng biểu thức sau là bội của 2017:

(a2017 +b2017 + c2017 +1)(x1 −x2)(x2 −x3)(x3 −x1).

Bài 26. (Quảng Nam) Cho số nguyên tố p và các số nguyên dương a,b,c phân biệtnhỏ hơn p. Chứng minh rằng nếu các số a3,b3,c3 có cùng số dư khi chia cho p thìa2 +b2 + c2 chia hết cho a +b + c.

Bài 27. (Quảng Ngãi)

1. Tìm mọi nghiệm nguyên của phương trình x2+2016x+2017y2+y = x y+2017x y2+2018.

2. Cho m,n là các số nguyên dương, m là số lẻ. Chứng minh rằng 4m | 3n +1 khi vàchỉ khi 2m | 3n +1 và 3 | (2m)n +1.

Bài 28. (Quảng Ninh) Cho số nguyên n ≥ 2 thỏa mãn điều kiện 2ϕ(n)+3ϕ(n)+ ...+nϕ(n)

chia hết cho n, trong đó ϕ(n) là số các số nguyên dương ko vượt quá n và nguyên tốcùng nhau với n.

1. Chứng minh rằng n ko có ước số lớn hơn 1 nào là số chính phương.

2. Biết rằng n có ko quá 3 ước nguyên tố, tìm tất cả các số n thỏa mãn điều kiệntrên.

Bài 29. (Thái Bình) Giải phương trình nghiệm nguyên dương

x y y x = (x + y)x+y .

7 Tổ hợp

Bài 1. (THPT chuyên KHTN, ĐH KHTN, ĐHQG HN ) Trên một giá sách có n cuốnsách được đánh số từ 1 đến n (n ∈N∗ cho trước), ban đầu các cuốn sách được sắp xếptheo thứ tự nào đó. Một thủ thư muốn xếp lại theo đúng thứ tự 1,2, ...,n từ trái quaphải, quy tắc xếp như sau: Chọn quyển sách đầu tiên có số không đúng vị trí (tínhtừ bên phải sang) và chuyển cuốn sách đó về đúng vị trí của nó. Ví dụ, trên giá có 4quyển sách theo thứ tự 3−1−4−2, sau một bước chuyển quyển số 2 về đúng vị trí củanó ta xếp lại thành 3−2−1−4. Chứng minh rằng người thủ thư có thể hoàn thànhcông việc của mình sau ít hơn 2n lần xếp theo quy tắc.

http://thcmn.wordpress.com 33

Page 34: t 2017 - WordPress.com...Bài 5. (B›c Ninh) Cho a, b c¨0 thäa mãn đi•u ki»n ¯ ˘9. Tìm giá trà lîn nh t cıabi”uthøc T ˘ ab 3a¯4b¯5c ¯ bc 3b¯4c¯5a ¯ ca 3c¯4a¯5b

Bài 2. (THPT chuyên KHTN, ĐH KHTN, ĐHQG HN ) Tìm số lớn nhất phần tử của mộttập hợp là tập con của

{1,2,3, ....2016

}thỏa mãn hiệu hai phần tử bất kỳ khác 4 và 7.

Bài 3. (THPT chuyên KHTN, ĐH KHTN, ĐHQG HN ) Cho n nguyên dương. Các tâmthẻ trong một bộ sưu tập có giá trị m! với m là số nguyên dương nào đó, Một bộ sưutập tốt là một bộ sưu tập sao cho với mọi số k thỏa mãn k ≤ n!, luôn tồn tại một sốtâm thẻ trong bộ sưu tập mà tồng giá trị các thẻ này bằng k. Tìm số tấm thẻ ít nhấtcủa bộ sưu tập tốt.

Bài 4. (Trường Phổ thông Năng khiếu - ĐHQG Tp. HCM ) Với các số nguyên a,b,c,dthỏa 1 ≤ a < b < c < d , kí hiệu T (a,b,c,d) = {x, y, z, t ∈ N |1 ≤ x < y < z < t , x ≤ a, y ≤ b, z ≤c, t ≤ d}.

1. Tính số phần tử của T (1,4,6,7).

2. Cho a = 1 và b ≥ 4. Gọi d1 là số phần tử của T (a,b,c,d) chứa 1 và không chứa 2,d2 là số phần tử chứa 1,2 nhưng không chứa 3, d3 là số phần tử chứa 1,2,3 nhưngkhông chứa 4. Chứng minh rằng d1 ≥ 2d2 −d3. Dấu bằng xảy ra khi nào?

Bài 5. (Trường Phổ thông Năng khiếu - ĐHQG Tp. HCM) Trong một hệ thống máytính, một máy tính có thể kết nối trực tiếp với ít nhất 30% máy tính khác của hệ thống.Hệ thống này có một chương trình ngăn chặn và cảnh báo khá tốt,do đó khi một máytính bị virus,nó chỉ đủ thời gian lây virus cho các máy tính được kết nối trực tiếp vớinó. Chứng minh rằng dù vậy, kẻ tấn công vẫn có thể chọn hai máy tính của hệ thốngmà nểu thả virus vào hai máy đó, ít nhất 50% máy tính của hệ thống sẽ bị nhiễm virus.

Bài 6. (THPT chuyên ĐH Vinh) Bạn An có 12 tấm thẻ, trên mỗi tấm thẻ ghi một sốnguyên từ 1 đến 12, các số trên các thẻ đều phân biệt.

1. Chứng minh rằng bạn An có thể chia 12 tấm thẻ đó thành một số nhóm thỏamãn tính chất (P ) như sau: trong mỗi nhóm có nhiều hơn một tấm thẻ đồngthời số lớn nhất ghi trên một tấm thể nào đó bằng tổng các số ghi trên tấm thẻcòn lại.

2. Nếu bạn An cho bạn Bình n tấm thẻ mang các số từ 1 đến n (1 ≤ n ≤ 12) thì vớinhững tấm thẻ còn lại bạn An có thể chia thành một số nhóm thỏa mãn tínhchất (P ) hay không?

Bài 7. (Bà Rịa - Vũng Tàu)

1. Tính số hoán vị f (1); f (2); ...; f (2016) sao cho biểu thức

T = | f (1)−1|+ | f (2)−2|+ ...+| f (2016)−2016|

đạt trá trị lớn nhất, trong đó f (1) là giá trị ở vị trí thứ i trong mỗi hoán vị,i =1,2,3, ...2016.

34 http://www.facebook.com/thcmn

Page 35: t 2017 - WordPress.com...Bài 5. (B›c Ninh) Cho a, b c¨0 thäa mãn đi•u ki»n ¯ ˘9. Tìm giá trà lîn nh t cıabi”uthøc T ˘ ab 3a¯4b¯5c ¯ bc 3b¯4c¯5a ¯ ca 3c¯4a¯5b

2. Trên mặt phẳng xét 42 điểm mà không có ba điểm nào thẳng hàng.Ta dựng cácđoạn thẳng nối hai điểm trong các điểm nói trên sao cho mọi bộ ba điểm đangxét luôn có hai điểm được nối với nhau. Tìm giá trị nhỏ nhất của số đoạn thẳngcần dựng.

Bài 8. (Bắc Ninh)

1. Xung quanh bờ hồ hình tròn có 2017 cây liễu. Người ta dự định chặt bớt 5 câyliễu sao cho không có 2 cây liễu nào kề nhau bị chặt. Hỏi có bao nhiêu cáchthực hiện khác nhau ?

2. Một cuộc họp có 12k( k ∈N∗) người, trong đó mỗi người bắt tay với đúng 3k +6người khác. Biết rằng với mọi cách chọn cặp hai người (A;B) thì số người bắt tayvới cả hai người A và B luôn là m(m ∈N∗,m ≤ 3k+6). Hỏi cuộc họp có bao nhiêungười ?

Bài 9. (Bình Dương ) Trên mặt phẳng cho 2017 điểm sao cho với ba điểm bất kì taluôn tìm được hai điểm để đoạn thẳng được tạo thành có độ dài bé hơn 1. Chứngminh rằng luôn tồn tại một đường tròn có bán kính bằng 1 chứa không ít hơn 1009điểm đã cho.

Bài 10. (Bình Dương ) Trên bảng cho các số1

2016,

2

2016, ...,

2016

2016. Mỗi lần thực hiện trò

chơi cho phép ta xóa đi hai số a,b bất kì và thay vào số mới bằng a +b −3ab. Hỏi sau2015 lần thực hiện phép xóa, số còn lại trên bảng là số nào?

Bài 11. (Bình Dương )

1. Trên mỗi ô vuông của một bảng 9x9, ta đặt một con châu chấu. Giả sử cứ saumột tiếng gõ, mỗi con châu chấu nhảy sang ô bên cạnh cùng một hàng hoặccùng một cột. Chứng minh rằng sau một tiếng gõ có ít nhất hai con ở cùng mộtô.

2. Trên mặt phẳng tọa độ Descartes Ox y , có một con châu chấu ở tọa độ (x, y)trong đó x, y ∈ Z. Với N là số nguyên dương cho trước, con châu chấu có thểnhảy từ điểm nguyên A đến điểm nguyên B nếu độ dài AB bằng N . Hỏi rằngcon châu chấu có thể nhảy đến bất kì điểm nguyên nào sau hữu hạn các bướcnhảy không nếu N = 2025. Vì sao? (Điểm nguyên là điểm có tung độ và hoànhđộ là các số nguyên).

Bài 12. (Đà Nẵng ) Trong mặt phẳng cho n ≥ 2 đường thẳng đôi một cắt nhau vàkhông có ba đường nào đồng quy. Các đường này chia mặt phẳng thành các miềnhữu hạn và vô hạn. Chứng minh ta có thể đánh dấu các miền đó bằng các số nguyênthỏa mãn cả ba điều kiện sau:

(i) Các số đó khác 0.

(ii) Trị tuyệt đối của mỗi số không lớn hơn n.

http://thcmn.wordpress.com 35

Page 36: t 2017 - WordPress.com...Bài 5. (B›c Ninh) Cho a, b c¨0 thäa mãn đi•u ki»n ¯ ˘9. Tìm giá trà lîn nh t cıabi”uthøc T ˘ ab 3a¯4b¯5c ¯ bc 3b¯4c¯5a ¯ ca 3c¯4a¯5b

(iii) Mỗi đường thẳng đã cho sẽ phân mặt phẳng làm hai phần mà tổng các số củamọi miền thuộc mỗi phần sẽ bằng 0.

Bài 13. (Đà Nẵng ) Cho bảng ô vuông 2017 x 2017, người ta điền vào mỗi ô của bảngmột số nguyên từ 1 đến 2017 sao cho mỗi số được điền vào bảng đúng một lần.

1. Chứng minh tồn tại hai số cạnh nhau trong bảng (tức thuộc hai ô chung cạnh)có hiệu không nhỏ hơn 2017.

2. Tìm k ∈N∗ nhỏ nhất sao cho tồn tại một cách điền để hiệu hai số cạnh nhau bấtkì trong bảng đều không lớn hơn k.

Bài 14. (Đồng Nai) Tìm số nguyên dương n nhỏ nhất sao cho: Với n số nguyên dương

a1, a2, ..., an đôi một khác nhau , luôn tồn tại hai chỉ số i , j ∈{

1,2,3, ...,n}

để ai + a j ≥2017(ai , a j ) với (a,b) là ước chung lớn nhất của hai số nguyên dương a,b.

Bài 15. (Đồng Nai) Có bao nhiêu hoán vị (a1, a2, ..., a10) của các số 1,2,3, ..,10 sao choai > a2i với 1 ≤ i ≤ 5 và a j > a2 j+1 với 1 ≤ i ≤ 4.

Bài 16. (Hà Nam) Gọi A là tập các bộ (x1, x2, x3) với x1, x2, x3 ∈ {0;1;2; ...;7

}. Bộ x =

(x1, x2, x3) ∈ A gọi là trội hơn bộ y = (y1, y2, y3) ∈ A nếu x 6= y và xi ≥ yi ∀i = 1;2;3. Khi đóta viết x > y . Tìm nmi n ∈N∗ sao cho mọi tập con có n ptử của A đều chứa ít nhất 2 bộx > y .

Bài 17. (Hà Tĩnh) Cho tập S = {1,2,3, ...,2016

}. Hỏi có bao nhiêu hoán vị (a1, a2, ..., a2016)

của tập S sao cho 2(a1 +a2 +a3 + ...+ak )... k với ∀k = 0,1,2, ...,2016

Bài 18. (Tp. HCM ) Xét tập hợp S gồm 2016 số nguyên dương đầu tiên. Gọi A,B ,C

là 3 tập con bất kỳ của S, đôi một không giao nhau sao cho |A| = |B | = |C | = 672 vàA∪B ∪C = S. Chứng minh rằng tồn tại 3 số a,b,c lần lượt thuộc 3 tập A,B ,C mà số nàybằng tổng hai số kia.

Bài 19. (Khánh Hòa) Cho tập hợp A ={

a1, a2, ..., a15

}gồm 15 phần tử. Chúng ta sẽ tạo

ra những tập hợp mà mỗi tập hợp này chỉ chứa một hay nhiều phần tử của A (có thểsử dụng tất cả các phần tử của tập A) và chỉ số dưới của mỗi phần tử trong mỗi tậphợp được tạo thành phải là bội của chỉ số dưới nhỏ nhất có trong tập hợp đó. Có baonhiêu tập hợp như hợp như thế được tạo thành? Chẳng hạn

{a2, a4, a8

},{

a6

}, ... là các

tập hợp thỏa mãn yêu cầu bài toán.

Bài 20. (Lạng Sơn) Cho tập Mn = {1;2; ...;n

}(n ∈N∗)

1. Gọi X là 1 tập con của M15 sao cho tích của 3 ptử bất kỳ của X không phải sốchính phương. Tìm max | X |.

2. Gọi Y là 1 tập con gồm có 15 phần tử của tập M25. Tập I ′ gọi là tập "tốt" nếu nhưkhông tồn tại 2 phần tử nào mà tích của chúng là số chính phương. Tính số tấtcả các tập "tốt".

36 http://www.facebook.com/thcmn

Page 37: t 2017 - WordPress.com...Bài 5. (B›c Ninh) Cho a, b c¨0 thäa mãn đi•u ki»n ¯ ˘9. Tìm giá trà lîn nh t cıabi”uthøc T ˘ ab 3a¯4b¯5c ¯ bc 3b¯4c¯5a ¯ ca 3c¯4a¯5b

Bài 21. (Nghệ An) Một số nguyên dương k được gọi là "đẹp" nếu có thể phân hoạchtập hợp các số nguyên dươngZ∗ thành tập hợp A1, A2, ..., Ak sao cho với mỗi số nguyêndương n ≥ 15 và với mọi i ∈ {

1;2; ...;k}

đều tồn tại hai sô thuộc tập Ai có tổng là n.

1. Chứng minh rằng k = 3 là đẹp.

2. Chứng minh rằng mọi k ≥ 4 đều không đẹp.

Bài 22. (Ninh Bình) Tìm kmax ∈ N∗ sao cho ta có thể phân hoạch tập hợp các sốnguyên dương thành k tập hợp A1, A2, ..., Ak thỏa mãn với mỗi n ∈ N∗,n > 14, trongmỗi tập Ai i = 1,k đều tồn tại 2 số có tổng bằng n.

Bài 23. (Phú Thọ) Một hàng cây bưởi Đoan Hùng gồm 17 cây thẳng hàng đánh sốcây theo thứ tự là các số tự nhiên từ 1 đến 17. Ban đầu mỗi cây có một con đậu trênđó để hút mật hoa. Sau đó, cứ mỗi giờ có hai con ong nào đó bay sang hai cây bêncạnh để tìm và hút mật nhưng theo hai chiều ngược nhau. Hỏi sau một số giờ có haykhông trường hợp mà

1. Không có con ong ở cây có số thứ tự chẵn.

2. Có 9 con ong ở cây cuối cùng.

Bài 24. (Quảng Bình) Cho số nguyên dương n ≥ 4 . Tìm số lớn nhất các cặp gồm haiphần tử phân biệt của tập Xn = {

1,2, ...,n}

sao cho tổng các cặp khác nhau là các sốnguyên khác nhau và không vượt quá n .

Bài 25. (Quảng Ninh) Giả sử S là tập hợp hữu hạn các điểm mà mỗi điểm của nóđược tô bởi một trong 2 màu đỏ hoặc xanh. Gọi A1, A2, ..., A68 là tập con của tập S màmỗi tập chứa đúng 5 điểm thỏa mãn đồng thời hai điều kiện sau:

i Mỗi tập A1, A2, ..., A68 chứa ít nhất một điểm màu đỏ.

ii Với ba điểm bất kì trong S, tồn tại chính xác 1 tập con Ai chứa 3 điểm đó.

Hỏi:

1. Tìm số phần tử của tập S.

2. Tồn tại hay không một tập con Ai chứa 4 hoặc 5 điểm đỏ. Vì sao ?

Bài 26. (Quảng Trị)

1. Cho tập hợp S = {1;2;3; ...;2016

}. Hỏi có bao nhiêu tập con của S có 3 phần tử mà

chúng là độ dài của 3 cạnh của một tam giác có chiều dài cạnh lớn nhất bằng1000 ?

2. Cho một hình vuông có cạnh bằng 1. Bên trong hình vuông này có n( n ∈ N∗)hình tròn có tổng diện tích lớn hơn n −1. Chứng minh rằng tồn tại một điểmcủa hình vuông nằm trong tất cả hình tròn này.

http://thcmn.wordpress.com 37

Page 38: t 2017 - WordPress.com...Bài 5. (B›c Ninh) Cho a, b c¨0 thäa mãn đi•u ki»n ¯ ˘9. Tìm giá trà lîn nh t cıabi”uthøc T ˘ ab 3a¯4b¯5c ¯ bc 3b¯4c¯5a ¯ ca 3c¯4a¯5b

Bài 27. (Thái Nguyên) Cho tập hợp E = {1;2;3;4;5

}. Gọi M là tập hợp tất cả các số tự

nhiên có ít nhất ba chữ số, các chữ số đôi một khác nhau thuộc E . Chọn ngẫu nhiênmột số thuộc M . Tính xác suất để số được chọn có tổng các chữ số bằng 10.

Bài 28. (Thanh Hóa) Tại bốn đỉnh của tứ diện ABC D có ghi tương ứng bốn số a,b,c,d

không đồng thời bằng nhau.Thực hiện phép biến đổi số tại các đỉnh của tứ diện nhưsau: Mỗi lần biến đổi ta xóa bộ các số cũ (x; y ; z; t ) và thay vào đó bộ bốn số mới(x+y+z−3t ; y+z+t−3x; z+t+x−3y ; t+x+y−3z) theo thứ tự bất kỳ. Chứng minh rằngkể từ sau lần biến đổi đầu tiên, trong bốn đỉnh của tứ điện có ít nhất một đỉnh đượcghi số dương và sau một số lần thực hiện phép biến đổi luôn có ít nhất một đỉnh củatứ diện được ghi số không nhỏ hơn 2016.

38 http://www.facebook.com/thcmn

Page 39: t 2017 - WordPress.com...Bài 5. (B›c Ninh) Cho a, b c¨0 thäa mãn đi•u ki»n ¯ ˘9. Tìm giá trà lîn nh t cıabi”uthøc T ˘ ab 3a¯4b¯5c ¯ bc 3b¯4c¯5a ¯ ca 3c¯4a¯5b

Phần II

LỜI GIẢI

1 Bất đẳng thức

1. Cho x, y là các số thực dương sao cho 2x + y và 2y +x khác 2. Tìm giá trị nhỏnhất của biểu thức

P = (2x2 + y)(4x + y2)

(2x + y −2)2+ (2y2 +x)(4y +x2)

(x +2y −2)2−3(x + y)

2. Cho a,b,c > 0 sao cho a +b + c = 3. Chứng minh rằng

a

b2(ca +1)+ b

c2(ab +1)+ c

a2(bc +1)≥ 9

(1+abc)(ab +bc + ca)

(THPT chuyên KHTN - ĐH KHTN, ĐHQG Hà Nội)

Bài 1

Lời giải

1. Ta sẽ chứng minh:(2x2 + y)(4x + y2)

(2x + y −2)2≥ 2x + y − 1

2

⇔ (2x y −6x −3y +2)2 ≥ 0 (đúng).

Chứng minh tương tự ta được: P ≥−1. Vậy GTNN của P là −1 khi x = y = 9+p65

4

hoặc x = y = 9−p65

4.

2. Theo BĐT Cauchy - Schwartz thì

∑c yc

c

a2(bc +1)= ∑

c yc

1a2

b + 1c

≥ ( 1a + 1

b + 1c )2

1a + 1

b + 1c +a +b + c

= ( 1a + 1

b + 1c )2

1a + 1

b + 1c +3

= (ab +bc + ca)2

abc(ab +bc + ca)+3a2b2c2.

Đặt ab +bc + ca = x, abc = y . BĐT ban đầu sẽ đúng nếu ta chứng minh được

x2

x y +3y2≥ 9

x(1+ y)⇔ x3 +x3 y ≥ 9x y +27y2

⇔ x(x2 −9y)+ y(x3 −27y) ≥ 0,

một điều luôn đúng vì x2−9y ≥ 0 và x3−27y ≥ 0. Vậy BĐT được chứng minh. Dấubằng xảy ra ⇔ a = b = c = 1.

http://thcmn.wordpress.com 39

Page 40: t 2017 - WordPress.com...Bài 5. (B›c Ninh) Cho a, b c¨0 thäa mãn đi•u ki»n ¯ ˘9. Tìm giá trà lîn nh t cıabi”uthøc T ˘ ab 3a¯4b¯5c ¯ bc 3b¯4c¯5a ¯ ca 3c¯4a¯5b

Tìm số nguyên dương k nhỏ nhất sao cho bất đẳng thức

xk yk zk (x3 + y3 + z3) ≤ 3

đúng với mọi số thực dương x, y, z thỏa mãn điều kiện x + y + z = 3.

(Trường Phổ thông Năng khiếu, ĐHQG Tp. HCM)

Bài 2

Lời giải

Lời giải sau đây trích từ trang nangkhieutoan.com

• Dễ dàng tìm được các bộ số để BĐT không đúng với k = 1 và k = 2.

• Nhận xét rằng nếu BĐT đúng với k = 3 thì BĐT sẽ đúng với mọi k > 3 vì xk yk zk (x3+y3 + z3) = x3 y3z3(x3 + y3 + z3).xk−3 yk−3zk−3 ≤ 3. Điều này gợi ý cho ta chứng minhrằng k = 3 là số nhỏ nhất cần tìm, bằng cách chứng minh

x3 y3z3(x3 + y3 + z3) ≤ 3. (1.1)

Thật vậy, giả sử z là số nhỏ nhât trong ba số x, y, z, suy ra z ≤ 1. Ta có

x3 + y3 = (x + y)3 −3x y(x + y) = (3− z)3 −3x y(3− z).

Khi đó(1.1) ⇔ (3− z)3 + z3 ≤ 3

x3 y3z3+3x y(x + y)

3z2 −9z +9 ≤ 1

x3 y3z3+x2 y +x y2. (1.2)

Để ý rằng

1

x3 y3z3+x2 y +x y2 ≥ 3 3

√x3 y3

x3 y3z3

đồng thời

3z2 −9z +9− 3

z= 3(z −1)3

z≤ 0

nên (1.2) đúng, và BĐT ban đầu được chứng minh. Vậy k = 3 là số nguyên dươngnhỏ nhất để BĐT ban đầu đúng. Dấu bằng xảy ra ⇔ x = y = z = 1.

40 http://www.facebook.com/thcmn

Page 41: t 2017 - WordPress.com...Bài 5. (B›c Ninh) Cho a, b c¨0 thäa mãn đi•u ki»n ¯ ˘9. Tìm giá trà lîn nh t cıabi”uthøc T ˘ ab 3a¯4b¯5c ¯ bc 3b¯4c¯5a ¯ ca 3c¯4a¯5b

Tìm tất cả các số thực k sao cho bất đẳng thức sau đúng với mọi số thực khôngâm a,b,c

ab +bc + ca ≤ (a +b + c)2

3+k.max{(a −b)2, (b − c)2, (c −a)2} ≤ a2 +b2 + c2.

(THPT chuyên Đại học Vinh)

Bài 3

Lời giải

Không mất tính tổng quát giả sử a ≥ b ≥ c. Khi đó max{(a−b)2, (b−c)2, (c−a)2} = (a−c)2.

Như vậy, ta sẽ tìm k sao cho: ab +bc + ca ≤ (a +b + c)2

3+k(a − c)2 ≤ a2 +b2 + c2.

Cho c = 0, a = 2b ta được−1

4≤ k ≤ 1

2. Ta sẽ chứng minh ab+bc+ca ≤ (a +b + c)2

3+k(a−

c)2 ≤ a2 +b2 + c2 với mọi−1

4≤ k ≤ 1

2.

Ta có

ab +bc + ca ≤ (a +b + c)2

3+k(a − c)2 ⇔

(k + 1

4

)(a − c)2 + 1

12(a + c −2b)2 ≥ 0,

nên BĐT đầu tiên đúng. Đồng thời

(a +b + c)2

3+k(a − c)2 ≤ a2 +b2 + c2 ⇔

(1

2−k

)(a − c)2 + 1

6(a + c −2b)2 ≥ 0

nên BĐT thứ hai cũng đúng.

1. Cho x, y, z là ba số thực dương thỏa mãn x y z = 1. Chứng minh bất đẳng thức

1

(2x + y + z)2+ 1

(2x + y + z)2+ 1

(2x + y + z)2≤ 3

16.

2. Cho x, y, z không âm và thỏa x2 + y2 + z2 = 1. Chứng minh bất đẳng thức

(x2 y + y2z + z2x)

(1p

x2 +1+ 1√

y2 +1+ 1p

z2 +1

)≤ 3

2.

(Bà Rịa - Vũng Tàu)

Bài 4

Lời giải

Trước hết xin phát biểu không chứng minh một bổ đề quen thuộc.

Bổ đề 1 Cho x, y, z > 0. Khi đó 9(x + y)(y + z)(z +x) ≥ 8(x + y + z)(x y + y z + zx).

http://thcmn.wordpress.com 41

Page 42: t 2017 - WordPress.com...Bài 5. (B›c Ninh) Cho a, b c¨0 thäa mãn đi•u ki»n ¯ ˘9. Tìm giá trà lîn nh t cıabi”uthøc T ˘ ab 3a¯4b¯5c ¯ bc 3b¯4c¯5a ¯ ca 3c¯4a¯5b

Trở lại bài toán.

1. Theo bất đẳng thức AM - GM, ta có∑ 1

(2x + y + z)2=∑ 1

((x + y)+ (x + z))2≤∑ 1

4(x + y)(x + z).

Do đó BĐT ban đầu sẽ đúng nếu ta chứng minh được∑ 1

4(x + y)(x + z)≤ 3

16⇔ 8

3(x + y + z) ≤ (x + y)(y + z)(z +x)

⇔ 8

3(x + y + z)(x y + y z + zx) ≤ (x + y)(y + z)(z +x)(x y + y z + zx),

Nhưng điều này đúng vì x y + y z + zx ≥ 3√

x2 y2z2 = 3 và theo bổ đề bên trên. Từ

đó ta có điều phải chứng minh. Dấu bằng xảy ra ⇔ a = b = c = 1.

2. Chúng tôi xin nêu hai cách chứng minh cho câu 2.

• Cách 1: Ta có

(x + y + z)(x2 + y2 + z2) = x3 + y3 + z3 +x2(y + z)+ y2(z +x)+ z2(x + y)

Áp dụng bất đẳng thức AM-GM và x2 + y2 + z2 = 1 ta có

x2 y + y2z + z2x ≤ 1

3(x + y + z)

Áp dụng bất đẳng thức Cauchy:(1p

x2 +1+ 1√

y2 +1+ 1p

z2 +1

)2

≤ 3

(1

x2 +1+ 1

y2 +1+ 1

z2 +1

)Do đó ta sẽ chứng minh

3

(1

x2 +1+ 1

y2 +1+ 1

z2 +1

)≤

(9

2(x + y + z)

)2

⇔ 27

4(x + y + z)2≥ 1

x2 +1+ 1

y2 +1+ 1

z2 +1

⇔ 27

4(x + y + z)2+ x2

x2 +1+ y2

y2 +1+ z2

z2 +1≥ 3

Áp dụng bất đẳng thức Cauchy - Schwartz ta có

V T ≥ (x + y + z)2

3+x2 + y2 + z2+ 27

4(x + y + z)2

⇒V T ≥ (x + y + z)2

4+ 27

4(x + y + z)2

Áp dụng bất đẳng thức AM-GM và x + y + z ≤√

3(x2 + y2 + z2) =p3 ta có

27

4(x + y + z)2+ (x + y + z)2

4= 9

4(x + y + z)2+ (x + y + z)2

4+ 18

4(x + y + z)2≥ 3

2+ 18

4.3= 3

Từ đó ta thấy đpcm là đúng.

42 http://www.facebook.com/thcmn

Page 43: t 2017 - WordPress.com...Bài 5. (B›c Ninh) Cho a, b c¨0 thäa mãn đi•u ki»n ¯ ˘9. Tìm giá trà lîn nh t cıabi”uthøc T ˘ ab 3a¯4b¯5c ¯ bc 3b¯4c¯5a ¯ ca 3c¯4a¯5b

• Cách 2: Ta có

x2 y + y2z + z2x ≤ (x + y + z)(x2 + y2 + z2)

3= x + y + z

3

Do đó cần chứng minh

x + y + zp1+x2

+ x + y + z√1+ y2

+ x + y + zp1+ z2

≤ 9

2

Ta có:x y + zx + zx ≤ x2 + y2 + z2 = 1

Do đóxp

1+x2≤ x√

(x + y)(x + z)≤ 1

2

(x

x + y+ x

x + z

)Do đó

xp1+x2

+ y√1+ y2

+ zp1+ z2

≤ 3

2

Hay là chứng minh

y + z√2x2 + y2 + z2

+ z +x√2y2 + z2 +x2

+ x + y√2z2 +x2 + y2

≤ 3

Ta có: (y + z√

2x2 + y2 + z2

)2

≤ y2

x2 + y2+ z2

x2 + z2

Suy ra: (y + z√

2x2 + y2 + z2

)2

+(

z +x√2z2 +x2 + y2

)2

+(

x + y√2z2 +x2 + y2

)2

≤ 3

Mặt khác:(y + z√

2x2 + y2 + z2+ z +x√

2y2 + z2 +x2+ x + y√

2z2 +x2 + y2

)2

≤ 3

(y + z√

2x2 + y2 + z2

)2

+(

z +x√2z2 +x2 + y2

)2

+(

x + y√2z2 +x2 + y2

)2≤ 9

Suy ray + z√

2x2 + y2 + z2+ z +x√

2y2 + z2 +x2+ x + y√

2z2 +x2 + y2≤ 3

Do đó ta có đpcm.

http://thcmn.wordpress.com 43

Page 44: t 2017 - WordPress.com...Bài 5. (B›c Ninh) Cho a, b c¨0 thäa mãn đi•u ki»n ¯ ˘9. Tìm giá trà lîn nh t cıabi”uthøc T ˘ ab 3a¯4b¯5c ¯ bc 3b¯4c¯5a ¯ ca 3c¯4a¯5b

Cho a,b,c > 0 thỏa mãn điều kiện a +b + c = 9. Tìm giá trị lớn nhất của biểu thức

T = ab

3a +4b +5c+ bc

3b +4c +5a+ ca

3c +4a +5b− 1p

ab(a +2c)(b +2c).

(Bắc Ninh)

Bài 5

Lời giải

Ta có

∑ ab

3a +4b +5c=∑ 2ab

5(a +b +2c)+ (a +3b)≤ 2

36

(∑ 5ab

a +b +2c+∑ ab

a +3b

).

Bây giờ ta sẽ chứng minh ∑ ab

a +b +2c≤ 9

4.

Ta có: ∑ ab

a +b +2c≤ 1

4

(∑ ab

c +a+∑ ab

b + c

)= 1

4(a +b + c) = 9

4

nên điều trên sẽ đúng nếu ta sẽ chứng minh:

∑ ab

a +3b≤ 9

4.

Đế ý rằng

∑ ab

a +3b≤ 1

16

(∑ ab

a+∑ 3ab

b

)= 1

16

(∑b +∑

3a)= 1

4(a +b + c) = 9

4

⇔∑ ab

3a +4b +5c≤ 1

18

(5.

9

4+ 9

4

)= 3

4(1.3)

Mặt Khác

1pab(a +2c)(b +2c)

= 2

2p

(ab +2bc)(ab +2ca)≥ 2

2(ab +bc + ca)

= 3

3(ab +bc + ca)≥ 3

(a +b + c)2= 3

92= 1

27(1.4)

Từ (1.3) và (1.4) ta được: T ≤ 3

4− 1

27= 77

108. Vậy GTLN của T là

77

108khi a = b = c = 3.

44 http://www.facebook.com/thcmn

Page 45: t 2017 - WordPress.com...Bài 5. (B›c Ninh) Cho a, b c¨0 thäa mãn đi•u ki»n ¯ ˘9. Tìm giá trà lîn nh t cıabi”uthøc T ˘ ab 3a¯4b¯5c ¯ bc 3b¯4c¯5a ¯ ca 3c¯4a¯5b

Cho a,b,c là các số thực dương . Tìm giá trị nhỏ nhất của biểu thức:

P = 1344

a +pab + 3pabc

− 2016pa +b + c

.

(Bến Tre)

Bài 6

Lời giải

Ta có: a+p

ab+ 3pabc = a+2

√a

4.b+ 3

√a

4.b.4c ≤ a+ a

4+b+ 1

3

(a

4+b +4c

)= 4

3

(a +b + c)

)⇒P ≥ 1008

a +b + c− 2016p

a +b + c= 1008

(1p

a +b + c−1

)2

− 1008 ≥ −1008. Vậy GTNN của P là

−1008 khi a = 16

21,b = 4

21,c = 1

21.

Cho các số thực dương x, y, z. Chứng minh rằng

x2

y + z+ y2

z +x+ z2

x + y≥ x + y + z

2≥ y z

y + z+ x y

x + y+ zx

z +x.

(Bình Thuận)

Bài 7

Hướng dẫn

•∑ x2

y + z≥ (

∑x)2∑

(y + z)= x + y + z

2.

•x + y + z

2≥ y z

y + z+ x y

x + y+ zx

z +x⇔∑ (x − y)2

4x y≥ 0 (đúng)

Cho các số thưc dương a,b,c thỏa mãn abc = 1. Chứng minh rằng :

√a

b + c+

√b

a + c+

√c

a +b≥ 3

p3p

a3 +b3 + c3 +3.

(Đồng Nai)

Bài 8

Lời giải

Ta có:∑√

a

b + c=∑ 1p

bc(b + c)≥ 9∑p

bc(b + c)≥ 9√

3(∑

bc(b + c))≥ 3

p3p

a3 +b3 + c3 +3abc=

3p

3pa3 +b3 + c3 +3

.

http://thcmn.wordpress.com 45

Page 46: t 2017 - WordPress.com...Bài 5. (B›c Ninh) Cho a, b c¨0 thäa mãn đi•u ki»n ¯ ˘9. Tìm giá trà lîn nh t cıabi”uthøc T ˘ ab 3a¯4b¯5c ¯ bc 3b¯4c¯5a ¯ ca 3c¯4a¯5b

Cho a,b,c ≥ 0. Tìm giá trị nhỏ nhất của:

P =√

a

b + c+

√b

a + c+

√c

a +b.

(Hà Nam)

Bài 9

Lời giải

Chuẩn hoá a +b + c = 3. Ta sẽ chứng minh rằnga

b + c=

√a

3−a≥ 2

3a. Điều này tương

đương với a(2a−3)2 ≥ 0, hiển nhiên đúng. Cộng lại ta được P ≥ 2

3.3 = 2. Vậy GTNN của

P bằng 2 khi có một số bằng 0 và hai số bằng nhau.

Nhận xét 1 Một số bạn sẽ giải bài này như sau: Ta có P = ∑√a

b + c= ∑ 2a

2p

a(b + c)≥∑ 2a

a +b + c= 2 là sai, vì để ý rằng khi nhân cả từ và mẫu của phân thức cho một số, số

đó phải khác 0.

Cho a,b,c > 0 thỏa mãn ab +bc + ca +2abc = 1 Tìm giá trị nhỏ nhất của

P = 1

a+ 1

b+ 1

c−2(a +b + c).

(Hà Nội)

Bài 10

Lời giải

Dự đoán GTNN của P là 3 đạt tại a = b = c = 1

2, vậy ta sẽ cố gắng chứng minh BĐT

P = 1

a+ 1

b+ 1

c−2(a +b + c) ≥ 3.

Từ giả thiết suy ra tồn tại các số x, y, z > 0 sao cho

a = x

y + z,b = y

z +x,c = z

x + y.

BĐT cần chứng minh trở thành

y + z

x+ z +x

y+ x + y

z≥ 2

(x

y + z+ y

z +x+ z

x + y

)+3.

46 http://www.facebook.com/thcmn

Page 47: t 2017 - WordPress.com...Bài 5. (B›c Ninh) Cho a, b c¨0 thäa mãn đi•u ki»n ¯ ˘9. Tìm giá trà lîn nh t cıabi”uthøc T ˘ ab 3a¯4b¯5c ¯ bc 3b¯4c¯5a ¯ ca 3c¯4a¯5b

Để ý rằngy + z

x+ z +x

y+ x + y

z≥ 4

(x

y + z+ y

z +x+ z

x + y

).

nên BĐT sẽ đúng nếu ta chứng minh được

x

y + z+ y

z +x+ z

x + y≥ 3

2.

Nhưng đây chính là BĐT Nesbitt quen thuộc, vì vậy BĐT ban đầu đúng..

Nhận xét 2 Cách đặt a = x

y + z,b = y

z +x,c = z

x + ykhá kinh điển trong việc đổi biến và

thuần nhất hoá để chứng minh bất đẳng thức, và nó giúp đưa về các dang bài toánquen thuộc. Ngoài ra, chúng ta còn những cách đặt khác cho các loại giả thiết tươngtự. Cụ thể như sau, nếu x, y, z là các số dương thi:

• x2 + y2 + z2 + x y z = 4 ⇒ x = 2cos A, y = 2cosB , z = 2cosC với A,B ,C là ba góc trongmột tam giác.

• x y z = x + y + z +2 ⇒ x = b + c

a, y = c +a

b, z = a +b

cvới a,b,c > 0.

Bạn đọc có thể dễ dàng kiểm tra các cách đặt trên.Ngoài ra còn một số bài toán khác liên quan đến cách đổi biến lượng giác như:

1. (USA 2001) Cho a,b,c không âm thỏa mãn a2 + b2 + c2 + abc = 4. Chứng minhrằng:

0 ≤ ab +bc + ca −abc ≤ 2

2. (Iran 2002) Cho a,b,c > 0 thỏa mãn a2 +b2 + c2 +abc = 4. Chứng minh rằng:

a +b + c ≤ 3

Cho các số thực dương a,b,c dương và thỏa mãn a5+b5+c5 = 3. Chứng minh rằng

a6b6 +b6c6 + c6a6 ≤ 3.

(Hà Tĩnh)

Bài 11

Lời giải

Đặt x = a5, y = b5, z = c5 thì x + y + z = 3. BĐT cần chứng minh tương đương với

x y 5p

x y + y z 5p

y z + zx 5p

zx ≤ 3.

Theo BĐT AM - GM thì∑x,y,z

x y 5p

x y ≤ ∑c yc

x y(x + y +3)

5= ∑

c yc

x y(x + y +x + y + z)

5= ∑

x,y,z

2x y(x + y)+x y z

5

http://thcmn.wordpress.com 47

Page 48: t 2017 - WordPress.com...Bài 5. (B›c Ninh) Cho a, b c¨0 thäa mãn đi•u ki»n ¯ ˘9. Tìm giá trà lîn nh t cıabi”uthøc T ˘ ab 3a¯4b¯5c ¯ bc 3b¯4c¯5a ¯ ca 3c¯4a¯5b

= 2(x + y + z)(x y + y z + zx)−3x y z

5= 6(x y + y z + zx)−3x y z

5.

Mặt khác theo BĐT Schur thì

(x + y + z)3 +9x y z ≥ 4(x + y + z)(x y + y z + zx) ⇔ 33 +9x y z ≥ 4.3(x y + y z + zx)

⇔ 9+3x y z ≥ 4(x y + y z + zx)

Suy ra6(x y + y z + zx)−3x y z

5≤ 6(x y + y z + zx)−4(x y + y z + zx)+9

5

= 2(x y + y z + zx)+9

5≤

2

3(x + y + z)2 +9

5= 3.

Vậy BĐT ban đầu được chứng minh. Dấu bằng xảy ra ⇔ a = b = c = 1.

Nhận xét 3 Bằng cách tương tự ta có thể giải đuợc bài toán tổng quát sau: Cho các sốthực dương a,b,c và k ≥ 3 thỏa mãn ak +bk + ck = 3. Chứng minh rằng

ak+1bk+1 +bk+1ck+1 + ck+1ak+1 ≤ 3.

Cho a,b,c ≥ 1

2thỏa mãn a +b + c = 6. chứng minh rằng

ab +bc + ca ≥ 3p

abc +ab +bc + ca −4.

(Hải Phòng)

Bài 12

Lời giải

Đặt p = a +b + c = 6, q = ab +bc + ca,r = abc. Từ giả thiết dễ dàng chứng minh đượcq ≤ 12.Ta có

(a −b)2(b − c)2(c −a)2 ≥ 0 ⇔−27r 2 +2p(9q −2p2)r +p2q2 −4q3 ≥ 0

⇒ r ≤ p(9q −2p2)+2(p2 −3q)√

p2 −3q

27= 18(q −8)+2(12−q)

√3(12−q)

9(1.5)

Ta cần chứng minhq2 ≥ 9(q + r −4) ⇔ q2 −9q +36 ≥ 9r. (1.6)

Điều này sẽ đúng nếu ta chứng minh được

q2 −9q +36 ≥ 18(q −8)+2(12−q)√

3(12−q)

⇔ (12−q)(√

12−q −p3)2 ≥ 0,

một điều hiển nhiên đúng.Từ (1.5) và (1.6) ta có điều phải chứng minh

48 http://www.facebook.com/thcmn

Page 49: t 2017 - WordPress.com...Bài 5. (B›c Ninh) Cho a, b c¨0 thäa mãn đi•u ki»n ¯ ˘9. Tìm giá trà lîn nh t cıabi”uthøc T ˘ ab 3a¯4b¯5c ¯ bc 3b¯4c¯5a ¯ ca 3c¯4a¯5b

Cho a,b,c là các số dương thỏa mãn abc = 1 và x, y, z thuộc R. Chứng minh rằng

x2(a +b)+ y2(b + c)+ z2(c +a) ≥ 2(x y + y z + zx).

(Hoà Bình)

Bài 13

Lời giải

Đặt a = m3,b = n3,c = p3. Do abc = 1 nên mnp = 1.Áp dụng BĐT quen thuộc m3 +n3 ≥ mn(m +n) và BĐT Cauchy - Schwartz, ta có

x2(m3 +n3)+ y2(n3 +p3)+ z2(p3 +m3) ≥ x2mn(m +n)+ y2np(n +p)+ z2pm(p +m)

= [x(m +n)]2

p(m +n)+ [y(n +p)]2

m(n +p)+ [z(p +m)]2

n(p +m)≥ [x(m +n)+ y(n +p)+ z(p +m)]2

2(mn +np +pm).

Như vậy BĐT ban đầu sẽ đúng nếu ta chứng minh được

[x(m +n)+ y(n +p)+ z(p +m)]2

2(mn +np +pm)≥ 2(x y + y z + zx)

⇔ [x(m +n)+ y(n +p)+ z(p +m)]2 ≥ 4(x + y + z)(m +n +p)

⇔ ∣∣x(m +n)+ y(n +p)+ z(p +m)∣∣≥ 2

√(x + y + z)(m +n +p)

⇔ ∣∣x(m +n)+ y(n +p)+ z(p +m)∣∣+ ∣∣xp + ym + zn

∣∣≥ 2

√(x + y + z)(m +n +p)+ ∣∣xp + ym + zn

∣∣ .

Để ý rằng ∣∣x(m +n)+ y(n +p)+ z(p +m)∣∣+ ∣∣xp + ym + zn

∣∣≤

√[x2 + y2 + z2 +2(x y + y z + zx)][m2 +n2 +p2 +2(mn +np +pm)]

=√

(x + y + z)2(m +n +p)2 = ∣∣(x + y + z)(m +n +p)∣∣≤∣∣x(m +n)+ y(n +p)+ z(p +m)

∣∣+ ∣∣xp + ym + zn∣∣

chính là điều phải chứng minh.Bài toán trên đã kết hợp BĐT trong đề Ukraine năm 2001 với một đánh giá quenthuộc là a3 +b3 ≥ ab(a +b)Nội dung bài BĐT trong đề Ukraine năm 2001 như sau:Cho a,b,c, x, y, z là các số thực dương và x + y + z = 1. Chứng minh rằng:

ax +by + cz +2√

(x y + y z + zx)(ab +bc + ca) ≤ a +b + c

Nhận xét 4 Có thể thấy bài toán này cũng đúng trong trường hợp x, y, z là các số thực,từ đó ta có được bài toán BĐT trong đề chọn đội tuyển của tỉnh Hòa Bình.

http://thcmn.wordpress.com 49

Page 50: t 2017 - WordPress.com...Bài 5. (B›c Ninh) Cho a, b c¨0 thäa mãn đi•u ki»n ¯ ˘9. Tìm giá trà lîn nh t cıabi”uthøc T ˘ ab 3a¯4b¯5c ¯ bc 3b¯4c¯5a ¯ ca 3c¯4a¯5b

Cho a,b,c là các số thực thỏa mãn a+b+c = 0. Tìm giá trị nhỏ nhất của biểu thức

(a2 +1)2 + (b2 +1)2 + (c2 +1)2 +6p

6abc

(Tp. HCM)

Bài 14

Lời giải

Chúng tôi xin nêu hai cách chứng minh cho bài này như sau.

• Cách 1: Đặt P = (a2+1)2+ (b2+1)2+ (c2+1)2+6p

6abc. Không mất tính tổng quátgiả sử ab ≥ 0.Thay c =−a −b vào P ta được

P = 3(a +b −p6ab)2 + (a2 −b2)2 + (a −b)2(a2 +b2 +6ab)+3 ≥ 3.

Vậy GTNN của P là 3, xảy ra khi a = b = c = 0 hoặc a = b =p

6

3,c = −2

p6

3.

• Cách 2: Dự đoán rằng bất đẳng thức tồn tại dấu bằng tại biên nên ta sẽ đi dồnbiến về biên. Theo nguyên lý Dirichlet trong 3 số a,b,c sẽ tồn tại 2 số cùng dấu,giả sử ab ≥ 0.Bỏ qua trường hợp đơn giản là c ≥ 0. Xét c < 0.Xét hiệu:

f (a,b,c)− f

(a +b

2,

a +b

2,c

)

= (a2 +1)2 + (b2 +1)2 + (c2 +1)2 +6p

6abc −2

((a +b)2

4+1

)2

− (c2 +1)2 −6p

6(a +b)2

4c

= 1

8(a −b)2(7a2 +10ab +7b2 +8− 3

p6

2c) ≥ 0 (do ab ≥ 0,c < 0)

Đặt t = a +b

2. Khi đó: c =−2t . Ta có:

P ≥ 2(t 2 +1)2 + (4t 2 +1)2 +12p

6t 3 = 3+2t 2(3t +p6)2 ≥ 3.

Vậy GTNN của P là 3, xảy ra khi a = b = c = 0 hoặc a = b =p

6

3,c = −2

p6

3

Cho hai số thực x và y thỏa mãn x2 +x y + y2 ≤ 2. Chứng minh rằng

5x2 +2x y +2y2 ≤ 12.

(Khánh Hòa)

Bài 15

Lời giải

Ta có: (x +2y)2 ≥ 0 ⇔ 5x2+2x y +2y2 ≤ 6(x2+x y + y2) ≤ 6.2 = 12 (điều phải chứng minh).

50 http://www.facebook.com/thcmn

Page 51: t 2017 - WordPress.com...Bài 5. (B›c Ninh) Cho a, b c¨0 thäa mãn đi•u ki»n ¯ ˘9. Tìm giá trà lîn nh t cıabi”uthøc T ˘ ab 3a¯4b¯5c ¯ bc 3b¯4c¯5a ¯ ca 3c¯4a¯5b

Cho x, y, z > 0 thỏa mãn x y z = 1. Tìm giá trị lớn nhất của

P = 1

x2 +2y2 +3+ 1

y2 +2z2 +3+ 1

z2 +2x2 +3.

(Lạng Sơn)

Bài 16

Lời giải

Ta có: P = ∑ 1

x2 +2y2 +3= ∑ 1

x2 + y2 +1+ y2 +2≤ ∑ 1

4

(1

x2 + y2 +1+ 1

y2 +2

). Ta sẽ lần

lượt chứng minh các BĐT phụ sau:

•∑ 1

x2 + y2 +1≤ 1. Ta có: x2+y2+1 =

(x

23

)3+(

y23

)3+x23 y

23 z

23 ≥ x

23 y

23 (x

23+y

23 )+x

23 y

23 z

23 =

x23 + y

23 + z

23

z23

⇒∑ 1

x2 + y2 +1≤∑ z

23

x23 + y

23 + z

23

= 1 (đpcm).

•∑ 1

y2 +2≤ 1. Thật vậy, điều này tương đương với

y2

y2 +2≥ 1. Để ý rằng

∑ y2

y2 +2≥

(x + y + z)2

x2 + y2 + z2 +6= x2 + y2 + z2 +2(x y + y z + zx)

x2 + y2 + z2 +6≥ x2 + y2 + z2 +2.3 3

√(x y z)2

x2 + y2 + z2 +6= 1 (đpcm)

Từ hai điều trên ta được: P ≤ 1

4(1+1) = 1

2. Vậy GTNN của P là

1

2khi x = y = z = 1.

Cho các số thực dương a,b,c thỏa mãn a +b + c = 3. Chứng minh rằng:

(a +pb)2

pa2 −ab +b2

+ (b +pc)2

pb2 −bc + c2

+ (c +pa)2

pc2 − ca +a2

≤ 12.

(Nam Định)

Bài 17

Lời giải

Ta có:∑ (a +p

b)2

pa2 −ab +b2

=∑ 2(a +pb)2√

(a +b)2 +3(a −b)2≤∑ 2(a +1)(a +b)

a +b=∑

2(a +1) = 12

http://thcmn.wordpress.com 51

Page 52: t 2017 - WordPress.com...Bài 5. (B›c Ninh) Cho a, b c¨0 thäa mãn đi•u ki»n ¯ ˘9. Tìm giá trà lîn nh t cıabi”uthøc T ˘ ab 3a¯4b¯5c ¯ bc 3b¯4c¯5a ¯ ca 3c¯4a¯5b

Cho x, y, z > 0 thỏa mãn x + y + z = 3. Chứng minh rằng:

1px +p

y+ 1p

y +pz+ 1p

z +px≥ 4

(1

x +7+ 1

y +7+ 1

z +7

).

(Ninh Bình)

Bài 18

Lời giải

Ta có∑ 4

x +7=∑ 4

(x +1)+ (x +1)+ (y +1)+ (z +1)≤∑ 4

2(p

x +py)+2(

pz +p

x)

≤∑(1

2(p

x +py)

+ 1

2(p

z +px)

)=∑ 1p

x +py

Cho a,b,c là độ dài ba cạnh tam giác và a ≥ b ≥ c. Chứng minh rằng√a(a +b −

pab)+

√b(a + c −p

ac)+√

c(c +b −p

bc) ≥ a +b + c.

(Quảng Bình)

Bài 19

Lời giải

Chúng tôi xin nêu hai cách chứng minh cho BĐT này.

• Cách 1: BĐT cần chứng minh trở thành

x√

x2 + y2 −x y + y√

x2 + z2 −xz + z√

y2 + z2 − y z ≥ x2 + y2 + z2.

Để ý rằng √x2 + y2 −x y =

√(x3 + y3)(x + y)

x + y≥ x2 + y2

x + y.

Chứng minh tương tự ta sẽ được

x√

x2 + y2 −x y+y√

x2 + z2 −xz+z√

y2 + z2 − y z ≥ x(x2 + y2)

x + y+ y(x2 + z2)

x + z+ z(y2 + z2)

y + z

Vậy BĐT ban đàu sẽ đúng nếu ta chứng minh được

x(x2 + y2)

x + y+ y(x2 + z2)

x + z+ z(y2 + z2)

y + z≥ x2 + y2 + z2

⇔ y(x − y)(y − z)

[x

(x + y)(x + z)+ z

(y + z)(x + z)

]≥ 0.

Nhưng điều này luôn đúng với mọi x ≥ y ≥ z > 0, nên BĐT ban đầu được chứngminh.

52 http://www.facebook.com/thcmn

Page 53: t 2017 - WordPress.com...Bài 5. (B›c Ninh) Cho a, b c¨0 thäa mãn đi•u ki»n ¯ ˘9. Tìm giá trà lîn nh t cıabi”uthøc T ˘ ab 3a¯4b¯5c ¯ bc 3b¯4c¯5a ¯ ca 3c¯4a¯5b

• Cách 2: Đặt f (y) = x√

x2 + y2 −x y+y√

x2 + z2 −xz+z√

y2 + z2 − y z−(x2+y2+z2) ⇒f ′(y) = x(2y −x)

2√

x2 + y2 −x y+ z(2y − z)

2√

y2 + z2 − y z+

√x2 −xz + z2 −2y

⇔ f ′′(x) = 3x3

4(√

x2 + y2 −x y)3+ 3z3

4(√

y2 + z2 − y z)3−2

Ta có:x2 −x y + y2 = (

x

2− y)2 + 3

4x2 ≥ 3

4x2

y2 − y z + z2 ≥ z2

⇒ f ′′(y) = 3

4(

√3

4)3

+ 3

4−2 < 0

Vậy f (y) là hàm lồi ⇒ f (y) đạt giá trị nhỏ nhất tại y = x hoặc y = z.Ta có

f (x) =√

(x + z)(x3 + z3)− (x2 + z2) ≥ 0

f (z) =√

(x + z)(x3 + z3)− (x2 + z2) ≥ 0

Từ đó ta có đpcm.

Nhận xét 5 Đây chính là BĐT trong đề Iran TST 2013, có thể thấy giả thiết a,b,clà 3 cạnh của tam giác là không cần thiết.

Cho các số thực không âm a,b,c,d . Chứng minh bất đẳng thức:

(a +b + c +d)3 ≤ 4(a3 +b3 + c3 +d 3)+24(abc +bcd + cd a +d ab)

(Quảng Nam)

Bài 20

Lời giải

Không mất tính tổng quát, giả sử d = min{a,b,c,d} ⇒ d ∈[

0,a +b + c

3

].

Xét f (d) = 4(a3+b3+c3+d 3)+24(abc+bcd +cd a+d ab)−(a+b+c+d)3 ⇒ f ′(d) = 12d 2−3(a+b+c+d)2+24(ab+bc+ca) ⇒ f ′′(d) = 24d−6(a+b+c+d) = 6(3d−a−b−c) ≤ 0 ⇒ f (d)

là hàm lồi ⇒ f (d) đạt giá trị nhỏ nhất tại d = 0 hoặc d = a +b + c

3. Ta có:

f (0) = 3[a3 +b3 + c3 +3abc −ab(a +b)−bc(b + c)− ca(c +a)]+9abc ≥ 0

f

(a +b + c

3

)= 16

9(a3 +b3 + c3)+ 104

3abc + 4

3[ab(a +b)+bc(b + c)+ ca(c +a)] ≥ 0

Từ đó ta có đpcm.

http://thcmn.wordpress.com 53

Page 54: t 2017 - WordPress.com...Bài 5. (B›c Ninh) Cho a, b c¨0 thäa mãn đi•u ki»n ¯ ˘9. Tìm giá trà lîn nh t cıabi”uthøc T ˘ ab 3a¯4b¯5c ¯ bc 3b¯4c¯5a ¯ ca 3c¯4a¯5b

Cho ba thực dương a,b,c thỏa mãn a +b + c = 3. Chứng minh rằng

1

a+ 1

b+ 1

c+ a +1

1+b2+ b +1

1+ c2+ c +1

1+a2≥ 6 ≥ 8

a2 +b2 +2+ 8

b2 + c2 +2+ 8

c2 +a2 +2

(Quảng Ngãi)

Bài 21

Lời giải

Trước hết xin phát biểu lại một bổ đề quen thuộc.

Bổ đề 2 Cho a,b,c là ba số dương. Khi đó

(1+ 1

a

)(1+ 1

b

)(1+ 1

c

)≥

(1+ 1

3pabc

)3

.

Chứng minh bổ đề này dễ dàng và xin giành lại cho bạn đọc.

Quay lại bài toán.

• Đầu tiên ta sẽ chứng minh

1

a+ 1

b+ 1

c+ a +1

1+b2+ b +1

1+ c2+ c +1

1+a2≥ 6.

Điều này tương đương với

⇔ 1

2a+ 1

2b+ 1

2c+ 1

2a+ 1

2b+ 1

2c+ a

2+ b

2+ c

2+ a +1

1+b2+ b +1

1+ c2+ c +1

1+a2≥ 6+ a +b + c

2

⇔ 1

2a+ 1

2b+ 1

2c+ a2 +1

2a+ b2 +1

2b+ c2 +1

2c+ a +1

1+b2+ b +1

1+ c2+ c +1

1+a2≥ 15

2

Ta có các BĐT sau:

1

2a+ 1

2b+ 1

2c≥ 9

2(a +b + c)= 9

6= 3

2(1.7)

a2 +1

2a+ b2 +1

2b+ c2 +1

2c+ a +1

1+b2+ b +1

1+ c2+ c +1

1+a2≥ 6

6

√(a +1)(b +1)(c +1)

8abc

= 6 6

√1

8(1+ 1

a)(1+ 1

a)(1+ 1

a) ≥ 6 6

√1

8(1+ 1

3pabc)3 ≥ 6 6

√1

8(1+1)3 = 6 (1.8)

Từ (1.7) và (1.8) ta có điều phải chứng minh.

• Bây giờ ta sẽ chứng minh

8

a2 +b2 +2+ 8

b2 + c2 +2+ 8

c2 +a2 +2≤ 6

54 http://www.facebook.com/thcmn

Page 55: t 2017 - WordPress.com...Bài 5. (B›c Ninh) Cho a, b c¨0 thäa mãn đi•u ki»n ¯ ˘9. Tìm giá trà lîn nh t cıabi”uthøc T ˘ ab 3a¯4b¯5c ¯ bc 3b¯4c¯5a ¯ ca 3c¯4a¯5b

⇔ a2 +b2

a2 +b2 +2+ b2 + c2

b2 + c2 +2+ c2 +a2

c2 +a2 +2≥ 3

2

Ta có:

a2 +b2

a2 +b2 +2+ b2 + c2

b2 + c2 +2+ c2 +a2

c2 +a2 +2≥ (

pa2 +b2 +

pb2 + c2 +

pc2 +a2)2

2(a2 +b2 + c2 +3).

Vậy BĐT ban đầu sẽ đúng nếu ta chứng minh được

(p

a2 +b2 +p

b2 + c2 +p

c2 +a2)2

2(a2 +b2 + c2 +3)≥ 3

2.

⇔ (√

a2 +b2+√

b2 + c2+√

c2 +a2)2 ≥ 3(a2+b2+c2)+9 = 3(a2+b2+c2)+ (a+b+c)2

⇔√

(a2 +b2)(b2 + c2)+√

(b2 + c2)(c2 +a2)+√

(c2 +a2)(a2 +b2) ≥ a2+b2+c2+ab+bc+ca

Để ý rằng √(b2 + c2)(b2 +a2) ≥ b2 + ca,√(c2 +a2)(c2 +b2) ≥ c2 +ab,√(a2 +b2)(a2 + c2) ≥ a2 +bc.

Cộng vế theo vế ta có điều phải chứng minh.

Vậy BĐT ban đầu của ta đúng.

Nhận xét 6 Đây chính là bài toán trong đề chọn đội tuyển Iran 2009, và cách giải củanó như sau: Không mất tính tổng quát, giả sử: a ≥ b ≥ c ⇒ a ≥ 1;b + c ≤ 2.Do điều kiện đề bài là a +b + c = 3 nên để giảm tính phức tạp, ta sẽ dồn như sau:

f(a,b,c

)≤ f

(a,

b + c

2,

b + c

2

)

vớif(a,b,c

)= 1

2+a2 +b2+ 1

2+b2 + c2+ 1

2+ c2 +a2

Vì b + c ≤ 2, a ≥ b ≥ c nên ta có:

f

(a,

b + c

2,

b + c

2

)− f

(a,b,c

)

=

2

2+a2 +(

b + c

2

)2 − 1

2+a2 +b2− 1

2+a2 + c2

+

− 1

2+b2 + c2+ 1

2+ (b + c)2

2

= (b − c)2(2a2 −b2 −4bc − c2 +4)(

2+a2 +b2)(

2+a2 + c2)(

8+4a2 + (b + c)2) + (b − c)2

(2+b2 + c2)(4+ (b + c)2)

= (b − c)2

[2(a2 −bc)+4− (b + c)2(

2+a2 +b2)(

2+a2 + c2)(

8+4a2 + (b + c)2) + 1

(2+b2 + c2)(4+ (b + c)2)

]≥ 0

http://thcmn.wordpress.com 55

Page 56: t 2017 - WordPress.com...Bài 5. (B›c Ninh) Cho a, b c¨0 thäa mãn đi•u ki»n ¯ ˘9. Tìm giá trà lîn nh t cıabi”uthøc T ˘ ab 3a¯4b¯5c ¯ bc 3b¯4c¯5a ¯ ca 3c¯4a¯5b

Do đó ta chỉ còn cần chứng minh f(a, t , t

)≤ 3

4với a +2t = 3

Điều này tương đương với:

(a −1)2(15a2 −78a +111) ≥ 0

Do đó ta có điều phải chứng minh là đúng.

Cho a,b,c > 0 thỏa mãn (a+b)(b+c)(c +a) = 1. Tìm giá trị nhỏ nhất của biểu thức:

P =p

a2 −ab +b2

pab +1

+p

b2 −bc + c2

pbc +1

+p

c2 − ca +a2

pca +1

.

(Quảng Ninh)

Bài 22

Lời giải

Ta có:√

(a2 −ab +b2)(a +b)2 =√

(a3 +b3)(a +b) ≥√

(a2 +b2)2 = a2+b2 ⇒√

a2 −ab +b2 ≥a2 +b2

a +b. Chứng minh tương tự ta được P ≥∑ a2 +b2

(p

ab +1)(a +b)≥

(∑pa2 +b2

)2

2∑

a +∑pab(a +b)

.

Ta sẽ chứng minh (∑pa2 +b2

)2

2∑

a +∑pab(a +b)

≥ 1

⇔ 2(a2 +b2 + c2)+2∑√

(a2 +b2)(b2 + c2) ≥ 2∑

a +∑pab(a +b).

Để ý rằng ∑pab(a +b) ≤∑ 1

2(a +b)2 = a2 +b2 + c2 +ab +bc + ca.

Vậy ta cần chứng minh

a2 +b2 + c2 +2∑√

(a2 +b2)(b2 + c2) ≥ 2(a +b + c)+ab +bc + ca.

Mà a2 +b2 + c2 ≥ ab +bc + ca nên ta chỉ cần chứng minh∑√(a2 +b2)(b2 + c2) ≥ a +b + c.

Ta có∑√(a2 +b2)(b2 + c2) ≥∑

(b2 + ca) = 1

2

∑(a +b)2 ≥ 1

6

(∑(a +b)

)2 = 2

3(a +b + c)2.

Vì thế ta sẽ đi chứng minh

2

3(a +b + c)2 ≥ a +b + c ⇔ a +b + c ≥ 3

2,

một điều dễ dàng suy được từ điều kiện ban đầu. Vậy GTNN của P là 1 khi a = b = c =1

2.

56 http://www.facebook.com/thcmn

Page 57: t 2017 - WordPress.com...Bài 5. (B›c Ninh) Cho a, b c¨0 thäa mãn đi•u ki»n ¯ ˘9. Tìm giá trà lîn nh t cıabi”uthøc T ˘ ab 3a¯4b¯5c ¯ bc 3b¯4c¯5a ¯ ca 3c¯4a¯5b

Cho ba số không âm x, y, z thỏa mãn x + y + z = 2. Chứng minh:

x2 y + y2z + z2x ≤ x3 + y3 + z3 ≤ 1+ 1

2(x4 + y4 + z4).

(Quảng Trị)

Bài 23

Lời giải

Nhận xét rằng vế đầu của BĐT cần chứng minh luôn đúng vì

3(x3 + y3 + z3)−3(x2 y + y2z + z2x) =∑(x − y)2(2x + y) ≥ 0.

Bây giờ ta sẽ chứng minh vế sau. Chúng tôi xin trình bày hai cách chứng minh choBĐT này.

• Cách 1: Ta cóx3 + y3 + z3 ≤ 1+ 1

2(x4 + y4 + z4)

⇔ 2(x3 + y3 + z3) ≤ 2+x4 + y4 + z4

⇔ (x + y + z)(x3 + y3 + z3) ≤ 2+x4 + y4 + z4

⇔ x y(x2 + y2)+ y z(y2 + z2)+ zx(z2 +x2) ≤ 2.

Để ý rằng

x y(x2+y2)+y z(y2+z2)+zx(z2+x2) ≤ x y(x2+y2)+y z(y2+z2)+zx(z2+x2)+x y z(x+y+z)

= (x y + y z + zx)(x2 + y2 + z2)

≤ 1

2

(2(x y + y z + zx)+x2 + y2 + z2

2

)2

= 1

8(x + y + z)4 = 2.

Vậy ta có điều phải chứng minh.

• Cách 2: Đặtp = x + y + z = 2; q = x y + y z + zx;r = x y z.

Khi đóx3 + y3 + z3 = p3 −3pq +3r = 8−6q +3r

x4 + y4 + z4 = (p2 −2q)2 −2(q2 −2pr ) = 2q2 −16q +8r +16.

Do đó ta cần chứng minh:

8−6q +3r ≤ q2 −8q +4r +8+1 ⇔ r + (q −1)2 ≥ 0,

một điều luôn đúng với mọi x, y, z ≥ 0. Vậy BĐT ban đầu được chứng minh.

http://thcmn.wordpress.com 57

Page 58: t 2017 - WordPress.com...Bài 5. (B›c Ninh) Cho a, b c¨0 thäa mãn đi•u ki»n ¯ ˘9. Tìm giá trà lîn nh t cıabi”uthøc T ˘ ab 3a¯4b¯5c ¯ bc 3b¯4c¯5a ¯ ca 3c¯4a¯5b

1. Cho a, b, c là các số thực dương. Chứng minh rằng

1

(1+a)3 + 1(1+b

)3 + 1

(1+ c)3 + 3

32

(ab +bc + ca

)Ê 21

32.

2. Cho x, y , z là ba số thực dương thoả mãn x y z Ê 1 và z É 1. Tìm GTNN củabiểu thức

F = x

1+ y+ y

1+x+ 4− z3

3(1+x y

) .

(Thái Nguyên)

Bài 24

Lời giải

1. Trước hết xin nhắc lại bổ đề quen thuộc sau.

Bổ đề 3 Cho a,b > 0. Khi đó

1

(1+a)2+ 1

(1+b)2≥ 1

1+ab.

Chứng minh bổ đề này dễ dàng, xin giành lại cho bạn đọc.

Quay lại bài toán. Ta có:

1

2(1+a)3+ 1

2(1+a)3+ 1

16≥ 3 3

√1

2(1+a)3.

1

2(1+a)3.

1

16= 3

4.

1

(1+a)2.

Chứng minh tương tự ta được:

∑ 1

(1+a)3+ 3

32(∑

ab) ≥ 3

4

(∑ 1

(1+a)2

)+ 3

32(∑

ab)− 3

16.

Áp dụng thêm bổ đề vừa nêu trên có được:

∑ 1

(1+a)3+ 3

32(∑

ab)− 3

16≥ 3

4

(∑ 1

(1+a)2

)+ 3

32(∑

ab)− 3

16

≥ 3

8

(∑ 1

1+ab

)+ 3

32(∑

ab)− 3

16≥ 27

8.

1

ab +bc + ca +3+ 3

32(ab+bc+ca+3)− 9

32− 3

16

≥ 2

√27

8.

3

32− 9

32− 3

16= 21

32

chính là đpcm.

2. Chúng ta tiếp tục có một bổ đề khác cho bài này như sau.

58 http://www.facebook.com/thcmn

Page 59: t 2017 - WordPress.com...Bài 5. (B›c Ninh) Cho a, b c¨0 thäa mãn đi•u ki»n ¯ ˘9. Tìm giá trà lîn nh t cıabi”uthøc T ˘ ab 3a¯4b¯5c ¯ bc 3b¯4c¯5a ¯ ca 3c¯4a¯5b

Bổ đề 4 Cho x, y là hai số thực dương thoả x y ≥ 1. Khi đó

x

1+ y+ y

1+x≥ 2

px y

px y +1

.

Chứng minh bổ đề này xin giành lại cho bạn đọc.

Quay lại bài toán. Dễ thấy z ≤ 1 nên x y ≥ 1. Ta có:

F = x

1+ y+ y

1+x+ 1

1+x y+ 1− z3

3(1+x y)≥ x

1+ y+ y

1+x+ 1

1+x y≥ 2

px y

px y +1

+ 1

1+x y.

Ta sẽ chứng minh:2p

x yp

x y +1+ 1

1+x y≥ 3

2

⇔ (p

x y −1)2 ≥ 0,

một điều hiển nhiên đúng. Do vậy F ≥ 3

2. Vậy GTNN của F là

3

2khi x = y = z = 1.

Ta có:∑√

a

b + c=∑ 1p

bc(b + c)≥ 9∑p

bc(b + c)≥ 9√

3(∑

bc(b + c))= 3

p3p

a3 +b3 + c3 +3abc=

3p

3pa3 +b3 + c3 +3

Cho x, y, z > 0 thỏa x + y + z = 1

x+ 1

y+ 1

z. Chứng minh rằng

1

(2x y + y z + zx)2+ 1

(2y z + zx +x y)2+ 1

(2zx +x y + y z)2≤ 3

16x2 y2z2.

(Thanh Hoá)

Bài 25

Lời giải

Theo BĐT AM - GM thì∑c yc

1

(2x y + y z + zx)2≤ ∑

c yc

1

4(zx +x y)(x y + y z)= ∑

c yc

1

4x y(y + z)(z +x)

nên BĐT ban đầu sẽ đúng nếu ta chứng minh được

∑c yc

1

4x y(y + z)(z +x)≤ 3

16x2 y2z2⇔ ∑

c yc

x

(z +x)(x + y)≤ 3

4x y z

⇔ 8x y z(x y + y z + zx) ≤ 3(x + y)(y + z)(z +x)

⇔ 8

(1

x+ 1

y+ 1

z

)≤ 3

(1

x+ 1

y

)(1

y+ 1

z

)(1

z+ 1

x

). (1.9)

http://thcmn.wordpress.com 59

Page 60: t 2017 - WordPress.com...Bài 5. (B›c Ninh) Cho a, b c¨0 thäa mãn đi•u ki»n ¯ ˘9. Tìm giá trà lîn nh t cıabi”uthøc T ˘ ab 3a¯4b¯5c ¯ bc 3b¯4c¯5a ¯ ca 3c¯4a¯5b

Đặt a = 1

x,b = 1

y,c = 1

z, ta có a +b + c = 1

a+ 1

b+ 1

c. Khi đó (1.9) trở thành

8(a +b + c) ≤ 3(a +b)(b + c)(c +a)

⇔ 8(a +b + c)(ab +bc + ca) ≤ 3(a +b)(b + c)(c +a)(ab +bc + ca) (1.10)

Vì theo bổ đề 1,

3(a +b)(b + c)(c +a) ≥ 8

3(a +b + c)(ab +bc + ca)

đồng thời

ab +bc + ca = abc(a +b + c

)≤ 1

3

(ab +bc + ca

)2 ⇒ ab +bc + ca ≥ 3

nên (1.10) đúng và ta giải quyết xong BĐT. Dấu bằng xảy ra ⇔ x = y = z = 1.

Chú ý 1 Các BĐT với điều kiện a +b + c = 1

a+ 1

b+ 1

cluôn khá khó chịu, đòi hỏi người

làm phải nắm chắc những kiến thức cơ bản cộng với một chút khéo léo trong biến đổi,tính toán. Nhưng bù lại, những bài toán có điều kiện này ẩn chứa những điều thú vịcó thể khai thác được từ giả thiết. Cụ thể như sau:

1. 3 ·max{a,b,c} ≥ a +b + c = 1

a+ 1

b+ 1

c≥ 3

max{a,b,c}nên max{a,b,c} ≥ 1. Tương tự,

min{a,b,c} ≤ 1.

2. ab+bc+ca = abc(a +b + c

)≤ 1

3

(ab +bc + ca

)2 ⇒ ab+bc+ca ≥ 3 hay ab+bc+ca ≥ 3.

Từ đó a +b + c ≥√

3(ab +bc + ca

)= 3.

3.1

3

(a +b + c

)2 ≥ ab+bc +ca = abc(a +b + c

)≥ 3abc ⇒ a+b+c ≥ 3abc, ab+bc +ca ≥3abc.

2 Đa thức

Tìm tất cả đa thức hệ số thực thỏa mãn

2

(P (x)−P

(1

x

))2

+3P (x2)P

(1

x2

)= 0.

(THPT chuyên KHTN, ĐH KHTN, ĐHQG HN)

Bài 1

Lời giải

1. P (x) ≡C ,∀x ∈R.Khi đó dễ thấy P (x) ≡ 0.

60 http://www.facebook.com/thcmn

Page 61: t 2017 - WordPress.com...Bài 5. (B›c Ninh) Cho a, b c¨0 thäa mãn đi•u ki»n ¯ ˘9. Tìm giá trà lîn nh t cıabi”uthøc T ˘ ab 3a¯4b¯5c ¯ bc 3b¯4c¯5a ¯ ca 3c¯4a¯5b

2. Xét deg P ≥ 1Thay x = 1 vào ta được: P (1) = 0Giả sử P (x) = (x −1)n .Q(x) sao cho Q(1) 6= 0.Khi đó ta có:

2

[(x −1)n .Q(x)−

(1

x−1

)n

.Q

(1

x

)]2

+3(x2 −1)n .Q(x2).

(1

x2−1

)n

.Q

(1

x2

)= 0

⇒ 2

[Q(x)− (−1)n

xnQ

(1

x

)]2

+3(x +1)n .(−1)n(1+x)n

x2n.Q(x2).Q

(1

x2

)= 0.

Thay x = 1 vào biểu thức trên ta được:

2[Q(1)− (−1)n .Q(1)

]2 +3.(−4)n .Q(1)2 = 0 ⇔ 2(1− (−1)n)2 +3.(−4)n = 0

Dễ thấy không có n thỏa mãn. Do đó không tồn tại đa thức P (x) có deqP ≥ 1.Vậy P (x) ≡ 0.

Nhận xét 7 Tương tự, ta có thể giải được bài toán sau: Tìm đa thức P (x) thỏa mãn

P 2(x)+P 2(

1

x

)= P (x2).P

(1

x2

)

Cho khai triển (1−2x +x3)n = a0 +a1x +a2x2 + ...++a3n x3n . Xác định hệ số a6 biết

rằng a0 + a1

2+ a2

22+ ...+ a3n

23n=

(1

2

)15

(Bến Tre)

Bài 2

Lời giải

Cho x = 1

2⇒

(1

2

)3n

= a0 + a1

2+ a2

22+ ...+ a3n

23n=

(1

2

)15

⇔ 3n = 15 ⇔ n = 5

Ta có (x3 +1−2x

)5 =5∑

k=0C k

5 x15−3k (1−2x)k =5∑

k=0C k

5 x15−3kk∑

i=0C i

k (−2x)i

=5∑

k=0

k∑i=0

C k5 .C i

k .(−2)i .x15−3k+i (0 ≤ i ≤ k ≤ 5

)15−3k + i = 6 ⇔ 3k − i = 9Ta có bảng sau:

k 3 4 5i 0 3 6

⇒ k = 3, i = 0 hoặc k = 4, i = 3Vậy a6 =C 3

5 .C 03 .(−2)0 +C 4

5 .C 34 .(−2)3 =−150.

http://thcmn.wordpress.com 61

Page 62: t 2017 - WordPress.com...Bài 5. (B›c Ninh) Cho a, b c¨0 thäa mãn đi•u ki»n ¯ ˘9. Tìm giá trà lîn nh t cıabi”uthøc T ˘ ab 3a¯4b¯5c ¯ bc 3b¯4c¯5a ¯ ca 3c¯4a¯5b

Cho phương trình

x5 − 1

2x4 −5x3 +x2 +4x −1 = 0 (2.1)

Chứng minh rằng phương trình (2.1) có đúng 5 nghiệm phân biệt. Với xi (i = 1,5)

là nghiệm của phương trình (2.1), tính tổng S biết: S =5∑

i=1

xi +1

2x5i−x4

i −2.

(Bến Tre)

Bài 3

Lời giải

f (x) là hàm số xác định và liên tục trên RTa có:

f (−2) =−5; f

(−3

2

)= 2; f (0) =−1; f

(1

2

)= 5

8; f (1) = −1

2; f (3) = 175

2

Khi đó:

f (−2). f

(−3

2

)< 0; f

(−3

2

). f (0) < 0; f (0). f

(1

2

)< 0; f

(1

2

). f (1) < 0; f (1). f (3) < 0

Do đó phương trình có 5 nghiệm phân biệt

−2 < x1 < −3

2< x2 < 0 < x3 < 1

2< x4 < 1 < x5 < 3

Ta có xi là nghiệm của (2.1) nên:

xi5 − 1

2xi

4 −5xi3 +x2

i +4xi −1 = 0

⇔ 2x5i −x4

i −2 = 2(5x3

i −x2i −4xi

)Do đó: S =

5∑i=1

xi +1

2(5x3

i −x2i −4xi

)Xét biểu thức:

g (x) = x +1

5x3 −x2 −4x= x +1

x (x −1)(5x +4)

Đồng nhất thức ta được:

g (x) =− 1

4x+ 2

9(x −1)+ 5

36(5x +4)

Do vậy:

S =−1

8

5∑i=1

1

xi+ 1

9

5∑i=1

1

xi −1+ 1

72

5∑i=1

1

xi + 45

Với x 6= xi ta có:f ′ (x)

f (x)=

5∑i=1

1

x −xi

62 http://www.facebook.com/thcmn

Page 63: t 2017 - WordPress.com...Bài 5. (B›c Ninh) Cho a, b c¨0 thäa mãn đi•u ki»n ¯ ˘9. Tìm giá trà lîn nh t cıabi”uthøc T ˘ ab 3a¯4b¯5c ¯ bc 3b¯4c¯5a ¯ ca 3c¯4a¯5b

vàf ′ (x) = 5x4 −2x3 −15x2 +2x +4

Do đó:f ′ (1)

f (1)=

5∑i=1

1

1−xi⇒

5∑i=1

1

xi −1=− f ′ (1)

f (1)=−12

f ′ (0)

f (0)=

5∑i=1

1

−xi⇒

5∑i=1

1

xi=− f ′ (0)

f (0)= 4

f ′(−4

5

)f(−4

5

) =5∑

i=1

1

−45 −xi

⇒5∑

i=1

1

xi + 45

=−f ′

(−4

5

)f(

45

) =−12900

4789

Vậy: S =−8959

4789.

Cho dãy các đa thức hệ số thực {Pn(x)},n = 1,2,3, ... thỏa mãn điều kiện Pn(2cos x) =2n cos(nx),∀x ∈R,∀n ∈N∗. Chứng minh rằng với mỗi n ∈N∗ thì Pn(x) là đa thức hệsố nguyên bậc n và x ≤ n

√Pn(x),∀x > 2.

(Bình Dương)

Bài 4

Lời giải

Dễ thấy: P1(x) = x;P2(x) = 2x2 −4Ta có:

Pn+1(2cos x) = 2n+1 cos[(n +1)x] = 2n+1[cos(nx).cos x − sin(nx).sin x]

= 2cos x.2n cos(nx)−2n+1 sin(nx).sin x

Suy ra:Pn+2(2cos x) = 2cos x2n+1 cos[(n +1)x]−2n+2 sin[(n +1)x]sin x

= 2cos xPn+1(2cos x)+2n+1 [cos[(n +2)x]−cos(nx)

]= 2cos xPn+1(2cos x)+ 1

2Pn+2(2cos x)−2Pn(2cos x)

Thay 2cos x bởi x ta được:

Pn+2(x) = 2xPn+1(x)−4Pn(x)

Do P1(x),P2(x) ∈ Z[x] nên theo công thức truy hồi tuyến tính cấp hai đã chứng minhở trên, ta có: Pn(x) ∈Z[x],∀n ∈N∗.Ta sẽ đi tìm công thức tổng quát của dãy đa thức Pn(x) như sau:Phương trình đặc trưng của dãy: X 2 −2x X +4 = 0 ⇔ X = x +

√x2 −4∨X = x −

√x2 −4

Do đó: Pn(x) = A(x +√

x2 −4)n +B(x −√

x2 −4)n ,∀x > 2

Thay n = 1,n = 2 vào ta tìm được A = 1

2;B = 1

2Do đó:

Pn(x) = 1

2(x +

√x2 −4)n + 1

2(x −

√x2 −4)n ,∀x > 2

http://thcmn.wordpress.com 63

Page 64: t 2017 - WordPress.com...Bài 5. (B›c Ninh) Cho a, b c¨0 thäa mãn đi•u ki»n ¯ ˘9. Tìm giá trà lîn nh t cıabi”uthøc T ˘ ab 3a¯4b¯5c ¯ bc 3b¯4c¯5a ¯ ca 3c¯4a¯5b

Áp dụng bất đẳng thức: an +bn ≥ (a +b)n

2n−1,∀a,b > 0;n ∈N∗

Ta có:

Pn(x) ≥ 1

2.(2x)n

2n−1= xn

Do đó:x ≤ n

√Pn(x),∀x > 2.

Nhận xét 8 Bằng cách chuyển từ dãy đa thức lượng giác sang dạng đại số, ta có thểgiải quyết bài toán một cách đơn giản hơn. Sau đây là một số bài toán về dãy đa thức:

1. (England, 1978)

• Chứng minh rằng với mọi số tự nhiên n tồn tại đa thức Pn(x) bậc n với hệsố nguyên thỏa mãn Pn(2cos x) = 2cos(nx),∀x ∈R.

• Với mọi số hữu tỉ α thì số cos(απ) hoặc trùng với một trong các số 0,±1,±1

2hoặc là số vô tỉ.

2. (VMO, 1989) Cho dãy đa thức Pn(x) xác định bởi

P0(x) = 0;Pn+1(x) = Pn(x)+ x −P 2n(x)

2,n ≥ 0

Chứng minh rằng với mọi x ∈ [0;1] và n ≥ 0 thì 0 ≤px −Pn(x) ≤ 2

n +1

Tìm số nguyên dương n nhỏ nhất sao cho tồn tại đa thức f (x) bậc n có hệ sốnguyên thỏa mãn: f (0) = 0, f (1) = 1 và với mọi m ∈ N∗, f (m)( f (m)− 1) là bội của2017.

(Đà Nẵng)

Bài 5

Lời giải

Ta sẽ chứng minh n ≥ 2016 thỏa mãn. Giả sử 1 ≤ n ≤ 2015 và tồn tại đa thức f (x) thỏamãn bài toán. Áp dụng công thức nội suy Lagrange:

f (x) =2015∑k=0

f (k).2015∏

i=0,i 6=k

x − i

k − i.

Suy ra:

f (2016) =2015∑k=0

f (k).2015∏

i=0,i 6=k

2016− i

k − i=

2015∑k=0

(−1)k+1C k2016 f (k).

Ta chứng minh C k2016 ≡ (−1)k ( mod 2017),k = 0,1, ...,2015

Với k = 0 thì C 02016 ≡ 1( mod 2017). Giả sử C k

2016 ≡ (−1)k ( mod 2017) thì C k+12016 =C k+1

2017−C k2016 ≡

64 http://www.facebook.com/thcmn

Page 65: t 2017 - WordPress.com...Bài 5. (B›c Ninh) Cho a, b c¨0 thäa mãn đi•u ki»n ¯ ˘9. Tìm giá trà lîn nh t cıabi”uthøc T ˘ ab 3a¯4b¯5c ¯ bc 3b¯4c¯5a ¯ ca 3c¯4a¯5b

(−1)k+1( mod 2017). Vậy f (k) ≡−2015∑k=0

f (k)( mod 2017) ⇒2016∑k=0

f (k) ≡ 0( mod 2017).

Nhưng2016∑k=0

f (k) = 1+2016∑k=2

f (k)

vàf (k) ≡ 0;1( mod 2017);k = 2,3, ...,2016

Do đó2016∑k=0

6≡ 0( mod 2017) (mâu thuẫn nhận xét trên).

Vậy n ≥ 2016. Với n = 2016 xét f (x) = x2016 thỏa mãn yêu cầu bài toán. Do đó nmi n = 2016

Chứng minh rằng với mọi m ∈N, tồn tại đa thức fm(x) có hệ số hữu tỉ thỏa mãnvới mọi n ∈N∗ thì: 12m+1 +22m+1 + ...+n2m+1 = fm(n(n +1)).

(Đà Nẵng)

Bài 6

Lời giải

• Với m = 0: f0(x) = x

2;m = 1 : f1(x) = x2

4.

• Giả sử khẳng định là đúng đến m −1, xét:

ϕ(x) = x(x2 −12)(x2 −22)...(x2 −m2) = x2m+1 −am−1x2m−1 −am−2x2m−3 − ...−a0x

⇔ϕ(x)+am−1x2m−1 +am−2x2m−3 + ...+a0x = x2m+1 (2.2)

Thay x lần lượt bởi 1,2, ...,n ta được:

n∑i=1

ϕ(i )+m−1∑i=0

ai fi (n(n +1)) =n∑

i=1i 2m+1 (2.3)

Ta có:

ϕ(x) = x(x −1)(x +1)...(x −m)(x +m) = (x −m)(x −m +1)...(x +m −1)(x +m)

Lại có:

ϕ(x) = 1

2m +2

[ϕ(x)(x +m +1)−ϕ(x)(x −m −1)

]Đặt d(x − m) = (x + m + 1)ϕ(x) = (x − m)(x − m + 1)...(x + m + 1) ⇒ d(x − m − 1) =(x−m−1)(x−m)...(x+m) = (x−m−1)ϕ(x) ⇒ϕ(x) = 1

2m +2

[d(x −m)−d(x −m −1)

].

Do đó

n∑i=1

ϕ(i ) = 1

2m +2

[d(n −m)−d(−m)

]= 1

2m +2d(n−m) = 1

2m +2(n−m)(n−m+1)...(n+m+1)

http://thcmn.wordpress.com 65

Page 66: t 2017 - WordPress.com...Bài 5. (B›c Ninh) Cho a, b c¨0 thäa mãn đi•u ki»n ¯ ˘9. Tìm giá trà lîn nh t cıabi”uthøc T ˘ ab 3a¯4b¯5c ¯ bc 3b¯4c¯5a ¯ ca 3c¯4a¯5b

Ta có: (n − i )(n + i +1) = n2 − i 2 +n − i = n(n +1)− i (i +1), i = 0,1, ...,mSuy ra:

n∑i=1

ϕ(i ) = 1

2m +2

m∏i=0

[n(n +1)− i (i +1)

]= hm(n(n +1)).

Khi đó hm có hệ số hữu tỉ và từ (2.3) chọn:

fm(x) = hm(x)+am−1 fm−1(x)+am−2 fm−2(x)+ ...+a0 f0(x).

Suy ra điều phải chứng minh.

Nhận xét 9 Một số bài toán tương tự:

1. Tìm số tự nhiên n để P (x) = x2n +xn +1 chia hết cho Q(x) = x2 +x +1.

2. Chứng minh rằng với mọi n ∈ Z+ ta có đa thức P (x) = (x +1)2n+1 + xn+2 chia hếtcho đa thức Q(x) = x2 +x +1.

Cho số tự nhiên n ≥ 2 và n số thực a1, a2, . . . , an sao cho a1 > −1, a2 ≥ n −1

2. Giả

sử phương trình xn +a1xn−1 +a2xn−2 + . . .+an−1x +an = 0 có đúng n nghiệm thực.Chứng minh rằng tất cả các nghiệm đó nằm trong đoạn [−a1, a1 +2].

(Đồng Nai)

Bài 7

Lời giải

Đặt P (x) = xn +a1xn−1 +a2xn−2 + . . .+an−1x +an.Gọi x1, x2, ..., xn là n nghiệm của phương trình P (x) = 0.Khi đó:

x1 +x2 + ...+xn =−a1

x1x2 +x1x3 + ...+xn−1xn = a2 ≥ 1

2

Mặt khác:

x2k ≤

n∑i=1

x2i = a2

1 −2a2 ≤ a21

Do đó:xk ∈ [−a1; a1],k = 1,2, ...,n

Vậy tất cả các nghiệm của P (x) = 0 đều nằm trong đoạn [−a1; a1 +2].

Nhận xét 10 Đôi lúc trong bài toán đa thức ta cần chú ý đến nghiệm của đa thức. Mộttrong những hướng xử lý là dùng định lý Bezout, định lý Viete,... Sau đây là một số bàitoán về nghiệm của đa thức:

1. (Costa Rica, 2009) Giả sử đa thức xn + an−1xn−1 + an−2xn−2 + ...+ a1x + a0 có thểphân tích thành (x + r1)(x + r2)...(x + rn), với r1,r2, ...,rn ∈R. Chứng minh rằng

(n −1)a2n−1 ≥ 2nan−2

66 http://www.facebook.com/thcmn

Page 67: t 2017 - WordPress.com...Bài 5. (B›c Ninh) Cho a, b c¨0 thäa mãn đi•u ki»n ¯ ˘9. Tìm giá trà lîn nh t cıabi”uthøc T ˘ ab 3a¯4b¯5c ¯ bc 3b¯4c¯5a ¯ ca 3c¯4a¯5b

2. (Taiwan TST Round 1, 2014) Cho đa thức hệ số thực P (x) = xn+an−2xn−2+an−3xn−3+·· ·+a1x +a0 (n ≥ 2). Giả sử xk nghiệm thực của phương trình P (x) = 0 k = 1,2, ...,n

. Chứng minh rằng |xk | ≤√

2(1−n)

nan−2.

3. Cho đa thức hệ số thực P (x) = an xn+an−1xn−1+...+a1x+a0, an 6= 0. Giả sử x0 là một

nghiệm tùy ý của P (x) = 0. Chứng minh rằng: |x0| < 1+max

∣∣∣∣ ai

an

∣∣∣∣ (i = 0,1, ...,n −1)

Cho P,Q,R là 3 đa thức hệ số thực thỏa mãn: P (Q(x))+P (R(x)) = c ∀x ∈ R với c =const ∈R Chứng minh rằng P (x) ≡ const hoặc [Q(x)+R(x)] ≡ const .

(Hà Nam)

Bài 8

Lời giải

Bỏ qua trường hợp P (x) là hằng số.Xét P (x) = an xn +an−1xn−1 + ...+a0(an 6= 0,n ≥ 1)Xét Q(x) = bm xm +bm−1xm−1 + ...+b0

R(x) = ck xk + ck−1xk−1 + ...+ c0

Giả sử m ≥ k. Ta có:P (Q(x))+P (R(x)) = c.

Đồng nhất hệ số của bậc cao nhất trong (*) ta được: an .bnm = 0

Do đó hoặc bm = 0 suy ra:degQ = deg R = 0 hoặc m = k.Khi m = k thì an .(bn

m + cnm) = 0 ⇒ bm =−cm và n lẻ.

Khi đóQ(x) = bm xm +bm−1xm−1 + ...+b0(bm 6= 0)

R(x) =−bm xm + cm−1xm−1 + ...+ c0(bm 6= 0)

Xét đa thức H(x) = an(Qn(x)+Rn(x)

)= an(Q(x)+R(x)

)S(x), trong đó

S(x) =Qn−1(x)−Qn−2(x)R(x)+Qn−3(x)R2(x)− ...−Q(x).Rn−2(x)+Rn−1(x)

Xét hệ số của x(n−1)m trong S(x): bn−1 −bn−2(−b)+·· ·−b(−b)n−2 + (−b)n−1 = nbn−1 6= 0Do đó deg S = m(n −1).Ta sẽ giả sử phản chứng rằng deg (Q(x)+R(x)) ≥ 1Khi đó: deg H ≥ m(n −1)+1

Xét T (x) = P (Q(x))+P (R(x))−H(x) =n−1∑i=0

ai

(Q i (x)+R i (x)

)Nhận thấy rằng deg T ≤ m(n −1) (Vô lí)Do đó ta có điều phải chứng minh là đúng.

Nhận xét 11 Khi gặp các dạng toán về đa thức, thông thường ta phải chú ý đến mộtsố vấn đề quan trọng như bậc của đa thức, mối quan hệ giữa các hệ số với nhau, ngoàira cũng nên chú ý đến các yếu tố giải tích. Sau đây là một số bài toán có liên quan.

http://thcmn.wordpress.com 67

Page 68: t 2017 - WordPress.com...Bài 5. (B›c Ninh) Cho a, b c¨0 thäa mãn đi•u ki»n ¯ ˘9. Tìm giá trà lîn nh t cıabi”uthøc T ˘ ab 3a¯4b¯5c ¯ bc 3b¯4c¯5a ¯ ca 3c¯4a¯5b

1. (Tiêu chuẩn Eisenstein) Cho đa thức hệ số nguyên P (x) = an xn + an−1xn−1 + ...+a0(an 6= 0) và số nguyên tố p sao cho thỏa mãn đồng thời cả 3 điều kiện:

• ap 6 ...p

• a0, a1, ..., an−1...p

• a0 6 ...p2

Chứng minh rằng khi ấy P (x) bất khả quy trên Z[x].

2. (Romania 1980) Tìm tất cả các đa thức hệ số thực thỏa mãn P (x2) = P 2(x),∀x ∈R

3. Cho đa thức P (x) và Q(x) là các đa thức monic thỏa mãn P (P (x)) =Q(Q(x)).Chứngminh rằng P (x) ≡Q(x)

4. (Albanian TST, 2009) Tìm tất cả đa thức P (x) khác đa thức không, có hệ số khôngâm và thỏa mãn:

P (x).P

(1

x

)≤ P 2(1),∀x > 0

Cho các đa thức P (x),Q(x),R(x) với hệ số thực có bậc tương ứng là 3,2,3 thỏa mãnđẳng thức P 2(x)+Q2(x) = R2(x)∀x ∈R. Hỏi đa thức T (x) = P (x).Q(x).R(x) có ít nhấtbao nhiêu nghiệm thực (kể cả nghiệm bội).

(Hà Tĩnh)

Bài 9

Lời giải

Đầu tiên ta có thể thấy rằng

Q2(x) = (R(x)−P (x))(R(x)+P (x))

Do deg (R+P ) = 3 nên phương trình R(x)+P (x) = 0 có ít nhất 1 nghiệm, điều này chứngtỏ rằng phương trình Q(x) = 0 có nghiệm. Mà do degQ = 2 nên Q(x) = 0 có 2 nghiệm.Mặt khác:

P 2(x) = (R(x)−Q(x))(R(x)+Q(x))

Do deg (R +Q) = deg (R −Q) = 3 nên P 2(x) = 0 có ít nhất 2 nghiệm.

1. Hai nghiệm của P 2(x) = 0 là phân biệt. Khi đó P (x) = 0 có ít nhất 2 nghiệm, màdeg P = 3 nên P (x) = 0 sẽ có 3 nghiệm thực. Mặt khác do deg R = 3 nên R(x) = 0có ít nhất 1 nghiệm. Do vậy mà T (x) = 0 có ít nhất 6 nghiệm.

2. Hai nghiệm là nghiệm kép x = a. Do đó:

68 http://www.facebook.com/thcmn

Page 69: t 2017 - WordPress.com...Bài 5. (B›c Ninh) Cho a, b c¨0 thäa mãn đi•u ki»n ¯ ˘9. Tìm giá trà lîn nh t cıabi”uthøc T ˘ ab 3a¯4b¯5c ¯ bc 3b¯4c¯5a ¯ ca 3c¯4a¯5b

• R(x)−Q(x) = 0 và R(x)+Q(x) = 0 đều nhận a làm nghiệm.Khi đó P (a) =Q(a) = R(a) = 0.Theo định lý Bezout ta sẽ phân tích được:

P (x) = (x −a)P1(x);Q(x) = (x −a)Q1(x);R(x) = (x −a)R1(x)

với P1(x),Q1(x),R1(x) là các đa thức có hệ số cao nhất dương.Do đó:

P 21 (x)+Q2

1(x) = R21(x)

Xét P1(x) = ax2 +bx + c;Q1(x) = d x +e;R1(x) = f x2 + g x +hĐồng nhất hệ số của x4 và x3 trong khai triển ta được:

a = f ;2ab = 2 f g ⇒ b = g

Do đóP1(x) = ax2 +bx + c;Q1(x) = d x +e;R1(x) = ax2 +bx +h

Ta có:(d x +e)2 = (h − c)(P1(x)+R1(x))

Mà deg (P1+R1) = 2 nên P1(x)+R1(x) = 2ax2+2bx+c +h có nghiệm kép. Suyra:

b2 = 2a(c +h)

Nếu cả hai đa thức P1(x) và R1(x) đều không có nghiệm thực thì

b2 < 4ac;b2 < 4ah

hay làb2 < 2a(c +h) (Vô lí)

Do đó một trong hai đa thức có 2 nghiệm. Điều này dẫn đến T (x) = 0 có 6nghiệm.

• Chỉ có một trong hai đa thức R(x)−Q(x);R(x)+Q(x) nhận a làm nghiệm.Khi đó do deg (R −Q) = deg (R +Q) = 3 nên R(x)−Q(x) = 0 hoặc R(x)+Q(x) sẽcó 3 nghiệm và P 2(x) = 0 cũng sẽ có ít nhất 3 nghiệm.Do deg P = 3 nên P 2(x) = 0 sẽ có 2 nghiệm hoặc có 6 nghiệm. Mà P 2(x) = 0có ít nhất 3 nghiệm nên P (x)2 = 0 phải có 6 nghiệm hay là P (x) = 0 có 3nghiệm.Do đó T (x) = 0 có ít nhất 6 nghiệm.

Nhận xét 12 Đối với những bài toán này ta cần phải để ý đến bậc, hệ số của bậc caonhất, nghiệm của đa thức. Ngoài ra cần kết hợp sử dụng các định lý về nghiệm nhưViete, Bezout,... hay đồng nhất hệ số. Sau đây là các bài toán tương tự:

1. Cho P (x),Q(x) là hai đa thức bậc n. Chứng minh rằng P 2(x) ≡ Q2(x) hoặc là đathức P 2(x)−Q2(x) có bậc không nhỏ hơn n.

2. Cho đa thức P (x) có bậc 2n, hệ số của bậc cao nhất là 1. Chứng minh rằng tồn tại2 đa thức Q(x) và R(x) sao cho degQ = n,deg R < n và P (x) =Q2(x)+R(x).

http://thcmn.wordpress.com 69

Page 70: t 2017 - WordPress.com...Bài 5. (B›c Ninh) Cho a, b c¨0 thäa mãn đi•u ki»n ¯ ˘9. Tìm giá trà lîn nh t cıabi”uthøc T ˘ ab 3a¯4b¯5c ¯ bc 3b¯4c¯5a ¯ ca 3c¯4a¯5b

Cho dãy đa thức hệ số thực{Pn (x)

}+∞n=0 xác định như sau P0 (x) = 2,P1 (x) = 2x,Pn+1 (x) =

2x.Pn (x)+(1−x2

)Pn−1 (x)∀n ≥ 1

1. Xác định công thức tổng quát của Pn (x).

2. Tìm tất cả các số tự nhiên n để Pn (x) chia hết cho x2 +3.

(Hải Phòng)

Bài 10

Lời giải

1. Đầu tiên ta xét phương trình đặt trưng của dãy đa thức trên:

t 2 −2xt +x2 −1 = 0 ⇔ t = x +1∨ t = x −1

Do đó dãy có dạng Pn(x) = a(x +1)n +b(x −1)n.Mặt khác do P0(x) = 2;P1(x) = 2x nên suy ra a = b = 1.Do đó Pn(x) = (x +1)n + (x −1)n . Thử lại thấy thỏa mãn.

2. Do đa thức Q(x) = x2 +3 có 2 nghiệm làp

3i và −p3i nên để Pn(x) chia hết choQ(x) thì P (x) nhận ±p3i làm nghiệm.

• P (p

3i ) = 0. Khi đó: (p

3i +1)n + (p

3i −1)n = 0

⇔(cos

π

3+ i sin

π

3

)n

+(cos

3+ i sin

3

)n

= 0

Theo công thức Moivre ta có:

cosnπ

3+ i sin

3+cos

2nπ

3+ i sin

2nπ

3= 0 ⇔ sin

3= 0 ⇔ n = 3k(k ∈Z)

• P (−p3i ) = 0. Khi đó: (−p3i +1)n + (−p3i −1)n = 0

⇔(cos

−π3

+ i sin−π3

)n

+(cos

3+ i sin

3

)n

= 0

Theo công thức Moivre ta có:

cosnπ

3+ i sin

−nπ

3+cos

4nπ

3+ i sin

4nπ

3= 0V n = 2k +1(k ∈Z)

Kết hợp cả hai dữ kiện trên ta được: n = 3(2k +1) = 6k +3.

Nhận xét 13 Một số bài toán tương tự:

1. (VMO, 2015) Cho dãy đa thức {Pn(X )} thỏa mãn điều kiện P0(x) = 2,P1(x) = 3x và

Pn(x) = 3xPn−1(x)+ (1−x −2x2)Pn−2(x),∀n ≥ 2

Tìm tất cả các số tự nhiên n để Pn(x) chia hết cho đa thức Q(x) = x3 −x2 +x.

70 http://www.facebook.com/thcmn

Page 71: t 2017 - WordPress.com...Bài 5. (B›c Ninh) Cho a, b c¨0 thäa mãn đi•u ki»n ¯ ˘9. Tìm giá trà lîn nh t cıabi”uthøc T ˘ ab 3a¯4b¯5c ¯ bc 3b¯4c¯5a ¯ ca 3c¯4a¯5b

2. (Trường Đông, 2015) Cho dãy đa thức {Pn(X )} thỏa mãn điều kiện P0(x) = 2,P1(x) =x và

Pn+1(x) = xPn(x)−Pn−1(x),∀n ≥ 2

• Chứng minh rằng tất cả đa thức của dãy đều thỏa mãn

P (x2 −2) = P 2(x)−2

• Chứng minh rằng với mọi n ≥ 1 thì Pn(x) có n nghiệm thực.

Cho đa thức P (x) = x4+ax3+bx2+cx+d và Q(x) = x2+px+q cùng thuộcQ[x] . Biếtrằng hai đa thức cùng nhận giá trị âm trên khoảng I có độ dài lớn hơn 2 và ngoàikhoảng I chúng đều nhận giá trị không âm . Chứng minh rằng tồn tại xo ∈ R đềP (xo) <Q(xo).

(Hòa Bình)

Bài 11

Lời giải

Gọi (x1; x2) là khoảng I có độ dài lớn hơn 2 thỏa đề. Do P (x) liên tục trên R và với 3giá trị xi , x j , xk sao cho chúng lần lượt thuộc các khoảng (−∞; x1); (x1; x2); (x2;+∞) nênP (xi ).P (x j ) < 0;P (x j ).P (xk ) < 0. Do đó P (x) có ít nhất 2 nghiệm, giả sử 2 nghiệm nàykhông phải là x1, x2.Do P (x) chỉ nhận giá trị âm trong (x1; x2) còn ngoài khoảng đó thì nhận giá trị khôngâm, đồng thời P (x) liên tục trên R nên không xày ra trường hợp P (x1),P (x2) 6= 0. Vậyx1, x2 là nghiệm của P (x) = 0. Xét các trường hợp sau:

• x1 không là nghiệm của Q(x) = 0 ⇒ P (x1) = 0 <Q(x1) (đpcm).

• x2 không là nghiệm của Q(x) = 0 ⇒ P (x2) = 0 <Q(x2) (đpcm).

• x1, x2 đều là nghiệm của Q(x) = 0. Do |x1 − x2| ≥ 2 nên p2 −4q ≥ 4. Theo định lýBezout và do P,Q đều monic nên P (x) =Q(x).H(x) với H(x) = x2 +mx +n. Giả sửphản chứng rằng: P (x) ≥Q(x),∀x ∈R. Khi đó: P (x)−Q(x) =Q(x).

[H(x)−1

].

Thay x = xi ta có: P (xi )−Q(xi ) = Q(xi )[H(xi )−1

] ≥ 0 ⇒ H(xi ) ≥ 1. Tương tự, vớimọi x ∈ (x1; x2) thì H(x) ∈ (0;1) và với x 6∈ (x1; x2) thì H(x) ≥ 1.Do H(x)−1 chỉ nhận giá trị âm trong (x1; x2) và ngoài đó thì nhận giá trị khôngâm nên nếu H(x1)−1, H(x2)−1 6= 0 thì vô lý vì H(x)−1 liên tục trên R. Vậy H(x)−1nhận x1, x2 làm nghiệm, dẫn tới P (x) −Q(x) có 2 nghiệm kép. Mà P (x) −Q(x)monic nên H(x)−1 ≡Q(x) hay P (x) =Q2(x)+Q(x).Để ý rằng nếu chọn β sao cho β ∈ I và P (β) = Q2(β)+Q(β) ≥ 0 thì Q2(β)+Q(β) =(β2 +pβ+q)2 + (β2 +pβ+q) ≥ 0 ⇔β2 +pβ+q ≤−1 (do Q(β) < 0)

⇔β ∈(−p −√

p2 −4(q +1)

2;−p −√

p2 −4(q +1)

2

)

Mặt khác dễ dàng chứng minh:

(−p −√

p2 −4(q +1)

2;−p −√

p2 −4(q +1)

2

)⊂ (x1; x2)

nên tồn tại β sao cho β ∈ I và P (β) ≥ 0. Điều này vô lí. Vậy không tồn tại x0 đểP (x0) <Q(x0).

http://thcmn.wordpress.com 71

Page 72: t 2017 - WordPress.com...Bài 5. (B›c Ninh) Cho a, b c¨0 thäa mãn đi•u ki»n ¯ ˘9. Tìm giá trà lîn nh t cıabi”uthøc T ˘ ab 3a¯4b¯5c ¯ bc 3b¯4c¯5a ¯ ca 3c¯4a¯5b

Nhận xét 14 Bài tập tương tự:

1. (Canada, 1981) Cho hai đa thức P (x),Q(x) thỏa mãn phương trình P (x) = Q(x)không có nghiệm thực và P (Q(x)) ≡ Q(P (x))∀x ∈ R. Chứng minh rằng phươngtrình P (P (x)) =Q(Q(x)) cũng không có nghiệm thực.

2. (VMO, 2003) Cho hai đa thức P (x) = 4x3−2x2−15x+9 và Q(x) = 12x3+6x2−7x+1.Chứng minh rằng P (x) = 0 và Q(x) = 0 đều có 3 nghiệm phân biệt. Chứng minhrằng tồn tại α là nghiệm của P (x) và β là nghiệm của Q(x) sao cho α2 +3β2 = 4.

3. Cho đa thức P (x) = x3+ax2+bx+c có 3 nghiệm phân biệt và Q(x) = x2+2x+2016.Biết rằng phương trình P (Q(x)) = 0 vô nghiệm. Chứng minh rằng P (2016) > 1.

Cho đa thức P (x) = x2016 + a2015x2015 + a2014x2014 + ...+ a1x + a0 có hệ số thực với

P (1)P (2) 6= 0 và 4P /(2)

P (2)> P /(1)

P (1)+2016. Giả sử P (x) có 2016 nghiệm thực, chứng minh

rằng trong số đó, có ít nhất một nghiệm thuộc khoảng (1;2).

(Tp. HCM)

Bài 12

Lời giải

Gọi x1, x2, ..., x2016 là nghiệm của P (x). Theo định lý Bezout ta có:

P (x) = (x −x1)(x −x2)...(x −xn)

Khi đó:

P ′(x) = (x −x1)(x −x2)...(x −xn)

(1

x −x1+ 1

x −x2+ ...+ 1

x −xn

)Do đó:

P ′(x)

P (x)=

n∑i=1

1

x −xi

Ta sẽ đi phản chứng rằng P (x) không có nghiệm nào thuộc khoảng (1;2).Ta có:

4.P ′(2)

P (2)− P ′(1)

P (1)=

2016∑i=1

(4

2−xi− 1

1−xi

)=

2016∑i=1

(2−3xi

(2−xi )(1−xi )

)Do xi > 2 hoặc xi < 1 nên (2−xi )(1−xi ) > 0. Ngoài ra:

2−3xi

(2−xi )(1−xi )≤ 1,∀xi 6∈ (1;2)

Suy ra:

4.P ′(2)

P (2)− P ′(1)

P (1)≤ 2016 (Vô lí)

Do đó tồn tại ít nhất một nghiệm thuộc khoảng (1;2).

Nhận xét 15 Đây là một dạng toán hay về hệ quả của định lý Bezout. Để xử lí đượcdạng toán này cần phải có những kiến thức về đại số, giải tích hỗ trợ. Sau đây là mộtsố bài toán về áp dụng định lý Bezout vào việc xử lý các bài toán bất đẳng thức trongđa thức:

72 http://www.facebook.com/thcmn

Page 73: t 2017 - WordPress.com...Bài 5. (B›c Ninh) Cho a, b c¨0 thäa mãn đi•u ki»n ¯ ˘9. Tìm giá trà lîn nh t cıabi”uthøc T ˘ ab 3a¯4b¯5c ¯ bc 3b¯4c¯5a ¯ ca 3c¯4a¯5b

1. Cho đa thức P (x) bậc n có hệ số thực, hệ số cao nhất là 1 và có n nghiệm thực saocho P (−1) 6= 0 và

−P ′(−1)

P (−1)≤ n

2.

Chứng minh rằng P (x) có ít nhất một nghiệm x0 với∣∣x0

∣∣≥ 1.

2. Cho đa thức P (x) = an xn + an−1xn−1 + ...+ a1x + a0,n ≥ 2, an a0 6= 0 và có n nghiệm

dương. Chứng minh rằng∣∣∣∣an−1a1

a0an

∣∣∣∣≥ n2

3. (Hong Kong, 1997) Cho P (x) = xn +an−1xn−1 + ...+a0 có bậc n ≥ 2 và có n nghiệmthực x1, x2, ..., xn. Chứng minh rằng:

P (x +1).P ′(x)

P (x)≥ 2n2,∀x > xi

4. (Hungary, 1983) Cho P (x) = xn +an−1xn−1 + ...+a1x +1 là đa thức hệ số dương vàphương trình P (x) = 0 có n nghiệm thực. Chứng minh rằng: P (2) ≥ 3n.

5. (Costa Rica, 2009) Giả sử đa thức xn + an−1xn−1 + an−2xn−2 + ...+ a1x + a0 có thểphân tích thành (x + r1)(x + r2)...(x + rn), với r1,r2, ...,rn ∈R. Chứng minh rằng

(n −1)a2n−1 ≥ 2nan−2

Cho P (x) là đa thức với hệ số nguyên. Chứng minh rằng tồn tại hai đa thức Q(x)và R(x) sao cho

1. P (x)Q(x) là các đa thức của x2

2. P (x)R(x) là các đa thức của x3

(Khánh Hòa)

Bài 13

Lời giải

1. Chú ý hằng đẳng thức sau:

a2 −b2 = (a +b)(a −b)

Với

P (x) =n∑

i=0ai xi =

[n2

]∑i=0

a2i x2i +

[n2

]∑i=0

a2i+1x2i+1

Ta chọn:

Q(x) =

[n2

]∑i=0

a2i x2i −

[n2

]∑i=0

a2i+1x2i+1

http://thcmn.wordpress.com 73

Page 74: t 2017 - WordPress.com...Bài 5. (B›c Ninh) Cho a, b c¨0 thäa mãn đi•u ki»n ¯ ˘9. Tìm giá trà lîn nh t cıabi”uthøc T ˘ ab 3a¯4b¯5c ¯ bc 3b¯4c¯5a ¯ ca 3c¯4a¯5b

Suy ra:

P (x)Q(x) =

[

n2

]∑i=0

a2i x2i

2

[

n2

]∑i=0

a2i+1x2i+1

2

= S(x2)

2. Chú ý hằng đẳng thức sau:

a3 +b3 + c3 −3abc = (a +b + c)(a2 +b2 + c2 −ab −bc − ca)

Ta xét:

P (x) =n∑

i=0ai xi =

[n3

]∑i=0

a3i x3i +

[n3

]∑i=0

a3i+1x3i+1 +

[n3

]∑i=0

a3i+2x3i+2

Đặt:

A(x) =

[n3

]∑i=0

a3i x3i ;B(x) =

[n3

]∑i=0

a3i+1x3i+1;C (x) =

[n3

]∑i=0

a3i+2x3i+2

Ta sẽ chọn

R(x) = A2(x)+B 2(x)+C 2(x)− A(x)B(x)−B(x)C (x)−C (x)A(x)

Suy ra:P (x)R(x) = A3(x)+B 3(x)+C 3(x)−3A(x)B(x)C (x) = T (x3)

Nhận xét 16 Một số bài toán tương tự:

1. Biểu diễn đa thức P (x) = x4 + x3 + x2 + x +1 dưới hạng hiệu bình phương của haiđa thức:P (x) = Q2(x)−R2(x) bậc khác nhau và với hệ số thực. Chứng minh rằngkhông tồn tại đa thức hệ số thực H(x) để P (x) = H 2(x).

2. Tồn tại hay không đa thức P (x) để ∀x ∈R thì P ′(x) > P ′′(x) và P (x) > P ′′(x).

3. (Iran, 2013) Cho a0, a1, . . . , an ∈N. Chứng minh rằng tồn tại các số tự nhiên b0,b1, . . . ,bn

thỏa mãn 0 ≤ i ≤ n : ai ≤ bi ≤ 2ai . Chứng minh rằng

P (x) = b0 +b1x +·· ·+bn xn

bất khả quy trên Z[x].

Tìm tất cả đa thức P (x) thỏa mãn

P (−x).P (3x)+ (P (2x)

)2 = P (x).P (5x),∀x ∈R. (2.4)

(Long An)

Bài 14

Lời giải

Thay x = 0 vào (2.4) ta được: P (0) = 0.

74 http://www.facebook.com/thcmn

Page 75: t 2017 - WordPress.com...Bài 5. (B›c Ninh) Cho a, b c¨0 thäa mãn đi•u ki»n ¯ ˘9. Tìm giá trà lîn nh t cıabi”uthøc T ˘ ab 3a¯4b¯5c ¯ bc 3b¯4c¯5a ¯ ca 3c¯4a¯5b

1. Xét P (x) ≡C . Khi đó ta có: C 2 +C 2 =C 2 ⇔C = 0, do đó P (x) ≡ 0.

2. Xét deg P ≥ 1P (x) = an xn +an−1X n−1 + ...+a1x +a0(an 6= 0)Khi đó xét hệ số của x2n trong khai triển (2.4) ta được:

an .(−1)n .an .3n + (an .2n)2 = an .an .5n ⇔ (−3)n +4n = 5n (2.5)

Nhận thấy rằng nếu n là số lẻ thì không thỏa mãn (2.5). Dễ thấy n = 2 là nghiệmcủa (2.5).Xét n ≥ 4. Khi đó:

3n +4n = 5n ⇔(

3

4

)n

+1 =(

5

4

)n

Mặt khác:(

3

4

)n

+1 < 1+1 = 2 và(

5

4

)n

≥ 2(n ≥ 4)

Do đó phương trình (2.5) có nghiệm duy nhất n = 2Suy ra:P (x) = a2x2 +a1x (do P (0) = 0).Từ đề bài, so sánh hệ số đứng trước x3 ta có :

3a1a2 −9a1a2 +16a1a2 = 5a1a2 +25a1a2

Suy ra:a1 = 0 (vì a2 6= 0)Thử lại ta thấy P (x) ≡ 0;P (x) = a2x2 (a2 6= 0) thỏa yêu cầu đề bài.

Nhận xét 17 Một số bài tập tương tự cho dạng này:

1. (KHTN, 2010) Tìm tất cả các đa thức P (x) hệ số thực thỏa mãn

P (x − y)+P (y − z)+P (z −x) = 3[P (x)+P (y)+P (z)

]với x + y + z = 0

2. (IMO, 2004) Tìm tất cả các đa thức hệ số thực P (x) thỏa mãn

P(x − y

)+P(y − z

)+P (z −x) = 2P(x + y + z

),∀x, y, z ∈R

sao cho x y + y z + zx = 0

3. (Nam Tư, 1982) Tìm tất cả đa thức hệ số nguyên P (x) sao cho

16P (x2) = [P (2x)

]2 ,∀x ∈R

4. (Ukraine, 2009) Tìm tất cả các đa thức hệ số thực P (x) thỏa mãn với x, y, z, t đôimột khác nhau:x2 + y2 + z2 = 2t 2,gcd(x, y, z, t ) = 1 và

2P 2(t )+2P (x y + y z + zx) = P 2(x + y + z)

5. (Costa Rica, 2008) Tìm đa thức P (x) hệ số thực thỏa mãn

P (p

3(a−b))+P (p

3(b−c))+P (p

3(c−a)) = P (2a−b−c)+P (−a+2b−c)+P (−a−b+2c)

với mọi a,b,c ∈R.

http://thcmn.wordpress.com 75

Page 76: t 2017 - WordPress.com...Bài 5. (B›c Ninh) Cho a, b c¨0 thäa mãn đi•u ki»n ¯ ˘9. Tìm giá trà lîn nh t cıabi”uthøc T ˘ ab 3a¯4b¯5c ¯ bc 3b¯4c¯5a ¯ ca 3c¯4a¯5b

Cho m là số nguyên dương thỏa mãn m ≡ 1( mod 2017). Chứng minh rằng đa thứcP (x) = x2017 −mx +2016 là đa thức bất khả quy trên Z[x].

(Nghệ An)

Bài 15

Lời giải

Ta sẽ đi chứng minh bài toán tổng quát hơn như sau: Cho a là một số nguyên dương vàp > 2 là số nguyên tố thỏa mãn (a, p) = 1. Khi đó đa thức P (x) = xp −mx+a bất khả quytrên Z[x] với m ≡ 1( mod p). Thật vậy, giả sử rằng P (x) khả quy, ta có: P (x) = Q(x).R(x)(degQ,deg R ≥ 1)Do P (x) là monic nên Q(x),R(x) cũng monic.Xét Q(x) = xn +an−1xn−1 + ...+a1x +a0 với an−1, an−2, ..., a0 ∈ZGọi αi là nghiệm (thực hoặc phức) của Q(x) = 0. Đặt

Sk =n∑

i=1αk

i

Do αi cũng là nghiệm của P (x) = 0 nên ta có:

Sp −mS1 +na = 0

Do S1 =n∑

i=1αi nên theo định lí Viete trong Q(X ) ta có S1 ∈Z. Do đó Sp ∈Z.

Ta có: Sp1 ≡ S1( mod p).

Mặt khác: Sp1 =

(n∑

i=1αi

)p

≡ Sp ( mod p)

Do đó: Sp −S1...p.

Mà Sp −mS1 ≡ Sp −S1 ≡ 0( mod p) nên na...p

Mà 1 < n < p và (a, p) = 1 nên vô lí. Do đó P (x) bất khả quy trên Z[x].

Nhận xét 18 Đa thức bất khả quy là một toán hay gặp trong các đề thi học sinh giỏi.Có nhiều phương pháp để giải quyết bài toán này như phản chứng, dùng số phức haydùng tiêu chuẩn Eisenstein. Sau đây là một số bài tập về sự khả quy cũng như bất khảquy của đa thức.

1. Cho a1, a2, ..., an ∈Z. Chứng minh rằng P (x) = (x −a1)(x −a2)...(x −an)−1 bất khảquy trên Z[x].

2. Chứng minh rằng xp−1 +xp−2 + ...+x +1 là bất khả quy trên Z[x].

3. (China TST, 2008) Chứng minh với mọi n ≥ 2, tồn tại đa thức P (x) = xn+aa−1xn−1+·· ·+a0 thỏa mãn đồng thời 3 điều kiện:

• a1, a2, · · · , an 6= 0

• P (x) bất khả quy trên Z[x]

76 http://www.facebook.com/thcmn

Page 77: t 2017 - WordPress.com...Bài 5. (B›c Ninh) Cho a, b c¨0 thäa mãn đi•u ki»n ¯ ˘9. Tìm giá trà lîn nh t cıabi”uthøc T ˘ ab 3a¯4b¯5c ¯ bc 3b¯4c¯5a ¯ ca 3c¯4a¯5b

• Với mọi số nguyên x thì |P (x)| không phải là số nguyên tố.

4. (Romania TST, 2006): Cho p là số nguyên tố, p ≥ 5. Xét đa thức P (x) = xp +p(xk +x l )+ 1 với 1 ≤ l ≤ k ≤ p − 1. Hỏi có bao nhiêu cặp (k, l ) để P (x) bất khả quy trênZ[x].

Tìm tất cả các đa thức P (x) hệ số thực thỏa mãn

(x2 −6x +8)P (x)− (x2 +2x)P (x −2) = 6x2 −12x.

(Phú Thọ)

Bài 16

Lời giải

Viết phương trình đã cho dưới dạng:(x2 −6x +8

)[P (x)+x

]= (x2 +2x

)[P (x −2)+x −2

],∀x ∈R.

Đặt Q (x) = P (x)+x, ta có Q ∈R [x] thỏa mãn(x2 −6x +8

)Q (x) =

(x2 +2x

)Q (x −2) ,∀x ∈R

Hay(x −2)(x −4)Q (x) = x (x +2)Q (x −2) ,∀x ∈R (2.6)

Cho x = 0,−2,4, từ (2.6) suy ra Q (0) = 0;Q (−2) = 0,Q (2) = 0.Suy ra Q (x) = x (x −2)(x +2)R (x) . Ta có R ∈R [x] thỏa mãn

(x −4)(x −2)2 (x) (x +2)R (x) = (x)2 (x +2)(x −4)(x −2)R (x −2) ,∀x ∈R⇒ (x −2)R (x) = xR (x −2) ,∀x 6= 0;±2;4 ⇒ (x −2)R (x) = xR (x −2) ,∀x ∈R. (2.7)

Cho x = 0,(2.7) ⇒ R (0) = 0; suy ra R (x) = xS (x) .Khi đó S ∈R [x] thỏa mãn

(x −2)(x)S (x) = x (x −2)S (x −2) ,∀x ∈R⇒ S (x) = S (x −2) ,∀x ∈R.

Từ đây suy ra S (x) = c, với c là hằng số.Do đó P (x) = cx2

(x2 −4

)−x = c

(x4 −4x2

)−x. Thử lại thoả mãn.

Nhận xét 19 Đây là một dạng toán tìm đa thức quen thuộc, tuy nhiên để xử lý bàitoán trên phải khéo léo. Sau đây là một số bài toán tương tự:

1. (Long An, 2012) Tìm tất cả các đa thức hệ số thực P (x) thỏa mãn

P (x)(x2 +2x +2

)x (x +2) =

(x2 +1

)(x2 −1

)P (x +1)∀x ∈R

2. (Moldova, 2004) Tìm tất cả đa thức hệ số thực P (x) thỏa mãn(x3 +3x2 +3x +2

).P (x −1) =

(x3 −3x2 +3x −2

).P (x) ,∀x ∈R

3. (Greece, 2014) Tìm tất cả các đa thức P (x) hệ số thực thỏa mãn

(x2 −6x +8)P (x) = (x2 +2x)P (x −2)

http://thcmn.wordpress.com 77

Page 78: t 2017 - WordPress.com...Bài 5. (B›c Ninh) Cho a, b c¨0 thäa mãn đi•u ki»n ¯ ˘9. Tìm giá trà lîn nh t cıabi”uthøc T ˘ ab 3a¯4b¯5c ¯ bc 3b¯4c¯5a ¯ ca 3c¯4a¯5b

Cho đa thứcf (x) = x2017 +ax2 +bx + c

có ba nghiệm nguyên x1, x2, x3, trong đó a,b,c ∈ Z . Chứng minh rằng biểu thứcsau là bội của 2017

(a2017 +b2017 + c2017 +1)(x1 −x2)(x2 −x3)(x3 −x1).

(Quảng Bình)

Bài 17

Lời giải

Xét f (x) = x2017 +ax2 +bx + cNếu x1, x2, x3 có cùng số dư khi chia cho 2017 thì ta có điều phải chứng minh.Xét x1, x2, x3 có số dư đôi một khác nhau đôi một khi chia cho 2017.Theo định lý Fermat, ta có:

f (x) ≡ ax2 +x(b +1)+ c

Mặt khác:

f (x1) = f (x2) = f (x3) = 0...2017

Ta có:f (x1)− f (x2) ≡ (x1 −x2)(ax1 +ax2 +b +1)

Suy ra:

ax1 +ax2 +b +1...2017

Tương tự:

ax2 +ax3 +b +1...2017

Do đó: (ax1 +ax2 +b +1)− (ax2 +ax3 +b +1) = a(x1 −x3)... 2017 ⇒ a

... 2017

Do f (x1)− f (x2) ≡ (x1 −x2)(ax1 +ax2 +b +1) nên b +1... 2017.

Ngoài ra: f (x1) ≡ ax21 +x1(b +1)+ c ≡ 0( mod 2017) nên c

... 2017Suy ra: a2017 +b2017 + c2017 +1 ≡ a +b + c +1 ≡ 0( mod 2017)Do đó ta có điều phải chứng minh.

Nhận xét 20 Đối với những bài toán như trên ta cần phải chú ý đến các tính chất củađa thức hệ số nguyên trong số học và một số hệ quả của định lý Bezout chẳng hạn như

P (a)−P (b)... a −b, a,b ∈ Z. Sau đây là một số bài toán liên quan đến đa thức trong số

học:

1. (Hong Kong 2001) Cho số tự nhiên k ≥ 4 và đa thức P (x) ∈Z[x] thỏa mãn điều kiện0 ≤ P (c) ≤ k với mọi c = 0,1, ...,k +1. Chứng minh rằng P (0) = P (1) = ... = P (k +1).

2. (China TST, 2009) Tìm tất cả các đa thức f (x) hệ số nguyên sao cho f (a)+ f (b)+f (c)

...a +b + c, với mọi số nguyên a,b,c.

78 http://www.facebook.com/thcmn

Page 79: t 2017 - WordPress.com...Bài 5. (B›c Ninh) Cho a, b c¨0 thäa mãn đi•u ki»n ¯ ˘9. Tìm giá trà lîn nh t cıabi”uthøc T ˘ ab 3a¯4b¯5c ¯ bc 3b¯4c¯5a ¯ ca 3c¯4a¯5b

Tìm tất cả các đa thức P (x) với hệ số thực thỏa mãn điều kiện

P (x2)+P (x).P (x +1) = 0,∀x ∈R. (2.8)

(Quảng Nam)

Bài 18

Lời giải

1. Nếu P (x) ≡C thì P (x) ≡ 0 hoặc P (x) ≡−1.

2. Nếu deg P = n ≥ 1 thì P (x) = an xn +an−1xn−1 + ...+a1x +a0(an 6= 0).Xét hệ số của x2n trong khai triển (2.8) ta có:

an +a2n = 0 ⇔ an = 0∨an =−1

Do an 6= 0 nên an =−1.Gọi α là nghiệm của P (x) = 0. Thay x =α vào (*) ta được:

P (α2) =−P (α)P (α+1) = 0

Do đó các giá trị α2k ,k = 1,2, ... cũng là nghiệm của P (x) = 0. Khi đó để các giátrị α2k là hữu hạn thì |α| = 0 hoặc |α| = 1. Mặt khác với x = α−1 thì P ((α−1)2) =−P (α− 1)P (α) = 0. Do đó (α− 1)2 là nghiệm của P (x) = 0. Tương tự ta cũng có(α−1)2k ;k = 1,2, ... cũng là nghiệm của P (x) = 0.Một cách tương tự ta sẽ có: |(α−1)2| = 0 hoặc |(α−1)2| = 1.

(a) Nếu |α| = 0 thì P (0) = 0 suy ra P ((0−1)2) = P (1) = 0.

(b) Nếu |α| = 1 và |(α−1)2| = 1.Xét α= cosϕ+ i si nϕ. Khi đó:

1−α= 1− cosϕ− i si nϕ

Suy ra:(1−α)2 = (1− cosϕ)2 −2i (1− cosϕ)si nϕ− si n2ϕ

= 2cosϕ(cosϕ−1)−2i si nϕ(1− cosϕ)

= 2(cosϕ−1)(cosϕ+ i si nϕ)

Do đó

1 = |(α−1)2| =√

4(cosϕ−1)2(cos2ϕ+ si n2ϕ) = 2(1− cosϕ)

⇔ cosϕ= 1

2⇔ϕ= π

3∨ϕ= 5π

3

Với ϕ = π

3thì α = 1

2+p

3

2i . Do α là nghiệm nên α2 cũng là nghiệm, suy ra:

(α−1)2 cũng là nghiệm.Mặt khác ta tính được: |(α−1)2| 6= 0,1 (Vô lí).

Tương tự với trường hợp ϕ= 5π

3.

http://thcmn.wordpress.com 79

Page 80: t 2017 - WordPress.com...Bài 5. (B›c Ninh) Cho a, b c¨0 thäa mãn đi•u ki»n ¯ ˘9. Tìm giá trà lîn nh t cıabi”uthøc T ˘ ab 3a¯4b¯5c ¯ bc 3b¯4c¯5a ¯ ca 3c¯4a¯5b

Vậy P (x) = 0 chỉ có nghiệm hai 0,1 kể cả nghiệm phức.Do đó theo Bezout ta có:

P (x) = xm(1−x)n ,m,n ∈N

Thay vào (2.8) ta được:

x2m(1−x2)n +xm .(1−x)n .(x +1)m(−x)n = 0

⇔ xm(1−x)n [xm(1+x)n + (x +1)m .(−x)n]= 0,∀x ∈R⇔ m = n

Khi đó:P (x) = xm(1−x)m ,m ∈N,∀x ∈R

.

Nhận xét 21 Cần nhận ra rằng phương trình P (x) = 0 không có nghiệm nào khácngoài 0,1 kể cả nghiệm phức, từ đó sử dụng kiến thức về số phức trong đa thức đểgiải quyết các bài toán. Sau đây là một số bài toán tương tự:

1. Tìm các đa thức hệ số thực P (x) sao cho

P (x)P (x +1) = P (x2 +x +1),∀x ∈R

2. (IMO 1983) Chứng minh rằng đa thức P (x) = xn +5xn−1 +3 là bất khả quy trênZ[x] với mọi n > 1.

3. (Bulgaria, 1979) Tìm tất cả các đa thức hệ số thực P (x) thỏa mãn

P (x).P (2x2) = P (2x3 +x),∀x ∈R

Cho Pi (x) = x2+bi x+ci (i = 1,2, ...,n) là n đa thức đôi một phân biệt với hệ số thựcsao cho với mọi 1 ≤ i < j ≤ n thì đa thức Qi , j (x) = Pi (x)+P j (x) có nghiệm thực duynhất. Tìm giá trị lớn nhất có thể của n.

(Vĩnh Phúc)

Bài 19

Lời giải

• Với n=3 ta chọn P1 (x) = x2 − 4;P2 (x) = x2 − 4x + 6;P3 (x) = x2 − 8x + 12 thì P1 (x)+P2 (x) = 2(x −1)2; P1 (x)+P3 (x) = 2(x −2)2; P2 (x)+P3 (x) = 2(x −3)2 thỏa mãn.

• Giả sử tồn tại 4 đa thức P1,P2,P3,P4 thỏa mãn điều kiện bài toán. Khi đóP1 +P2 = 2

(x − t12

)2; P3 +P4 = 2(x − t34

)2; P1 +P3 = 2(x − t13

)2; P2 +P4 = 2(x − t24

)2,trong đó ti j là nghiệm duy nhất của Pi +P j .Đặt Q = P1 +P2 +P3 +P4 thì Q có hai biểu diễn

Q = 2(x − t12

)2 +2(x − t34

)2

80 http://www.facebook.com/thcmn

Page 81: t 2017 - WordPress.com...Bài 5. (B›c Ninh) Cho a, b c¨0 thäa mãn đi•u ki»n ¯ ˘9. Tìm giá trà lîn nh t cıabi”uthøc T ˘ ab 3a¯4b¯5c ¯ bc 3b¯4c¯5a ¯ ca 3c¯4a¯5b

vàQ = 2

(x − t13

)2 +2(x − t24

)2.

So sánh hệ số ta được

t12 + t34 = t13 + t24 và t 212 + t 2

34 = t 213 + t 2

24.

Suy rat12 + t34 = t13 + t24 = a; t12.t34 = t13.t24 = b.

Do đó: t12; t34 và t13, t24 là nghiệm của phương trình x2 −ax +b = 0Nếu t12 = t13 thì P2 = P3, nếu t12 = t24 thì P1 = P4 đều dẫn đến mâu thuẫn với giảthiết.Vậy giá trị lớn nhất có thể của n là 3.

3 Giải tích

Cho dãy số (xn) thỏa {x1 = 3, x2 = 7

xn+2 = x2n+1 −x2

n +xn

Đặt dãy

yn =n∑

k=1

1

xk

Chứng minh (yn) có giới hạn hữu hạn và tìm giới hạn đó.

(THPT chuyên KHTN - ĐH KHTN, ĐHQG HN)

Bài 1

Lời giải

Ta sẽ cố gắng rút gọn biểu thức để có thể quy về dãy tuyến tính cấp 2 rồi sau đó sẽ rútgọn yn .Ta có

xn+2 −x2n+1 +xn+1 = xn+1 −x2

n +xn = ... = x2 −x21 +x1 = 1

Do đó ta có đượcxn+1 = x2

n −xn +1

Khi đó:1

xk= 1

xk −1− 1

xk+1 −1

Do đó sai phân và rút gọn, ta tính được

yn = 1

x1 −1− 1

xn −1

http://thcmn.wordpress.com 81

Page 82: t 2017 - WordPress.com...Bài 5. (B›c Ninh) Cho a, b c¨0 thäa mãn đi•u ki»n ¯ ˘9. Tìm giá trà lîn nh t cıabi”uthøc T ˘ ab 3a¯4b¯5c ¯ bc 3b¯4c¯5a ¯ ca 3c¯4a¯5b

Tiếp theo ta sẽ tìm lim xn. Ta có xn+1 −xn = (xn −1)2 ≥ 0. Do đó xn tăng.Giả sử xn bị chặn trên thì khi đó xn có giới hạn hữu hạn. Đặt L = lim xn thì giải ra đượcL = 1 vô lí. Do đó lim xn =+∞. Khi đó

lim yn = 1

2.

Nhận xét 22 Khi gặp những bài dạng tính tổng sai phân mà ta đã có phương trìnhsai phần cấp 2 thì thường ta có 2 cách làm thông dụng như sau:

1. Đồng nhất hệ số: Ta sẽ tìm α,β thỏa mãn

1

xk= α

xk −β− α

xk+1 −β.

2. Sử dụng điểm cố định của hàm số: Khi đó ta sẽ giải phương trình sai phân códạng f (x) = x để tìm ra điểm cố định đó, sau đó đặt vào lại biểu thức sai phân làđượcVí dụ bài trên là ta giải phương trình x = x2 − x +1 ta tìm được nghiệm x = 1 nênta có cách phân tích như trên.

Ta có một số bài tập ứng dụng sau đây:

1. Cho a là một số thực dương và dãy (xn) được xác định bởi:

x1 = 1

2a, xn+1 = xn

1+axn +√

a2x2n +4axn

,∀n ∈N∗

Đặt Sn =n∑

k=1xk . Tính lim

n→+∞Sn.

2. Cho m là số thực lớn hơn 1 và a là số thực lớn hơn 4. Xét dãy số (xn) được xácđịnh bởi:

x1 = a, xn+1 =xm+1

n +3xn +16

xmn −xn +11

,∀n ∈N∗

Đặt yn =n∑

i=1

1

xmi +7

,∀n ∈N∗. Tìm limn→+∞ yn.

3. Cho a ∈R. Xét dãy số (un) được xác định bởi: u1 =−a4 −2a2 −2 và:

un+1 =un +5−

√u2

n −14un +21

5,∀n ∈N∗

Đặt yn =n∑

i=1

1

u2i+1 −4ui+1 −5

,∀n ∈N∗. Tính các giới hạn lim yn và limun

n.

4. Cho dãy số thực (xn) được xác định bởi:

x0 = 2

xn+1 = 2xn +1

xn +2,n ∈N .

Với mỗi số nguyên dương n, đặt An =n∑

k=1xk . Tính bAnc.

82 http://www.facebook.com/thcmn

Page 83: t 2017 - WordPress.com...Bài 5. (B›c Ninh) Cho a, b c¨0 thäa mãn đi•u ki»n ¯ ˘9. Tìm giá trà lîn nh t cıabi”uthøc T ˘ ab 3a¯4b¯5c ¯ bc 3b¯4c¯5a ¯ ca 3c¯4a¯5b

Tìm a để dãy số (un) hội tụ, biết u1 = a và:

un+1 =

2un −1 khi un > 0

−1 khi −1 ≤ un ≤ 0

u2n +4un +2 khi un <−1

,n ∈N∗

(Trường Phổ thông Năng khiếu, ĐHQG Tp. HCM)

Bài 2

Lời giải

Đây là dạng toán cần phải xét nhiều trường hợp do đó điều cần nhất là việc chiakhoảng sao cho hợp lí.Ta có nhận xét sau: Nếu tồn tại xi ∈ [−1;0] thì dãy sẽ có giới hạn hữu hạn và limun =−1.Ta sẽ chia khoảng vào những điểm nhạy cảm của bài toán để từ đó đi ra được lời giải.

• a > 1 ⇒ u2 = 2u1 −1 > 1. Bằng quy nạp suy ra un > 1,∀n. Từ đó suy ra công thứctổng quát của dãy là un = 2n−1(a −1)+1, tức là dãy (un) không có giới hạn hữuhạn.

• a = 1 ⇒ u2 = 1 ⇒ un = 1. Vậy dãy có giới hạn L = 1.

• 0 < a < 1: Ta sẽ chứng minh trong dãy un luôn có 1 số hạng âm. Thật vậy giả sử

un > 0,∀n. Khi đó u2 ∈ [1

2;1]. Cứ chia nửa khoảng nửa khoảng rồi kẹp lim thì suy

ra được un = 1 với n đủ lớn (vô lí). Do đó tồn tại k : uk < 0 và −1 ≤ uk ≤ 0. Như vậytheo nhận xét thì ta có limun = 1.

• −1 ≤ a ≤ 0 thì hiển nhiên theo nhận xét.

• a <−1: Đây cũng chính là trường hợp chính của bài toán vì chỗ xác định dãy làkhá khó. Với a <−1, u2 = a2+4a+2. Nếu u2 > 1 ⇔ a <−2−p

3 thì theo trường hợp1, dãy không có giới hạn.Ta đi chứng minh −2−p

3 ≤ a < −1 thì dãy có giới hạn. Nhưng tới đây thì vẫnchưa chia khoảng được nên ta tiếp tục xét thêm các điểm nhạy cảm là −2;−1.

– Nếu −2−p3 ≤ a ≤ −2 thì −1 ≤ u2 ≤ 1. Theo các trường hợp trên thì dãy hội

tụ.

– Nếu −2 < a <−1, xét hàm số f (x) = x2+4x+2 trên (−2;−1) thì f ′(x) = 2x+4 > 0,mà u2 < u1 nên dãy giảm. Mặt khác un bị chặn dưới bởi −2 nên tồn tại giớihạn của un .

Vậy dãy có giới hạn khi −2−p3 ≤ a ≤ 1.

Nhận xét 23 Dạng toán trên thuộc lớp bài toán biện luận sự hội tụ của dãy số. Trongcác kì thi học sinh giỏi, đặc biệt là kì thi học sinh giỏi quốc gia ta thường gặp dạngtoán sau: Cho dãy số (xn) như sau:

x1 = a và xn+1 = f (xn),∀n ∈N∗ (a là tham số thực).

Tìm a để dãy số (xn) có giới hạn hữu hạn khi n →+∞ và tìm giới hạn trong các trườnghợp đó. Phương pháp giải bài toán trên gồm hai bước.

http://thcmn.wordpress.com 83

Page 84: t 2017 - WordPress.com...Bài 5. (B›c Ninh) Cho a, b c¨0 thäa mãn đi•u ki»n ¯ ˘9. Tìm giá trà lîn nh t cıabi”uthøc T ˘ ab 3a¯4b¯5c ¯ bc 3b¯4c¯5a ¯ ca 3c¯4a¯5b

1. Xét hàm số y = f (x). Tìm nghiệm của phương trình x = f (x), lập bảng biến thiêncủa hàm số y = f (x) và dấu của f (x)−x.

2. Sử dụng bảng vừa lập để xét tính đơn điệu và bị chặn của dãy số này vào điềukiện của a.

Ta có một số bài tập áp dụng:

1. Cho dãy số (xn) như sau: x1 = a ∈ R và xn+1 = 3√

7xn −6,∀n ∈ N∗. Tìm a để dãy số(xn) có giới hạn hữu hạn và tìm giới hạn trong các trường hợp đó.

2. Với mỗi cặp số thực (a;b). Xét dãy số (xn) như sau:x0 = a

xn+1 = xn +b sin xn ,∀n ∈N

(a) Cho b = 1. Chứng minh rằng với mọi số thực a, dãy (xn) có giới hạn hữu hạnkhi n →+∞. Hãy tính giới hạn đó theo a.

(b) Chứng minh rằng với mỗi số thực b > 2 cho trước, tồn tại số thực a sao chodãy số (xn) tương ứng không có giới hạn hữu hạn.

3. Cho dãy số (xn) như sau:x1 = a

xn+1 = 3x3n −7x2

n +5xn ,∀n ∈N∗

Tìm a để dãy số (xn) có giới hạn hữu hạn khi n →+∞ và tìm giới hạn trong cáctrường hợp đó.

Tìm tất cả các hàm số f :R→R thỏa mãn:

1. f (1) > 0.

2. f (x y −1)+2 f (x) f (y) = 3x y −1∀x, y ∈R.

(THPT chuyên ĐH Vinh)

Bài 3

Lời giải

Kí hiệu P (u, v) là phép thế x bởi u,y bởi v vào 2..Từ P (x,0) ta có: f (−1)+2 f (0) f (x) =−1∀x ∈R.Từ đây ta thấy f (0) = vì nếu ngược lại, ta có f (x) = c∀x ∈ R (c là hằng số), không thỏamãn đề bài.P (1,1) ⇒ f (0)+2 f (1)2 = 2 ⇒ f (1) = 1( f (1) > 0).P (x +1,1)

f (x)+2 f (x +1) = 3x +2∀x ∈R. (3.1)

84 http://www.facebook.com/thcmn

Page 85: t 2017 - WordPress.com...Bài 5. (B›c Ninh) Cho a, b c¨0 thäa mãn đi•u ki»n ¯ ˘9. Tìm giá trà lîn nh t cıabi”uthøc T ˘ ab 3a¯4b¯5c ¯ bc 3b¯4c¯5a ¯ ca 3c¯4a¯5b

Trong đẳng thức trên thay x =−1 ⇒ f (−1) =−1Từ P (−x −1,−1) ta có:

f (x)−2 f (−x −1) = 3x +2∀x ∈R. (3.2)

Từ (3.1) và (3.2) ⇒ f (x) =− f (−x)∀x ∈RP (−x −1,−1)

f (−x −1)+2 f (−x) =−3x −1 ⇒ f (x +1)+2 f (x) = 3x +1∀x ∈R. (3.3)

Từ (3.1)(3.3), ta có f (x) = x∀x ∈R. Thử lại thỏa mãn.

Cho số thực a ≥ 2 và dãy số(un

)xác đinh bởi:

u1 = a

un+1 = un + lnun +1

2un −3,n ∈N∗

Chứng minh rằng dãy un có giới hạn hữu hạn và tìm giới hạn đó.

(Trường THPT chuyên ĐH Vinh)

Bài 4

Lời giải

Ta cóln

x +1

2x −3> 0 ⇔ x +1

2x −3> 1 ⇔ 4−x

2x −3> 0 ⇔ 3

2< x < 4. (3.4)

Do đó ta xét hàm số f (x) = x+lnun +1

2un −3trên

(3

2,+∞

). Có f ′(x) = 1+

−5

(2x −3)2

x +1

2x −3

= 2x2 −x −8

(x +1)(2x −3).

f ′(x) = 0 ⇔ x = 1+p65

4.

Do khi giải phương trình giới hạn ta suy ra được limun = 4 nên ta sẽ xét 2 trường hợpsau:

TH1: a ≥ 4. Lập bảng biến thiên, ta suy ra u2 = f (u1) = f (a) ≥ f (4) = 4.. Tiếp đó, bằngquy nạp suy ra

un ≥ 4∀n ≥ 1. (3.5)

Từ (3.4) và (3.5) suy ra un+1 −un ≤ 0 với mọi n ≥ 1. Do đó (un) là dãy giảm và bịchặn dưới nên có giới hạn hữu hạn.

TH2: 2 ≤ a < 4.Ta có f (2) = 2+ ln3 < 4 và f

(1+p

65

4

)> 1+p

65

4> 2. Lập bảng biến thiên,

ta suy ra u2 = f (u1) = f (a) ∈ [2; 4). Bằng quy nạp suy ra

un ∈ [2; 4)∀n ≥ 1. (3.6)

Từ (3.4) và (3.6) suy ra un+1 −un > 0 với mọi n ≥ 1.Do đó (un) là dãy tăng và bị chặn trên nên có giới hạn hữu hạn.

http://thcmn.wordpress.com 85

Page 86: t 2017 - WordPress.com...Bài 5. (B›c Ninh) Cho a, b c¨0 thäa mãn đi•u ki»n ¯ ˘9. Tìm giá trà lîn nh t cıabi”uthøc T ˘ ab 3a¯4b¯5c ¯ bc 3b¯4c¯5a ¯ ca 3c¯4a¯5b

Vậy với a ≥ 2 thì (un) có giới hạn hữu hạn. Giả sử limun = L, khi đó L = L + lnL+1

2L−3⇔

L = 4. Vậy limun = 4.

1. Chứng minh rằng không tồn tại hàm số f :R→R thỏa mãn:

f (x −2016 f (y)) = y −2017 f (x) ∀x, y ∈R. (3.7)

2. Tìm tất cả các hàm số f :R→R thỏa mãn

f (x + y f (x)) = x f (y)+ f (x) ∀x, y ∈R. (3.8)

(Bà Rịa - Vũng Tàu)

Bài 5

Lời giải

1. Giả sử tồn tại hàm f : R→ R thỏa mãn. Thay x = 0 vào (3.7) ta có f (−2016 f (y)) =y −2017 f (0) ∀y ∈R⇒ f (x) toàn ánh.Do đó ∃a sao cho f (a) = 0.Thay y bởi a vào phương trình ban đầu, ta có f (x) = a − 2017 f (x) ∀x ∈ R hayf (x) = c ∀x ∈R (c là hằng số thực).Thử lại ta thấy không thỏa mãn.Vậy không tồn tại hàm số thỏa mãn f (x−2016 f (y)) = y−2017 f (x) với mọi x, y ∈R.

2. Giả sử tồn tại hàm f :R→R thỏa mãn. Đặt P (x, y) là phép thế của x, y vào (3.8).Xét x 6= 0: P (x,0) ⇒ f (0) = 0.

• Ta có f (x) ≡ 0 là một nghiệm hàm.

• Xét f (x) 6≡ 0. Gỉa sử tồn tại a 6= 0 là số thỏa mãn f (a) = 0.

– P (a, y) ⇒ a f (y) = 0 ⇒ f (y) = 0 ∀y ∈R vô lý. Vậy f (x) = 0 ⇔ x = 0.– P (−1,−1) ⇒ f (−1) =−1.– P (1,−1) ⇒ f (1− f (1)) = f (1)−1.– P (1− f (1),1) ⇒ ( f (1)−1)2 = 0 ⇒ f (1) = 1.– P (1, y) ⇒ f (y +1) = f (y)+1 ∀y ∈R.– P (x, y+1) ⇒ f (x+y f (x)+ f (x)) = x f (y)+ f (x)+x = f (x+y f (x))+x ∀x, y ∈R.

Mặt khác vì x + y f (x) toàn ánh( cho x=1, y chạy) nên thay x + y f (x) bởiy ở (3.8), ta có

f (y + f (x)) = f (y)+x ∀x, y ∈R. (3.9)

Thay y = 0 vào (3.9) ta có: f ( f (x)) = x ∀x ∈R.Thay x bởi f (x) vào (3.9) ta có

f (x + y) = f (x)+ f (y) ∀x, y ∈R. (3.10)

P ( f (x), y) kết hợp (3.10) ta có

f (x y) = f (x). f (y) ∀x, y ∈R. (3.11)

Từ (3.10) và (3.11) ta có nghiệm hàm là f (x) = x ∀x ∈R và f (x) ≡ 0.

86 http://www.facebook.com/thcmn

Page 87: t 2017 - WordPress.com...Bài 5. (B›c Ninh) Cho a, b c¨0 thäa mãn đi•u ki»n ¯ ˘9. Tìm giá trà lîn nh t cıabi”uthøc T ˘ ab 3a¯4b¯5c ¯ bc 3b¯4c¯5a ¯ ca 3c¯4a¯5b

Cho dãy số xn xác định bởi:x1 = 1

2

xn+1 =nx2

n

1+ (n +1)xn,n ∈N∗

1. Chứng minh xn ≤ 1

n(n +1),∀n ≥ 1.

2. Với mỗi số nguyên dương n, đặt yn =n∑

k=1

kxk

1+ (k +1)xk. Chứng minh dãy số có

giới hạn hữu hạn và tính giới hạn đó.

(Bà Rịa - Vũng Tàu)

Bài 6

Lời giải

1. Đầu tiên ta dễ chứng minh 0 < xn < 1,∀n. Ta sẽ quy nạp để chứng minh xn ≤1

n(n +1).

• Với u1 = 1

2≤ 1

1.2.

• Giả sử đúng tới n, ta chứng minh đúng với n +1 tức là chứng minh

xn+1 ≤ 1

(n +1)(n +2)

⇔ n(n +1)(n +2)x2n − (n +1)xn ≤ 1

⇔ (n +1)xn(n(n +2)xn −1) ≤ 1

Áp dụng giả thiết quy nạp ta cần chứng minh xn ≤ 1, một điều hiển nhiênđúng. Do đó ta có điều phải chứng minh.

2. Để ý rằng so với công thức truy hồi thì bậc của xk giảm đi 1 đơn vị, do đó ngaylập tức thì ta sẽ lập hiệu có dạng f (x +1)xk+1 − f (x).xk

Ta sẽ chọn f (x) = x. Khi đó ta sẽ có

(k +1)xk+1 −kxk = −kxk

1+ (k +1)xk

Đổi dấu lại thì

kxk − (k +1)xk+1 =kxk

1+ (k +1)xk

Cho k chạy từ 1 → n −1 thì

x1 −nxn =∑ kxk

1+ (k +1)xk

http://thcmn.wordpress.com 87

Page 88: t 2017 - WordPress.com...Bài 5. (B›c Ninh) Cho a, b c¨0 thäa mãn đi•u ki»n ¯ ˘9. Tìm giá trà lîn nh t cıabi”uthøc T ˘ ab 3a¯4b¯5c ¯ bc 3b¯4c¯5a ¯ ca 3c¯4a¯5b

Mà từ câu a thì ta có0 < nxn ≤ 1

n +1⇐ limnxn = 0

Do đó lim yn = x1 = 1

2.

Tìm tất cả các hàm số f :R→R thỏa mãn

1

3f (x y)+ 1

3f (xz)− f (x) f (y z) ≥ 1

9∀x, y, z ∈R

(Bình Dương)

Bài 7

Lời giải

Kí hiệu P (u, v, w) là phép thế x bởi u, y bới v , z bởi w vào đề bài.

• P (1,1,1) ⇒(

f (1)− 1

3

)2

≤ 0 ⇒ f (1) = 1

3.

• P (0,0, z) ⇒(

f (0)− 1

3

)2

≤ 0 ⇒ f (0) = 1

3.

• P (x,1,1) ⇒ f (x) ≥ 1

3∀x ∈R.

• P (0,1, x) ⇒ f (x) ≤ 1

3∀x ∈R.

Vậy ta có f (x) = 1

3∀x ∈R, Thử lại thấy đúng.

1. Tìm limun với un = 1

2· 3

4· · · 2n +1

2n +2n ∈N.

2. Cho dãy số (un) xác định bởi:u1 = 1

un+1 =√

1+u2n −1

un,n ∈N∗

Tìm công thức tổng quát của (un).

(Bình Thuận)

Bài 8

Lời giải

88 http://www.facebook.com/thcmn

Page 89: t 2017 - WordPress.com...Bài 5. (B›c Ninh) Cho a, b c¨0 thäa mãn đi•u ki»n ¯ ˘9. Tìm giá trà lîn nh t cıabi”uthøc T ˘ ab 3a¯4b¯5c ¯ bc 3b¯4c¯5a ¯ ca 3c¯4a¯5b

1. Chúng tôi xin nêu hai cách giải cho câu 1.

Cách 1: Nếu 0 < a < b thìa

b< a +1

b +1. Do đó

un < 2.4.6...(2n +2)

3.5....(2n +3)=> 0 < u2

n < 1

2n +3.

Lấy lim hai vế ta được limun = 0.

Cách 2: Ta sẽ quy nạp để chứng minh un ≤ 13p

n,∀n ≥ 3.

– Với n = 3 thì u3 = 35

128< 1

3p

3.

– Giả sử đúng tới n, ta chứng minh đúng với n +1. Thật vậy,

un+1 = un .2n +3

2n +4< 1

3p

n.2n +3

2n +4< 1

3p

n +1.

Nhân chéo lên, điều trên tương đương với

(2n +4)3.n > (2n +3)3.(n +1) ⇔ 4n3 +6n2 −17n −27 > 0,

một điều đúng với mọi n ≥ 3. Do đó ta có 0 < un < 13p

n. Lấy lim2 vế ta

được limun = 0.

2. Đây là 1 câu khá quen thuộc và nhìn hình thức thì ta sẽ nghĩ ngay tới đặt lượng

giác. Do đó quy nạp dễ chứng minh un = tanπ

2n+1.

Cho dãy Fibonacci xác định như sau:{u1 = u2 = 1

un = un−1 +un−2

Chứng minh rằng với mọi số nguyên tố p ≥ 7 thì có đúng 1 trong 2 số up−1,up+1

là bội của p.

(Đà Nẵng)

Bài 9

Lời giải

Trong bài này chúng ta sẽ dùng tới các bổ đề quen thuộc sau:

Bổ đề 5 Với {un} là dãy Fibonacci thì:

http://thcmn.wordpress.com 89

Page 90: t 2017 - WordPress.com...Bài 5. (B›c Ninh) Cho a, b c¨0 thäa mãn đi•u ki»n ¯ ˘9. Tìm giá trà lîn nh t cıabi”uthøc T ˘ ab 3a¯4b¯5c ¯ bc 3b¯4c¯5a ¯ ca 3c¯4a¯5b

1. Công thức tổng quát của un là

un = 1p5

(1+p

5

2

)n

− 1−p5

2)n

.

2. u2n −un−1un+1 = (−1)n−3.

Bổ đề 6 C kp

...p,∀k = 2,3, ..., p −1.

Quay lại bài toán. Dễ thấy nếu tồn tại uk−1,uk+1 đều chia hết cho p thì uk chia hết chop nên suy ra được un chia hết cho p với mọi n vô líDo đó chỉ có duy nhất 1 trong 2 số up−1,up+1 chia hết cho p ≥ 7

Thay n = p ≥ 7 trong bổ đề 5 ⇒ u2p −1 = up−1up+1. Do đó ta cần chứng minh u2

p −1...p.

Ta có

u2p −1 = 1

5

(1+p

5

2

)2p

+ 1−p5

2)2p −3

Do p ≥ 7 nên ta chỉ cần chứng minh(

1+p5

2

)2p

+(

1−p5

2

)2p

−3...p.

Mà khai triển trực tiếp và quy đồng, chú ý là rút gọn, ta thu đươc tử sẽ là

2(C 02p +C 2

2p .5+C 42p .52 + ...+C 2p

2p .5p )−3.22p

Mà theo bổ đề 6 thì ta có C 22p ,C 4

2p , ...C 2p−22p

...p nên ta cần chứng minh

2(C 02p +C 2p

2p .5p )−3.22p ...p,

một điều hiển nhiên theo định lý Fermat. Do đó tử chia hết cho p ≥ 7, còn mẫu sẽ là

22p không chia hết cho p. Do đó ta có u2p −1

...p nên ta có điều phải chứng minh.

Nhận xét 24 Dãy số với các tính chất số học bao giờ cũng là một trong những đề tài"nóng hổi" được quan tâm trong các kì thi học sinh giỏi. Hơn nữa, bài toán trên thuộcmột trong những tính chất đẹp của dãy Fibonacci. Để bạn đọc có thể nắm vững mộtsố tính chất về dãy đặc biệt này, hãy thử sức ở các bài toán sau:

1. Cho (un) là dãy số Fibonacci. Chứng minh rằng nếu n là bội của k thì un là bộicủa uk .

2. Cho (un) là dãy Fibonacci. Với mọi số tự nhiên k, chứng minh phân số sau đâytối giản:

kun+2 +un

kun+3 +un−1,∀n ∈N∗

90 http://www.facebook.com/thcmn

Page 91: t 2017 - WordPress.com...Bài 5. (B›c Ninh) Cho a, b c¨0 thäa mãn đi•u ki»n ¯ ˘9. Tìm giá trà lîn nh t cıabi”uthøc T ˘ ab 3a¯4b¯5c ¯ bc 3b¯4c¯5a ¯ ca 3c¯4a¯5b

Tìm tất cả các hàm f :R→R thỏa mãn

f (x2 −2y f (x))+ f (y2) = f 2(x − y),∀x, y ∈R. (3.12)

(Đồng Nai)

Bài 10

Lời giải

Giả sử tồn tại hàm f :R→R thỏa mãn (3.12).Thay y = 0 vào (3.12) ta có

f (x2)+ f (0) = f 2(x) ∀x ∈R. (3.13)

Thay x = 0 vào đẳng thức trên ta có 2 f (0) = f 2(0) ⇒ f (0) = 0 hoặc f (0) = 2.

TH1: f (0) = 2.Thay x = 0, y bởi −y vào (3.12) ta có f (4y)+ f (y2) = f 2(y) ∀y ∈R⇒ f (4y) = f (0) = 2 ∀y ∈R⇒ f (x) ≡ 2 Thử lại đúng.

TH2: f (0) = 0.(3.12) ⇒ f (x2) = f 2(x) ∀x ∈R. (3.14)

Ta có f (x) ≡ 0 là một nghiệm hàm.Xét f (x) 6≡ 0.Nếu tồn tại a 6= 0 sao cho f (a) = 0:(3.14) ⇒ f (a2) = 0 và f (−a) = 0 do đó không giảm tổng quát giả sử a > 0Từ (3.14) ta cũng có f (x) ≥ 0 ∀x > 0.Thay x = a, y bởi −y vào (3.12) ta có f (a2)+ f (y2) = f 2(a + y) ∀y ∈R⇒ f 2(y) = f 2(a + y) ∀y ∈R⇒ f (y) = f (y +a) ∀y > 0Xét x > 0 thay x bởi x +a vào (3.12) ta có

f ((x +a)2 −2y f (x +a))+ f (y2) = f 2(x +a − y),∀x > 0, y ∈R

⇒ f ((x +a)2 −2y f (x))+ f (y2) = f 2(x − y),∀x > 0, y ∈R⇒ f (x2 −2y f (x)+2ax +a2) = f (x2 −2y f (x)),∀x > 0, y ∈R. (3.15)

Do f (x) 6≡ 0 nên tồn tại x0 > 0 sao cho f (x0) 6= 0.

Thay x = x0, y bởix2

0 − y

2 f (x0)vào (3.15) ta có

f (y +2ax0 +a2) = f (y) ∀y ∈R. (3.16)

Đặt c = 2ax0 +a2. Do a, x0 > 0 nên c > 0.Với mỗi y ∈R tồn tại n ∈N∗ sao cho y +nc > 0.Từ (3.16) ta suy ra f (y) = f (y +nc) ∀y ∈RKết hợp với (3.14) suy ra f (y) ≥ 0 ∀y ∈RDo đó f (−x) = f (x) ∀x ∈RThay x bởi −x vào (3.12) ta suy ra f 2(x − y) = f 2(−x − y) ∀x, y ∈R⇒ f (x − y) = f (x + y) ∀x, y ∈R

http://thcmn.wordpress.com 91

Page 92: t 2017 - WordPress.com...Bài 5. (B›c Ninh) Cho a, b c¨0 thäa mãn đi•u ki»n ¯ ˘9. Tìm giá trà lîn nh t cıabi”uthøc T ˘ ab 3a¯4b¯5c ¯ bc 3b¯4c¯5a ¯ ca 3c¯4a¯5b

Thay x bởix + y

2, y bởi

x − y

2ta có f (x) = f (y) ∀x, y ∈R

Do đó f (x) là hàm hằng. Thử lại suy ra f (x) ≡ 2 hoặc f (x) ≡ 0 vô lý.Vậy f (x) = 0 ⇔ x = 0.Thay x = 2y vào (3.12) ta có

f (4y2 −2y f (2y)) = 0 ∀y ∈R

⇒ 4y2 = 2y f (2y) ∀y ∈R⇒ f (2y) = 2y ∀y 6= 0.

Mà f (0) = 0 suy ra f (x) = x ∀x ∈R. Thử lại đúng.

Vậy bài toán có 3 nghiệm hàm f (x) ≡ 2, f (x) ≡ 0 và f (x) = x ∀x ∈R.

Cho dãy số (un) xác định bởi:u1 ∈

(1;2

)un+1 = 1+un − u2

n

2,∀n ∈N∗

Chứng minh rằng(un

)có giới hạn hữu hạn và tìm giới hạn đó.

(Đồng Nai)

Bài 11

Lời giải

Xét hàm số f (x) = 1+ x − x2

2trên (1;2). Có f ′(x) = 1− x < 0,∀x ∈ (1;2) nên f (x) nghịch

biến. Mặt khác 1 ≤ un ≤ 3

2nên dãy chẵn tăng bị chặn trên, dãy lẻ giảm bị chặn dưới.

Đặt giới hạn từng dãy là A,B thì ta có hệ phương trìnhA = 1+B − B 2

2

B = 1+ A− A2

2

Trừ 2 vế ta được

2(A−B) = A2 −B 2

2

Suy ra A = B hoặc 2 = A+B

2. Nhưng vì A,B ≤ 3

2nên A = B

Do đó L =p2

92 http://www.facebook.com/thcmn

Page 93: t 2017 - WordPress.com...Bài 5. (B›c Ninh) Cho a, b c¨0 thäa mãn đi•u ki»n ¯ ˘9. Tìm giá trà lîn nh t cıabi”uthøc T ˘ ab 3a¯4b¯5c ¯ bc 3b¯4c¯5a ¯ ca 3c¯4a¯5b

Cho hai dãy số được xác định bởi:x1 = y1 =

p3

xn+1 = xn +√

1+x2n

yn+1 = yn

1+√

1+ y2n

,∀n ∈N∗

1. Chứng minh rằng xn yn ∈ (2;3) ∀n ≥ 2.

2. Tính limn→+∞ yn.

(Hà Nam)

Bài 12

Lời giải

Ta sẽ chứng minh bằng quy nạp rằng

xn = cotπ

3.2n, yn = tan

π

3.2n.

• Dễ thấy điều trên đúng với n = 1.

• Giả sử điều trên đúng tới n.

• Bây giờ ta sẽ chứng minh nó đúng với n +1. Thật vậy,

xn +√

1+x2n =

1+cosπ

3.2n

sinπ

3.2n

= cotπ

3.2n+1= xn+1.

Tương tự với yn. Khi đó ta có

xn yn = 2

1− tan2 π

3.2n

∈ (2,3).

Dễ dàng tính được lim yn = 0.

Cho dãy số(un

)có u1 = 1,un = n

n −1un−1 +n với n ∈N và n ≥ 2.

1. Xác định công thức của(un

).

2. Chứng minh u1 +u2 + ...+u2016 < 20163.

(Hà Nội)

Bài 13

http://thcmn.wordpress.com 93

Page 94: t 2017 - WordPress.com...Bài 5. (B›c Ninh) Cho a, b c¨0 thäa mãn đi•u ki»n ¯ ˘9. Tìm giá trà lîn nh t cıabi”uthøc T ˘ ab 3a¯4b¯5c ¯ bc 3b¯4c¯5a ¯ ca 3c¯4a¯5b

Lời giải

1. Tính toán vài giá trị đầu: u2 = 4;u3 = 9 làm ta dự đoán được un = n2, và ta sẽ tiếnhành quy nạp với mong muốn khẳng định đó là đúng.

• Với n = 1,u1 = 12 khẳng định đúng.

• Giả sử khẳng định đúng tới n −1, (n > 1). Khi đó un−1 = (n −1)2.

• Giờ ta sẽ chứng minh khẳng định đúng với n. Thật vậy,

un = n

n −1un−1 +n = n

n −1(n −1)2 +n = n2.

Do đó ta có điều phải chứng minh.

Vậy un = n2.

2. Đề bài tương đương với việc chứng minh

12 +22 + ...+20162 < 20163.

Rất dễ thấy số 2016 mang tính tượng trưng, và nó gợi ý cho ta đi chứng minh 1kết quả tổng quát hơn:

12 +22 + ...+n2 < n3. (3.17)

• Với n = 2 thì (3.17) đúng.

• Giả sử (3.17) đúng tới n. Khi đó 12 +22 + ...+n2 < n3.

• Bây giờ ta sẽ chứng minh (∗) đúng tới n +1. Thật vậy,

12 +22 + ...+n2 + (n +1)2 < n3 + (n +1)2

nên điều cần chứng minh sẽ đúng nếu cón3+ (n+1)2 < (n+1)3 ⇔ 3n2+3n >0,một điều hiển nhiên đúng.

Do đó ta có điều phải chứng minh.

Với mỗi số nguyên dương n, xét hàm số fn trên R được xác định bởi fn(x) = x2n +x2n−1 + ...+x2 +x +1.

1. Chứng minh hàm số fn đạt giá trị nhỏ nhất tại một điểm duy nhất.

2. Gọi giá trị nhỏ nhất của hàm số fn là sn. Chứng minh dãy số (sn) có giới hạnhữu hạn.

(Hà Tĩnh)

Bài 14

Lời giải

Ở câu 1, ta dễ dàng suy ra các nhận xét sau:

94 http://www.facebook.com/thcmn

Page 95: t 2017 - WordPress.com...Bài 5. (B›c Ninh) Cho a, b c¨0 thäa mãn đi•u ki»n ¯ ˘9. Tìm giá trà lîn nh t cıabi”uthøc T ˘ ab 3a¯4b¯5c ¯ bc 3b¯4c¯5a ¯ ca 3c¯4a¯5b

• fn(x) ≥ 1∀x ≥ 0.

• fn(x) ≥ 1∀x ≤−1.

• fn(x) < 1∀x ∈ (−1;0)

Do đó min fn(x) ∈ (−1;0). Khi đó ta có

fn(x) = 1−x2n+1

1−x⇒ f ′

n(x) = 2nx2n+1 − (2n +1)x2n +1

(1−x)2.

Xét hàm số gn(x) = 2nx2n+1−(2n+1)x2n +1∀x ∈ (−1;0) có g ′n(x) = 2n(2n+1)x2n−1(x−1) >

0∀x ∈ (−1;0). Dễ chứng minh gn(x) có đúng 1 nghiệm xn trên (−1;0). Lập bảng biếnthiên, thấy được fn(x) đạt GTNN tại 1 điểm duy nhất.Với mọi x ∈ (−1;0), ta có

fn+1(x) = fn(x)+x2n+1(x +1) ≤ fn(x) ⇒ sn+1 = fn+1(xn+1) ≤ fn(xn+1) ≤ fn(xn) = sn .

Do đó dãy sn giảm. Mặt khác

fn = fn(xn) = 1−x2n+1n

1−xn> 1

1−xn> 1

2

nên dãy giảm bị chặn dưới nên tồn tại L = lim sn.

Nhận xét 25 Đây là một trong những bài toán về các dãy số xác định bởi dãy cácphương trình với tham số n nguyên dương fn(x) = 0. Yêu cầu bài toán đặt ra rằng mộtdãy số (xn) được sinh bởi nghiệm, giá trị nhỏ nhất, giá trị lớn nhất,... của phương trìnhfn(x) = 0. Chứng minh rằng dãy số (xn) có giới hạn hữu hạn và tính giới hạn đó.Đối với dạng toán này, ta thường khảo sát hàm số f (x) để nhận xét một số tính chấtliên quan đến dãy (xn) rồi sau đó biện luận miền của các số hạng trong dãy và tínhđơn điệu của dãy. Đôi khi ta cũng cần kết hợp một số định lý có liên quan đến phầngiải tích như định lý Lagrange, định lý Rolle,... để biện luận dãy (xn). Sau đây là mộtsố bài tập ứng dụng:

1. (Trường Đông viện Toán 2016 - Ngày 1) Ký hiệu xn là nghiệm dương duy nhấtcủa phương trình:

xn +xn−1 +·· ·+x = n +2

Chứng minh rằng dãy (xn) hội tụ tới một số thực dương. Tìm giới hạn đó.

2. (Moscow University Entrance Exam 2000) Ký hiệu xn là nghiệm của phươngtrình:

1

x+ 1

x −1+·· ·+ 1

x −n= 0

thuộc khoảng (0;1). Chứng minh dãy số (xn) hội tụ. Hãy tìm giới hạn đó.

3. Cho n là một số nguyên dương lớn hơn 1. Chứng minh rằng phương trình xn =x +1 có một nghiệm dương duy nhất, ký hiệu là xn. Chứng minh rằng lim xn = 1và tính limn(xn −1).

http://thcmn.wordpress.com 95

Page 96: t 2017 - WordPress.com...Bài 5. (B›c Ninh) Cho a, b c¨0 thäa mãn đi•u ki»n ¯ ˘9. Tìm giá trà lîn nh t cıabi”uthøc T ˘ ab 3a¯4b¯5c ¯ bc 3b¯4c¯5a ¯ ca 3c¯4a¯5b

Cho dãy số (un) thỏa: u1 = 1

un+1 =u2

n +n

2un,n ∈N∗

Chứng minh rằng dãy

(unp

n

)có giới hạn hữu hạn.

(Hải Phòng)

Bài 15

Lời giải

Ta chứng minh quy nạpp

n −1 ≤ un ≤pn ∀n.

• Dễ thấy khẳng định đã đúng với n = 1,2.

• Nếu đã cóp

k −1 ≤ uk ≤p

k ∀k ≥ 2 thì:

Với f (x) = x2 +k

2x= x

2+ k

2xta có

f ′ (x) = 1

2− k

2x2= x2 −k

2x2≤ 0 ∀

pk −1 ≤ x ≤

pk

nênuk+1 = f

(uk

)≥ f(p

k)=p

k

uk+1 = f(uk

)≤ f(p

k −1)= 2k −1

2p

k −1.

Ta có2k −1

2p

k −1≤p

k +1 ⇔ (2k −1

)2 ≤ 4(k2 −1

)⇔ 4k −5 ≥ 0

đúng ∀ k ≥ 2.

Từ đó theo nguyên lý kẹp ta có limunp

n= 1.

Xác định tất cả các hàm f :R→R thỏa mãn:

f ([x]y) = f (x)[ f (y)]∀x, y ∈R (3.18)

với [x] là số nguyên lớn nhất không vượt quá x.

(Hòa Bình)

Bài 16

Lời giải

96 http://www.facebook.com/thcmn

Page 97: t 2017 - WordPress.com...Bài 5. (B›c Ninh) Cho a, b c¨0 thäa mãn đi•u ki»n ¯ ˘9. Tìm giá trà lîn nh t cıabi”uthøc T ˘ ab 3a¯4b¯5c ¯ bc 3b¯4c¯5a ¯ ca 3c¯4a¯5b

Giả sử tồn tại hàm f :R→R thỏa mãn (3.18). Thay x = 0 vào (3.18) ta có

f (0) = f (0)[ f (y)] ∀y ∈R.

TH1: f (0) 6= 0 ⇒ [ f (x)] = 1 ∀x ∈R⇒ f ([x]y) = f (x) ∀x, y ∈R.

Xét x 6∈ [0;1) thay y bờiy

[x]vào đẳng thức trên ta có f (y) = f (x) = c ∀y ∈ R (c là

hằng số).Kết hợp với [ f (x)] = 1 ∀x ∈ R ta có 1 ≤ c < 2. Vậy f (x) = c với mọi số thực x, y ,1 ≤ c < 2 là hằng số. Thử lại đúng.

TH2: f (0) = 0.Xét x ∈ (0;1) ta có [x] = 0. Khi đó f (0) = f (x)[ f (x)] ∀x ∈ (0;1) ⇒ f (x)[ f (x)] = 0 ∀x ∈(0;1)Từ đây suy ra [ f (x)] = 0 ∀x ∈ (0;1)

Xét x > 1. Thay y bởi1

[x]vào (∗) ta có f (1) = f (x)[ f (

1

[x])] ∀x > 1 ⇒ f (1) = 0

Thay x = 1 vào (3.18) ta được f (y) = 0 ∀y ∈R. Thử lại đúng.

Vậy bài toán có 2 nghiệm hàm f (x) = c(1 ≤ c < 2) ∀x ∈R và f (x) = 0 ∀x ∈R.

Cho (xn) được xác định như sau:

x0 > 0; xn+1 = xn

1+x2n

,n ∈N

Tìm limp

2nxn.

(Hòa Bình)

Bài 17

Lời giải

Đặt xn = 1

yn. Khi đó

1

yn+1= 1

yn(1+ 1

y2n

)⇒ yn+1 = yn + 1

yn⇒ y2

n+1 = y2n +2+ 1

y2n

.

Ta cần tìm lim2n

y2n

.Để ý rằng lim xn = 0 ⇒ lim yn =+∞. Áp dụng định lý Stolz, ta có

lim2n

yn= 2n +2−2n

2+ 1

y2n

= 1 ⇒ limp

2nxn = 1.

http://thcmn.wordpress.com 97

Page 98: t 2017 - WordPress.com...Bài 5. (B›c Ninh) Cho a, b c¨0 thäa mãn đi•u ki»n ¯ ˘9. Tìm giá trà lîn nh t cıabi”uthøc T ˘ ab 3a¯4b¯5c ¯ bc 3b¯4c¯5a ¯ ca 3c¯4a¯5b

Cho dãy số (xn) xác định bởi:xo = 1

x1 = 41

xn+2 = 3xn +√

8(x2n+1 +x2

n),n ∈N

Chứng minh rằng mọi số hạng của dãy đều là số nguyên.

(Hòa Bình)

Bài 18

Lời giải

Ta có

xn+2 −3xn =√

8(x2n+1 +x2

n)

x2n+2 −6xn+2xn +x2

n = 8x2n+1

Tương tự ta cũng sẽ cóx2

n+1 −6xn+1xn−1 +x2n−1 = 8x2

n

. Trừ vế theo vế ta sẽ được

(xn+2 −3xn)2 = (3xn+1 −xn−1)2

Đến đây ta có nhận xét rằng nếu x0; x1; x2 nguyên thì mọi số hạng của dãy đều nguyên.Tính được x2 = 239, do đó ta có điều phải chứng minh.

Nhận xét 26 Việc chứng minh dãy số có các số hạng đều nguyên quả là một vấn đềthú vị. Ngoài cách xác định xn+2 theo hệ thức truy hồi như bài toán vừa xong, ta còncó thể dùng một số tính chất số học để chứng minh các số hạng của một dãy luôn là sốnguyên. Sau đây là một số bài tập liên quan:

1. Dãy số (un) được xác định bởi:u1 = 1

un+1 =(1+ 3

n

)un +2− 3

n,n ∈N∗

Chứng minh rằng mọi số hạng của dãy đều là số nguyên.

2. Cho a, b, c là ba số nguyên thỏa mãn điều kiện a2 = b +1. Dãy số (xn) xác địnhbởi: xo = 0

xn+1 = axn +√

bx2n + c2,n ∈N

Chứng minh rằng mọi số hạng của dãy đều là số nguyên.

98 http://www.facebook.com/thcmn

Page 99: t 2017 - WordPress.com...Bài 5. (B›c Ninh) Cho a, b c¨0 thäa mãn đi•u ki»n ¯ ˘9. Tìm giá trà lîn nh t cıabi”uthøc T ˘ ab 3a¯4b¯5c ¯ bc 3b¯4c¯5a ¯ ca 3c¯4a¯5b

3. Dãy số (un) có tính chất sau:u1 = 2,u2 = 7

−1

2< un+1 −

u2n

un−1< 1

2,∀n ∈N,n ≥ 2

Chứng minh rằng khi n ≥ 2 thì mọi số hạng un của dãy đều là số nguyên lẻ.

Cho dãy số un xác định bởi công thứcu1 = 1,u2 = 3

2un+2 = un+1 +2

un +2,n ∈N∗

Chứng minh dãy số un có giới hạn hữu hạn.

(Tp. HCM)

Bài 19

Lời giải

Nhận xét rằng giới hạn của (un), nếu có, là limun = 1, do đó ta sẽ thử tiến hành kẹp

1− 1

n≤ un ≤ 1+ 1

n.

• Với n = 1,n = 2 ta có điều hiển nhiên.

• Giả sử điều trên đúng tới n +1. Ta có 1− 1

n +1≤ un+1 ≤ 1+ 1

n +1.

• Bây giờ ta sẽ chứng min điều trên đúng với n +2. Thật vậy,

un+2 = un+1 +2

un +2>

3− 1

n +1

3+ 1

n

.

nên điều trên sẽ đúng nếu ta chứng minh được

3− 1

n +1

3+ 1

n

> 1− 1

n +2

⇔ 3(n +2)− n +2

n +1> 3(n +1)+ n +1

n⇔ 3 ≥ n +1

n+ n +2

n +1,

một điều hiển nhiên. Tương tự ta cũng chứng minh được un ≤ 1+ 1

n. Do đó

limun = 1.

http://thcmn.wordpress.com 99

Page 100: t 2017 - WordPress.com...Bài 5. (B›c Ninh) Cho a, b c¨0 thäa mãn đi•u ki»n ¯ ˘9. Tìm giá trà lîn nh t cıabi”uthøc T ˘ ab 3a¯4b¯5c ¯ bc 3b¯4c¯5a ¯ ca 3c¯4a¯5b

Tìm các hàm số f : R→ R thỏa mãn: f (x y)+ f (x − y)+ f (x + y +1) = x y +2x +1 vớimọi x, y ∈R

(Khánh Hòa)

Bài 20

Lời giải

Đặt g (x)+x = f (x) thì phương trình đã cho trở thành:

g (x y)+ g (x − y)+ g (x + y +1) = 0∀x, y ∈R.

Kí hiệu P(u,v) là phép thế x bởi u, y bởi v trong phương trình trên.

• P (0,0),P (0,1),P (0,2),P (1,1) ⇒ g (0) = 0.

• P (x,0) ⇒ g (x)+ g (x +1) = 0∀x ∈R.

• P (x,−1) ⇒ g (−x)+ g (x +1)+ g (x) = 0∀x ∈R.

Từ hai đẳng thức trên ta có g (−x) = 0∀x ∈ R hay g (x) = 0∀x ∈ R. Suy ra f (x) = x∀x ∈ R.Thử lại thấy đúng.

Cho dãy số (un) xác định bởi:u1 = 1

un+1 = un

n+ n

un,n ∈N∗

Chứng minh rằng[

u2n

]= n khi n ≥ 4.

(Khánh Hòa)

Bài 21

Lời giải

u2n+1 =

u2n

n2+2+ n2

u2n

Ta sẽ chứng minh quy nạp rằng n ≤ u2n < n + 1

2với mọi n ≥ 4.

Tính được u4 = 13

6và u2

4 =169

36. Ta có

u2n+1 = 2+ u2

n

n2+ n2

u2n< 2+ (n +1)2

n2+ n2

n2= 3+ (n +1)2

n2< n +2

nên điều cần chứng minh sẽ đúng nếu ta chỉ ra được

(n +1)2 < (n −1).n2,

một điều luôn đúng với mọi n ≥ 4.Tương tự vế kia.Do đó ta có điều phải chứng minh.

100 http://www.facebook.com/thcmn

Page 101: t 2017 - WordPress.com...Bài 5. (B›c Ninh) Cho a, b c¨0 thäa mãn đi•u ki»n ¯ ˘9. Tìm giá trà lîn nh t cıabi”uthøc T ˘ ab 3a¯4b¯5c ¯ bc 3b¯4c¯5a ¯ ca 3c¯4a¯5b

Tìm tất cả các hàm số f :R→R đơn điệu trên R thỏa mãn:

f (x3 + f (y)) = f 3(x)+ y∀x, y ∈R

(Lạng Sơn)

Bài 22

Lời giải

Kí hiệu P (u, v) là phép thế x bởi u, y bởi v vào đề bài.

• P (0, y) ⇒ f ( f (y)) = y + f 3(0) với mọi số thực y .Ta thấy vế phải của phương trìnhnày có tập giá trị là R nên f toàn ánh.

• Giả sử có f (a) = f (b), làn lượt thực hiện P (x, a),P (x,b) ta có a=b. Vậy f đơn ánh.Suy ra f song ánh. Vậy tồn tại duy nhất giá trị t sao cho f (t ) = 0.

• P (0, y) ⇒ f ( f (y)) = f 3(0)+ y∀y ∈R.Trong đẳng thức trên thay y bởi f (y) và sử dụng lại chính đẳng thức trên ta cóf (y + f 3(0)) = f 3(0)+ f (y)∀y ∈R.

• P (0,0) ⇒ f ( f (0)) = f 3(0).

• P ( f (0), y) ⇒ f ( f (y)+ f 3(0)) = f 3( f (0))+ y ∀y ∈ R⇒ f 3(0)+ f ( f (y)) = f 9(0)+ y ∀y ∈R⇒ 2 f 3(0) = f 9(0). Do đó f (0) = 0 hoặc f 6(0) = 2.

• P (t ,0) ⇒ f (t 3 + f (0)) = 0 = f (t ) ⇒ t 3 + f (0) = t .

• P (0, t ) ⇒ f (0) = f 3(0)+ t (∗). Suy ra t 3 =− f 3(0) ⇒ t =− f (0) ⇒ 2 f (0) = f 3(0) ⇒ f (0) =0 hoặc f 2(0) = 2. Kết hợp với trên suy ra f (0) = 0.Từ đó ta có f ( f (x)) = x ∀x ∈R và f (x3) = f 3(x) ∀x ∈R (2).P ( 3

px, f (y)) ⇒ f (x + y) = f (x)+ f (y) ∀x, y ∈R.

Đặt f (1) = k ta có k = k3, thay x bởi x +1 vào (2) ta có ∀x ∈R

f (x3 +3x2 +3x +1) = ( f (x)+ f (1))3

⇒ f (x3)+3 f (x2)+3 f (x) = f 3(x)+3k f 2(x)+3k2 f (x)

⇒ f (x2)+ f (x) = k f 2(x)+k2 f (x)

Thay x bởi x −1 vào (2) trên ta có ∀x ∈R

− f (x2)+ f (x) =−k f 2(x)+k2 f (x)

Do đó f (x2) = k f 2(x) ∀x ∈R Ta xét 2 trường hợp của k:

TH1: k = 1. Ta có f (x2) = f 2(x) ∀x ∈R⇒ f (x) > 0 ∀x > 0 do f (x) song ánh và f (0) = 0Xét x > y ta có: f (x) = f (y)+ f (x − y) > f (y). Suy ra f (x) đồng biến. Kết hợp vớif (x) cộng tính và f (1) = 1 ta có f (x) = x ∀x ∈R.Thử lại đúng.

http://thcmn.wordpress.com 101

Page 102: t 2017 - WordPress.com...Bài 5. (B›c Ninh) Cho a, b c¨0 thäa mãn đi•u ki»n ¯ ˘9. Tìm giá trà lîn nh t cıabi”uthøc T ˘ ab 3a¯4b¯5c ¯ bc 3b¯4c¯5a ¯ ca 3c¯4a¯5b

TH2: k =−1. Ta có f (x2) =− f 2(x) ∀x ∈R⇒ f (x) < 0 ∀x > 0 do f (x) song ánh và f (0) = 0Xét x > y ta có: f (x) = f (y)+ f (x − y) < f (y). Suy ra f (x) nghịch biến. Kết hợp vớif (x) cộng tính và f (1) =−1 ta có f (x) =−x ∀x ∈R.Thử lại đúng.

Vậy bài toán có 2 nghiệm hàm f (x) = x ∀x ∈R và f (x) =−x ∀x ∈R

Cho dãy số (un) xác định bởiu1 =−1

3un+1 +1 = un +1√

u2n +1

∀n ∈N∗.

1. Chứng minh rằng un+1 +1 < 3(un +1)p10

∀n ∈N∗.

2. Chứng minh rằng dãy (un) hội tụ. Tính limn→+∞un.

(Lạng Sơn)

Bài 23

Lời giải

1. Ý tưởng đầu tiên sẽ là chứng minh bằng quy nạp, tức là ta cần chứng minh

3(un +1)p10

≥ un +1√u2

n +1.

Cũng bằng quy nạp, dễ nhận thấy un +1 ≥ 0. Do đó ta cần chứng minh

3√

u2n +1 ≥p

10.

Bình phương 2 vế, điều cần chứng minh trở thành

9(u2n +1) ≥ 10 ⇔ u2

n ≥ 1

9.

Dễ chứng minh un giảm, nên bằng quy nạp, ta sẽ chứng minh được

u2n ≥ 1

9∀n,

và ta có điều cần chứng minh ban đầu.

2. Dãy đã cho giảm và bị chặn dưới bởi −1 nên tồn tại L = limun. Chuyển sang giới

hạn: L+1 = L+1pL2 +1

⇒ L =−1.

102 http://www.facebook.com/thcmn

Page 103: t 2017 - WordPress.com...Bài 5. (B›c Ninh) Cho a, b c¨0 thäa mãn đi•u ki»n ¯ ˘9. Tìm giá trà lîn nh t cıabi”uthøc T ˘ ab 3a¯4b¯5c ¯ bc 3b¯4c¯5a ¯ ca 3c¯4a¯5b

Cho dãy số thực (xn) được xác định bởix1 = 5

2

xn+1 =√

x3n −12xn + 20n +21

n +1

.

Chứng minh rằng dãy số (xn) có giới hạn hữu hạn và tìm giới hạn đó .

(Lào Cai)

Bài 24

Lời giải

Chuyển sang giới hạn thì

L =√

L3 −12L+20 ⇔ L = 2.

Do đó ta sẽ quy nạp để chứng minh xn ≥ 2.

• Với n = 1 thì x1 = 5

2≥ 2.

• Giả sử điều cần chứng minh đúng tới n.

• Bây giờ ta sẽ chứng minh điều này cũng đúng với n + 1, tức là ta sẽ đi chứngminh

x3n −12xn + 20n +21

n +1≥ 4.

Nhưng điều này sẽ là hiển nhiên do

x3n −12xn + 20n +21

n +1−4 ≥ x3

n −12xn +16 ≥ (xn −2)2(xn +4) ≥ 0.

Do đó xn ≥ 2

Xét hàm số f (x) = x3 −12x có f ′(x) = 3x2 −12 ≥ 0, và x2 < x1 nên dãy giảm.

Ta cũng có hàm số g (n) = 20n +21

n +1là hàm giảm, do đó xn vì là tổng của hai dãy giảm

nên cũng là dãy giảm. Hơn nữa nó bị chặn dưới bởi 2 nên tồn tại L = lim xn. Dễ dàngtìm được L = 2.

Nhận xét 27 Việc đánh giá hàm ở đây phải khéo léo vì việc xét một hàm là sai phâncủa xn như trên khá "chặt" do hàm sinh bởi công thức truy hồi xn trong bài toán vừaxong. Trong đề chọn đội tuyển quốc gia Cần Thơ năm nay cũng có một bài tương tựnhư sau, bạn đọc hãy giải nó như một bài tập áp dụng:

Cho dãy số thực(un

)được xác định bởi:

u1 = 3

2và un+1 =

√u3

n +3u2n −9un + 9n +10

n +1−1, ∀n ∈N∗

Tìm limn→+∞un .

http://thcmn.wordpress.com 103

Page 104: t 2017 - WordPress.com...Bài 5. (B›c Ninh) Cho a, b c¨0 thäa mãn đi•u ki»n ¯ ˘9. Tìm giá trà lîn nh t cıabi”uthøc T ˘ ab 3a¯4b¯5c ¯ bc 3b¯4c¯5a ¯ ca 3c¯4a¯5b

Ngoài hai bài toán trên, bạn đọc có thể xây dựng những bài toán tương tự với bậc caohơn thông qua hàm số đảm bảo tính đơn điệu và bị chặn trong công thức truy hồi đốivới số hạng của dãy.

Cho hàm số f :N∗ →N∗ thỏa mãn các điều kiện sau:

i. f (m) < f (n) ∀m,n ∈N∗;m < n.

ii. f (mn) = f (m) f (n) ∀m,n ∈N∗; (m,n) = 1.

iii. ∃i ∈N∗, i > 1 sao cho f (i ) = i .

1. Chứng minh rằng f (1) = 1 , f (3) = 3.

2. Tìm tất cả các hàm f (n) thỏa mãn yêu cầu đề bài.

(Ninh Bình)

Bài 25

Lời giải

1. Thay m = n = 1 vào 2. ta được f (1) = 1.Do i > 1 mà f (i ) = i nên ta suy ra f (n) = n với mọi n = ¯1, i (do i.).

• Nếu i Ê 3 thì f (3) = 3.

• Nếu i = 2 thì f (2) = 2 và f (3). f (5) = f (15) < f (18) = f (2). f (9) < 2. f (10) = 4. f (5).Do đó 2 < f (3) < 4 nên f (3) = 3.

2. Xét dãy (an) : a1 = 3, an = an−1(an−1 −1

).

Khi đó f (a1) = a1, nếu f (ak ) = ak thì f (ak − 1) = ak − 1 (do f tăng ngặt) nênf (ak+1) = f (ak ) f (ak −1) = ak+1.Vậy bằng quy nạp t có thể chứng minh f (ak+1) = ak+1∀k ∈N∗.Dựa vào tính chất đầu tiên ta cũng dễ dàng chứng minh rằng nếu f (ak ) = ak (k >1) thì f ( j ) = j với mọi 1 ≤ j ≤ ak . Mà dãy trên tăng vô hạn nên suy ra f (n) = n vớimọi số tự nhiên n.

104 http://www.facebook.com/thcmn

Page 105: t 2017 - WordPress.com...Bài 5. (B›c Ninh) Cho a, b c¨0 thäa mãn đi•u ki»n ¯ ˘9. Tìm giá trà lîn nh t cıabi”uthøc T ˘ ab 3a¯4b¯5c ¯ bc 3b¯4c¯5a ¯ ca 3c¯4a¯5b

Cho dãy số (xn) xác định bởi hệ thức:x1 = 1

xn+1 =√

xn(xn +1)(xn +2)(xn +3)+1,n ∈N∗

Đặt yn =n∑

i=1

1

xi +2. Tính lim yn.

(Ninh Bình)

Bài 26

Lời giải

Dễ thấy xn > 0,∀n ∈NTa sẽ cố gắng thu gọn biểu thức tính của xn+1

Ta cóx2

n+1 = xn(xn +3)(xn +1)(xn +2)+1

⇔ x2n+1 = (x2

n +3xn +1)2

⇔ xn+1 = x2n +3xn +1

Do đó ta sẽ tính được

1

xi +2= 1

xi +1− 1

xi+1 +1

Mặt khác, ta tính được lim xn =+∞Do đó lim yn = lim

1

x1 +1− 1

xn +1= 1

2.

Xét dãy số thực vô hạn x1, x2, · · · , xn thỏa mãn

|xm+n −xm −xn | < 1

m +n

với mọi số nguyên dương m,n. Chứng minh rằng (xn) là cấp số cộng.

(Phú Thọ)

Bài 27

Lời giải

Để chứng minh cấp số cộng thì ta nên tìm cách chứng minh xm+1 − xm = d : cố định.Do đó, ta sẽ tìm cách làm như sau:

|(xm+1 −xm)− (xn+1 −xn)| = |(xm+n+1 −xm −xn+1)− (xm+n+1 −xm+1 −xn)|< |(xm+n+1 −xm −xn+1)|+ |(xm+n+1 −xm+1 −xn)|< 2

m +n +1

.

http://thcmn.wordpress.com 105

Page 106: t 2017 - WordPress.com...Bài 5. (B›c Ninh) Cho a, b c¨0 thäa mãn đi•u ki»n ¯ ˘9. Tìm giá trà lîn nh t cıabi”uthøc T ˘ ab 3a¯4b¯5c ¯ bc 3b¯4c¯5a ¯ ca 3c¯4a¯5b

Tới đây thì vẫn chưa ổn vì vậy ta sẽ cố tìm cách chặn thêm 1 hằng số tự do nữa, cụthể như sau:

|(xm+1 −xm)− (xn+1 −xn)| = |(xm+1 −xm −xa+1 +xa)− (xn+1 −xn −xa+1 +xa)|< |(xm+1 −xm)− (xa+1 −xa)|+ |(xn+1 −xn)− (xa+1 −xa)|< 2

a +m +1+ 2

a +n +1

.

Cho a đủ lớn thì suy ra được

xm+1 −xm = xn+1 −xn = d ,∀m,n ∈N∗.

Do đó xn là cấp số cộng.

Tìm tất cả hàm số f : N∗ → N∗ sao cho ba số a, f (b), f (b + f (a)−1) luôn là độ dàiba cạnh của một tam giác với mọi a,b ∈ N∗

(Quảng Bình)

Bài 28

Lời giải

Giả sử tồn tại hàm f thỏa mãn đề bài. Với mọi số tự nhiên a,b, ta có: a + f (b) ≥ f (b +f (a)−1)+1 ⇒ f (b) ≥ f (b+ f (1)−1). Ta cũng có: a+ f (b+ f (a)−1) ≥ f (b)+1 ⇒ f (b+ f (1)−1) ≥f (b). do đó f (b) = f (b + f (1)−1).

1. f (1) 6= 1. Ta có f tuần hoàn. Mà f có tập giá trị là các số nguyên dương nên f bịchặn. Vậy tồn tại M = max f (x).Từ đề bài chọn b = 1, a = 2M ta có f ( f (2M)), f (1) và 2M là độ dài 3 cạnh của mộttam giác. Suy ra 2M ≥ f ( f (M +1))+ f (1) ≥ 2M +1, vô lý!

2. f (1) = 1Từ đề bài, chọn b = 1, ta có hai điều sau:

a + f (1) ≥ f ( f (a))+1 ⇒ a ≥ f ( f (a))∀a ∈N∗

f ( f (a))+ f (1) ≥ a +1 ⇒ f ( f (a)) ≥ a∀a ∈N∗.

Vậy f ( f (a)) = a∀a ∈N∗, từ đây dễ có f song ánh. Đặt f (2) = t ta có f (t ) = 2. Thaya = 2,b = t ta có 2,2, f (t − 1+ f (2)) là độ dài ba cạnh của một tam giác. Suy ra3 ≥ f (t −1+ f (2)) ≥ 1.

• Nếu f (t −1+ f (2)) = 1 = f (1), ta có f (2) = 1 vô lý.

• Nếu f (t −1+ f (2)) = 2 = f ( f (2)), ta có f (2) = t = 1 vô lý.

Vậy f (t −1+ f (2)) = 3. Ta chứng minh:

f (t −1+ f (n)) = n +1∀n ∈N∗ (3.19).

• Ta thấy (3.19) đúng với n = 1,2.

106 http://www.facebook.com/thcmn

Page 107: t 2017 - WordPress.com...Bài 5. (B›c Ninh) Cho a, b c¨0 thäa mãn đi•u ki»n ¯ ˘9. Tìm giá trà lîn nh t cıabi”uthøc T ˘ ab 3a¯4b¯5c ¯ bc 3b¯4c¯5a ¯ ca 3c¯4a¯5b

• Giả sử (3.19) đúng với n < k, ta chứng minh (3.19) đúng với n = k. Thật vậy,thay a = k,b = t ta có k,2, f (t −1+ f (k)) là độ dài ba cạnh của một tam giácnên k +2 > f (t −1+ f (k)) > k −2.Nếu f (t −1+ f (k)) = k −1 = f (t −1+ f (k −2)) ta có điều vô lý. Tương tự ta cóf (t −1+ f (k)) 6= k. Do đó f (t −1+ f (k)) = k +1.Vậy f (t −1+ f (n)) = n +1∀n ∈N∗

f (t − 1+ f (n)) = n + 1 = f ( f (n + 1)) ⇒ f (n + 1) = t − 1+ f (n)∀n ∈ N∗. Từ đó ta cóf (n + 1) = n(t − 1)+ 1∀n ∈ N∗. Mặt khác n + 1 = f ( f (n + 1)) = f (n(t − 1)+ 1) = (t −1)2n +1∀n ∈N∗, do đó t = 2, f (n) = n∀n ∈N∗.Thử lại, với hàm f (n) = n∀n ∈ N∗, điều kiện đề bài trở thành a,b, a +b −1 luônlà độ dài ba cạnh của một tam giác (đúng). Vậy chỉ có hàm số f (n) = n∀n ∈ N∗

thỏa mãn.

Cho a là một số thực và dãy số thực (xn) xác định bởi

xn = 2016n +a.3√

n3 +1.

1. Tìm a sao cho dãy số (xn) có giới hạn hữu hạn.

2. Tìm a để dãy số là dãy tăng từ một lúc nào đó.

(Quảng Bình)

Bài 29

Lời giải

1. xn = 2016n −20163√

n3 +1+ (a +2016).3√

n3 +1

xn = 2016 · −1

n2 +n3p

n3 +1+ 3√

(n3 +1)2+ (a +2016) · 3

√n3 +1

Khi đó nếu a 6= −2016 thì lim xn =+∞ hoặc là lim xn =−∞Do đó để xn có giới hạn hữu hạn thì a =−2016.

2. Ta dễ chứng minh lim

(3√

(n +1)3 +1− 3√

n3 +1

)= 1.

Ta có xn+1 −xn ≥ 0,∀n ≥ n0

Tức là a ≥ −20163√

(n +1)3 +1− 3pn3 +1

Lấy lim hai vế được a ≥−2016.Với a =−2016 dễ thấy thỏa mãn.Với a >−2016 thì ta có luôn điều phải chứng minh theo giới hạn.Do đó a ≥−2016.

http://thcmn.wordpress.com 107

Page 108: t 2017 - WordPress.com...Bài 5. (B›c Ninh) Cho a, b c¨0 thäa mãn đi•u ki»n ¯ ˘9. Tìm giá trà lîn nh t cıabi”uthøc T ˘ ab 3a¯4b¯5c ¯ bc 3b¯4c¯5a ¯ ca 3c¯4a¯5b

Cho a,b là các số thực dương. Xét dãy số un được xác định bởi un = a2n2+bn, vớin ∈N∗. Tính lim

{pun

}.

(Quảng Ninh)

Bài 30

Lời giải

Đặt b = 2aq + r với q,r ∈ N ;0 ≤ r ≤ 2a −1.

TH1: r = 0 thì b = 2aq và

un = a2n2 +2aqn = (an +q)2 −q2 < (an +q)2.

Lại cóun − (an +q −1)2 = 2an − (q −1)2.

Từ đây suy ra

un > (an +q −1)2; ∀n > (q −1)2

2a.

Điều này dẫn tới [pun

]= an +q −1∀n > (q −1)2

2

Khi ấy

limn→+∞

{pun

}= limn→+∞

(pun − [p

un])= lim

n→+∞

(√a2n2 +2aqn − (an +q −1)

)

= limn→+∞

2an − (q −1)2√a2n2 +2aqn +an +q −1

= limn→+∞

2a − (q−1)2

n√a2 + 2aq

n +a + q−1n

= 1.

TH2: 1 ≤ r ≤ 2a −1. Để ý rằng

un − (an +q +1)2 = (r −2a)n − (q +1)2 < 0; ∀n ∈N∗

vàun − (an +q)2 = r n −q2.

Suy ra

(an +q)2 < un < (an +q +1)2; ∀n > q2

r

⇒ [pun

]= an +q ; ∀n > q2

r.

Do đó limun ={

b

a

}.

108 http://www.facebook.com/thcmn

Page 109: t 2017 - WordPress.com...Bài 5. (B›c Ninh) Cho a, b c¨0 thäa mãn đi•u ki»n ¯ ˘9. Tìm giá trà lîn nh t cıabi”uthøc T ˘ ab 3a¯4b¯5c ¯ bc 3b¯4c¯5a ¯ ca 3c¯4a¯5b

Cho dãy số xn xác định bởi x1 = 2

xn+1 = 2xn +1

xn +2

Tìm số hạng tổng quát xn và tìm lim xn .

(Quảng Trị)

Bài 31

Lời giải

Đây là một bài khá quen thuộc và điển hình trong dạng bài tìm công thức tổng quátcủa dãy số. Ta có

xn+1 −1 = 2xn +1

xn +2−1 = xn −1

xn +2, (3.20)

xn+1 +1 = 2xn +1

xn +2+1 = 3(xn +1)

xn +2. (3.21)

Do đó lấy (3.21) chia (3.20), ta được

xn +1

xn −1= 3

xn−1 +1

xn−1 −1= ... = 3n−1.

x1 +1

x1 −1= 3n .

Quy đồng lên ta tính được xn = 3n +1

3n −1⇒ lim xn = 1.

Cho dãy số (an) có a1 ∈R và an+1 =∣∣∣an −21−n

∣∣∣ ,∀n ∈N∗. Tìm lim an.

(Thái Bình)

Bài 32

Lời giải

TH1: a1 ≥ 2 Khi đó a2 = a1 −1 ≥ 1 = 20. Khi đó a3 = a2 −2−1 ≥ 1− 1

2≥ 2−1

Do đó quy nạp thì ta có an+1 = an −21−n ⇒ an+1 = a1 −1− 1

2− ...− 1

2n−1= a1 −2−

1

2n−1⇒ lim an = a1 −2.

TH2: a1 ≤ 0, Khi đó a2 = 1−a1 ≥ 1 = 20, a3 =∣∣∣∣a2 − 1

2

∣∣∣∣= a2− 1

2. Quy nạp tương tự như trên,

ta được

an+1 = a1 −1− 1

2− ...− 1

2n−1= a1 −2− 1

2n−1.

TH3: a1 ∈ (0;2). Khi đó ta sẽ chứng minh quy nạp rằng 0 ≤ an ≤ 1

2n−2,∀n ≥ 2.

– Với n = 2 thì a2 = |a1 −1| ≤ 1.

http://thcmn.wordpress.com 109

Page 110: t 2017 - WordPress.com...Bài 5. (B›c Ninh) Cho a, b c¨0 thäa mãn đi•u ki»n ¯ ˘9. Tìm giá trà lîn nh t cıabi”uthøc T ˘ ab 3a¯4b¯5c ¯ bc 3b¯4c¯5a ¯ ca 3c¯4a¯5b

– Giả sử đúng tới n, ta chứng minh đúng với n +1. Ta có an+1 = |an −21−n | ≤1

2n−1. Để ý rằng

an −21−n ≤ 21−n ⇔ an ≤ 22−n đúng,

21−n −an ≤ 21−n ⇔ an ≥ 0 đúng.

Do đó theo quy nạp ta có điều phải chứng minh.

Như vậy 0 < an ≤ 1

2n−2. Lấy lim hai vế ta được lim an = 0.

Tìm tất cả các hàm số f :R→R thỏa mãn:

f ( f (x)+ f (y)) = f (x2)+2x2 f (y)+ ( f (y))2∀x, y ∈R

(Thanh Hoá)

Bài 33

Lời giải

Kí hiệu P (u, v) là phép thế x bởi u,y bởi v vào phương trình đề bài cho.

• P (1,0) ⇒ f ( f (1)+ f (0)) = f (1)+2 f (0)+ f 2(0). (3.22)

• P (0,1) ⇒ f ( f (0)+ f (1)) = f (0)+ f 2(1).

Từ đó suy ra f (1) =− f (0) hoặc f (1) = f (0)+1.

• Nếu f (1) =− f (0), ta có f (0) = f (1)+2 f (0)+ f 2(0) ⇒ f (0) = f (1) = 0.

– P (0, y) ⇒ f ( f (y)) = f 2(y)∀y ∈R.

– P (1, y) ⇒ f ( f (y)) = 2 f (y)+ f 2(y)∀y ∈R.

Từ đó suy ra f (y) = 0∀y ∈R• Nếu f (1) = f (0)+1: P (y, x) ⇒ f (x2)+2x2 f (y)+ f 2(y) = f (y2)+2y2 f (x)+ f 2(x) (1)∀x, y ∈R. Trong phương trình trên lần lượt thay x = 0, x = 1 ta được:

f (0)+ f 2(y) = f (y2)+2y2 f (0)+ f 2(0)∀y ∈R,

f (1)+2 f (y)+ f 2(y) = f (y2)+2y2 f (1)+ f 2(1)∀y ∈RVới chú ý f (1) = f (0)+1, từ hai phương trình trên ta thu được f (x) = x2+ f (0)∀x ∈R.

Thử lại thấy f (0) = 0. Hàm f (x) = x2∀x ∈ R thỏa mãn. Vậy có hai hàm thỏa mãn bàitoán là f (x) = 0∀x ∈R và f (x) = x2∀x ∈R

110 http://www.facebook.com/thcmn

Page 111: t 2017 - WordPress.com...Bài 5. (B›c Ninh) Cho a, b c¨0 thäa mãn đi•u ki»n ¯ ˘9. Tìm giá trà lîn nh t cıabi”uthøc T ˘ ab 3a¯4b¯5c ¯ bc 3b¯4c¯5a ¯ ca 3c¯4a¯5b

Với số thực a 6= −1p2

cho trước ,xét dãy số an cho bởi

a1 = a

an+1 =7√

2a2n +7−14

4an +√

2a2n +7

Xác định a để dãy có giới hạn hữu han.

(Thanh Hoá)

Bài 34

Lời giải

Xét hàm f (x) = 7p

2x2 +7−14

4x +p

2x2 +7thì an+1 = f (an).

Xét hàm số g (x) = f (x)− x có g (x) = 0 ⇔ x = 1; x = −3. Mà hàm số g (x) liên tục trênmỗi khoảng (−∞,−3) và (1;+∞) nên dễ chứng minh được f (x) > x,∀x ∈ (−∞;−3) vàf (x) < x,∀x ∈ (1;+∞).

Tiếp theo, dễ chứng minh được f (x) ≥ 1,∀x > −7

2. Dấu bằng xảy ra khi x = 1.

Tương tự f (x) ≤−3,∀x < −7

2. Dấu bằng xảy ra khi x =−3.

Từ các kết quả trên, ta thu được:

TH1: x1 = a = 1 Khi đó xn = 1 nên lim xn = 1.

TH2: x1 = a =−3 Khi đó xn =−3 nên lim xn =−3.

TH3: x1 = a > −7

2. Khi đó x2 ≥ 1 ⇒ xn ≥ 1 ⇒ xn+1 = f (xn) < xn. Do dãy giảm bị chặn dưới

bởi 1 nên lim xn = 1.

TH4: x1 = a < −7

2. Khi đó x2 ≤−3 ⇒ xn ≤−3 ⇒ xn+1 = f (xn) > xn. Do đó xn tặng bị chặn

trên bởi −3. Giải ra được lim xn =−3.

http://thcmn.wordpress.com 111

Page 112: t 2017 - WordPress.com...Bài 5. (B›c Ninh) Cho a, b c¨0 thäa mãn đi•u ki»n ¯ ˘9. Tìm giá trà lîn nh t cıabi”uthøc T ˘ ab 3a¯4b¯5c ¯ bc 3b¯4c¯5a ¯ ca 3c¯4a¯5b

4 Hình học

4ABC nhọn (AB < AC ) có H ,O lần lượt là trực tâm và tâm đường tròn ngoại tiếp.Điểm E thuộc cạnh AC sao cho OE//BC . OE cắt đường tròn ngoại tiếp 4EBC tạiF . Tiếp tuyển tại F của đường tròn ngoại tiếp 4EBC cắt BC , AH ở P,Q.

1. Chứng minh đường tròn (K ) ngoại tiếp 4BPQ đi qua trung điểm M của AH .

2. PA,PH cắt (K ) ở S,T khác P . Chứng minh hai tiếp tuyển của (K ) tại S,T cắtnhau tại một điểm trên ME .

(THPT chuyên KHTN - ĐH KHTN, ĐHQG HN )

Bài 1

Hướng dẫn

1. Do EF //BC nên �F PC = �FC E . Mặt khác dễ thấy 4OC E ∼ 4B AH nênOE

EC= B H

AH.

Từ đó suy raM H

HB= EC

EFnên 4F EC ∼4B H M (c.g.c). Do đó �B MQ = �FC E = �F PC

nên tứ giác MQBP nội tiếp.

2. Gọi R là giao điểm của E M với (K ). Dễ thấy chỉ cần chứng minh tứ giác RSMTđiều hòa ⇔ P (RM AH) = −1. Mặt khác do M là trung điểm AH nên ta chỉ cần

112 http://www.facebook.com/thcmn

Page 113: t 2017 - WordPress.com...Bài 5. (B›c Ninh) Cho a, b c¨0 thäa mãn đi•u ki»n ¯ ˘9. Tìm giá trà lîn nh t cıabi”uthøc T ˘ ab 3a¯4b¯5c ¯ bc 3b¯4c¯5a ¯ ca 3c¯4a¯5b

chứng minh PR//AH . Điều này tương đương với chứng minh �PQM + �E MQ =180◦. Do �PQM = 90◦+ �BPQ = 90◦+ �B M H nên ta chỉ cần chứng minh �B ME = 90◦,một kết quả quen thuộc.

Tứ giác ABC D nội tiếp (O) sao cho ABC D không phải hình thang. Tiếp tuyến tạiC ,D của (O) cắt nhau tại T . T A cắt BD tại S, E dối xứng với D qua S. AB cắt đườngtròn ngoại tiếp 4EBC tại F . EC cắt T A tại P .

1. Chứng minh rằng PF tiếp xúc đường tròn ngoại tiếp 4EBC .

2. Giả sử PF cắt AC tại Q, H ,K lần lượt là hình chiếu của Q lên F A,FC . M làtrung điểm F A. Chứng minh rằng tiếp tuyến qua A của (O) và đường thẳngqua Qv song song AO cắt nhau trên đường tròn ngoại tiếp 4M HK .

(THPT chuyên KHTN, ĐH KHTN, ĐHQG HN)

Bài 2

Lời giải

1. • Dễ dàng chứng minh được 4EC F v4DC A.

• AT cắt C D ở N , R ∈ C D sao cho ER//AT . Như vậy N là trung điểm DR. Để

ý rằng AN là đường đối trung nênN D

NC= AD2

AC 2⇒ PE

PC= N R

NC= N D

BC= AD2

AC 2=

F E 2

FC 2⇒ PF tiếp xúc đường tròn ngoại tiếp 4EBC tại F .

http://thcmn.wordpress.com 113

Page 114: t 2017 - WordPress.com...Bài 5. (B›c Ninh) Cho a, b c¨0 thäa mãn đi•u ki»n ¯ ˘9. Tìm giá trà lîn nh t cıabi”uthøc T ˘ ab 3a¯4b¯5c ¯ bc 3b¯4c¯5a ¯ ca 3c¯4a¯5b

2. Đường thẳng qua I song song AO cắt tiếp tuyến qua (O) của A tại L.

• PF tiếp xúc đường tròn ngoại tiếp 4EBC tại F nên �L AQ = �ADC = �F E A =�K FQ ⇒4QL A ∼4QK F .

• Theo kết quả quen thuộc thì đường tròn ngoại tiếp 4HK L đi qua trungđiểm M của AF , từ đó L nằm trên đường tròn ngoại tiếp 4M HK .

4ABC nhọn nội tiếp (O) có H là trực tâm. P là một điểm nằm trên trung trực củaBC và nằm trong 4ABC . Đường thẳng qua A song song PH cắt (O) tại E khác A.Đường thẳng qua E song song AH cắt (O) tại F khác E . Gọi Q là điểm đối xứng vớiP qua O. Đường thẳng qua F song song với AQ cắt PH tại G.

1. Chứng minh rằng B ,C ,P,G cùng thuộc một đường tròn tâm K .

2. AQ cắt (O) tại R khác A. PQ cắt F R tại L. Chứng minh K L =OP .

(THPT chuyên KHTN, ĐH KHTN, ĐHQG HN)

Bài 3

Lời giải

1. AH cắt (O) tại điểm thứ hai là A′. HG cắt BC tại F ′ Ta sẽ chứng minh F,F ′, A′

thẳng hàng. Thật vậy, �F ′H A = àF ′A′H = �E AH = �F A′H ( vì HP//AE và AEF A′ là

114 http://www.facebook.com/thcmn

Page 115: t 2017 - WordPress.com...Bài 5. (B›c Ninh) Cho a, b c¨0 thäa mãn đi•u ki»n ¯ ˘9. Tìm giá trà lîn nh t cıabi”uthøc T ˘ ab 3a¯4b¯5c ¯ bc 3b¯4c¯5a ¯ ca 3c¯4a¯5b

hình thang cân do EF //A A′) , suy ra F,F ′, A′ thẳng hàng. Khi đó, ta có: �FGF ′ =�Q AE = �F A′P (dựa vào tính song song của gt và AQPA′ là hình thang cân), suy raPFG A′ nội tiếp, nên theo tính chất phương tích thì F ′P.F ′G = F ′F.F ′A′ = F ′B.F ′C ,suy ra: BPCG nội tiếp.

2. Gọi A′F cắt trung trực của BC tại S. Theo định lý Reim ta có QS//A A′;F R A′A nộitiếp nên F RSQ nội tiếp.

• LR.LF = LS.LQ = LO2 −R2 (Phương tích của L tới (O) và (QF RS)) ⇔ LS.LQ =LO2 −R2 ⇔ (LO −OS)(LO +OQ) = LO2 −R2 ⇔ LO(OQ −OS)−OS.OQ = −R2 ⇔LO.PS +OS.OQ = R2.

• OS = 2OM −OP ;PS = 2OM −2OP ⇒ 2.LO.OM −2.LO.OP +2OM .OP −OP 2 = R2

⇔ LO = R2 +OP 2 −2.OM .OP/2OM −2OP .

• Ta có: K P = C P

2sinPBC= C P

2P MC P

= C P 2

2P M. Vì P M = OM −OP nên ta cần phải

chứng minh K P = LO hay⇔ R2+OP 2−2OM .OP =C P 2⇔ 2.R.OP.cos A = 2OM .OP⇔R cos A =OM , luôn đúng.

4ABC nội tiếp (O), ngoại tiếp (I ). Đường tròn qua B ,C tiếp xúc (I ) tại P . AI giaoBC tại X . Tiếp tuyến qua X của (I ) khác BC , giao tiếp tuyến tại (I ) tại P tại S. ASgiao (O) tại T khác A. Chứng minh rằng �AT I = 90o

(THPT chuyên KHTN, ĐH KHTN, ĐHQG HN)

Bài 4

Lời giải

Gọi tiếp điểm của (I ) với BC ,C A, AB lần lượt là D,E ,F , K là hình chiếu của D trên EF .

• DK cắt (I ) tại điểm thứ hai Q thì DL ⊥ DK ⇒QL là đường kính của (I ).

http://thcmn.wordpress.com 115

Page 116: t 2017 - WordPress.com...Bài 5. (B›c Ninh) Cho a, b c¨0 thäa mãn đi•u ki»n ¯ ˘9. Tìm giá trà lîn nh t cıabi”uthøc T ˘ ab 3a¯4b¯5c ¯ bc 3b¯4c¯5a ¯ ca 3c¯4a¯5b

• EF cắt BC tại G, tiếp tuyến chung tại P cắt BC tại M thì MD2 = MP 2 = MB.MC ,mà (B ,C ,D,G) = −1 nên M là trung điểm DG ⇒ 4GPD vuông tại P ⇒ Tứ giácGPK D nội tiếp. Ngoài ra, vì BC tiếp xúc (I ) tại D nên �PKQ = �PGD = 90o − �PDG =90o −�PQD ⇒ �K PQ = 90o ⇒ P,K ,L thẳng hàng.

• Lấy J thuộc BC sao cho I J ⊥ I A//QD thì J là cực của QD. Để ý rằng LP là đốicực của S và EF là đối cực của A, mà DQ,EF,PL đồng quy tại K nên A, J ,S thẳnghàng. Từ đó JT.J A = JB.JC = J I 2 ⇒4AT I vuông ở T .

4ABC có �B AC tù,AH ⊥ BC H ∈ BC . Điểm M thay đổi trên cạnh AB . Dựng điểm Nsao cho 4B M N ∼4HC A (H và N khác phía đối với AB).

1. C M cắt đường tròn ngoại tiếp 4B M N tại K khác M . Chứng minh đườngthẳng N K luôn đi qua một điểm cố định.

2. N H cắt AC tại P . Dựng Q sao cho 4HPQ ∼4H N M (Q và M khác phía so vớiP N). Chứng minh Q thuộc một đường thẳng cố định.

(Trường Phổ thông Năng khiếu, ĐHQG Tp. HCM)

Bài 5

Lời giải

Chúng tôi xin trích đăng lời giải của thầy Nguyễn Tăng Vũ (trường Phổ thông Năngkhiếu - ĐHQG Tp. HCM).

1. Gọi X là giao điểm của đường thẳng vuông góc với BC tại B và đường thẳngAC , K ′ là giao điểm của N X và C M . 4B M N ∼4BC X nên có một phép vị tự quaytâm H biến M thành N , biến C thành X . Vì K ′ là giao điểm của C M và N X nên K ′

thuộc đường tròn ngoại tiếp 4B M N hay K trùng K ′. Vậy N K luôn đi qua điểmX cố định.

116 http://www.facebook.com/thcmn

Page 117: t 2017 - WordPress.com...Bài 5. (B›c Ninh) Cho a, b c¨0 thäa mãn đi•u ki»n ¯ ˘9. Tìm giá trà lîn nh t cıabi”uthøc T ˘ ab 3a¯4b¯5c ¯ bc 3b¯4c¯5a ¯ ca 3c¯4a¯5b

2. Xét phép vị tự tâm H biến N thành P , biến M thành Q, biến B thành F . Có4B M N ∼4FQP ⇒ �FQP = �B M N = �AC B = �FC P ⇒ Tứ giác C F PQ nội tiếp ⇒ �QC P =�QF P = �MB N = 90o ⇒Q thuộc đường thẳng qua C vuông góc với AC cố định.

Cho 4ABC nhọn. Đường tròn (I ) có tâm I thuộc BC và tiếp xúc với các cạnhAB , AC lần lượt tại E ,F .Llấy hai điểm M , N bên trong tứ giác BC EF sao cho tứ giácEF N M nội tiếp (I ) và các đường thẳng BC , M N ,EF đồng quy. MF cắt N E tại P , APcắt BC tại D.

1. Chứng minh A,D,E ,F cùng thuộc một đường tròn.

2. Trên đường thẳng B N ,C M lấy các điểm H ,K sao cho �AC H = �ABK = 90o. LấyT là trung điểm HK . Chứng minh T B = TC .

(Trường Phổ thông Năng khiếu, ĐHQG Tp. HCM)

Bài 6

Lời giải

Chúng tôi xin trích đăng lời giải của thầy Nguyễn Tăng Vũ (trường Phổ thông Năngkhiếu - ĐHQG Tp. HCM).

Các bổ đề dùng trong bài như sau:

Bổ đề 7 Cho đường tròn (O;R) và điểm Q nằm ngoài đường tròn. Cát tuyến qua Q cắt(O) tại M , N . Tiếp tuyến tại M , N cắt nhau A, vẽ AD ⊥OQ. Khi đó OD.OQ = R2.

Bổ đề 8 (Định lý Brocard) Cho tứ giác ABC D nội tiếp. Gọi P,Q, I lần lượt là giao điểmcủa AD và BC , AB và C D, AC và BD. Khi đó PI ⊥OQ và nếu D là giao điểm của PI vàOQ thì OD.OQ = R2.

Bổ đề 9 (Định lí Desargue) Cho hai tam giác ABC và A′B ′C ′. Gọi M , N ,P lần lượt làgiao điểm của AB và A′B ′, AC và A′C ′, BC và B ′C ′. Khi đó M , M ,P thẳng hàng ⇔A A′,BB ′,CC ′ đồng quy.

http://thcmn.wordpress.com 117

Page 118: t 2017 - WordPress.com...Bài 5. (B›c Ninh) Cho a, b c¨0 thäa mãn đi•u ki»n ¯ ˘9. Tìm giá trà lîn nh t cıabi”uthøc T ˘ ab 3a¯4b¯5c ¯ bc 3b¯4c¯5a ¯ ca 3c¯4a¯5b

Bổ đề 10 Cho 4ABC , hai tia Ax, Ay đối xứng nhau qua phân giác trong góc A. GọiH ,K là hình chiếu của B trên Ax, Ay ; P,Q là hình chiếu của C trên Ax, Ay . Khi đóH ,K ,P,Q cùng thuộc một đường tròn có tâm là trung điểm BC .

Chứng minh các bổ đề trên xin dành cho bạn đọc.

1. Gọi D ′ là hình chiếu của A trên BC . Theo bổ đề 7, ta có I D ′.IQ = R2.Gọi X ,D" là giao điểm của ME và N F , X P và BC thì X P ⊥ BC và I D".IQ = R2 ⇒D ′ ≡ D" ⇒ X , A,P,D thẳng hàng và tứ giác EF I D nội tiếp.

2. Gọi S là giao điểm của B M và C N . Áp dụng định lí Desargue cho hai tam giácPEF và SBC ta được A,P,S thẳng hàng ⇒ S ∈ AD. Chứng minh được �B AK =�C AH ⇒ AK , AH đối xứng nhau qua phân giác trong góc �B AC . Đến đây theo bổđề 10, trung điểm của HK cách đều BC và ta có đpcm.

Cho 4ABC . Đường tròn (I ) nội tiếp 4ABC tiếp xúc với BC ,C A, AB tại D,E ,F .Đường thẳng D I cắt đường tròn tâm A bán kính AE tại M , N (N nằm giữa Mvà D). Các đường thẳng AD,EF cắt nhau tại P . các đường thẳng M A, N P cắt nhautại Q. Gọi H là giao điểm thứ hai của AD và (I ). Đường thẳng qua trung điểm củaD H ,DE cắt AC tại L. Chứng minh rằng:

1. QH ⊥ AD.

2. DL//EF .

(THPT chuyên Đại học Sư phạm Hà Nội)

Bài 7

Lời giải

1. Gọi J là giao điểm D I và (I ).

• AN 2 = AM 2 = AH .AD ⇒ �AN H = �AM H = �ADN ⇒ Tứ giác AH N M nội tiếp, màAM = AN nên H A là phân giác ngoài của àN H M hay H J là phân giác ngoài

của àN H M ⇒ N J

N D= M J

MD.

118 http://www.facebook.com/thcmn

Page 119: t 2017 - WordPress.com...Bài 5. (B›c Ninh) Cho a, b c¨0 thäa mãn đi•u ki»n ¯ ˘9. Tìm giá trà lîn nh t cıabi”uthøc T ˘ ab 3a¯4b¯5c ¯ bc 3b¯4c¯5a ¯ ca 3c¯4a¯5b

• Tứ giác HEDF điều hoà ⇒ PH

PD= AH

AD.

• Bằng định lý Menelaus, chứng minh được N P, J H , AM đồng quy hay Q, J , Hthẳng hàng ⇒QH ⊥ AD.

2. Gọi K là trung điểm D H ⇒ K L//HE ⇒ AH

AK= AE

AL. Lại có AH .AD = AP.AK ⇒ AH

AK=

AP

AD⇒ AE

AL= AP

AD⇒ DL//PE ⇒ DL//EF .

Hai đường tròn (O1) và (O2) tiếp xúc ngoài nhau tại A. BC là một tiếp tuyến chungngoài của (O1) và (O2), với B ∈ (O1) và C ∈ (O2). Gọi M là trung điểm BC , P,Q theothứ tự là điểm đối xứng của B ,C qua O1,O2. MP theo thứ tự cắt BO2,B A tại X ,Y .MQ cắt CO1,C A tại Z ,T . Chứng minh rằng:

1. Các tứ giác B Z T P,C X Y Q nội tiếp.

2. AM , Z T, X Y đồng quy.

(THPT chuyên Đại học Sư phạm Hà Nội)

Bài 8

Lời giải

http://thcmn.wordpress.com 119

Page 120: t 2017 - WordPress.com...Bài 5. (B›c Ninh) Cho a, b c¨0 thäa mãn đi•u ki»n ¯ ˘9. Tìm giá trà lîn nh t cıabi”uthøc T ˘ ab 3a¯4b¯5c ¯ bc 3b¯4c¯5a ¯ ca 3c¯4a¯5b

1. Giả sử MQ giao với (O2) tại Z ′, ta chứng minh Z ≡ Z ′. Dễ thấy P ∈ (O1),Q ∈(O2),BP//CQ.

• Dễ chứng minh M năm trên trục đẳng phương của (O1), (O2) ⇒ AM⊥O1O2.Khi đó MB 2 = MC 2 = M Z ′.MQ ⇒ 4MB Z ′ ∼ 4MQB ⇒ àMB Z ′ = �Z ′Q A. Màtheo tính chất tiếp tuyến, ta có: àMC Z ′ = �CQ Z ′ = �C AZ ′ ⇒ �B Z ′A = 360o −�B Z ′C−�AZ ′C = 2.(180o−�Z ′C A−�Z ′AC ) = 2.(180O−�CQ A) = 180O−(180O−2. �ABO1) =180O − �BO1 A ⇒ B Z ′AO1 là tứ giác nội tiếp. Từ đây dễ dàng suy ra được:àO1Z A = �AQC = 180o − �AZC ⇒ O1, Z ,C thẳng hàng ⇒ Z ′ ≡ Z Kết hợp với trênsuy ra B Z AO1 là tứ giác nội tiếp. Chứng minh tương tự: CO2X A là tứ giácnội tiếp.

• B Z ′AO1 là tứ giác nội tiếp ⇒ �Z BO1 = �Z AO2 = 180O−_

ZC + _AQ

2= 180o− �Z T A ⇒

B Z T P là tứ giác nội tiếp. Chứng minh tương tự, C X Y Q là tứ giác nội tiếp.

2. Gọi D là giao điểm của Y Z và PC và E là giao điểm của QB và P Z . Theo hàngđiểm điều hòa cơ bản, (MT ZQ) =−1 ⇒ P (MT ZQ) =−1 ⇒ (Y AEQ) =−1. Do đó, tacũng sẽ có được (MT ZQ) = (AY EQ) =−1 nên AM ,Y T ,Z P đồng quy ⇒ (PT AD) =−1.Mà (P X MY ) =−1 nên Y Z ≡ Y D,M A,X T đồng quy.

120 http://www.facebook.com/thcmn

Page 121: t 2017 - WordPress.com...Bài 5. (B›c Ninh) Cho a, b c¨0 thäa mãn đi•u ki»n ¯ ˘9. Tìm giá trà lîn nh t cıabi”uthøc T ˘ ab 3a¯4b¯5c ¯ bc 3b¯4c¯5a ¯ ca 3c¯4a¯5b

4ABC vuông tại A nội tiếp đường tròn (O). Đường thẳng qua A song song BC cắt(O) tại điểm thứ hai là D. Gọi I là giao điểm của AC và BD. Đường thẳng qua Isong song AB cắt AD tại J . Đường tròn tâm C bán kính C I cắt (O) tại E và F (Ethuộc cung B AC .

1. Gọi S là giao điểm của I J với C D. Chứng minh S,E ,F thẳng hàng.

2. Chứng minh E J ⊥ AF .

(THPT chuyên ĐH Vinh)

Bài 9

Lời giải

1. SI ⊥ AC và I D ⊥ SD nên SI 2 = SD.SC ⇒ S thuộc trục đẳng phương của (O) và(C ) ⇒ S,E ,F thẳng hàng.

2. Dễ thấy 4S JD cân với trung trực SE ⇒�JEF = �DEF = �D AF và SE ⊥ AD ⇒ AF ⊥ E J .

http://thcmn.wordpress.com 121

Page 122: t 2017 - WordPress.com...Bài 5. (B›c Ninh) Cho a, b c¨0 thäa mãn đi•u ki»n ¯ ˘9. Tìm giá trà lîn nh t cıabi”uthøc T ˘ ab 3a¯4b¯5c ¯ bc 3b¯4c¯5a ¯ ca 3c¯4a¯5b

4ABC có M di chuyển trên cạnh AC . Đường tròn ngoại tiếp 4AB M cắt cạnh BCtại điểm thứ hai là D. Đường tròn ngoại tiếp 4BC M cắt cạnh AB tại điểm thứ hailà E .

1. Gọi O là giao điểm của AD và C E . Chứng minh A,E ,O, M cùng thuộc mộtđường tròn.

2. Gọi I , J , N lần lượt là giao điểm của các cặp đường thẳng AB và DM , BC vàE M , AJ và C I . Chứng minh đường thẳng M N luôn đi qua điểm cố định.

(THPT chuyên ĐH Vinh)

Bài 10

Lời giải

1. �AEC = �AMB = �ADB ⇒ Tứ giác BDOE nội tiếp ⇒ �AOE = �ABC = �AME ⇒ tứ giácAEOM nội tiếp.

2. Theo định lý Pappus có O, M , N thẳng hàng ⇒ �C M N = �AMO = �BEC = �B MC ⇒M N luôn đi qua điêm đối xứng của B qua AC . Ta có đpcm.

122 http://www.facebook.com/thcmn

Page 123: t 2017 - WordPress.com...Bài 5. (B›c Ninh) Cho a, b c¨0 thäa mãn đi•u ki»n ¯ ˘9. Tìm giá trà lîn nh t cıabi”uthøc T ˘ ab 3a¯4b¯5c ¯ bc 3b¯4c¯5a ¯ ca 3c¯4a¯5b

Trên nửa đường tròn tâm O đường kính AB lấy hai điểm M , N sao cho AM < ANvà M N không song song với AB . Đường tròn ngoại tiếp 4OM N cắt AB tại điểmD khác O. Đường thẳng AN cắt đường tròn ngoại tiếp 4MBD tại hai điểm E ,F(E nằm giữa A và N ). Đường thẳng B M cắt đường tròn ngoại tiếp 4N AD tại haiđiểm P,Q (P nằm giữa B và M).

1. Chứng minh E ,F,P,Q cùng thuộc một đường tròn.

2. Đường thẳng AP cắt BE tại điểm X , đường thẳng BE cắt AQ tại điểm Y .Chứng minh bốn đường thẳng EP,QF, X Y , AB đồng quy.

(Bà Rịa - Vũng Tàu)

Bài 11

Lời giải

1. Dùng phương tích.

2. EP cắt QF tại U , PF cắt EQ taị V , AN cắt B M tại T . Khi đó UV là đường đối cựccủa T đối với (EF PQ).

• (ADN ) cắt (BDM) tại K ⇒ T là trực tâm của 4K AB ⇒ T D ⊥ AB . Từ đâyđược (AT EF ) = (BT PQ) = −1 (1) ⇒ A,B thuộc đường đối cực của T đối với(EF PQ) ⇒ A,B ,U ,V thẳng hàng. Ngoài ra , từ (1) ⇒ AB ,PE ,QF đồng quy tạiU .

• AQ cắt BF tại H áp dụng định lý Desargues cho 2 tam giác AQE BPF đượcH V T thẳng hàng.

http://thcmn.wordpress.com 123

Page 124: t 2017 - WordPress.com...Bài 5. (B›c Ninh) Cho a, b c¨0 thäa mãn đi•u ki»n ¯ ˘9. Tìm giá trà lîn nh t cıabi”uthøc T ˘ ab 3a¯4b¯5c ¯ bc 3b¯4c¯5a ¯ ca 3c¯4a¯5b

• AP cắt BE tại Z . Có (UV AB) = −1 ⇒ T, Z ,V thẳng hàng ⇒ T, Z ,V , H thẳnghàng. Áp dụng định lý Desargues cho 2 tam giác AEY và BP X ⇒ AB ,PE , X Yđồng quy. Từ đó ta có đpcm.

Tứ giác ABC D nội tiếp đường tròn (O). Giả sử AD cắt BC tại N , AB cắt C D tại M ,AC cắt BD tại E . Đường thẳng OE cắt M N tại K . Chứng minh KO là phân giác của�BK D.

(Bắc Ninh)

Bài 12

Lời giải

Gọi I là giao điểm của BD và M N thì (B ,D,E ,T ) =−1.Áp dụng định lý Brocard cho tứ giác toàn phần ABC DM N ⇒ OK ⊥ M N , do đó EK làphân giác góc �BK D.

124 http://www.facebook.com/thcmn

Page 125: t 2017 - WordPress.com...Bài 5. (B›c Ninh) Cho a, b c¨0 thäa mãn đi•u ki»n ¯ ˘9. Tìm giá trà lîn nh t cıabi”uthøc T ˘ ab 3a¯4b¯5c ¯ bc 3b¯4c¯5a ¯ ca 3c¯4a¯5b

Cho đường tròn (O1), (O2) tiếp xúc ngoài tại điểm T . Một đường thẳng cắt đườngtròn (O1) tại hai điểm A,B phân biệt và tiếp xúc với (O2) tại X . đường thẳng X Tcắt (O1) Tại S ( S khác T và C là một điểm trên cung TS không chứa A và B . ChoC Y là tiếp tuyến của (O2) tại Y sao cho các đoạn thẳng C Y và ST không cắt nhau.Cho I là giao điển của các đường thẳng X Y và SC . Chứng minh rằng:

1. C ,T,Y và I cùng thuộc một đường tròn.

2. S A = SI

(Bến Tre)

Bài 13

Lời giải

1. Vì (O1), (O2) tiếp xúc ngoài nhau tại T nên dễ dàng chứng minh được: �SO1T =àT O2X ⇒ �T AS = �T Y X = �T Y I . Mà: �T AS = �TC I (Do C thuộc cung T S không chứaAB và theo tính chất góc ngoài của tứ giác nội tiếp) ⇒ �TC I = �T Y I ⇒ T C I Y là tứgiác nội tiếp.

2. Dễ chứng minh: O1S//XO2, mà XO2⊥AB ⇒ SO1⊥AB ⇒ S A = SB .Ta lại có: T I S = �T IC = �T Y C = �T X I , mà T SI =�X SI ⇒4ST I ∼4SI X ⇒ SI 2 = ST.SX .Ta có: �T BS = �T AS = �TC I = �T Y I = �B X S, mà: �BST = �BSX ⇒⇒ 4ST B ∼ 4SB X ⇒SB 2 = ST.SX .TỪ ba điều trên suy ra S A = SI .

http://thcmn.wordpress.com 125

Page 126: t 2017 - WordPress.com...Bài 5. (B›c Ninh) Cho a, b c¨0 thäa mãn đi•u ki»n ¯ ˘9. Tìm giá trà lîn nh t cıabi”uthøc T ˘ ab 3a¯4b¯5c ¯ bc 3b¯4c¯5a ¯ ca 3c¯4a¯5b

4ABC nội tiếp đường tròn (O). Điểm P nằm trong 4ABC sao cho PA,PB ,PC cắt(O) lần lượt tại D,E ,F . Tiếp tuyến tại A của (O) cắt BC tại T . Chứng minh rằng nếuT A = T P thì DE = DF .

(Bình Dương)

Bài 14

Lời giải

Gọi giao điểm của EF và AD là H . Yêu cầu đề bài tương đương với chứng minh 4DEFcân tại D, hay OD ⊥ F E .Dễ dàng chứng minh được: T P⊥OD nên điều trên sẽ đúng nếu ta chứng minh đượcT P//F E . Thật vậy, T P 2 = T A2 = T B.TC ⇒ 4T BP ∼ 4T PC ⇒ �T BP = �T PC ⇒ �F PT =

180o − �T PC = �PBC ⇒ �APT = �F PT + �APF = �PBC + �APF =_

EC + _C D + _

AF

2= �AHF . Theo

tính chất hai góc đồng vị, ta có đpcm.

Cho 2 đường tròn (O), (O′) cắt nhau tại A,B . trên tia B A lấy M (M nằm ngoài (O′)).Từ M Kẻ 2 tiếp tuyến MC , MD của (O′) (C ,D là 2 tiếp điểm). AC , AD lần lượt cắt(O) tại P,Q.

1. Chứng minhD A

DQ= C A

C P.

2. Chứng minh C đi qua trung điểm PQ.

3. Chứng minh C D đi qua một điểm cố định khi M di chuyển trên tia B A.

(Bình Định)

Bài 15

126 http://www.facebook.com/thcmn

Page 127: t 2017 - WordPress.com...Bài 5. (B›c Ninh) Cho a, b c¨0 thäa mãn đi•u ki»n ¯ ˘9. Tìm giá trà lîn nh t cıabi”uthøc T ˘ ab 3a¯4b¯5c ¯ bc 3b¯4c¯5a ¯ ca 3c¯4a¯5b

Lời giải

1. Dễ dàng chứng minh được

4PBC ∼4QBC ⇒ QD

PC= BD

BCAD

AC= BD

BC

⇒ D A

DQ= C A

C P(đpcm).

2. Giả sử C D giao với PQ tại T , ta cần chứng minh T P = TQ.Xét tam giác Q AD có T,C ,D thẳng hàng. Theo định lý Menelaus, ta có

T Q

T P.D A

DQ.C P

C A= 1.

Kết hợp với kết quả câu 1, ta có: TQ = T P ⇒ (đpcm).

3. Gọi giao điểm của C D, AB với O1O2 lần lượt là X ,E . Trung điểm EO′ là F . Tachứng minh X cố định. Thật vậy, dễ thấy: O′,E ,F cố định.Ta có: O′X 2 = CO′2 −C M 2 + M X 2 = CO′2 − M A.MB + ME 2 + E X 2 = CO′2 − ME 2 +E A2 + ME 2 +E X 2 = CO′2 +E A2 ⇒ XO′2 − X E 2 = CO′2 +E A2 = 2.X F ′.EO′ ⇒ X F ′ =CO′2 +E A2

2.EO′ . Từ đây, ta có được X là điểm cố định.

Cho tam giác ABC với I là tâm đường tròn nội tiêp và M là một điểm nằm tronggiác. Gọi A1,B1,C1 là các điểm đối xứng với điểm M lần lượt qua các đường thẳngAI ,B I ,C I . Chứng minh rằng các đường thẳng A A1,BB1,CC1 đồng quy.

(Bình Thuận)

Bài 16

Hướng dẫn

Áp dụng Ceva − si n và các góc đối xứng.

http://thcmn.wordpress.com 127

Page 128: t 2017 - WordPress.com...Bài 5. (B›c Ninh) Cho a, b c¨0 thäa mãn đi•u ki»n ¯ ˘9. Tìm giá trà lîn nh t cıabi”uthøc T ˘ ab 3a¯4b¯5c ¯ bc 3b¯4c¯5a ¯ ca 3c¯4a¯5b

Cho tứ giác ABC D nội tiếp đường tròn (O). Gọi M , N ,P lần lượt là giao điểm củaAB và C D, AC và BD. Lấy K là trung điểm của đoạn M N . Đoạn PK cắt (O) tại H ,M H cắt (O) tại I khác H , N H cắt (O) tại J khác H . Hãy phân tích

−−→PK theo hai vectơ−−→

M I ,−−→M J .

(Bình Thuận)

Bài 17

Lời giải

Tước hết, ta cần chứng minh: P là trung điểm I J .Gọi giao điểm của tia PK với (O) là E (E 6= H), bán kính của (O) là R,OP giao M N tại F .

Theo tính chất đối cực và định lý Brocard, ta có:

P là trực tâm 4MON

OF⊥M NOP .OF = R2

⇒ MF .F N = F P .FO = FO2 −OP .OF = FO2 −R2

⇒ FO2 +F K 2 −R2 =OK 2 −R2 = MF .F N +F K 2 = MK 2 −F K 2 +F K 2 = MK 2

⇒ K M 2 = K H .K E = K N 2

Từ P ta kẻ đường vuông góc với OF giao (O) lần lượt tại hai điểm I ′, J ′, (I ′ nằm trên nữamặt phẳng bờ EK chứa M). Ta chứng minh I ′ ≡ I , J ≡ J ′.Từ cách vẽ và do F nằm trên đường đối cực tương ứng với điểm P của (O), nên tachứng minh được: P J ′ = PI ′, I ′ J ′//M N , (K PHE) =−1 .

Ta có: (K PHE) =−1 nênEK

EP=−HK

HP.

Do K H .K E = K N 2; PH .PE = PI ′2, nên kết hợp với (4.5) ta sẽ chứng minh được tỉ sốsau:

EK

EP= N K

PI ′(4.5)

128 http://www.facebook.com/thcmn

Page 129: t 2017 - WordPress.com...Bài 5. (B›c Ninh) Cho a, b c¨0 thäa mãn đi•u ki»n ¯ ˘9. Tìm giá trà lîn nh t cıabi”uthøc T ˘ ab 3a¯4b¯5c ¯ bc 3b¯4c¯5a ¯ ca 3c¯4a¯5b

Từ đây ta chứng minh được E , I ′, N thằng hàng. Tương tự, E , J ′, M thẳng hàng.Ngoài ra thì K M 2 = K H .K E nên ta có: �K M H = �ME H = �J ′I H ′. Mà I ′ J ′//M N , nên M , H , I ′

thẳng hàng.Từ đây dễ thấy: I ≡ I ′. Tương tự: J ≡ J ′. Mà P là trung điểm J ′I ′ ⇒ P là trung điểm I J .Vậy:

−−→PK =−−→

PH +−−→HK =

−→J H +−→

I H +−−→H N +−−→

H M

2=−

−−→M I +−→

N J

2

Cho tam giác ABC nhọn nội tiếp (O), H là trực tâm tam giác. Đường thẳng qua Avuông góc OH cắt BC tại D. K .L là tâm (ADB), (ADC ).

1. Chứng minh A,K ,L,O thuộc một đường tròn gọi là (S).

2. AH cắt lại (S) tại E .F đối xứng với E qua BC . Chứng minh H A = HF.

(Đà Nẵng)

Bài 18

Lời giải

1. Theo giả thiết thìLK ⊥ AD,LO ⊥ AC ⇒ �K LO = �D AC . (4.6)

Do 4ADC có tâm nội tiếp K nên

�AK D = 2 �ABC = �AOC ⇒4K AD ∼4O AC (c.g.c)⇒ �K AD = �O AC ⇒ �K AO = �D AC .(4.7)

Từ (4.6) và (4.7) ⇒ AK LO nội tiếp (dpcm).

2. Gọi M , N ,P là trung diểm của AD, AB , AC .

• AK M N nội tiếp do �AMK = �AN K = 90◦. Lại có �AHO = �ADC do AH vuônggóc DC ,OH vuông góc AD ⇒ �AHO = �ADC = �AMP = �AM N = �AK N = �AKO =�OE H ⇒4OE H cân tại O.

http://thcmn.wordpress.com 129

Page 130: t 2017 - WordPress.com...Bài 5. (B›c Ninh) Cho a, b c¨0 thäa mãn đi•u ki»n ¯ ˘9. Tìm giá trà lîn nh t cıabi”uthøc T ˘ ab 3a¯4b¯5c ¯ bc 3b¯4c¯5a ¯ ca 3c¯4a¯5b

• Đặt AH cắt (O) tại Q ⇒ AE = HQ. Lại có H ,Q;E ,F đối xứng qua BC ⇒ E H =FQ. Vậy AH = HF .

Cho tứ giác ABC D lồi, P là điểm nằm bên trong tứ giác thỏa �PAD = �C AB .M , N đốixứng C qua AB , AD. (C P M), (C P N ) cắt đoạn AB , AD tại S,T.X ,Y tâm (PSC ), (PTC ).Qlà giao của X Y và trung trực AP.

1. Chứng minh AQ là tiếp tuyến đường tròn ngoại tiếp 4AX Y .

2. Tiếp tuyến tại P,C của (PST ), (C ST ) cắt nhau ở G .(PST ), (C ST ) cắt lại AP, ACở U ,V. Chứng minh tâm (AUV ) thuộc AG.

(Đà Nẵng)

Bài 19

Lời giải

Lời giải

1. Vì Q là tâm của (APC ) nên ⇒ (QP,QX ) ≡ (AP, AC )Ta chứng minh: (P X ,PQ) ≡ (Y Q,Y P ) ( mod π) ⇔ (P X ,PC )+(PC ,PQ) ≡ (Y Q,Y P ) ( mod

π) ⇔ π

2−(QP,QY )+(P X ,PC ) ≡ (Y Q,Y P ) ( mod π). Thật vậy, điều trên tương đương

với

(PY ,PC ) ≡ (QP,QY )+ (PC ,P X )( modπ) ⇔ (PY ,PC ) ≡ (AP, AC )+ (PC ,P X ) ( modπ)

⇔ (N P, NC ) ≡ (AP, AC )+ (MC , MP ) ( modπ). (4.8)

Dễ thấy rằng: AP là phân giác �M AN và AM = AC = AN ⇒ (N A, N P ) ≡ (MP, M A)( modπ)

130 http://www.facebook.com/thcmn

Page 131: t 2017 - WordPress.com...Bài 5. (B›c Ninh) Cho a, b c¨0 thäa mãn đi•u ki»n ¯ ˘9. Tìm giá trà lîn nh t cıabi”uthøc T ˘ ab 3a¯4b¯5c ¯ bc 3b¯4c¯5a ¯ ca 3c¯4a¯5b

(4.8)⇔ (N P, NC )+(N A, N P ) ≡ (AP, AC )+(MC , MP )+(MP, M A)( mod π) ⇔ (N A, NC ) ≡(AP, AC )+(MC , M A)( mod π) ⇔ (AP, AC ) ≡ (N A, N D)−(AB , AM) ≡ (AD, AC )−(AD, AP ) ( modπ).Vì điều trên hiển nhiên đúng, nên ta chứng minh được: (P X ,PQ) ≡ (Y Q,Y P )(mod π).Từ đây ta chứng minh được: Q A2 =QP 2 =QX .QY .Từ đây ta suy ra được đpcm.

Cho tam giác ABC nhọn (AB < AC ) có M là trung điểm BC , các đường cao AD,BE ,C Fcắt nhau tại H . Gọi K là trung điểm AH , L là giao điểm EF và AH , N là giao điểmcủa đoạn AM và đường tròn ngoại tiếp 4BC H .

1. Chứng minh rằng 5 điểm A,E , N , H ,F cùng thuộc một đường tròn.

2. Chứng minh rằng �H M A = �LN K .

(Đồng Nai)

Bài 20

Lời giải

http://thcmn.wordpress.com 131

Page 132: t 2017 - WordPress.com...Bài 5. (B›c Ninh) Cho a, b c¨0 thäa mãn đi•u ki»n ¯ ˘9. Tìm giá trà lîn nh t cıabi”uthøc T ˘ ab 3a¯4b¯5c ¯ bc 3b¯4c¯5a ¯ ca 3c¯4a¯5b

Gọi S là giao điểm của EF và BC , SH cắt AM tại N ′, ta chứng minh N ≡ N ′.

• Định lý Brocard cho ta SN ′ ⊥ AM , do đó N ′ ∈ (AHEF ) ⇒ SH .SN ′ = SE .SF = SB.SC ⇒N ′ ∈ (HBC ) ⇒ N ′ ≡ N hay A,E ,F, H , N đồng viên.

• Ta có (AHLD) = −1, K là trung điểm AH ⇒ AK 2 = K H 2 = K L.K D = K N 2 ⇒ K N làtiếp tuyến tại N của (LN D) ⇒ �K N H =�LDN = àH M N .

Gọi O, I lần lượt là tâm đường tròn ngoại tiếp và nội tiếp của 4ABC không đều.Chứng minh rằng �AIO ≤ 900 ⇔ 2BC ≤ AB + AC .

(Đồng Nai)

Bài 21

Lời giải

Gọi D là giao điểm thứ hai của đường thẳng AI với đường tròn ngoại tiếp tam giácABC .

• �B I D = �B AD+�AB I = �C AD+�C B I = �DBC+�C B I = �DB I ⇒ DB = D I . Dễ thấy DB = DC .Vậy có DB = DC = D I .

• Vì ABDC là tứ giác nội tiếp nên theo định lý Ptolemy ta có AD.BC = AB.DC +AC .BD = (AB + AC ).D I ⇒ D I = AD.BC

AB + AC.

• Vì tam giác AOD cân tại O nên �AIO ≤ 900 ⇔ D I ≤ AD

2.

• Từ hai điều trên ta cóAD.BC

AB + AC≤ AD

2⇔ BC

AB + AC≤ 1

2⇔ AB + AC ≥ 2BC .

4ABC nhọn (AB < AC ) có trung tuyến AM . Đường thẳng AM cắt đường trònngoại tiếp 4ABC tại điểm thứ hai D. Đường thẳng AB và C D cắt nhau tại E ,đường thẳng AC và BD cắt nhau tại F . ĐƯờng tròn ngoại tiếp 4ABF cắt đườngtròn ngoại tiếp 4AC E tại điểm thứ hai P . Gọi (S1) là đường tròn đi qua C và tiếpxúc với AB tại A, (S2) là đường tròn đi qua B và tiếp xúc AC tại A. (S1) cắt (S2) tạiđiểm thứ hai Q. Chứng minh 4OPQ vuông.

(Hà Nội)

Bài 22

Lời giải

132 http://www.facebook.com/thcmn

Page 133: t 2017 - WordPress.com...Bài 5. (B›c Ninh) Cho a, b c¨0 thäa mãn đi•u ki»n ¯ ˘9. Tìm giá trà lîn nh t cıabi”uthøc T ˘ ab 3a¯4b¯5c ¯ bc 3b¯4c¯5a ¯ ca 3c¯4a¯5b

Gọi P ′ là giao điểm thứ hai của EF và (ABF ).

• EP ′.EF = E A.EB = EC .ED ⇒ Tứ giác DC F P ′ nội tiếp ⇒ �EBD + �EP ′D = 180o ⇒ Tứgiác EBDP ′ nội tiếp ⇒ �C EP ′ = �P ′BF = �P ′AC ⇒ Tứ giác E AC P ′ nội tiếp ⇒ P ≡ P ′ vàE ,P,F thẳng hàng.

• Để ý rằngE A

EB.

C B

C M.DM

D A= 1 và

F A

FC.

BC

B M.DM

D A= 1 nên

E A

EB= F A

FChay BC //EF .

• �PBD = �DEF = �BC D ⇒ PB là tiếp tuyến của (O). Tương tự, PC cũng là tiếp tuyếncủa (O).

• Gọi Q ′ là hình chiếu vuông góc của O trên AP . Ta có O,Q ′,B ,C ,P cùng thuộcđường tròn đường kính OP ⇒ �ABQ ′ = �ABC − �Q ′PC = �ABC − �APC = �ABC − �AEC =�B AD = �BC D = �C EP = �C AQ ′ ⇒ AC là tiếp tuyến của (ABQ ′), do đó (ABQ ′) ≡ (S2).Tương tự, (ACQ ′) ≡ (S1) ⇒Q ≡Q ′ ⇒4OPQ vuông tại Q.

http://thcmn.wordpress.com 133

Page 134: t 2017 - WordPress.com...Bài 5. (B›c Ninh) Cho a, b c¨0 thäa mãn đi•u ki»n ¯ ˘9. Tìm giá trà lîn nh t cıabi”uthøc T ˘ ab 3a¯4b¯5c ¯ bc 3b¯4c¯5a ¯ ca 3c¯4a¯5b

4ABC nhọn (AB < AC ) có AD là phân giác trong đỉnh A (D ∈ BC ). E là điểm trênđoạn BC sao cho BD = C E . Phân giác ngoài đỉnh A cắt đường thẳng qua D vàvuông góc với BC tại F . Gọi I là trung điểm DF , đường thẳng E I cắt AD tại M ,đường thẳng EF cắt đường thẳng qua M và vuông góc với BC tại K .

1. Đường thẳng AF cắt đường thẳng BC tại P , K D cắt đường tròn đường kínhDF tại L khác D. Chứng minh đường thẳng PL tiếp xúc với đường trònđường kính DF .

2. Chứng minh I là tâm đường tròn nội tiếp 4K BC .

(Hải Phòng)

Bài 23

Lời giải

1. K M cắt BC tại H , ta có M là trung điểm HK và HK //DF nên (DP,DK ,D A,DF ) = 1hay D ALF là tứ giác điều hòa. Do PD tiếp xúc với (I ) và p ∈ AF nên PL tiếp xúcvới (I ).

2. Kẻ các tiếp tuyến K X ,K Y tới (I ), K X cắt BC tại B ′, K Y cắt BC tại C ′ (theo thứ tựtrên đường thẳng là PB ′DC ′).

• Do D X LY là tứ giác điều hòa nên P, X ,Y thẳng hàng.

• Do (I ) là đường tròn nội tiếp và tiếp xúc với các cạnh tam giác K B ′C ′ tạiD,Y , X nên K D,B ′Y ,C ′X đồng quy. Từ đó suy ra (PDB ′C ′) =−1.

134 http://www.facebook.com/thcmn

Page 135: t 2017 - WordPress.com...Bài 5. (B›c Ninh) Cho a, b c¨0 thäa mãn đi•u ki»n ¯ ˘9. Tìm giá trà lîn nh t cıabi”uthøc T ˘ ab 3a¯4b¯5c ¯ bc 3b¯4c¯5a ¯ ca 3c¯4a¯5b

• Ngoài ra với DF là đường kính đường tròn nội tiếp (I ) của 4K B ′C ′ và thìta có tính chất quen thuộc B ′D = C ′E . Vậy các cặp điểm B ,C và B ′,C ′ cùngthỏa mãn hai điều kiện (PDBC ) = (PDB ′C ′) =−1 và BD =C E ,B ′D =C ′E nêndễ dàng suy ra B ≡ B ′,C ≡C ′ hay (I ) nội tiếp 4K BC .

4ABC nhọn (AB < AC ) có trực tâm H , D,E là chân các đường cao kẻ từ B và Cxuống C A, AB . Gọi M là trung điểm BC , F là giao điểm của hai đường thẳng DEvà BC , K là giao điểm thứ hai của AM với đường tròn (C ) ngoại tiếp 4ADE .

1. Chứng minh F, H ,K thẳng hàng.

2. Gọi S là trung điểm MF , T là giao điểm của đường thẳng DE với đườngthẳng qua A và song song với BC . Chứng minh đường thẳng ST tiếp xúc vớiđường tròn (C ).

(Hải Phòng)

Bài 24

Lời giải

1. Gọi giao điểm của đường tròn (ADE) với đường tròn (ABC ) là G. Sử dụng tínhchất tâm đẳng phương của (ABC ), (ADE), (BEDC ), ta suy ra: AG ,ED,BC đồngquy tại F . Từ đây, ta chứng minh được rằng: H , M ,G thẳng hàng ⇒ M H⊥AK ,mà HK⊥AK ⇒ F, H ,K thẳng hàng.

http://thcmn.wordpress.com 135

Page 136: t 2017 - WordPress.com...Bài 5. (B›c Ninh) Cho a, b c¨0 thäa mãn đi•u ki»n ¯ ˘9. Tìm giá trà lîn nh t cıabi”uthøc T ˘ ab 3a¯4b¯5c ¯ bc 3b¯4c¯5a ¯ ca 3c¯4a¯5b

2. Gọi giao điểm của AM với DE là X , do ADK E là tứ giác điều hòa ⇒ F (M X K A) =−1 ⇒ F (C T M A) =−1. Mà AM//FC ⇒ T là trung điểm AM . Từ đây dễ chứng minh:S,K ,T thẳng hàng. Mà ta lại có: �F K S = �K F S = �H AK ⇒ ST là tiếp tuyến của (C )

4ABC nhọn không cân nội tiếp đường tròn (O). Gọi H là trực tâm 4ABC , A′,B ′,C ′

lần lượt là chân các đường vuông góc kẻ từ A,B ,C xuống BC ,C A, AB . Gọi A1,B1,C1

là các điểm trên (O) sao cho A A1//BC ,BB1//C A,CC1//AB . A′1,B ′

1,C ′1 là các điểm

trên (O) sao cho A1 A′1//A A′, B1B ′

1//BB ′,C1C ′1//CC ′.

1. Chứng minh các đường tròn ngoại tiếp các tam giác O A1 A′1,OB1B ′

1,OC1C ′1

cùng đi qua một điểm K khác O.

2. Chứng minh OK .OH = abc

p, trong đó a,b,c là độ dài 3 cạnh của 4ABC và p

là chu vi tam giác A1B1C1.

(Hà Tĩnh)

Bài 25

Lời giải

1. A1 A′1 ⊥ BC ⇒ A1 A′

1 ⊥ B ′1C ′

1. Chứng minh tương tự suy ra A1 A′1,B1B ′

1,C1C ′1 đồng

quy tại H ′ là trực tâm của tam giác A′1B ′

1C ′1. Mặt khác H ′A1.H ′A′

1 = H ′B1.H ′B ′1 =

H ′C1.H ′C ′1 nên OH ′ là trục đẳng phương của 3 đường tròn (O A1 A′

1), (OB1B ′1), , (OC1C ′

1).Do đó ba đường tròn (O A1 A′

1), (OB1B ′1), (OC1C ′

1) cùng đi qua điểm K khác O.

2. Dễ thấy tam giác A′1B ′

1C ′1 là ảnh của tam giác ABC qua phép đối xứng tâm O nên

a,b,c cũng là độ dài 3 cạnh của tam giác A′1B ′

1C ′1 và OH =OH ′.

Gọi p ′ là chu vi của tam giác A2B2C2. Vì 4A2B2C2 là ảnh của tam giác A′1B ′

1C ′1 qua

phép vị tự tâm H tỉ số1

2nên p ′ = P

2. Do đó ta chỉ cần chứng minh OK .OH ′ = abc

2p ′ .

136 http://www.facebook.com/thcmn

Page 137: t 2017 - WordPress.com...Bài 5. (B›c Ninh) Cho a, b c¨0 thäa mãn đi•u ki»n ¯ ˘9. Tìm giá trà lîn nh t cıabi”uthøc T ˘ ab 3a¯4b¯5c ¯ bc 3b¯4c¯5a ¯ ca 3c¯4a¯5b

Dễ thấy O A′1 ⊥ B2C2 nên SOB2 A′

1C2= R.B2C2

2trong đó R là bán kính của (O).

Chứng minh tương tự và cộng lại suy raabc

4R= S A′

1B ′1C ′

1= R.p ′

2⇒ abc

2p ′ = R2 Lại có:àO A1 A′1 = àO A′

1 A1 =àOK A1 nên O A1 là tiếp tuyến của (H ′A1K ) ⇒OK .OH ′ =O A21 = R2

Do đó ta có điều phải chứng minh

4ABC nhọn nội tiếp đường tròn (O), E là trung điểm cung BC không chứa A. GọiD là giao điểm AE và BC . P,Q lần lượt là hai điểm di động trên đoạn AD sao cho�ABQ = �DBP và Q nằm giữa A,P . Lấy điểm S sao cho QS//AO và DS ⊥ AO. Gọi Mlà trung điểm BC , N là điểm đối xứng của M qua AE , R là hình chiếu vuông góccủa Q lên BC .

1. Chứng minhM N

MR= 2ME

QE.

2. Chứng minh đường thẳng qua P vuông góc SM đi qua một điểm cố địnhkhi P,Q thay đổi.

(Hà Tĩnh)

Bài 26

Lời giải

http://thcmn.wordpress.com 137

Page 138: t 2017 - WordPress.com...Bài 5. (B›c Ninh) Cho a, b c¨0 thäa mãn đi•u ki»n ¯ ˘9. Tìm giá trà lîn nh t cıabi”uthøc T ˘ ab 3a¯4b¯5c ¯ bc 3b¯4c¯5a ¯ ca 3c¯4a¯5b

1. Gọi J là hình chiếu của M trên AE . Do tính đối xứng nên M N = 2M J . Do đó bài

toán quy về chứng minhM J

MR= E M

EQ. Thật vậy, QR J M nội tiếp nên �JRM = �EQM .

Lại có �RM J = �DE M . Do đó 4RM J v4QE M suy raM J

MR= E M

EQsuy ra điều phải

chứng minh.

2. Kẻ đường cao AL . Gọi U là hình chiếu của P trên BC

• AL.AO đẳng giác nên dễ suy ra QD là phân giác của �RQS. Mà DS ⊥QS nênDS = DR và R,S đối xứng qua A,E .

• Gỉa sử E N cắt AL tại T . Do AE là phân giác của �T AO và �T EO nên dễ dàngsuy ra O,T đối xứng qua A,E suy ra T cố định.

• Bây giờ ta sẽ chứng minh PT ⊥ SM ⇔ T S2−T M 2 = PS2−P M 2. Nhưng vì PS =PR,T S =OR (phép đối xứng trục AE) nên chỉ cần chứng minh OR2−T M 2 =PR2−P M 2 ⇔OM 2+RM 2−T M 2 = RU 2−MU 2. Thật vậy, ⇔ T M 2−OM 2 = RM 2+MU 2−RU 2 = (RM −RU )(RM +RU )+MU 2 = MU (RM +RU +MU ) = 2MR.MU .

Chứng minh tương tự phần a ta cũng có:M J

MU= ME

PE. Do đó 2MU .MR =

2M J 2.EP.EQ

ME 2. Bằng cộng góc dễ dàng chứng minh được EB là tiếp tuyến

của (BPQ) nên 2MU .MR = 2.M J 2.EB 2

ME 2là số cố định nên ta chỉ cần chứng

minh đẳng thức trên khi P ≡Q là tâm nội tiếp của tam giác.

• Gọi tâm nội tiếp của tam giác là I và tiếp điểm trên BC là X Cần chứngminh

T M 2 −OM 2 = 2M X 2,

mà T M 2 −OM 2 = T M 2 −T D2 +OD2 −OM 2 = ML2 −DL2 +DM 2 = DM(ML +DL+MD) = 2MD.ML nên chỉ cần chứng minh

MD.ML = M X 2 ⇔ ML

MD= M X 2

MD2.

Mặt khácML

MD= E A

ED

vàM X 2

MD2= E I 2

ED2

nên chỉ cần chứng minhE I 2 = E A.ED,

một tính chất quen thuộc.

Vậy ta có đường thẳng qua P vuông góc với SM đi qua điểm T cố định.

138 http://www.facebook.com/thcmn

Page 139: t 2017 - WordPress.com...Bài 5. (B›c Ninh) Cho a, b c¨0 thäa mãn đi•u ki»n ¯ ˘9. Tìm giá trà lîn nh t cıabi”uthøc T ˘ ab 3a¯4b¯5c ¯ bc 3b¯4c¯5a ¯ ca 3c¯4a¯5b

Cho 4ABC có tâm đường tròn nội tiếp I . Gọi M , N ,P lần lượt là các điểm nằmtrên các cạnh AB ,BC ,C A sao cho AN − AP = BP −B M = C P −C M . Gọi X ,Y , Z lầnlượt là tâm đường tròn nội tiếp của 4AN P,4BP M ,4C M N . Chứng minh rằng I làtâm đường tròn nội tiếp của 4X Y Z .

(Tp. HCM)

Bài 27

Lời giải

Gọi D,E ,F lần lượt là hình chiếu của I lên BC ,C A, AB ⇒ I D = I E = I F và AE = AF,BD =BF,C D =C E .

• AN −AP = BP −B M =C M −C N ⇒ (AE +E N )−(AF −PF ) = (BF +F P )−(BD−MD) =(C D+DM)−(C E−E N ) ⇒ E N+PF = PF+MD = MD+E N ⇒ E N = MD = PF4I E N =4I F P =4I DM ⇒ I N = I P = I M .

• Tứ giác AF I E nội tiếp ⇒ Tứ giác API N nội tiếp. Ta có I là điểm chính giữa củacung P N không chứa A của đường tròn (API N ) và X là tâm đường đường trònnội tiếp 4AP N ⇒ I P = I N = I X . Chứng minh tương tự ta có I P = I M = I Y vàI M = I N = I Z ⇒ I X = I Y = I Z ⇒ I là tâm đường tròn ngoại tiếp tam giác X Y Z .

Đường tròn nội tiếp 4ABC lần lượt tiếp xúc với BC ,C A, AB tại A1,B1,C1.

1. Chứng minh các đường thẳng A A1,BB1,CC1 đồng quy tại một điểm T .

2. Đường tròn ω1 đi qua T tiếp xúc với C A,C B tại B2, A3. Các điểm C2, A2,B3,C3

xác định tương tự. Chứng minh sáu điểm A2,B2,C2, A3,B3,C3 cùng thuộcmột đường tròn.

(Tp. HCM)

Bài 28

Lời giải

http://thcmn.wordpress.com 139

Page 140: t 2017 - WordPress.com...Bài 5. (B›c Ninh) Cho a, b c¨0 thäa mãn đi•u ki»n ¯ ˘9. Tìm giá trà lîn nh t cıabi”uthøc T ˘ ab 3a¯4b¯5c ¯ bc 3b¯4c¯5a ¯ ca 3c¯4a¯5b

1. Từ tính chất của đường tròn nội tiếp tam giác ta suy raA1B

A1C.B1C

B1 A.C1 A

C1B= 1 nên

A A1,BB1,CC1 đồng quy.

2. Gọi I là tâm đường tròn nội tiếp tam giác ABC .

• Do C là giao điểm của hai tiếp tuyến chung ngoài của hai đường tròn(A1B1C1) và (ω1) nên C là tâm vị tự ngoài của hai đường tròn này. Từ đó suyra tồn tại phép vị tự tâm C biến đường tròn (A1B1C1) thành đường tròn (ω2).Khi đó phép vị tự này cũng biến 4A1B1C1 thành 4A3B2T. Từ đó suy đượcA1B1//A3B2,B1C1//B2T,C1 A1//T A3. Tương tự, A1B1//A2T,B1C1//T C3,C1 A1//C3 Avà A1B1//T B3,B1C1//B3C2,C1 A1//C2T . Từ đó ta suy ra các đường thẳng A2B3,B2C3,C2 A3 cùng đi qua T và lần lượt song song với A1B1,B1C1,C1 A1.

• Tiếp theo để ý rằng đường phân giác C I của �AC B là đường trung trực củaA1B1 mà A2B3//A1B1//A3B2 nên C I cũng là đường trung trực của A2B3 vàA3B2, B I là trung trực của C2 A3 và C3 A2 ⇒ I A2 = I B2 = IC2 = I A3 = I B3 =IC3 ⇒ A2, A3,B2,B3,C2,C3 cùng thuộc một đường tròn.

140 http://www.facebook.com/thcmn

Page 141: t 2017 - WordPress.com...Bài 5. (B›c Ninh) Cho a, b c¨0 thäa mãn đi•u ki»n ¯ ˘9. Tìm giá trà lîn nh t cıabi”uthøc T ˘ ab 3a¯4b¯5c ¯ bc 3b¯4c¯5a ¯ ca 3c¯4a¯5b

Cho tam giác ABC nhọn nội tiếp đường tròn (O) và P là một điểm nằm trên cungnhỏ BC . Tiếp tuyến tại B ,C cắt nhau tại T . Đường thẳng qua O và vuông góc vớiPT cắt C A, AB lần lượt tại E ,F . Hai đường thẳng PE ,PF cắt đường tròn (O) lầnlượt tại M , N khác P . Lấy K ,L sao cho �K AC = �K N P = �L AB =�LMP = 90o.

1. Chứng minh rằng �BQF = �K AB với Q là giao của EF với PT .

2. Chứng minh rằng K B và LC vắt nhau tại 1 điểm thuộc (O).

(Hoà Bình)

Bài 29

Lời giải

1. �OQT = �OBT = �OC T , nên O,B ,C ,T,Q cùng thuộc 1 đường tròn ⇒ �FQB = �OC B =90o − �B AC = �K AB .

2. Gọi I ,G , H lần lượt là giao điểm của các cặp sau: N K và ML, AK và (O), AL và (O).Khi đó dễ chứng minh được rằng I ,O,P thẳng hàng; G ,O,C thẳng hàng; H ,O,Bthẳng hàng.Áp dụng định lý Pascal cho 3 bộ điểm sau:(N , A, M ,B ,P,C ); (A, I ,B , M , H ,P ); (A, I ,C , N ,G ,P ), ta có được K ,F,Q,O,E ,L thẳnghàng.Từ kết quả câu 1 có K AQB là tứ giác nội tiếp nên

(AQ; AB) ≡ (K L;K B)( modπ). (4.9)

Tương tự:(AC ; AQ) ≡ (LC ;LK )( modπ). (4.10)

Từ (4.9) và (4.10) ⇒ (LC ;K B) = (AC ; AB)( modπ) và ta có đpcm.

http://thcmn.wordpress.com 141

Page 142: t 2017 - WordPress.com...Bài 5. (B›c Ninh) Cho a, b c¨0 thäa mãn đi•u ki»n ¯ ˘9. Tìm giá trà lîn nh t cıabi”uthøc T ˘ ab 3a¯4b¯5c ¯ bc 3b¯4c¯5a ¯ ca 3c¯4a¯5b

Cho hai đường tròn (O1) và (O2) cắt nhau tại A,B . C D là tiếp tuyến chung của haiđường tròn (O1) và (O2) với C ∈ (O1) , D ∈ (O) , và B gần C D hơn A.

1. Gọi E là giao điểm của BC và AD, F là giao điểm của DB và AC . Chứng minhrằng EF //C D.

2. Gọi N là giao điểm của AB và EF . Lấy K trên C D sao cho �B AC = �D AK . Chứngminh rằng K E = K F .

(Hoà Bình)

Bài 30

Lời giải

1. Để ý rằng �BC D = �C AB và �BDC = �B AD, mà �BC D + �BDC + �C BD = 180o ⇒ �E AF +�EBF = 180o ⇒ Tứ giác AEBF nội tiếp ⇒ �EF B = �B AE = �BDC ⇒ EF ⊥C D.

2. Gọi B ′ là điểm đối xứng với B qua C D, AN cắt C D tại M thì MC 2 = MB.M A =MD2 ⇒ M là trung điểm của C D và N là trung điểm của EF .

• Giả sử K ′ là điểm thuộc C D sao cho K ′N ⊥ EF tại N . Xét tứ giác toàn phầnAF BEC D có (AB N M) = −1 ⇒ K ′(AB N M) = −1. Mà K ′N ⊥ K ′M ⇒ K ′M làphân giác ngoài của �BK ′A. Lại có K ′M là phân giác �BK B ′ ⇒ A,K ′,B thẳnghàng.

• �C B ′D = �C BD = 180o − �C AD ⇒ �B ′C D = �B ′AD ⇒ Tứ giác AC B ′D nội tiếp. Mà�B ′C D = �BC D = �B AC và �B ′C D = �K ′AD ⇒ �B AC = �K ′AD ⇒ K ≡ K ′ ⇒ K N ⊥ EF ⇒K E = K F .

Tứ giác ABC D nội tiếp đường tròn (O). Đường thẳng qua C cắt các tia đối của tiaB A,D A lần lượt tại M , N . Chứng minh

4S ABC

S AM N≤

(BD

AC

)2

.

(Khánh Hoà)

Bài 31

Lời giải

142 http://www.facebook.com/thcmn

Page 143: t 2017 - WordPress.com...Bài 5. (B›c Ninh) Cho a, b c¨0 thäa mãn đi•u ki»n ¯ ˘9. Tìm giá trà lîn nh t cıabi”uthøc T ˘ ab 3a¯4b¯5c ¯ bc 3b¯4c¯5a ¯ ca 3c¯4a¯5b

Qua C kẻ C F //AB , F ∈ AD.

• �F AC = �DBC , �AC F = �BC D ⇒4AFC =4BC D ⇒ SBC D

S AFC=

(BD

AC

)2

.

• S AM N = 1

2.AM .AN . sin �B AD ,S AFC = 1

2.AF.AC . sin �B AD.

•4S ABC

S AM N≤

(BD

AC

)2

⇔ AM .AN ≥ 4AF.AC ⇔(

AM

FC

).AN ≥ 4AF ⇔

(AN

F N

).AN ≥ 4AF ⇔

AN 2 ≥ 4AF.F N ⇔ (AF +F N )2 ≥ 4AF.F N , đúng theo BĐT AM - GM.

4ABC nhọn có AM ,B N ,C P là các trung tuyến. Gọi R và r lần lượt là bán kínhđường tròn ngoại tiếp và đường tròn nội tiếp 4ABC . Chứng minh

AM +B N +C P ≤ 4R + r.

(Khánh Hoà)

Bài 32

http://thcmn.wordpress.com 143

Page 144: t 2017 - WordPress.com...Bài 5. (B›c Ninh) Cho a, b c¨0 thäa mãn đi•u ki»n ¯ ˘9. Tìm giá trà lîn nh t cıabi”uthøc T ˘ ab 3a¯4b¯5c ¯ bc 3b¯4c¯5a ¯ ca 3c¯4a¯5b

Gọi M , N ,P lần lượt là trung điểm của BC ,C A, AB . Theo bđt tam giác ta có:

AM +B N +C P ≤ (O A+OM)+ (OB +ON )+ (OC +OP ) = 3R +R(cos A+ cosB + cosC ).

Mặt khác dễ chứng minh

cos A+ cosB + cosC = 1+ r

R

nên AM +B N +C P ≤ 3R +R(1+ r

R) = 4R + r. Dấu "=" xảy ra ⇔4ABC đều.

Cho tam giác nhọn ABC có đường cao AH , trực tâm K . Đường thẳng BK cắtđường tròn đường kính tại D,E (BD < BE). Đường thẳng C K cắt (AB) tại F,G (C F <CG). Và (D HF ) cắt BC tại điểm thứ hai là P .

1. Chứng minh rằng các điểm G , H ,P,E cùng thuộc một đường tròn.

2. Chứng minh rằng các đường thẳng BF,C D,PK đồng quy.

(Lào Cai)

Bài 33

Hướng dẫn

144 http://www.facebook.com/thcmn

Page 145: t 2017 - WordPress.com...Bài 5. (B›c Ninh) Cho a, b c¨0 thäa mãn đi•u ki»n ¯ ˘9. Tìm giá trà lîn nh t cıabi”uthøc T ˘ ab 3a¯4b¯5c ¯ bc 3b¯4c¯5a ¯ ca 3c¯4a¯5b

1. Áp dụng hệ thức lượng và hệ thức bằng nhau của hai dường cao, dễ chứng minhđược (KCGF ) = (K BED) =−1Từ tính chất đồng quy của hàng điểm điều hòa, ta suy ra: GE ,DF,BC đồng quytại X . Ta có: KG .K F = K A.K H = K D .K E ⇒GDF E là tứ giác nội tiếp.Kết hợp hai điều trên, ta chứng minh được rằng: XG .X E = X H .X P ⇒ G HPE làtứ giác nội tiếp (đpcm)

2. E (KCGF ) = −1 ⇒ (X PBC ) = −1 Áp dụng tính chất của hàng điểm điều hòa cho4K BC , ta có điều phải chứng minh.

Cho 4ABC nhọn nội tiếp (O) với I là tâm nội tiếp tam giác. Đường tròn đi qua Ctiếp xúc với AI tại I cắt AC tại E và cắt (O) tại H (E , H 6=C ).

1. Chứng minh E H đi qua trung điểm của AI .

2. Đường tròn đi qua B tiếp xúc với AI tại I cắt AB tại F và cắt (O) tại G (G ,F 6=B). Chứng minh rằng hai đường tròn (E I F ) và (G I H) tiếp xúc nhau.

(Lạng Sơn)

Bài 34

Lời giải

1. Ta sẽ chứng minh rằng AI cũng là tiếp tuyến tại A của (AHE). Thật vậy, �H I A =�HC I = 1

2�AC B ⇒ �H IC = �AIC − �AI H = 90o + 1

2�ABC − 1

2�AC B . Mặt khác �HEC = 180o −

http://thcmn.wordpress.com 145

Page 146: t 2017 - WordPress.com...Bài 5. (B›c Ninh) Cho a, b c¨0 thäa mãn đi•u ki»n ¯ ˘9. Tìm giá trà lîn nh t cıabi”uthøc T ˘ ab 3a¯4b¯5c ¯ bc 3b¯4c¯5a ¯ ca 3c¯4a¯5b

�H IC = 90o − 1

2�ABC + 1

2�AC B ⇒�AE H = �AEC −�HEC = 90o − 1

2�ABC − 1

2�AC B = 1

2�B AC =�I AH . Vậy I A cũng là tiếp tuyến của (AHC ). Tới đây, theo tính chất phương tích,

nếu gọi M là giao điểm của HE và AI thì M là trung điểm AI .

2. Ta sẽ chứng minh AI là đường nối tâm của (E I F ) và (G I H). Gọi giao điểm thứhai của AI và (AEF ) là J thì �E JF = 180o − �B AC ,

mà AI lại là phân giác trong góc �E AF nên J là trung điểm cung EF của (AEF )hay JE = JF . Mà �E I F =

�ABC + �AC B

2= 1

2(180o − �B AC )

và I J không phải là phân giác góc �E I F nên J là tâm (E I F ).Gọi S và T lần lượt là tâm của (B IG) và (C I H). Vì EF và G H là hai đường đối song

nên �EF M = �G H M . Ta có G I S = 90o −�GB I = 90o − �I F M = 90o − 1

2�ABC − �JF M = 90o −

1

2�ABC−�JF E+�MF E = 90o− 1

2�ABC− 1

2�B AC+ �G H M = 1

2�AC B+ �HGM = �M H I + �M HG =�G H I ⇒ ST là tiếp tuyến của (G I H) tại I . hay tâm của (G I H) nằm trên AI .

Cho 4ABC nhọn nội tiếp (O). Các đường cao AD,BE ,C F cắt nhau tại H (D ∈BC ,E ∈ C A,F ∈ AB). Gọi M là trung điểm của BC . 2 đường tròn (DEF ) và (HBC )cắt nhau tại X và Y

1. Chứng minh AX = AY .

2. Gọi R là trung điểm của X Y . AR cắt H M tại S. Chứng minh tứ giác HDSRnội tiếp.

(Lạng Sơn)

Bài 35

Lời giải

146 http://www.facebook.com/thcmn

Page 147: t 2017 - WordPress.com...Bài 5. (B›c Ninh) Cho a, b c¨0 thäa mãn đi•u ki»n ¯ ˘9. Tìm giá trà lîn nh t cıabi”uthøc T ˘ ab 3a¯4b¯5c ¯ bc 3b¯4c¯5a ¯ ca 3c¯4a¯5b

1. Gọi O9 là tâm đường tròn Euler của 4ABC , O′ đối xứng với O qua BC . Khi đódễ chứng minh được O′ là tâm đường tròn ngoại tiếp 4N HC và AOO′H là hìnhbình hành ⇒ A,O9,O′ thẳng hàng. Vậy hiển nhiên AX = AY .

2. Chúng tôi xin nêu hai cách chứng minh cho câu này.

Cách 1: AH .AD = AF.AB vàAS

SO′ =AH

MO′ =AH

MO= 2 ⇒ AS

AO′ =2

3⇒ AS

AO9= 4

3.

– RO′ = O′O29 +R2 − R2

4

2O′O9= AO2

9 + 3R2

4

2AO9⇒ AR = AO′−RO′ = 2AO9−

AO29 + 3

4 R2

2AO9⇒

AS.AR = 4

3AO9.

3AO29 − 3R2

4

2AO9= 2AO2

9 −R2

2.

– AO29 −

R2

4= AF.AN = 1

2AF.AB (N là trung điểm AB) nên 2AO2

9 −R2

2=

AF.AB = AH .AD ⇒ Tứ giác HDSR nội tiếp.

Cách 2: Từ câu 1 dễ thấy R, A,O′,O9 thằng hàng. Áp dụng tích chất đẳng phương cho4 đường tròn (DEF ), (HBC ), (HDM), (AF E), ta chứng minh được: EF, X Y ,BCđồng quy tại K .Cho H M giao AK tại G, dễ chứng minh được rằng: AK⊥H M tại G ,G ∈ (ABC ).⇒ KG .K A = K X .K Y . Bằng tính toán, ta chứng minh được: AX 2 = AG .AK =AR.AS = AH .AD ⇒ HDSR là tứ giác nội tiếp.(đpcm)

Từ một điểm M tùy ý trong tam giác ABC , các đường thẳng M A, MB , MC lần lượtcắt BC ,C A, AB tại A1,B1,C1. Chứng minh rằng

M A1

A A1+ MB1

BB1+ MC1

CC1= 1.

(Long An)

Bài 36

Hướng dẫn

Bằng biến đổi tỉ số, ta có:M A1

A A1= S A1MC

S A A1C= S A1MB

S A A1B= SMBC

S ABC. Tương tự với

MB1

BB1và

MC1

CC1.

Cộng ba tỉ số theo diện tích đó sẽ ra điều phải chứng minh.

http://thcmn.wordpress.com 147

Page 148: t 2017 - WordPress.com...Bài 5. (B›c Ninh) Cho a, b c¨0 thäa mãn đi•u ki»n ¯ ˘9. Tìm giá trà lîn nh t cıabi”uthøc T ˘ ab 3a¯4b¯5c ¯ bc 3b¯4c¯5a ¯ ca 3c¯4a¯5b

Cho tam giác nhọn ABC không cân nội tiếp đường trònω tâm O. Một đường trònω, đi qua B ,C cắt các cạnh AB , AC lần lượt ở E ,F (E ,F 6= A). Đường tròn ngoại tiếptam giác AEF cắt lại đường trònω tại K (A 6= K ). K E ,K F lần lượt cắt lại đường trònω tại Q,P (P,Q 6= K ). Gọi T là giao điểm của BQ và C P ; M , N lần lượt là trung điểmBF,C E .

1. Chứng minh rằng A,O,T thẳng hàng.

2. Chứng minh rằng K A là tiếp tuyến của đường tròn ngoại tiếp tam giácAM N .

(Nam Định)

Bài 37

Hướng dẫn

Ta có: �AQC = �ABC = �AF E = �ACQ ⇒ AQ =QC .Mặt khác: �BKQ = �PKC ⇒ BQC P là hình thang cân nội tiếp (O) ⇒ TQ = T C .Từ đây, suy ra A,O,T cùng năm trên đường trung trực đoạn QC ⇒ đpcm.

148 http://www.facebook.com/thcmn

Page 149: t 2017 - WordPress.com...Bài 5. (B›c Ninh) Cho a, b c¨0 thäa mãn đi•u ki»n ¯ ˘9. Tìm giá trà lîn nh t cıabi”uthøc T ˘ ab 3a¯4b¯5c ¯ bc 3b¯4c¯5a ¯ ca 3c¯4a¯5b

4ABC nhọn nội tiếp đường tròn (O) ngoại tiếp đường tròn (I ). Gọi P là trungđiểm cung BC không chứa A của (O), J là điểm đối xứng với I qua O. Tiếp tuyếntại I của đường tròn ngoại tiếp 4I BC cắt BC tại M , H là hình chiếu của M trênOI . Gọi D là trung điểm BC và K là giao điểm thứ hai của I D với đường tròn ngoạitiếp 4OD H .

1. Chứng minh rằng 4JP M vuông ở J .

2. Chứng minh H , A,K thẳng hàng.

(Nam Định)

Bài 38

Lời giải

1. PQ là đường kính (O).Gọi giao điểm của AI và BC là F .Dễ chứng minh được tích sau: I F.AP = AI .I P . Ngoài ra thì 4M I F ∼ 4PAQ ⇒M I

I F= PA

AQ. Kết hợp hai điều trên suy ra:

AQ

I P= AI

M I⇒ ∆M I P ∼ ∆I AQ. Từ đây dễ

chứng minh bằng cộng góc: ∆MP J vuông tại P .

2. M A cắt (O) tại T .

• Bằng cộng góc, chứng minh được �AQ J = 90o ⇒Q J//AP .

• M I 2 = MB.MC = MT.M A ⇒ I T ⊥ M A.

• 4Q AT ∼4PI T ⇒ Q A

AT= PI

I T= Q J

I T⇒4AQ J ∼4AT I ⇒ �AJQ = �AI T = �AM I hay

J AI = �I M A ⇒ M A⊥AJ . Từ đây A, J ,P, H , M cùng thuộc một đường tròn ⇒�K HO = �K DO = �AP J = �AH J hay A,K , H thẳng hàng (đpcm)

http://thcmn.wordpress.com 149

Page 150: t 2017 - WordPress.com...Bài 5. (B›c Ninh) Cho a, b c¨0 thäa mãn đi•u ki»n ¯ ˘9. Tìm giá trà lîn nh t cıabi”uthøc T ˘ ab 3a¯4b¯5c ¯ bc 3b¯4c¯5a ¯ ca 3c¯4a¯5b

Cho hai đường tròn (O1) và (O2) cắt nhau tại A và B . Một đường thẳng qua B cắthai đường tròn (O1), (O2) lần lượt tại C và D. Gọi M là trung điểm C D. Đường thẳngAM cắt (O1) tại điểm thứ hai là K (K và C khác phía so với AB), cắt đường tròn(O2) tại điểm thứ hai P . Đường thẳng (d) qua M vuông góc với O1M cắt đườngthẳng AC tại Q và cắt đường thẳng K B tại R.

1. Gọi I , J lần lượt là giao điểm củaAB ,C K với đường thẳng (d). Chứng minhM là trung điểm I J vàRQ.

2. Chứng minh đường thẳng PQ đi qua một điểm cố định.

(Nghệ An)

Bài 39

Hướng dẫn

1. Hiển nhiên đúng theo định lý con bướm.

2. Gọi N là giao điểm thứ 2 của BC với (AC P ) ≡ (O3).

• MB .MD = MP .M A = MC .M N =−MD .M N ⇒ M là trung điểm của B N .

• MK .M A = MB .MC =−M N .MC =−MP .M A ⇒ M là trung điểm của PK .

150 http://www.facebook.com/thcmn

Page 151: t 2017 - WordPress.com...Bài 5. (B›c Ninh) Cho a, b c¨0 thäa mãn đi•u ki»n ¯ ˘9. Tìm giá trà lîn nh t cıabi”uthøc T ˘ ab 3a¯4b¯5c ¯ bc 3b¯4c¯5a ¯ ca 3c¯4a¯5b

• Xét phép đối xứng DM : K 7→ P,B 7→ N ,C 7→ D ⇒ (O1) 7→ (P N D) ≡ (O4) ⇒O1,O4, M . Mà MQ ⊥ O1O4,PM/(O1) = PM/(O4) nên MQ là trục đẳng phươngcủa (O1) và (O4). Xét 3 đg tròn (O1), (O3), (O4) có 3 trục đẳng phương lần lượtlà AC , MQ, N P nên theo định lý về tâm đẳng phương thì AC , MQ, N P đồngquy hay N ,P,Q.

• Gọi T là giao điểm thứ 2 của PQ với (O2). Bằng biến đổi góc ta suy ra BT làtt của (O1) tại B ⇒ T cố định. Vậy PQ đi qua điểm T cố định.

4X Y Z đều, các đỉnh X ,Y , Z lần lượt nằm trên các cạnh BC ,C A, AB của 4ABCnhọn. Chứng minh tâm đường tròn nội tiếp 4ABC nằm trong 4X Y Z .

(Nghệ An)

Bài 40

Lời giải

Ta chứng minh rằng tâm nội tiếp của 4ABC nằm trong đường tròn nội tiếp 4X Y Z . Kíhiệu d(U ,W V ) là khoảng cách giữa điểm U và đường thẳng W V.O là tâm đường trònnội tiếp 4X Y Z và r,r ′,R ′ lần lượt là bán kính đường tròn nội tiếp tam giác ABC , X Y Zvà đường tròn ngoại tiếp tam giác X Y Z . Khi đó R ′ = 2r ′ và điều cần chứng minh làOI ≤ r ′.

• Nếu O ≡ I , ta có ngay điều cần chứng minh.

http://thcmn.wordpress.com 151

Page 152: t 2017 - WordPress.com...Bài 5. (B›c Ninh) Cho a, b c¨0 thäa mãn đi•u ki»n ¯ ˘9. Tìm giá trà lîn nh t cıabi”uthøc T ˘ ab 3a¯4b¯5c ¯ bc 3b¯4c¯5a ¯ ca 3c¯4a¯5b

• Giả sử O 6= I . Đường tròn nội tiếp tam giác ABC tiếp xúc với các cạnh BC ,C A, ABtại lần lượt là A1,B1,C1. Các đường thẳng I A1, I B1, IC1 chia mặt phẳng thằng 6góc nhọn, mỗi góc chứa một trong các điểm A1,B1,C1 trên biên của nó.Giả sử rằng O nằm trong góc xác đỉnh bởi các đường thằng I A1.I B1. Gọi A′,B ′

lần lượt là hình chiếu của O lên các đường thằng I A1, I B1.Ta có:

OX = R ′ ⇒ d(O,BC ) ≤ R ′.O A′//BC ⇒ d(A′,BC ) = A′I + r = d(O,BC ) ≤ R ′ ⇒ A′I ≤ R ′− r.

Mặt khác đường tròn nội tiếp 4Z Y X nằm trong 4ABC , suy ra

r − I B ′ = B ′B1 = d(O, AC ) ≥ r ′ ⇒ I B ′ ≤ r − r ′.

Cuối cùng I A′OB ′ là tứ giác nội tiếp, suy ra_

A′OB ′<_

OB ′I⇒_

A′O +_

OB ′<_

OB ′ +_

B ′I⇒_

A′O<_

B ′I⇒ A′O < B ′I .

Vậy suy ra OI ≤ I A′+ A′O < I A′+ I B ′ ≤ R ′− r + r − r ′ = r ′ (đpcm).

Cho 4ABC . Đường tròn (O) đi qua A và C cắt các cạnh AB ,BC tại K , N . Đườngtròn (K B N ) cắt đường tròn (ABC ) tại B và M . Tính �B MO.

(Ninh Bình)

Bài 41

Lời giải

Gọi D là giao điểm thứ hai của MK và đường tròn tâm O.

• (B M ,B N ) = (K M ,K N ) = (C D,C N ) ⇒ B M//C D.

• (DC ,BK ) = (AC , AK ) = (MC , MB) = (C M ,C D) ⇒ MC = MD, mà OC = OD ⇒ OM làtrung trực của C D ⇒OM ⊥C D ⇒OM ⊥ B M ⇒ �B MC = 90o.

152 http://www.facebook.com/thcmn

Page 153: t 2017 - WordPress.com...Bài 5. (B›c Ninh) Cho a, b c¨0 thäa mãn đi•u ki»n ¯ ˘9. Tìm giá trà lîn nh t cıabi”uthøc T ˘ ab 3a¯4b¯5c ¯ bc 3b¯4c¯5a ¯ ca 3c¯4a¯5b

Cho 4ABC và điểm (O) nằm trog 4ABC Đường thẳng d1 đi qua O song song vớiBC lần lượt cắt AB , AC tại J ,G. Đường thẳng d2 đi qua O song song với C A lần lượtcắt BC ,C A tại F, J . Đường thẳng d3 đi qua O song song với AB lần lượt cắt C A,C Btại H ,E . Dựng các hình bình hành OE A1F,OGB1H ,OIC1 J . Chứng minh rằng cácđường thẳng A A1,BB1,CC1 đồng quy.

(Ninh Bình)

Bài 42

Lời giải

Gọi M là giao điểm của A A1 và BC , N là giao điểm của BB1 và C A, P là giao điểm củaCC1 và AB , D là giao của AO và BC .

• E A1//AC ⇒ E M

MC= E A1

AC= OF

AC= DO

D A. Tương tự,

F M

MB= DO

D A⇒ E M

MC= F M

MB⇒ MB

MC=

F M

E M= F M +MB

E M +MC= F B

EC. Tương tự,

NC

N A= HC

G A.

• 4AG J ∼ 4HC E ⇒ HC

G A= EC

G J⇒ NC

N A= EC

G J. Tương tự,

PA

PB= J A

I B= G J

BF. Do đó

MB

MC.NC

N A.PA

PB= 1 ⇒ MB

MC.NC

N A.PA

PB=−1 ⇒ A A1,BB1,CC1 đồng quy.

http://thcmn.wordpress.com 153

Page 154: t 2017 - WordPress.com...Bài 5. (B›c Ninh) Cho a, b c¨0 thäa mãn đi•u ki»n ¯ ˘9. Tìm giá trà lîn nh t cıabi”uthøc T ˘ ab 3a¯4b¯5c ¯ bc 3b¯4c¯5a ¯ ca 3c¯4a¯5b

Cho tam giác ABC ngoại tiếp (I ). Các cạnh AB , AC tiếp xúc với (I ) tại E ,F . Đườngthằng qua B song song với AC cắt EF tại K . C K cắt AB tại G. Chứng minh tamgiác AIG vuông.

(Phú Thọ)

Bài 43

Lời giải

Đặt a = BC ,b =C A,c = AB . Yêu cầu đề bài tương đương với chứng minh: F K //G I . Gọigiao điểm của AI và EF là H . Do K B//AC nên 4K BE cân tại B ⇒ K B = BE . Áp dụngđịnh lý Thales và phép biến đổi tỉ số, ta có

AE

AG= AE

AB.

AB

AG= AE

AB.

AC +K B

AC=

AB+AC−BC2

AB.

AC + AB+BC−AC2

AC= (c +b −a)(c +b +a)

4bc.

Mà ta có:AH

AI= cos �B AC +1

2= b2 + c2 −a2 +2bc

4bc

nên trên suy raAE

AG= AH

AI⇒ K F //G I (đpcm).

154 http://www.facebook.com/thcmn

Page 155: t 2017 - WordPress.com...Bài 5. (B›c Ninh) Cho a, b c¨0 thäa mãn đi•u ki»n ¯ ˘9. Tìm giá trà lîn nh t cıabi”uthøc T ˘ ab 3a¯4b¯5c ¯ bc 3b¯4c¯5a ¯ ca 3c¯4a¯5b

Cho đường tròn (O) và dây cung AB . Các đường tròn (O1) và (O2) nằm về mộtphía đối với đường thẳng AB , Tiếp xúc với nhau tại T đồng thời tiếp xúc trongvới đường tròn (O). Tiếp tuyến chung tại T của (O1), (O2) cắt đường tròn (O) tại C(Với C thuộc nửa mặt phẳng bờ AB chứa (O1), (O2)). Chứng minh rằng T là tâmđường tròn nội tiếp tam giác ABC .

(Phú Thọ)

Bài 44

Lời giải

(O1), (O2) lần lượt tiếp xúc với (O) tại F,D. Gọi giao điểm của FG , HD với (O) là I , I ′.Bằng định lý Thales, ta dễ dàng chứng minh được: I ≡ I ′ là điểm chính giữa cungAB không chứa C . Ta cũng chứng minh được I A2 = IG .I F = I H .I D = I T 2 = I B 2 ⇒ Tứgiác FG HD nội tiếp. Từ đây, sử dụng tính chất tâm đẳng phương cho 3 đường tròn(FG HD), (O1), (O2), ta suy ra được: I ,T,C thẳng hàng. Kết hợp với hệ thức trên, ta suyra được T là tâm đường tròn nội tiếp 4ABC .

http://thcmn.wordpress.com 155

Page 156: t 2017 - WordPress.com...Bài 5. (B›c Ninh) Cho a, b c¨0 thäa mãn đi•u ki»n ¯ ˘9. Tìm giá trà lîn nh t cıabi”uthøc T ˘ ab 3a¯4b¯5c ¯ bc 3b¯4c¯5a ¯ ca 3c¯4a¯5b

Cho tam giác ABC nhọn không cân . P là một điểm bất kì trên cạnh BC và khôngtrùng với B ,C . Đường tròn ngoại tiếp tam giác ABP cắt AC tại Y khác A . Tươngtự xác định Z . Gọi BY cắt C Z tại K . Gọi T là hình chiếu của A lên BC , H là trựctâm tam giác ABC , A′ là điểm đối xứng của A qua BC .

1. Chứng minh A′,P,K thẳng hàng.

2. Chứng minh khi P di chuyển trên BC , tâm đường tròn ngoại tiếp tam giácAY Z luôn di chuyển trên một đường thẳng cố định.

(Quảng Bình)

Bài 45

Lời giải

1. Bằng biến đổi góc đơn giản suy ra AZ K Y nội tiếp từ đó PK Y C , Z K PB nội tiếptừ đó suy ra �BPA′ = �APB = �K PC suy ra đpcm.

2. B N , C M là các đường cao của tam giác ABC .Trung tuyến At của tam giác ABCgiao (AH) tại I . Ta có hai tam giác Z I M , M I Y đồng dạng suy ra M I N = Z I Y suyra I thuộc (AY Z ) do đó SI ′ vuông góc AI (S là tâm (AY Z ), I ′ là trung điểm AH)Ta có dpcm.

156 http://www.facebook.com/thcmn

Page 157: t 2017 - WordPress.com...Bài 5. (B›c Ninh) Cho a, b c¨0 thäa mãn đi•u ki»n ¯ ˘9. Tìm giá trà lîn nh t cıabi”uthøc T ˘ ab 3a¯4b¯5c ¯ bc 3b¯4c¯5a ¯ ca 3c¯4a¯5b

Cho đường tròn (O) và dây cung BC khác đường kính. Điểm A di động trên cunglớn BC sao cho tam giác ABC không cân tại A. Gọi M là giao điểm của hai tiếptuyến với (O) tại B và C , AM cắt (O) tại D khác A. Dựng đường kính DE của (O).Các đường thẳng BD,C E cắt nhau tại X , các đường thẳng BE ,C D cắt nhau tại Y .

1. Chứng minh rằng M X = MY .

2. Gọi N là giao điểm của AE và X Y . Chứng minh rằng N nằm trên một đườngthẳng cố định.

(Quảng Nam)

Bài 46

Lời giải

1. Bằng tính góc, ta chứng minh được:�B MC

2= �BY C = �B XC . Mà 4B MC cân ⇒ M là

tâm đường tròn (Y BC X ) có đường kính X Y ⇒ M X = MY .

2. Lấy F đối xứng với D qua M . Ta chứng minh được: �E AF = �EY F = �E X F = 90o

⇒ E AY X nội tiếp.Áp dụng tính chất tâm đẳng phương cho ba đường tròn (O), (AE X Y ), (Y BC X ), tasuy ra được: N thuộc đường thẳng BCVậy N luôn chạy trên đường thẳng BC cố định khi A di chuyển trên cung BCkhông chứa A của (O).

http://thcmn.wordpress.com 157

Page 158: t 2017 - WordPress.com...Bài 5. (B›c Ninh) Cho a, b c¨0 thäa mãn đi•u ki»n ¯ ˘9. Tìm giá trà lîn nh t cıabi”uthøc T ˘ ab 3a¯4b¯5c ¯ bc 3b¯4c¯5a ¯ ca 3c¯4a¯5b

4ABC nội tiếp đường tròn (O;R), D là điểm di chuyển trên cung BC không chứaA (D không trùng B ,C ).

1. Gọi E ,F lần lượt là giao điểm của các đường thẳng AD và BC , AB và C D. GọiP là trung điểm EF , M là điểm đối xứng với B qua P . Chứng minh đườngthẳng DM luôn đi qua một điểm cố định.

2. Gọi H , I ,K lần lượt là chân đường vuông góc kẻ từ D đến các cạnh BC ,C A, AB .

Cho BC = Rp

3. Tìm GTNN của tổngAB

DK+ AC

D I+ BC

D H.

(Quảng Ngãi)

Bài 47

Lời giải

1. Gọi K ′, J lần lượt là trung điểm BD, AC . Lấy H ′ đối xứng với J qua BC thì H ′ cốđịnh.Ta chứng minh: MD đi qua H ′ cố định. Áp dụng tính chất đường thẳng Gausscho tứ giác toàn phần BEF D với P,K ′, J lần lượt trung điểm F E ,BD, AC ⇒ P,K ′, Jthẳng hàng .Từ đây, áp dụng tính chất đường trung bình, ta suy ra được MD đi qua H ′.

2. Gọi tổng cần tìm GTNN là T . Ta chứng minh được: D H .D A = D I .DB = DK .DC ⇒T = 2.

BC

D H.

Đồng thời, do BC = Rp

3, nên ta cũng chứng minh được: D H ≤ R

2⇒ T ≥ 4.

p3

Cho 4ABC nhọn ko cân tại A nội tiếp (O) . Gọi M là trung điểm của BC . 2 tiếptuyến tại B và C của (O) cắt nhau tại P . Gọi D,E lần lượt là hình chiếu vuông góccủa B va C lên AP , Q là giao điểm của AP và BC . Đường tròn (CQD) và (BQE) lầnlượt cắt (O) tại điểm thứ 2 là K ,L.

1. Chứng minh �MKC =�MLB .

2. Kẻ đường kính AT của (O). Giả sử các đường thẳng T B ,TC , AK , AL đôi 1 cắtnhau tại 4 điểm phân biệt. Chứng minh rằng 4 giao điểm này là 4 đỉnh của1 tứ giác nội tiếp.

(Quảng Ninh)

Bài 48

Lời giải

158 http://www.facebook.com/thcmn

Page 159: t 2017 - WordPress.com...Bài 5. (B›c Ninh) Cho a, b c¨0 thäa mãn đi•u ki»n ¯ ˘9. Tìm giá trà lîn nh t cıabi”uthøc T ˘ ab 3a¯4b¯5c ¯ bc 3b¯4c¯5a ¯ ca 3c¯4a¯5b

1. Ta có: �QDK = �QC K = �PBK ⇒ Tứ giác DBPK nội tiếp. Mà tứ giác BDMP nội tiếpnên B ,D, M ,K ,P cùng thuộc một đường tròn. Khi đó �MKC = �BKC−�BK M = 180o−�BP M − �PB M = 90o. Chứng minh tương tự, �MLB = 90o ⇒ �MKC =�MLB = 90o.

2. Xét phép đối xứng trục MP , thấy được L,K đối xứng với nhau qua đường thẳngMP ⇒ BLKC là hình thang cân.Gọi X ,Y lần lượt là giao điểm của T B với AK , AL; Z ,W lần lượt là giao điểm củaTC với AL, AK . Khi đó bằng biến đổi góc, chứng minh được �X Y Z = �X W T ⇒ Tứgiác X Y Z W nội tiếp.

Cho 4ABC nhọn nội tiếp (O) (AB < AC ), trực tâm H . Lấy điểm T trên (O) sao choAT ∥ BC . Giả sử AH cắt (O) tại K , T H cắt (O) tại D trên cung nhỏ BC . Gọi I là trungđiểm của HT .

1. Chứng minh 5 điểm A,L,O,K ,D cùng thuộc 1 đường tròn.

2. Gọi P là giao điểm thứ 2 của AO với (O). Đường thẳng qua H song song vớiBC cắt PD tại X . Chứng minh rằng X A là tiếp tuyến của (O).

(Quảng Ninh)

Bài 49

Lời giải

http://thcmn.wordpress.com 159

Page 160: t 2017 - WordPress.com...Bài 5. (B›c Ninh) Cho a, b c¨0 thäa mãn đi•u ki»n ¯ ˘9. Tìm giá trà lîn nh t cıabi”uthøc T ˘ ab 3a¯4b¯5c ¯ bc 3b¯4c¯5a ¯ ca 3c¯4a¯5b

1. Gọi S là giao điểm của AH với (O). Vì AT ⊥ BC nên suy ra �S AT = 90o, do đó S,O,Tthẳng hàng. Theo tính chất đối xứng quen thuộc của trực tâm ta có K S = K H .

• HD.HT = H A.HS ⇒ HD.HL = H A.HK ⇒ Tứ giác ALK D nội tiếp.

• Dễ chứng minh được ALOK là hình thang. Vì AL là trung tuyến tam giácvuông AHT nên �H AL = �AHL = �AKO ⇒ ALOK là hình thang cân ⇒ ALOKnội tiếp đường tròn.

2. Gọi E ,F là giao của X H với AC , AB ; I , J là giao của B H ,C H với (O). Gọi P ′ là giaocủa I E với (O). Dễ thấy H , I đối xứng qua AC , �AC B = �AH I ,�B AH = �C AO, nên ta có:_AI +

_

C P ′

2= �AE I = �AE H = �AC B = �AB H + �B AH = �AB H + �C AO =

_AI + _

C P

2, suy ra

_C P=

_

C P ′, tức là P ≡ P ′, hay P,E , I thẳng hàng.Tương tự, suy ra J ,F,P thẳng hàng. Do đó,

�EPF =_AI + _

J A

2=�AB H +�AC H = 180o −2.A (4.12)

Mặt khác, dễ thấy �BD H =_

BT

2=

_AC

2= �ABC = �AF E , suy ra BD HF nội tiếp, tượng

tự C D HE nội tiếp. Suy ra

�EDF = �ED H +�F D H =�EC H +�F B H = 180o −2.A (4.13)

Từ (4.12) và (4.13) suy ra �EPF = �EDF , tức tứ giác EF D H nội tiếp.

Vì EF //BC , nên (AEF ) tiếp xúc với (O) tại A. Xét ba đường tròn (AEF ), (O), (EF DP )có ba trục dẳng phương là: tiếp tuyến tại A của (O),EF,PD. Vậy X là tâm đẵngphương nghĩa là X A là tiếp tuyến của (O).

160 http://www.facebook.com/thcmn

Page 161: t 2017 - WordPress.com...Bài 5. (B›c Ninh) Cho a, b c¨0 thäa mãn đi•u ki»n ¯ ˘9. Tìm giá trà lîn nh t cıabi”uthøc T ˘ ab 3a¯4b¯5c ¯ bc 3b¯4c¯5a ¯ ca 3c¯4a¯5b

Cho tứ giác ABC D nội tiếp đường tròn (O). Gọi I là giao điểm AC và BD, H và K lầnlượt là trực tâm của 4I AD và 4I BC . M và N lần lượt là trung điểm AB và C D; PvàQ lần lượt là chân đường vuông góc kẻ từ I đến BC và AD.

1. Chứng minh HK ⊥ M N .

2. Chứng minh M N đi qua trung điểm PQ.

(Quảng Trị)

Bài 50

Lời giải

Gọi J là trung điểm I A ⇒ JQ = I A

2, J M = I B

2⇒ JQ

J M

I A

I B= IQ

I P. Mặt khác, �Q J M = �QI P ,

do �QI A = �PI B ⇒ 4PIQ ∼ 4P J M . Do đó, theo phép vị tự quay, 4Q J I ∼ 4QMP , mà4Q J I cần tại J nên 4QMP cân tại M . Chứng minh tương tự, 4QN P cân tại N , suy rađpcm.

http://thcmn.wordpress.com 161

Page 162: t 2017 - WordPress.com...Bài 5. (B›c Ninh) Cho a, b c¨0 thäa mãn đi•u ki»n ¯ ˘9. Tìm giá trà lîn nh t cıabi”uthøc T ˘ ab 3a¯4b¯5c ¯ bc 3b¯4c¯5a ¯ ca 3c¯4a¯5b

Cho tam giác ABC nội tiếp đường tròn (O). Đường tròn (O′) tiếp xúc với hai cạnhAB , AC theo thứ tự tại P.Q và tiếp xúc trong với (O) tại S. Hai đường thẳng SP.SQcắt lại (O) theo thứ tự tại M , N . Gọi E ,D,F theo thứ tự là hình chiếu vuông góc củaS trên các đường thẳng AM , M N , AN .

1. Chứng minh SM .AN = SN .AM .

2. Chứng minh DE = DF .

(Quảng Trị)

Bài 51

Lời giải

1. Gọi I là tâm (SPQ)

•SP

SM= SI

SO= SQ

SN⇒ PQ//M N .

• Tiếp tuyến tại M , N cắt nhau tại T . Ta có �MSN = 180◦− �MON = �PAQ ⇒ I A

OT=

IQ

ON= SI

SO⇒ A,S,O thẳng hàng ⇒ AMSN là tứ giác điều hòa.

2. DE = SM .si n �DSE = SM .si n �AM N ,DF = SN .si n �DSF = SN .si n �AN M ⇒ DE

DF= SM .si n �AM N

SN .si n �AN M=

AN .si n �AM N

AM .si n �AN M= 1

162 http://www.facebook.com/thcmn

Page 163: t 2017 - WordPress.com...Bài 5. (B›c Ninh) Cho a, b c¨0 thäa mãn đi•u ki»n ¯ ˘9. Tìm giá trà lîn nh t cıabi”uthøc T ˘ ab 3a¯4b¯5c ¯ bc 3b¯4c¯5a ¯ ca 3c¯4a¯5b

Cho đường tròn (O) có hai đường kính AB và C D. Tiếp tuyến với đường tròn (O)tại B cắt AC tại P . Gọi G là giao điểm thứ hai của đường thẳng DP với (O), I làtrung điểm AP . Chứng minh rằng:

1. O,B ,C , I cùng thuộc một đường tròn.

2. AG ,BC ,OP đồng quy.

(Thái Bình)

Bài 52

Lời giải

1. OI //BP ⇒ �IOB = �OBP = 90o. Mà �BC I = 90o nên O,B ,C , I cùng nằm trên đườngtròn đường kính B I .

2. Gọi I ′ là trung điểm của PC . Ta có OI ′//DP nên �COI ′ =�C DG . Mà �C DG = �C AG , �COI ′ =�I ′OG ⇒ I ′, A,O,G nằm trên một đường tròn (ω′).

Để ý rằng PO/(ω) = PO/(ω′) = 0, hơn nữa PP/(ω) = PI .PC = 1

2PA.PI ′ = PA.PI ′ = PP/(ω′) ⇒

OP là trục đẳng phương của (ω) và (ω′). Lại có AG là trục đẳng phương của haiđường tròn (C ) và (ω), BC là trục đẳng phương của hai đường tròn (C ) và (ω) nênba đường thẳng AG ,BC ,OP đồng qui tại S là tâm đẳng phương của ba đườngtròn (C ), (ω) và (ω′).

http://thcmn.wordpress.com 163

Page 164: t 2017 - WordPress.com...Bài 5. (B›c Ninh) Cho a, b c¨0 thäa mãn đi•u ki»n ¯ ˘9. Tìm giá trà lîn nh t cıabi”uthøc T ˘ ab 3a¯4b¯5c ¯ bc 3b¯4c¯5a ¯ ca 3c¯4a¯5b

Cho tam giác ABC nhọn, có AC > AB . Gọi D là hình chiếu của A trên BC và E làhình chiếu của D trên AC . Xét điểm F trên đoạn DE . Chứng minh rằng

AF ⊥ BF ⇐⇒ EF.EC = BD.DE

.

(Thái Nguyên)

Bài 53

Hướng dẫn

Ta chứng minh được AF⊥F B ⇔ Tứ giác AF DB nội tiếp ⇔ 4AEF ∼ 4ADB ⇔ E A

AD=

EF

BD⇔ ED

EC= AE

ED= EF

BD(Do hệ thức lượng của 4ADC với đường cao DE ⇔ EF.EC =

BD.DE .

Cho tam giác ABC không cân tại A.Gọi D,E ,F lần lượt là tiếp điểm của đường trònnội tiếp 4ABC với BC ,C A, AB . Đường tròn ngoại tiếp 4ABC cắt đường thẳng EFtại Pvà Q. Gọi M là trung điểm BC , O1 và O2 lần lượt là tâm đường tròn ngoại tiếpcác tam giác I AB và I AC .

1. Chứng minh rằng D,P,Q, M nằm trên một đường tròn.

2. Chứng minh rằng tâm đường tròn ngoại tiếp 4DPQ nằm trên đường thẳngO1O2.

(Thanh Hoá)

Bài 54

Lời giải

164 http://www.facebook.com/thcmn

Page 165: t 2017 - WordPress.com...Bài 5. (B›c Ninh) Cho a, b c¨0 thäa mãn đi•u ki»n ¯ ˘9. Tìm giá trà lîn nh t cıabi”uthøc T ˘ ab 3a¯4b¯5c ¯ bc 3b¯4c¯5a ¯ ca 3c¯4a¯5b

1. Kéo dài M N cắt BC tại J . Dễ chứng minh (JDBC ) =−1, từ đó kết hợp với hệ thứcMaclaurin suy ra JD.JG = J M .J N và ta có đpcm.

2. OH cắt (O) tại P suy ra PH = I D, mà PH//I D nên PH I D là hình bình hành, suyra trung điểm L của D H cũng là trung điểm L của PI .Mặt khác dễ chứng minh góc I AP vuông (dựa vào tính chất AI cắt (O) tại điểmchính giữa cung BC ) nên dễ suy ra AP N M là hình thang cân.L thuộc trung trực của AP nên L cũng thuộc đường trung trực của M N , màL cũng là đường trung trực của DG (dễ chứng minh), do đó L ≡ K hay K ,D, Hthằng hàng (đpcm)

4ABC nhọn nội tiếp đường tròn (O) có AB < AC . Gọi D,E lần lượt là trung điểmAB , AC . Các đường tròn ngoại tiếp 4ABE và 4AC D cắt nhau tại K khác A. Đườngthẳng LB cắt đường tròn ngoại tiếp 4ABE tại điểm thứ hai M , đường thẳng LCcắt đường tròn ngoại tiếp AC D tại điểm thứ hai N .

1. Chứng minh M ,K ,L thẳng hàng và M N ⊥OL.

2. Chứng minh K là trung điểm M N .

(Vĩnh Phúc)

Bài 55

http://thcmn.wordpress.com 165

Page 166: t 2017 - WordPress.com...Bài 5. (B›c Ninh) Cho a, b c¨0 thäa mãn đi•u ki»n ¯ ˘9. Tìm giá trà lîn nh t cıabi”uthøc T ˘ ab 3a¯4b¯5c ¯ bc 3b¯4c¯5a ¯ ca 3c¯4a¯5b

Lời giải

1. Gọi G là giao của BE và C D. Tứ giác ABLC nội tiếp nên �AK N = 180o −�AC N =�ABL = 180o − �AB M = 180o − �AK M ⇒ M ,K , N thẳng hàng.

• Tứ giác AK NC và AK B M nội tiếp nên LN .LC = LK .L A = LB.LM ⇒ Tứ giácMB NC nội tiếp ⇒ �C BL = �LN M .

• Gọi P là giao điểm của OL và M N . Ta có

�PLN = �OLC = 90o −�LOC

2= 90o − �LBC = 90o − �M N L ⇒OL ⊥ M N .

2. Áp dụng định lý Miquel cho tứ giác toàn phần ADGEBC ta được K là điểmchung của hai đường tròn (BDG) và (C EG). Từ đó dễ thấy hai tam giác BK Dvà EKC đồng dạng.

•AB

AC= BD

C E= K B

K E= sin �B AK

sin �E AK= sin �B AL

sin �C AL= LB

LC⇒ Tứ giác ABLC điều hoà ⇒ L A

là đường đối trung của 4LBC .

• Vì 4LBC ∼ 4LN M nên đường đối trung của 4LBC là đường trung tuyếncủa 4LN M , tức là L A đi qua trung điểm của M N hay K là trung điểm M N .

4ABC nhọn nội tiếp đường tròn (O), P nằm trong 4ABC nằm trên đường phângiác trong góc �B AC . Gọi K ,L lần lượt là giao điểm khác P của BP với (APC ), C Pvới (APB). Gọi E ,F lần lượt là giao điểm của các phân giác trong góc �ABC , �AC Bvới (O). các đường thẳng AE , AF cắt (APC ), (APB) lần lượt tại M , N khác A. Chứngminh K ,L, M , N cùng thuộc một đường tròn.

(Vĩnh Phúc)

Bài 56

166 http://www.facebook.com/thcmn

Page 167: t 2017 - WordPress.com...Bài 5. (B›c Ninh) Cho a, b c¨0 thäa mãn đi•u ki»n ¯ ˘9. Tìm giá trà lîn nh t cıabi”uthøc T ˘ ab 3a¯4b¯5c ¯ bc 3b¯4c¯5a ¯ ca 3c¯4a¯5b

Lời giải

Gọi Y , Z lần lượt là giao điểm của (APC ) với C F , (APB) với BE , I là tâm nội tiếp 4ABC .

• �N Z B =�N AB = �F EB ⇒ N Z //EF . Tương tự Y M//EF .

• Do A,P, I thẳng hàng nên I nằm trên trục đẳng phương của (APB) và (APC ) ⇒I Y .IC = I Z .I B ⇒ Tứ giác B Z Y C nội tiếp ⇒ �I Z Y = �IC B = �F E Z ⇒ Y Z //EF . VậyM , N ,Y , Z thẳng hàng. Mặt khác, �BLC = �B AP = �C AP = �C K B ⇒ Tứ giác BLKC nội

tiếp ⇒ �LK B = �LC B . Từ đó �LK M = �LK B + �BK M = �PC B + �PAE = �PC B +�B AC

2+

�ABC

2.

Lại có �LN M = �Z PC = �Z PI +�I PC = �Z B A+ �PAC + �PC A =�ABC

2+

�B AC

2+ �PC A. Do đó�LK M + �LN M = �B AC + �ABC + �AC B = 180o ⇒ Tứ giác LK M N nội tiếp.

5 Phương trình và hệ phương trình

Giải hệ phương trình :{x3 − y3 +6y2 +2(x −7y)+12 = 0p

3−x +√y −3 = x2 + y2 −10x −5y +22

(x, y ∈R).

(Bến Tre)

Bài 1

Lời giải

Điều kiện xác định: x ≤ 3 ≤ yPhương trình đầu có thể được viết lại thành:

x3 +2x = y3 −6y2 +14y −12 ⇔ x3 +2x = (y −2)3 +2(y −2).

http://thcmn.wordpress.com 167

Page 168: t 2017 - WordPress.com...Bài 5. (B›c Ninh) Cho a, b c¨0 thäa mãn đi•u ki»n ¯ ˘9. Tìm giá trà lîn nh t cıabi”uthøc T ˘ ab 3a¯4b¯5c ¯ bc 3b¯4c¯5a ¯ ca 3c¯4a¯5b

Để ý hàm f (t ) = t 3 +2t đồng biến trên R nên suy ra x = y −2.Thay vào phương trình sau rồi thu gọn, ta được:√

5− y +√y −3 = 2y2 −19y +46

(3 ≤ y ≤ 5

)Trừ 2 ở cả hai vế:

(y −4)(1√

y −3+1− 1√

5− y +1) = (y −4)(2y −11)

Nếu y 6= 4 thì:1√

y −3+1− 1√

5− y +1= 2y −11

Để ý trên [3,5] vế trái là hàm nghịch biến nên nó không nhỏ hơn −2+p2 (đạt được

khi y = 5. Lại để ý trên cùng đoạn thì vế phải làm hàm đồng biến nên không lớn hơn2.5−11 =−1 <−2+p

2.Vậy phương trình có duy nhất nghiệm y = 4, x = 4−2 = 2.

Nhận xét 28 Mấu chốt bài toán trên là việc sử dụng bổ đề: Cho hàm số f khả vi vàđơn điệu trên D. Khi đó với u, v ∈D: f (u) = f (v) ⇔ u = v . Kết quả trên được vận dụng đểgiải quyết phương trình đầu. Ta chuyển hai biến về hai vế, giữ lại vế có cấu trúc đơngiản hơn (cụ thể bài này là vế trái) và sau đó dùng biến đổi đại số để đưa vế còn lại vềcấu trúc này.

Giải hệ phương trình:6x(x +1)+ y(y −4x −1) = 2

[(x +1)

√x2 + y −1

]2x +5x = 13− y

6+44log2

(2+ 131x

3−5x

) (x, y ∈R).

(Bắc Ninh)

Bài 2

Lời giải

Điều kiện xác định: x2 + y ≥ 0,5x ≤ 2+ 131x

3.

Khi đó theo bất đẳng thức AM-GM:

(x +1)2 + (x2 + y) ≥ 2(x +1)√

x2 + y

Suy ra:6x(x +1)+ y(y −4x −1) ≤ (x +1)2 +x2 + y −2

⇔ (2x − y +1)2 ≤ 0

Vậy y = 2x +1. Thay vào phương trình sau, ta được:

f (x) = 2x +5x + x

3−2−44log2(2+ 131x

3−5x) = 0

168 http://www.facebook.com/thcmn

Page 169: t 2017 - WordPress.com...Bài 5. (B›c Ninh) Cho a, b c¨0 thäa mãn đi•u ki»n ¯ ˘9. Tìm giá trà lîn nh t cıabi”uthøc T ˘ ab 3a¯4b¯5c ¯ bc 3b¯4c¯5a ¯ ca 3c¯4a¯5b

Đặt g (x) = 2x +5x + x

3−2;h(x) = ln(2+ 131x

3−5x) thì khi đó:

f (x) = g (x)− 44

ln2h(x)

Để ý g ′′(x) > 0,h′′(x) < 0 nên f ′′(x) = 0 vô nghiệm, tức f (x) = 0 có không quá hai nghiệmphân biệt. Lại có f (0) = f (3) = 0 nên kết luận nghiệm hệ đã cho:(x, y) = (0,1); (3,7)

Nhận xét 29 Bài toán trên có hai ý:

1. Khai thác phương trình đầu để có y = 2x +1. Bước này cần vận dụng bất đẳngthức khéo léo để khử căn và ép được một bình phương không dương.

2. Thay vào phương trình sau và giải. Bước này vẫn không hề đơn giản khi có xuấthiện cả hàm mũ và hàm logarit. Ta đã vận dụng hệ quả định lý Rolle:

Bổ đề 11 (Định lý Rolle) Cho hàm số f (x) khả vi n lần trên miền xác định củanó. Khi đó nếu phương trình f (n)(x) = 0 có tối đa k nghiệm thực phân biệt thìphương trình f (x) = 0 có không quá k +n nghiệm thực phân biệt.

Hệ quả trên tỏ ra khá hiệu quả với các bài phương trình phức tạp mà ta đã nhẩmđược nghiệm. Khi đó ta sẽ khảo sát đạo hàm cấp phù hợp để giới hạn số nghiệmphương trình ban đầu.

1. Giải phương trình√2x2 −2x +4+

√5x2 +4+x2 −7x +1 = 0

2. Giải hệ phương trình {√2x + y +√

x +4y = 5√x +4y +2x − y = 3

(Hà Nội)

Bài 3

Lời giải

1. Để ý từ vế trái phương trình đã cho không nhỏ hơn 2+x2 −7x +1 nên suy ra:

x2 −7x +3 ≤ 0

Vậy x > 0.Viết lại phương trình đã cho:

[√

2x2 −2x +4− (x +1)]+ [√

5x2 +4− (2x +1)]+x2 −7x +1+3x +2 = 0

⇔ (x2 −4x +3)[1p

2x2 −2x +4+x +1+ 1p

5x2 +4+2x +1+1] = 0

http://thcmn.wordpress.com 169

Page 170: t 2017 - WordPress.com...Bài 5. (B›c Ninh) Cho a, b c¨0 thäa mãn đi•u ki»n ¯ ˘9. Tìm giá trà lîn nh t cıabi”uthøc T ˘ ab 3a¯4b¯5c ¯ bc 3b¯4c¯5a ¯ ca 3c¯4a¯5b

Lại do1p

2x2 −2x +4+x +1+ 1p

5x2 +4+2x +1+1 = 0(x > 0)

nên phương trình trên tương đương x2 − 4x + 3 = 0, tức x = 1, x = 3. Đó cũng lànghiệm phương trình đã cho.

2. Điều kiện xác định: 2x + y ≥ 0, x +4y ≥ 0Đặt

√2x + y = a ≥ 0,

√x +4y = b ≥ 0

Khi đó

x = 4a2 −b2

7, y = 2b2 −a2

7

Hệ đã cho được viết lại: a +b = 5

b + 9a2 −4b2

7= 3

Thay phương trình đầu vào phương trình sau, ta được:

b + 9(5−b)2 −4b2

7= 3

Giải được các nghiệm b = 68

5,b = 3, lần lượt tương ứng a = −43

5, a = 2.

Kết hợp điều kiện, chỉ chọn (a,b) = (2,3).Suy ra nghiệm của hệ là (x, y) = (1,2).

Nhận xét 30 Câu 3.2 đã được xử lí bằng cách đặt ẩn là căn thức. Cách này giúp takhử căn, và thật may mắn là hệ với ẩn mới gồm các phương trình có bậc không quálớn, và phép thế giúp giải quyết gọn gàng.

Giải hệ phương trình { √y −3x +4+√

y +5x +4 = 4√5y +3−p

7x −2 = 2x −1−4y

(Hòa Bình)

Bài 4

Lời giải

Điều kiện xác định:

x ≥ 2

7

y ≥ −3

5y −3x +4 ≥ 0

y +5x +4 ≥ 0

170 http://www.facebook.com/thcmn

Page 171: t 2017 - WordPress.com...Bài 5. (B›c Ninh) Cho a, b c¨0 thäa mãn đi•u ki»n ¯ ˘9. Tìm giá trà lîn nh t cıabi”uthøc T ˘ ab 3a¯4b¯5c ¯ bc 3b¯4c¯5a ¯ ca 3c¯4a¯5b

Theo điều kiện xác định, x > 0 nên√

y −3x +4 <√y +5x +4.

Thực hiện nhân liên hợp với phương trình đầu, ta có:

8x√y +5x +4−√

y −3x +4= 4

⇔√y +5x +4−√

y −3x +4 = 2x

⇔√y +5x +4 = x +2

⇔ y = x2 −x

Thay vào phương trình sau, ta được:√5x2 −5x +3−p

7x −2 = 2x −1−4(x2 −x)

⇔√

5x2 −5x +3−p7x −2 =−4x2 +6x −1

⇔ [√

5x2 −5x +3− (x +1)]+ (2x −p7x −2) =−(4x2 −7x +2)

⇔ (4x2 −7x +2)[1p

5x2 −5x +3+ (x +1)+ 1

2x +p7x −2

+1] = 0

Để ý với điều kiện x ≥ 2

7thì mỗi số hạng trong thừa số thứ hai đều dương nên đẳng

thức trên tương đương:4x2 −7x +2 = 0

hay x = 7+p

17

8

x = 7−p17

8

Từ đó kết luận bộ (x, y) là nghiệm của hệ là: (7+p

17

8,

5+3p

17

32), (

7−p17

8,

5−3p

17

32)

Nhận xét 31 Bài giải trên có hai ý:

1. Nhân liên hợp để tìm ra mối quan hệ hai biến. Ý tưởng khá rõ ràng khi để ý haicăn thức có cùng hệ số của y và hệ số tự do.

2. Nhân liên hợp tìm nghiệm ở phương trình sau.Khi nhẩm nghiệm, nhận thấy phương trình sau nhiều khả năng không có nghiệm“đẹp” (nguyên hay hữu tỉ) nên ta nghĩ tới khả năng có nghiệm vô tỉ, tức nhânliên hợp với một biểu thức để xuất hiện hàm bậc hai làm thừa số chung. Gọiax+b,cx+d lần lượt là các hàm được nhân liên hợp với hai số hạng ở vế trái. Khiđó, ta thu được các hàm bậc hai:

5x2 −5x +3− (ax +b)2

(cx +d)2 − (7x −2)

−4x2 +6x −1+ (cx +d)− (ax +b)

Cho các hàm này bằng nhau (hoặc đối nhau) để tìm được a = b = 1,c = 2,d = 0.

http://thcmn.wordpress.com 171

Page 172: t 2017 - WordPress.com...Bài 5. (B›c Ninh) Cho a, b c¨0 thäa mãn đi•u ki»n ¯ ˘9. Tìm giá trà lîn nh t cıabi”uthøc T ˘ ab 3a¯4b¯5c ¯ bc 3b¯4c¯5a ¯ ca 3c¯4a¯5b

Giải phương trình trên tập số thực

(x −4)p

x −2−1p4−x +x −5

= 2+ (2x −4)p

x −2

x −1.

(Khánh Hòa)

Bài 5

Lời giải

Điều kiện xác định: x ≥ 2, x 6= 1,p

4−x +x −5 6= 0 Khi đó, phương trình tương đương:

(x −2)p

x −2−2p

x −2−1p4−x +x −5

= 2[(x −2)p

x −2−1]

x −1

⇔ (p

x −2+1)(x −3−px −2)p

4−x +x −5= 2(

px −2+1)(x −1−p

x −2)

x −1

⇔ x −3−px −2p

4−x +x −5= 2(x −1−p

x −2)

x −1

Bớt 1 ở hai vế:2−p

x −2−p4−xp

4−x +x −5= x −1−2

px −2

x −1

Để ý vế phải không âm (do(p

x −2−1)2

x −1≥ 0).

Ta chứng minh vế trái không dương.Thật vậy, theo bất đẳng thức Cauchy-Schwarz:

px −2+p

4−x ≤√

2(x −2+4−x) = 2

nên tử số không âm.

Lại để ý với mọi số thực a, a2 −a +1 > (a − 1

2)

2

≥ 0, tức a2 +1 > a. Áp dụng kết quả này,

ta có: p4−x +x < 4−x +1+x = 5

nên mẫu số luôn âm.Vậy đẳng thức xảy ra giữa hai vế khi và chỉ khi:

x −2 = 4−xp4−x = 1

(p

x −2−1)2 = 0

tức x = 3.Vậy phương trình có nghiệm duy nhất x = 3.

Nhận xét 32 Mấu chốt bài toán là phân tích nhân tử được tử số ở hai vế, sau đó làphép thêm bớt phù hợp để dễ đánh giá hơn.

172 http://www.facebook.com/thcmn

Page 173: t 2017 - WordPress.com...Bài 5. (B›c Ninh) Cho a, b c¨0 thäa mãn đi•u ki»n ¯ ˘9. Tìm giá trà lîn nh t cıabi”uthøc T ˘ ab 3a¯4b¯5c ¯ bc 3b¯4c¯5a ¯ ca 3c¯4a¯5b

Giải hệ phương trìnhx +√

x(x2 +3x +3) = 3√

y +4+√y +3−1 =

7p

x +1−√

x2 +18x +18 = 3√

y +4+1

(Khánh Hòa)

Bài 6

Lời giải

Điều kiện xác định: x ≥−1, y ≥−3, x(x2 +3x +3) ≥ 0, x2 +18x +18 ≥ 0Phương trình đầu tương đương:

3√

(x3 +3x2 +3x)+1+√

x3 +3x2 +3x = 3√

y +4+√y +3

Để ý hàm3√

t 2 +1+pt đồng biến trên (0,+∞) nên đẳng thức trên cho ta:√

x3 +3x2 +3x =√y +3

tức y +4 = (x +1)3, x > 0 (do y ≥−3)Thay vào phương trình sau, ta có:

7p

x +1 = x +2+√

x2 +18x +18

⇒ 7p

x +1− (x +2) =√

x2 +18x +18

⇒ 49(x +1)+ (x +2)2 −14(x +2)p

x +1 = x2 +18x +18

⇒ 35x +35 = 14(x +2)p

x +1

⇒ 25(x +1)2 = 4(x +2)2(x +1)

⇒ (x +1)(4x2 −9x −9) = 0

⇒ (x +1)(x −3)(4x +3) = 0

Thử lại, chọn nghiệm x = 3 (khi đó y = 60).Kết luận: (x, y) = (3,60).

Nhận xét 33 Một lần nữa, dạng f (u) = f (v) đã thể hiện vai trò trong cách khai thácphương trình đầu.

Giải hệ phương trình(x +1)√

y2 + y +2+ (y −1)√

x2 +x +1 = x + y

(x2 +x)√

x − y +3 = 2x2 +x + y +1

(Lào Cai)

Bài 7

http://thcmn.wordpress.com 173

Page 174: t 2017 - WordPress.com...Bài 5. (B›c Ninh) Cho a, b c¨0 thäa mãn đi•u ki»n ¯ ˘9. Tìm giá trà lîn nh t cıabi”uthøc T ˘ ab 3a¯4b¯5c ¯ bc 3b¯4c¯5a ¯ ca 3c¯4a¯5b

Lời giải

Điều kiện xác định: x ≥ y −3Viết lại phương trình thứ hai dưới dạng:

(x2 +x)√

x − y +3 = 2x2 +2x − (x − y −1)

⇔ (x2 +x)x − y −1√

x − y +3+2+ (x − y −1) = 0

⇔ (x − y −1)x2 +x +2+√

x − y +3√x − y +3

= 0

Để ý x2 +x +2 > (x − 1

2)

2

≥ 0 nênx2 +x +2+√

x − y +3√x − y +3

> 0, suy ra y = x −1.

Thay vào phương trình đầu tiên, ta có:

(x +1)√

x2 −x +2+ (x −2)√

x2 +x +1 = 2x −1

⇔ (2x −1)√

x2 −x +2+ (x −2)(√

x2 +x +1−√

x2 −x +2) = 2x −1

⇔ (2x −1)√

x2 −x +2+ (x −2)2x −1p

x2 +x +1+p

x2 −x +2= 2x −1

Vậy x = 1

2hoặc

√x2 −x +2+ x −2p

x2 +x +1+p

x2 −x +2= 1.

Trường hợp sau tương đương:√(x2 −x +2)(x2 +x +1)+x2 −x +2+x −2 =

√x2 +x +1+

√x2 −x +2

Đặt a =√

x2 +x +1,b =√

x2 −x +2 thì x2 = a2 +b2 −3

2.

Đẳng thức trên trở thành:

ab + a2 +b2 −3

2= a +b

⇔ (a +b)2 −2(a +b)−3 = 0

Lại do a +b ≥ 0 nên suy ra a +b = 3, tức:√x2 +x +1+

√x2 −x +2 = 3

⇔[√

x2 +x +1−(

1

3x + 4

3

)]+

[√x2 −x +2−

(−1

3x + 5

3

)]= 0

⇔ (8x −7)(x +1).

1p

x2 +x +1+ 1

3x + 4

3

+ 1p

x2 −x +2− 1

3x + 5

3

= 0. (5.1)

Để ý: √x2 +x +1 =

√3

4x2 + 1

4(x +2)2 ≥

p3

2|x| > 1

3.(−x)

⇒√

x2 +x +1+ 1

3x + 4

3> 0

174 http://www.facebook.com/thcmn

Page 175: t 2017 - WordPress.com...Bài 5. (B›c Ninh) Cho a, b c¨0 thäa mãn đi•u ki»n ¯ ˘9. Tìm giá trà lîn nh t cıabi”uthøc T ˘ ab 3a¯4b¯5c ¯ bc 3b¯4c¯5a ¯ ca 3c¯4a¯5b

Và: √x2 −x +2 >

√x2 −x +1 =

√3

4x2 + 1

4(x −2)2 ≥

p3

2|x| > 1

3x

⇒√

x2 −x +2− 1

3x + 5

3> 0

Nên (5.1) cho ta x =−1 (khi đó y =−2) hoặc x = 7

8(khi đó y = −1

8)

Kết luận: Các bộ (x, y) cần tìm: (1

2,−1

2), (−1,−2), (

7

8,−1

8).

Nhận xét 34 Khai thác phương trình đầu không hề dễ vì nó không có dạng f (u) = f (v).Khi đó, ta có thể cho một biến nhận giá trị đặc biệt, rồi giải ra giá trị biến còn lại. Mụcđích là để “tiên đoán” quan hệ hai biến, và rồi ý tưởng phân tích nhân tử trở nên rõràng.

Giải hệ phương trình: {y3 +4x2 y +3x y2 = x6 +3x5 +4x4√

x2 +3+√3y +1 = 4

(Ninh Bình)

Bài 8

Lời giải

Điều kiện xác định: y ≥ −1

3.

• Nếu x = 0, thay vào phương trình đầu tìm được y = 0. Nhưng cặp số này khôngthỏa phương trình sau.

• Nếu x 6= 0, phương trình đầu tiên tương đương

(y −x2)(y2 + (x2 +3x)y +x4 +3x3 +4x2) = 0

Xét f (y) = y2 + (x2 +3x)y +x4 +3x3 +4x2 là tam thức bậc hai theo biến y , có:

∆= (x2 +3x)2 −4(x4 +3x3 +4x2) =−x2(3x2 +6x +7) < 0

Vậy: f (y) > 0Tức phương trình đầu cho ta:

y = x2

Thay vào phương trình sau: √y +3+√

3y +1 = 4

Vế trái là hàm đồng biến theo y nên phương trình trên có tối đa một nghiệm.Lại có y = 1 thỏa đẳng thức trên nên nó chính là nghiệm duy nhất phương trình.Kết luận: (x, y) = (1,1).

Nhận xét 35 Đánh giá tổng quan hệ số tương ứng từng hạng tử ở hai vế là quan trọngđể nhận ra mối tương quan hai biến được “che giấu” trong phương trình đầu.

http://thcmn.wordpress.com 175

Page 176: t 2017 - WordPress.com...Bài 5. (B›c Ninh) Cho a, b c¨0 thäa mãn đi•u ki»n ¯ ˘9. Tìm giá trà lîn nh t cıabi”uthøc T ˘ ab 3a¯4b¯5c ¯ bc 3b¯4c¯5a ¯ ca 3c¯4a¯5b

1. Giải phương trìnhp

x +3+p3x +1 = 2

px +p

2x +2

2. Giải hệ phương trình x3 + y = 2(x2 +1)

2y3 + z = 2(2y2 +1)3z3 +x = 2(3z2 +1)

(Quảng Trị)

Bài 9

Lời giải

1. Điều kiện xác định: x ≥ 0Phương trình đã cho được viết lại thành:

px +3+p

3x +1 =p4x +p

2x +2

⇔ (p

4x −p3x +1)+ (

p2x +2−p

x +3) = 0

⇔ (x −1)

(1p

4x +p3x +1

+ 1p2x +2+p

x +3

)= 0

Lại do1p

4x +p3x +1

+ 1p2x +2+p

x +3> 0 nên x = 1.

Kết luận: S = {1}

2. Viết lại hệ đã cho dưới dạng: y −2 = x2(2−x)

z −2 = 2y2(2− y)

x −2 = 3z2(2− z)

Nếu trong ba biến có một biến bằng 2 thì từ hệ trên suy ra các biến còn lại cũngbằng 2.Nếu cả ba biến đều khác 2, nhân vế theo vế ba phương trình rồi thu gọn, tađược: x2.2y2.3z2 =−1Rõ ràng phương trình trên vô nghiệm trên R.Kết luận nghiệm của hệ (x, y, z) = (2,2,2).

176 http://www.facebook.com/thcmn

Page 177: t 2017 - WordPress.com...Bài 5. (B›c Ninh) Cho a, b c¨0 thäa mãn đi•u ki»n ¯ ˘9. Tìm giá trà lîn nh t cıabi”uthøc T ˘ ab 3a¯4b¯5c ¯ bc 3b¯4c¯5a ¯ ca 3c¯4a¯5b

Giải bất phương trình

2(1−x)√

x2 +2x −1 É x2 −2x −1.

(Thái Nguyên)

Bài 10

Lời giải

Điều kiện xác định:

x ≥p2−1

x ≤−(p

2+1). Đặt A là vế trái, B là vế phải.

Để ý đẳng thức:

A−B = (√

x2 +2x −1−2)(√

x2 +2x −1+2x)

Nếu x ≥p2−1, ta có

√x2 +2x −1+2x > 0.

Khi đó bất phương trình đã cho tương đương:x2 +2x −1 ≥ 4, hay suy ra x ≥p

6−1.Nếu x ≤−(

p2+1), để ý khi đó

√x2 +2x −1+2x < 0 (tương đương 3x2 −2x +1 > 0, hiển

nhiên đúng), nên bất phương trình đã cho tương đương x2 +2x −1 ≤ 4, hay suy ra:

−(p

6+1) ≤ x ≤−(p

2+1)

Bài toán được giải quyết xong.

Nhận xét 36 Hướng suy nghĩ đầu tiên xuất hiện là bình phương hai vế khử căn. Tuynhiên, việc mơ hồ về dấu khiến việc bình phương này dẫn đến việc xét nhiều trườnghợp. Như bài 7, ta lại phải để ý mối quan hệ giữa các biểu thức trong và ngoài căn, đểtừ đó có được phân tích nhân tử “đẹp mắt” và việc xét dấu trở nên nhẹ nhàng hơn.

Giải hệ phương trình 2y3 + y +2xp

1−x = 3p

1−x√9−4y2 = 2x2 +6y2 −7

(x, y ∈R)

.

(Thái Nguyên)

Bài 11

Lời giải

Điều kiện xác định: x ≤ 1, y2 ≤ 9

4.

Phương trình đầu tương đương:

2y3 + y = 2(p

1−x)3 +p

1−x

http://thcmn.wordpress.com 177

Page 178: t 2017 - WordPress.com...Bài 5. (B›c Ninh) Cho a, b c¨0 thäa mãn đi•u ki»n ¯ ˘9. Tìm giá trà lîn nh t cıabi”uthøc T ˘ ab 3a¯4b¯5c ¯ bc 3b¯4c¯5a ¯ ca 3c¯4a¯5b

Để ý hàm f (t ) = 2t 3 + t đồng biến trên R nên suy ra y =p1−x, tức y2 = 1− x. Thay vào

phương trình sau, ta được: p4x +5 = 2x2 −6x −1

⇔ 4x +5 = (2x2 −6x −1)2

⇔ (x2 −2x −1)(x2 −4x +1) = 0

Vậy:

x = 1+p

2

x = 1−p2

x = 2+p3

x = 2−p3

Đối chiếu điều kiện xác định, suy ra tập nghiệm:(x, y) = (1−p2,

4p2); (2−p

3,4p

3)

Giải hệ phương trình2x2 +7x +3+px +1 = y2 +2y −x y +p

y −x

3√

6+x − y +3√

3x + y −5 = y +6(x, y ∈R).

(Vĩnh Phúc)

Bài 12

Lời giải

Điều kiện xác định: 6+x ≥ y ≥ x ≥−1, y ≥ 5−3x.Viết lại phương trình đầu dưới dạng:

[(2x2 +7x +3)− (y2 +2y −x y)]+ [p

x +1−py −x] = 0

⇔ (2x +1− y)(y +x +3)+ (2x +1− y).1p

x +1+py −x

= 0

Từ điều kiện xác định, y ≥ x ≥−1 nên y + x +3 > 0. Kết hợp với1p

x +1+py −x

> 0, suy

ra đẳng thức trên cho ta y = 2x +1.Thay vào phương trình sau, ta được:

3p

5−x +3p

5x −4 = 2x +7 (điều kiện:4

5≤ x ≤ 5)

⇔ 9[5−x +5x −4+2√

(5−x)(5x −4)] = 4x2 +28x +49

⇔ 18√

(5−x)(5x −4) = 4x2 −8x +40

⇔ 9√

(5−x)(5x −4) = 2x2 −4x +20

⇔ 81(5−x)(5x −4) = (2x2 −4x +20)2

⇔ (x −1)(x −4)(4x2 +4x +505) = 0

Để ý khi4

5≤ x ≤ 5 thì 4x2 +4x +505 > 0 nên suy ra phương trình trên có nghiệm x =

1, x = 4 tương ứng lần lượt y = 3, y = 9.Kết luận: Nghiệm (x, y) của hệ phương trình là (1,3), (4,9).

178 http://www.facebook.com/thcmn

Page 179: t 2017 - WordPress.com...Bài 5. (B›c Ninh) Cho a, b c¨0 thäa mãn đi•u ki»n ¯ ˘9. Tìm giá trà lîn nh t cıabi”uthøc T ˘ ab 3a¯4b¯5c ¯ bc 3b¯4c¯5a ¯ ca 3c¯4a¯5b

Nhận xét 37 Cần một sự nhạy cảm để phát hiện mối quan hệ giữa hai biến rút ra từphương trình đầu. Ta có thể dự đoán khi cho hai căn thức bằng nhau để tìm mối quanhệ, và kiểm tra lại mối quan hệ đó trong cả phương trình, từ đó có cách nhân liên hợpnhư trên. Phần còn lại trở nên đơn giản.

6 Số học

1. Giải phương trình nghiệm nguyên dương (a, p,n) trong đó p là một số nguyêntố thỏa mãn

a2(a2 +1) = 5n(5n+1 −p3).

2. Một số nguyên dương n ≥ 2 được gọi là tốt nếu với mọi 2 ≤ k ≤ n thì n códạng n = a1 + a2 + ....+ ak trong đó (n, ak ) = 1 và các số ai là nguyên dương.Tính tổng tất cả các số tốt nhỏ hơn 2016.

(Trường THPT chuyên KHTN, ĐH KHTN, ĐHQG Hà Nội)

Bài 1

Lời giải

1. Với n = 0 ta cóa2(a2 +1) = 5−p3,

vô lý do vế phải âm.Với n ≥ 1, do (a2; a2 +1) = 1 nên ta xét 2 trường hợp:

TH1: 5n | a2

Khi đó đặt a2 = k.5n ⇒ k(k.5n +1) = 5n+1 −p3 ⇒ p3 +k = 5n(5−k2).Dễ thấy vế phải là một số dương nênk = 1 hoặc k = 2.

• k = 1 ⇒ p3 + 1 = 4.5n ⇒(

p +1

4

)(p2 − p + 1) = 5n. Mà (

p +1

4, p2 − p + 1) =

1 ⇒ p +1

4= 1 ⇒ p = 3 hoặc p2 − p + 1 = 1 (vô lý do p > 1). Vậy p = 3 ⇒

p2 −p +1 = 7 không thỏa mãn nên phương trình vô nghiệm.• k = 2 ⇒ a2 = 2.5n ⇒ a2 ≡ 2( mod 4), vô lý.

TH2: 5n | a2 +1. Đặt a2 +1 = k.5n thì p3 −k = 5n(5−k2).Nếu p3 < k ⇒ k −p3 ≥ 5n ⇒ k > 5n ⇒ a2 +1 > 52n ⇒ a2(a2 +1) > 52n(52n −1) >52n+1 > 5n(5n+1 −p3). Do đó p3 > k nên k = 1 hoặc k = 2.

• k = 1 ⇒ p3 = 4.5n+1. Chứng minh tương tự TH1, ta có PT này vô nghiệm.• k = 2 ⇒ p3 = 5n +2.

* n chẵn ta có 3 | p ⇒ (p,n) = (3,2).

* n lẻ ta đặt n = 2t +1. Khi đó

5n +2 ≡ 52t+1 +2( mod 13) ⇒ p3 ≡ 5.(−1)t +2( mod 13)

⇒ p3 ≡ 7,10( mod 13) (6.1)

http://thcmn.wordpress.com 179

Page 180: t 2017 - WordPress.com...Bài 5. (B›c Ninh) Cho a, b c¨0 thäa mãn đi•u ki»n ¯ ˘9. Tìm giá trà lîn nh t cıabi”uthøc T ˘ ab 3a¯4b¯5c ¯ bc 3b¯4c¯5a ¯ ca 3c¯4a¯5b

Nhưng khi cho p chạy trên mod13, ta thu được kết quả là p3 ≡0,1,8,5,12( mod 13). Do đó (6.1) vô lý.

Vậy p = 3,n = 2, a = 7.

2. Rõ ràng mọi số tự nhiên lớn hơn 2, nếu muốn là số tốt thì nó phải lẻ (vì ta thấynếu nó chẵn thì các ai phải lẻ và khi đó dễ thấy nếu chọn k lẻ thì vô lý). Điềunày gợi ý ta chứng minh mọi số tự nhiên lẻ lớn hơn 2 đều là số tốt.Thật vậy xét n lẻ, n > 2. Ta chứng minh với mọi 2 ≤ k ≤ n thì đều có thể viếtn = a1 +a2 + ...+ak trong đó (n, ai ) = 1Thật vậy, ta sẽ viết n = (n −2x)+2x . Bằng quy nạp, ta dễ dàng chứng minh

2x = a1 +a2 + ...+ak

với mọi 1 ≤ k ≤ 2x và ai đều là lũy thừa của 2. Do đó với 2 ≤ k ≤ 2x +1 thì bài toánđược chứng minh.

Chọn x là số lớn nhất thỏa mãn 2x < n < 2x+1 ⇒ 2x > n

2nên chỉ cần chứng minh

nó đúng với k > n

2.

Mặt khác ta cũng viết được n = 2+2+ ...+2+1 nên rõ ràng bằng quy nạp theo k

với k > n

2thì cũng thỏa mãn

Vậy tất cả các số tốt là 2 và các số lẻ lớn hơn 2.Từ đó ta tính được tổng bằng 1+10082.

Tìm tất cả các số nguyên tố p sao cho 2p2 −1 là một lũy thừa của 7.

(Trường THPT chuyên KHTN, ĐH KHTN, ĐHQG Hà Nội)

Bài 2

Lời giải

2p2 −1 = 7t

TH1: p = 2 thỏa mãn do 2.22 −1 = 7

TH2: p ≥ 3. Khi đó xét mod 4 ta thấy t chẵn.

Đặt t = 2s a với s ≥ 1 và a 6 ... 2.Xét a ≥ 3 ta có:

2p2 = 72s a +1

⇔ 2p2 = (72s +1)(72s (a−1) −72s (a−2) + . . .−72s +1)

Đặt A = 72s (a−1) −72s (a−2) + . . .−72s +1. Do s ≥ 1 nên 72s +1 > 2, hơn nữa 72s +1... 2

nên 72s +1 = 2p hoặc 72s +1 = 2p2.

– Nếu 72s +1 = 2p ⇒ A = p. Nhưng a ≥ 3 thì A > 72s +1 ⇒ p > 2p vô lý.

180 http://www.facebook.com/thcmn

Page 181: t 2017 - WordPress.com...Bài 5. (B›c Ninh) Cho a, b c¨0 thäa mãn đi•u ki»n ¯ ˘9. Tìm giá trà lîn nh t cıabi”uthøc T ˘ ab 3a¯4b¯5c ¯ bc 3b¯4c¯5a ¯ ca 3c¯4a¯5b

– Nếu 72s +1 = 2p2 ⇒ A = 1. Nhưng a ≥ 3 thì A > 1 nên vô lý. Do đó a = 1. Ta có

72s +1 = 2p2 ⇔ p −1

2.p +1

2= 72s−1 +1

2.72s−1 −1

4

Lại có(

p −1

2;

p +1

2

)= 1 nên

p −1

2

...72s−1 +1

2hoặc

p +1

2

...72s−1 +1

2

Nếup −1

2

...72s−1 +1

2⇒ p +1

2| 72s−1 −1

4.

Như thế thìp −1

2≥ 72s−1 +1

2> 72s−1 −1

4≥ p +1

2⇒ p −1 > p +1 vô lý.

Do đóp +1

2

...72s−1 +1

2⇒ 72s−1 −1

4

...p −1

2

⇒ p +1

2≥ 72s−1 +1

2> 2.

72s−1 −1

4≥ 2.

p −1

2⇒ p +1 > 2(p −1) ⇒ 3 > p ⇒ p = 2 vô

lý.

Vậy p = 2

Cho n nguyên dương. Chứng minh rằng tồn tại số nguyên dương m có n chữ số,chỉ gồm các chữ số 1,2,3 và chia hết cho S(m)(tổng các chữ số của m).

(Trường THPT chuyên KHTN, ĐH KHTN, ĐHQG Hà Nội)

Bài 3

Lời giải

TH1: n là lũy thừa của 3. Chọn m = 111...11 (n số 1). Đặt n = 3t . Theo bổ đề LTE ta có

v3(103t −1) = v3(10−1)+ v3(3t ) = t +2.

Do đó m = 10n −1

9

... n.

TH2: n không là lũy thừa của 3 thì tồn tại số x sao cho 3x < n < 3x+1. Khi đó ta sẽ chọn

S(m) = 3x+1, tức là chỉ ra tồn tại số m có tổng các chữ số là 3x+1 và m... 3x+1.

– Nếu n ≤ 2.3x , đặt n = 3x + t . Ta chọn m = 111...1333...3222...2, trong đó gồm t

số 1, t số 2 và 3x−t số 3. Sau khi biến đổi, ta thu được m = (10t +2)(103x −1)

9

... 3x+1

(đúng theo bổ đề LT E).

– Nếu n > 2.3x . Đặt n = 2.3x + tTa chọn m = 111...1222...21.....1 thứ tự từ trái sang phải là gồm 3x+t số 1, 3x−t

số 2 rồi lại t số 1. Sau khi biến đổi ta thu được m = (103x+t +10t +1)(103x −1)

9

...3x+1

(đúng theo bổ đề LT E).

Vậy ta có đpcm.

http://thcmn.wordpress.com 181

Page 182: t 2017 - WordPress.com...Bài 5. (B›c Ninh) Cho a, b c¨0 thäa mãn đi•u ki»n ¯ ˘9. Tìm giá trà lîn nh t cıabi”uthøc T ˘ ab 3a¯4b¯5c ¯ bc 3b¯4c¯5a ¯ ca 3c¯4a¯5b

Cho hàm số f :N∗ →N∗ thỏa mãn các điều kiện: f tăng thực sự và f (2n) = 2 f (n)với mọi số n nguyên dương.

1. Giả sử f (1) = 3 và p là số nguyên tố lớn hơn 3. Chứng minh tồn tại n sao chof (n) chia hết cho p.

2. Cho q là một số nguyên tố lẻ. Hãy xây dựng một hàm f thỏa mãn bài toánmà f (n) không chia hết cho q với mọi số nguyên dương n.

(Trường Phổ thông Năng khiếu, ĐHQG Tp.HCM)

Bài 4

Lời giải

Chúng tôi xin đăng lời giải từ trang nangkhieutoan.com.

1. Đặt A = {f (n +1)− f (n)|n ∈N∗}

, vì f tăng thực sự nên A ⊆N∗. Khi đó tồn tại phầntử k nhỏ nhất của A và n thỏa mãn k = f (n +1)− f (n).

Khi đó f (2n+2)− f (2n) = 2( f (n+1)− f (n)) = 2k mà

f (2n) < f (2n +1) < f (2n +2)

f (2n +1)− f (2n) ≥ kf (2n +2)− f (2n +1) ≥ k

⇒ f (2n +1)− f (2n) = k

Bằng quy nạp ta chứng minh được

f (2i n) = 2i f (n)

f (2i n +1) = 2i f (n)+k...

f (2i n + i ) = 2i f (n)+ i k

Do f (1) = 3, f (2) = 6 nên k ≤ 3 ⇒ (k, p) = 1Khi đó p số f (2p + i ) với i = 0,1,2, ..., p −1 sẽ tạo thành một hệ thặng dư đầy đủmod p nên sẽ có một số chia hết cho p.

2. Ta xây dựng hàm f như sau:

f (1) = 2a > qf (2n) = 2 f (n)

f (2n +1) = f (2n)+q

Ta chứng minh f thỏa đề bài.

(a) f là hàm tăng thực sự. Ta chứng minh f (n +1)− f (n) ≥ q bằng quy nạp.Với n = 1 ta có f (2)− f (1) = 2a > q. Giả sử đúng dến k. Ta xét f (k +1).

• Nếu k chẵn, dễ thấy ta có điều phải chứng minh thỏa theo cách xâydựng hàm.

• Nếu k lẻ, ta có f (k +1) = 2 f

(k +1

2

)≥ f

(k −1

2+q

)= f (k −1)+2q và ta có

điều phải chứng minh.

(b) Ta chứng minh không tồn tại n thỏa q| f (n) theo quy nạp.

• Ta có f (1) = 2a không chia hết cho q.• Giả sử đúng đến k.

– Nếu k chẵn thì f (k +1) = f (k)+q.

182 http://www.facebook.com/thcmn

Page 183: t 2017 - WordPress.com...Bài 5. (B›c Ninh) Cho a, b c¨0 thäa mãn đi•u ki»n ¯ ˘9. Tìm giá trà lîn nh t cıabi”uthøc T ˘ ab 3a¯4b¯5c ¯ bc 3b¯4c¯5a ¯ ca 3c¯4a¯5b

– Nếu k lẻ thì f (k +1) = 2 f

(k +1

2

)không chia hết cho q.

Vậy hàm f thỏa đề.

Với mỗi số nguyên dương n ,tồn tại duy nhất số tự nhiên a thỏa mãn a2 ≤ n <(a +1)2. Đặt ∆n = n −a2.

1. Tìm giá trị nhỏ nhất của ∆n khi n thay đổi và luôn thỏa n = 15m2 với m là sốnguyên dương.

2. Cho p, q là các số nguyên dương và d = 5(4p +3)q2. Chứng minh ∆d ≥ 5.

(Trường Phổ thông Năng khiếu, ĐHQG Tp.HCM)

Bài 5

Lời giải

Chúng tôi xin trích đăng lời giải từ trang nangkhieutoan.com.

1. Ta chỉ cần tìm ∆n nhỏ nhất để phương trình 15m2 −a2 = ∆n có nghiệm nguyêndương. Để ý rằng 15− 32 = 6, điều này gợi ý cho ta chứng minh phương trìnhkhông có nghiệm với ∆n < 6.Ta có 3|a2 +∆n nên suy ra

3|∆n hoặc 3|∆n +1. (6.2)

Mặt khác 5|a2 +∆n nên suy ra

∆n ≡ 0,1,4 ( mod 5). (6.3)

Từ (6.2) và (6.3) ta có nếu ∆n < 6 thỏa đề thì ∆n = 5.Ta có 15m2 −a2 = 5 nên 5|a, đặt a = 5s ta có 3m2 −5s2 = 1.Mà 3m2 −1 ≡−1,2,11 ( mod 5) nên phương trình vô nghiệm.

2. Xin nhắc lại một bổ đề quen thuộc như sau.

Bổ đề 12 Nếu p = 4k +3 nguyên tố, a2 +b2 ≡ 0 ( mod p) thì p|a và p|b.

Quay lại bài toán. Xét phương trình 5(4p+3)q2−a2 =∆d . Để ý rằng a2 ≡ 0,1,4 ( mod5) ⇒∆d ≡ 0,1,4 ( mod 5). Mặt khác 5(4p+3) ≡ 3 ( mod 4) nên khác số chính phương.Nếu ∆d = 1 ta có a2 +1 = 5(4p +3)q2. Ta có 5(4p +3) là số có dạng 4k +3 nên tồntại ước nguyên tố r có dạng 4k +3. Suy ra a2 +1 ≡ 0 ( mod r )Áp dụng bổ đề trên, ta có: 1 ≡ 0 ( mod r ) (Vô lý)Tương tự cho trường hợp ∆d = 4 thì cũng có điều vô lý. Vậy ∆d ≥ 5.

http://thcmn.wordpress.com 183

Page 184: t 2017 - WordPress.com...Bài 5. (B›c Ninh) Cho a, b c¨0 thäa mãn đi•u ki»n ¯ ˘9. Tìm giá trà lîn nh t cıabi”uthøc T ˘ ab 3a¯4b¯5c ¯ bc 3b¯4c¯5a ¯ ca 3c¯4a¯5b

Tìm các số nguyên dương a,b,c,d thỏa mãn

a +2b +3c = 3d !+1,

biết rằng tồn tại các số nguyên tố p, q sao cho a = (p +1)(2p +1) = (q +1)(q −1)2.

(Trường THPT chuyên ĐH Vinh)

Bài 6

Lời giải

Ta chia làm hai ý như sau:

1. Tìm tất cả các số a thoả mãn đề bài.Khai triển và thu gọn, ta có p(2p +3) = q(q2 −q −1).Do (q, q2 −q −1) = 1 và p là số nguyên tố nên p|q hoặc p|q2 −q −1.

• Nếu p|q thì do p, q là số nguyên tố nên p = q. Thay vào tính được p = q = 4hoặc p = q =−1, đều loại.

• Nếu p|q2−q−1 thì q|2p+3. Đặt 2p+3 = kq(k ∈N,k ≥ 1). Thay vào đẳng thứcban đầu, khai triển và thu gọn thì

2q2 − (2+k2)q +3k −2 = 0

Xem đây là phương trình bậc hai theo q, để có nghiệm nguyên thì cần có4 là số chính phương. Khi đó ∆= k4 +4k2 −24k +20.Thử với k từ 1 đến 5, do k lẻ ta chọn k ∈ {

1,3,5}. Thế vào tính lại, nhận k = 5,

q = 13, p = 31, a = 2016.Với k ≥ 6 thì (k2+2)2 > k4+4k2−24k+20 > (k2)2 nên k4+4k2−24k+20 = (k2+1)2,Phương trình này không cho nghiệm nguyên nên loại.

Vậy a = 2016.

2. Tìm tất cả các số b,c,d thoả mãn đề bài.Từ giả thiết, 3d !+1 > 2016 nên suy ra d ≥ 6 hay 2|d !.Ta có đẳng thức 2015+2b +3c = 3d !.

Từ đây ta có 2b ≡ 1( mod 9) và 3c ≡ 1( mod 16). Vì thế b... 6 và c

... 4.Đặt b = 6x; c = 4y ta có

2015+64x +81y = 3d !.

• Nếu d = 6 ⇒ x = y = 1 ⇒ b = 6,c = 4.

• Nếu d ≥ 7 ⇒ 3d !... 7, mà 2015+64x +81y ≡ 4y ( mod 7) hay không chia hết cho

7, vô lý.

Vậy a = 2016, b = 6, c = 4.

184 http://www.facebook.com/thcmn

Page 185: t 2017 - WordPress.com...Bài 5. (B›c Ninh) Cho a, b c¨0 thäa mãn đi•u ki»n ¯ ˘9. Tìm giá trà lîn nh t cıabi”uthøc T ˘ ab 3a¯4b¯5c ¯ bc 3b¯4c¯5a ¯ ca 3c¯4a¯5b

Cho dãy số (an) xác định bởi{a1 = 34

an+1 = 4a3n −104a2

n −107an.

với mọi số n nguyên dương. Tìm tất cả các số nguyên tố p thỏa mãn hai điều kiện

p ≡ 3( mod 4) và a2017 +1... p.

(Bắc Ninh)

Bài 7

Lời giải

Từ công thức truy hồi ta có

an+1 −27 = (an −27)(2an +1)2.

Do đó an −27 = (a1 −27)n−1∏k=1

(2ak +1

)2 = 7n−1∏k=1

(2ak +1

)2.

Do đó an +1 = 7

[n−1∏k=1

(2ak +1

)2 +4

].

Xét số nguyên tố p |n−1∏k=1

(2ak +1

)2 +4 (dĩ nhiên ta thấy chỉ có p lẻ).

Ta có

(−4

p

)= 1 ⇐⇒

(−1

p

)= 1 hay p ≡ 1 (mod 4)

Do đó số cần tìm là 7.

Nhận xét 38 Lời giải trên cho ta thấy được ngoài những yếu tố giải tích phức tạp, cácbài toán dãy số còn mang "hương vị" số học. Như vậy, ngoài việc khảo sát hàm, biệnluận phương trình,... ta còn phải nắm vững những tính chất cơ bản nhưng vô cùngquan trọng của số học. Mời bạn đọc thử sức với các bài toán dãy số mang tính số họcsau:

1. Dãy số (un) được xác định như sau:u1 = 1,u2 = 2,u3 = 24

un = 6u2n−1un−3 −8un−1u2

n−2

un−2un−3,∀n ≥ 4

Chứng minh rằng với mọi n là số nguyên dương thì un... n.

2. Cho dãy số (un) được xác định bởi: un = 2n +1,∀n ∈N∗. Tìm số hạng up của dãysao cho p|up với p là số nguyên tố.

3. Dãy số (un) được xác định bởi:u0 = 0,u1 = 1

un+2 = 1999un+1 −un ,∀n ∈N

Tìm tất cả các số tự nhiên n sao cho un là số nguyên tố.

http://thcmn.wordpress.com 185

Page 186: t 2017 - WordPress.com...Bài 5. (B›c Ninh) Cho a, b c¨0 thäa mãn đi•u ki»n ¯ ˘9. Tìm giá trà lîn nh t cıabi”uthøc T ˘ ab 3a¯4b¯5c ¯ bc 3b¯4c¯5a ¯ ca 3c¯4a¯5b

Cho p là số nguyên tố lẻ và T =p∑

k=0C k

pC kp+k − (2p +1). Chứng minh rằng T chia hết

cho p2 .

(Bến Tre)

Bài 8

Lời giải

T =p∑

k=0C k

p .C kp+k − (2p +1) =

p∑k=0

C kp .C k

p+k − (p∑

k=0C k

p +1) =p−1∑k=1

C kpC k

p+k +1+C p2p − (

p∑k=0

C kp +3) =

p−1∑k=1

C kp (C k

p+k −1)+ (C p2p −2). Đặt A =

p−1∑k=1

C kp (C k

p+k −1), B =C p2p −2

Ta có C kp

... p với p là số nguyên tố và 1 ≤ k ≤ p −1.

• Xét C kp+k −1 = (p +1)...(p +k)−k !

k !. Ta có

(p +1)(p +2)...(p +k)−k ! ≡ k !−k ! ≡ 0 (mod p)

cộng với việc(p,k !) = 1

nên ta cóp2|A.

• Xét (1+ x)2p = (1+ x)p .(1+ x)p ⇔2p∑

k=0C k

2p .xk = (p∑

i=0C i

p .xi )2. Đồng nhất hệ số của xp

hai vế ta có

C p2p = 2+

p−1∑k=0

.(C kp )2 ⇒C p

2p −2 =p−1∑k=0

.(C kp )2 ... p2 ⇒ p2|B.

Vậy ta có điều phải chứng minh.

Tìm tất cả các số nguyên dương n sao cho phương trình sau có nghiệm nguyêndương

(x + y)2 = n(4x y +1).

(Bình Dương)

Bài 9

Lời giải

186 http://www.facebook.com/thcmn

Page 187: t 2017 - WordPress.com...Bài 5. (B›c Ninh) Cho a, b c¨0 thäa mãn đi•u ki»n ¯ ˘9. Tìm giá trà lîn nh t cıabi”uthøc T ˘ ab 3a¯4b¯5c ¯ bc 3b¯4c¯5a ¯ ca 3c¯4a¯5b

(x + y)2 = n(4x y +1) ⇔ x2 +2x y(1−2n)+ y2 −n = 0. (6.4)

Gọi (x0; y0) là nghiệm nguyên dương của phương trình (6.4) thỏa mãn x0+y0 nhỏ nhất.Do vai trò x, y như nhau nên giả sử y0 ≤ x0

Coi phương trình (6.4) là phương trình bậc 2, ẩn x. Gọi (x1; y0) là một nghiệm kháccủa (6.4). Theo hệ thức Viete ta có:{

x0 +x1 = 2y0(2n −1)x0x1 = y2

0 −n

với x1 nguyên.Nếu y2

0 < n thì x1 < 0 ⇒ x1 ≤−1 ⇒ x21 +2x1 y0(1−2n)+ y2

0 −n ≥ 2y0(2n −1)+ y20 −n > 0 do

2y0(2n −1) ≥ 2n > n ta có điều vô lý.Suy ra x1 nguyên không âm. Xét các trường hợp:

• x1 = 0 ⇒ x0 = 2y0(2n−1) và n = y20 . Thay vào phương trình (6.4) có x0 = 2y0(2y2

0−1).Do đó mọi n chính phương đều thoả mãn.

• x1 nguyên dương thì (x1; y0) là một nghiệm nguyên dương của phương trình(6.4), do đó:

x1 + y0 ≥ x0 + y0 ⇒ x1 ≥ x0 ⇒ y20 −n ≥ x2

0 ≥ y20 ⇒ −n ≥ 0

vô lý.

Vậy n chính phương để phương trình (6.4) có nghiệm nguyên dương.

Tìm tất cả các bộ ba số nguyên (x; y ; z) thỏa mãn 12x + y4 = 56z .

(Hà Nội)

Bài 10

Lời giải

Nếu 1 trong 3 số x, y, z bằng 0 ta dễ dàng suy ra 2 số còn lại cũng bằng 0.Xét x, y, z khác 0. Ta có x, z phải nguyên dươngXét mod 3, ta thấy z chẵn và y không chia hết cho 3. Đặt z = 2k. Ta có: 12x = (56k −y2)(56k + y2). Vì (56k − y2)+ (56k + y2) không chia hết cho 3 nên có dạng:56k + y2 = 3x .4a1 ,56k − y2 = 4a2 hoặc 56k − y2 = 3x .4a1 ,56k + y2 = 4a2 .

TH1: 56k − y2 = 3x .4a1 , 56k + y2 = 4a2

Do đó: 2.y2 ≡ 4a2 (mod 3) ( Vô lý vì 2.y2 ≡ 2 (mod 3) mà 4a2 ≡ 1 (mod 3))

TH2: 56k + y2 = 3x .4a1 , 56k − y2 = 4a2

Khi đó y2+22a2 = 56k . Mặt khác 56 có ước 7 là số nguyên tố dạng 4t+3 ⇒ y...7; 2

...7(vô lý)

Vậy PT có nghiệm x = y = z = 0.

http://thcmn.wordpress.com 187

Page 188: t 2017 - WordPress.com...Bài 5. (B›c Ninh) Cho a, b c¨0 thäa mãn đi•u ki»n ¯ ˘9. Tìm giá trà lîn nh t cıabi”uthøc T ˘ ab 3a¯4b¯5c ¯ bc 3b¯4c¯5a ¯ ca 3c¯4a¯5b

Tìm tất cả các cặp số nguyên dương (x; y) thỏa mãn

x19 −1 = (x −1)(y12 −1).

(Hà Tĩnh)

Bài 11

Lời giải

Trước hết xin phát biểu và chứng minh bổ đề sau đây.

Bổ đề 13 Với x là số nguyên dương, p | x19 −1

x −1thì p=19 hoặc p = 19n +1.

Chứng minh 1 Giả sử p 6≡ 1(mod19) do đó (p − 1;19) = 1 ⇒ Tồn tại k,m thỏa mãn:

19k + (p −1)m = 1 ⇒ x ≡ x19k+(p−1)m = (x19)k .(xp−1)m ≡ 1( mod p) ⇒ x19 −1

x −1= x18 + ...+1 ≡

19( mod p). Vậy p = 19.

Quay lại bài toán. Từ bổ đề suy ra mỗi ước d củax19 −1

x −1thỏa mãn d = 19k, hoặc

d = 19k +1.

Giả sử (x, y) là nghiệm của PT, ta có y −1 | x19 −1

x −1y = 1 → x = 1Xét y ≥ 2. Theo bổ đề thì y = 19k +1, hoặc y = 19k +2.Do đó y11 + y10 + ...+1 ≡ 0,1( mod 19)

Mặt khác y11 + y10 + ...+1 | x19 −1

x −1nên y11 + y10 + ...+1 ≡ 1,2( mod 19) (mâu thuẫn)

Vậy phương trình chỉ có nghiệm (x; y) là (1;1).

Tìm số nguyên dương n nhỏ nhất sao cho 1n +2n + . . .+2016n không chia hết cho2017.

(Hải Dương)

Bài 12

Lời giải

Ta chứng minh bài toán tổng quát: Cho p là số nguyên tố lẻ. Chứng minh rằng 1k +2k + . . .+ (p −1)k ... p với mọi k 6 ... p −1. Thật vậy, mỗi số nguyên tố p có ít nhất một cănnguyên thủy m. Khi đó {m;m2; . . . ;mp−1} ≡ {1;2; . . . ; p −1}.Vì thế

1k +2k + . . .+ (p −1)k = mk + (m2)k + . . .+ (mp−1)k = mk (mk )p−1 −1

mk −1.

Do m là căn nguyên thủy của p và k 6 ... p −1 nên mk −1 6 ... p, lại có (mk )p−1 −1... p. Suy ra

đpcm.Từ bài toán trên suy ra số n nhỏ nhất cần tìm là 2016.

188 http://www.facebook.com/thcmn

Page 189: t 2017 - WordPress.com...Bài 5. (B›c Ninh) Cho a, b c¨0 thäa mãn đi•u ki»n ¯ ˘9. Tìm giá trà lîn nh t cıabi”uthøc T ˘ ab 3a¯4b¯5c ¯ bc 3b¯4c¯5a ¯ ca 3c¯4a¯5b

1. Cho p là một số nguyên tố lẻ. Chứng minh rằng nếu or d p2 (2) = p(p −1) và

vp (2p(p−1) −1) = 2 thì or d pk (2) = pk−1(p −1) và vp (2pk−1(p−1) −1) = k,∀k ≥ 2,trong đó or db(a) là cấp của a theo mod b và vp (a) là số mũ đúng của a theomod b.

2. Có bao nhiêu số nguyên dương n ≤ 10.3310 thỏa mãn đồng thời

2n ≡ 2017 ( mod 311) ; 2n ≡ 2016 ( mod 113)?

(Hải Phòng)

Bài 13

Lời giải

1. Giả sử khẳng định đúng với k ≥ 2, đặt a = 2pk−1(p−1) ta có

al −1 = (a −1)(al−1 +al−2 + ...+a +1

)≡ (a −1) .l

(mod pk+1

).

Do đó

al −1 ≡ 0(

mod pk+1)⇔ l ≡ 0

(mod p

)⇒ or dpk+1 (2) = (p −1

).pk

vàvp

(ap −1

)= vp (a −1)+1 ⇒ vp

(2(p−1).pk −1

)= k +1.

2. Để ý rằngor d32 (2) = 2.3 = 6; v3

(26 −1

)= 2

vàor d112 (2) = 10.11 = 110; v11

(2110 −1

)= 2

nên ⇒{

20;21;22; ...;22.310−1}

và{

20;21;22; ...;210.112−1}

lần lượt là các hệ thặng dư

thu gọn theo các module 311 và module 113. Vì vậy ∃n1,n2 sao cho 2n1 ≡ 2017(

mod 311)

2n2 ≡ 2016(

mod 113) ⇒

{2n1 ≡ 1

(mod 3

)2n2 ≡ 3

(mod 11

) ⇒{

n1 = 2k1

n2 = 10k ′+8 = 2k2

Mà(310;5.112

)= 1 nên theo định lý thặng dư Trung Hoa ∃k :

k ≡ k1

(mod 310

)k ≡ k2

(mod 5.112

) .

Đặt n = 2k thì

n ≡ n1

(mod 2.310

)n ≡ n2

(mod 10.112

) ⇒ 2n ≡ 2n1 ≡ 2017

(mod 311

)2n ≡ 2n2 ≡ 2016

(mod 113

) . Suy ra hệ

trên có nghiệm duy nhất theo module BC N N[

2.310;10.112]= 10.310.112. Như vậy

có 118 số nguyên dương n thỏa mãn.

http://thcmn.wordpress.com 189

Page 190: t 2017 - WordPress.com...Bài 5. (B›c Ninh) Cho a, b c¨0 thäa mãn đi•u ki»n ¯ ˘9. Tìm giá trà lîn nh t cıabi”uthøc T ˘ ab 3a¯4b¯5c ¯ bc 3b¯4c¯5a ¯ ca 3c¯4a¯5b

Với mỗi số nguyên dương n lớn hơn 1, số1

nđược biểu diễn dưới dạng thập phân

như sau1

n= 0, a1a2 . . .. Tìm tất cả các số n sao cho n = a1 +a2 + . . ..

(Tp.HCM)

Bài 14

Lời giải

Vì1

n= 0, a1a2 . . . có hữu hạn số sau dấu phẩy nên ta đặt

1

n= 0, a1a2 . . . am = a1a2 . . . am

10m

⇒ n.a1a2 . . . am = 10m cho nên n chỉ có ước nguyên tố là 2 và 5. Đặt n = 2p 5q . Kí hiệus(x) là tổng các chữ số của x trong hệ thập phân. Ta có s(x) ≤ x.

• Nếu q ≥ p ⇒ 2p 5q = s(2q−p)≤ 2q−p < 2p 5q (vô lý).

• Nếu p > q ⇒ 2p 5q = s(5p−q)

. Với p ≥ 6 ⇒ 2p ≤ 2p 5q = s(5p−q) ≤ 9

(p −q

) ≤ 9p < 2p

(vô lý). Do đó 1 ≤ p ≤ 5 ⇒ s(5p−q) ∈ {

5,7,8,13,11}. Mặt khác n chẵn nên s

(5p−q)

chẵn. Vì thế s(5p−q)= 8 ⇒ p −q = 3. Thử các trường hợp ta thấy p = 3, q = 0 thỏa

mãn.

Vậy n = 8.

Cho các số nguyên tố thỏa mãn p1 < p2 < p3 < p4 và p4 − p1 = 8. Giả sử p1 > 5.Chứng minh rằng p1 chia 30 dư 11.

(Khánh Hòa)

Bài 15

Lời giải

Từ giả thiết thì p2, p3 chỉ có thể nhận 2 trong 3 giá trị {p1 +2; p1 +4; p1 +6}.Nếu p2 = p1 +2; p3 = p1 +4 ⇒ Trong 3 số p1, p2, p3 có 1 số chia hết cho 3, vô lý.Nếu p2 = p1 +4; p3 = p1 +6 ⇒ Trong 3 số p2, p3, p4 có 1 số chia hết cho 3, vô lý.Do đó p2 = p1 +2; p3 = p1 +6; p4 = p1 +8. Từ đó suy ra p1 ≡ 2( mod 3) và p1 ≡ 1( mod 5).Cộng thêm việc p1 lẻ suy ra p1 ≡ 11( mod 30).

Chứng minh rằng nếu n và m = 2+2√

12n2 +1 là các số nguyên dương thì m là sốchính phương.

(Kiên Giang)

Bài 16

Lời giải

190 http://www.facebook.com/thcmn

Page 191: t 2017 - WordPress.com...Bài 5. (B›c Ninh) Cho a, b c¨0 thäa mãn đi•u ki»n ¯ ˘9. Tìm giá trà lîn nh t cıabi”uthøc T ˘ ab 3a¯4b¯5c ¯ bc 3b¯4c¯5a ¯ ca 3c¯4a¯5b

m ∈N∗ ⇔ 12n2 +1 = (2k +1)2 với k nguyên dương.Ta có

3n2 = k(k +1)

(k;k +1) = 1 nên ta có 2 trường hợp sau:

TH1:

{k = 3a2

k +1 = b2 Với (a;b) = 1, ab = n ⇒ 3a2 +1 = b2. Do đó 12n2 +1 = (2k +1)2 =

(3a2 +b2)2 = (2b2 −1)2 ⇒ m = 2+2(2b2 −1) = (2b)2. Ta có m là số chính phương.

TH2:

{k = b2

k +1 = 3a2 Với (a;b) = 1, ab = n ⇒ 3a2 = b2 + 1. Điều này vô lí vì b2 + 1 ≡1,2( mod 3).

Vậy ta có đpcm.

Tìm tất cả các cặp số nguyên (x; y) thỏa mãn

9x2 +24x +15 = y3.

(Lào Cai)

Bài 17

Lời giải

Đặt a = 3x +4. Phương trình được viết lại như sau

a2 −1 = y3 ⇔ a2 = (y +1)(y2 − y +1).

Do a ≡ 1 (mod 3 ) nên VP không chia hết cho 3. Do đó UC LN (y +1; y2 − y +1) = 1, tứclà y +1 = b2 ; y2 − y +1 = c2.Thay y = b2 −1 vào, ta được

(2b2 −3)2 +3 = 4c2 ↔ (2c −2b2 +3)(2c +2b2 −3) = 3.

Ta có 4 trường hợp sau:

TH1:

{2c −2b2 +3 = 12c +2b2 −3 = 3

⇒{

c = 1b2 = 2

(loại).

TH2:

{2c −2b2 +3 = 32c +2b2 −3 = 1

⇒{

c = 1b2 = 1

⇒{

y = 0a = 1

⇒{

y = 0x =−1

TH3:

{2c −2b2 +3 =−12c +2b2 −3 =−3

⇒{

c =−1b2 = 1

⇒{

y = 0a = 1

⇒{

y = 0x =−1

TH4:

{2c −2b2 +3 =−32c +2b2 −3 =−1

⇒{

c =−1b2 = 2

(loại).

Vậy phương trình có nghiệm nguyên (x; y) là (−1;0).

http://thcmn.wordpress.com 191

Page 192: t 2017 - WordPress.com...Bài 5. (B›c Ninh) Cho a, b c¨0 thäa mãn đi•u ki»n ¯ ˘9. Tìm giá trà lîn nh t cıabi”uthøc T ˘ ab 3a¯4b¯5c ¯ bc 3b¯4c¯5a ¯ ca 3c¯4a¯5b

Cho đa thức P (x) = 4x3 −18x2 +27x +m. Chứng minh rằng với mỗi m ∈ Z, ∃n ∈ Zsao cho P (n)

...107.

(Lạng Sơn)

Bài 18

Lời giải

Trước hết xin phát biểu lại một bổ đề quen thuộc.

Bổ đề 14 Cho p là số nguyên tố dạng 3k +2. Khi đó

x2 +x y + y2 ... p ⇔ x, y... p.

Chứng minh bổ đề này dễ dàng xin dành lại cho bạn đọc.

Quay lại bài toán. Ta chứng minh

A = {4x3 −18x2 +27x|x ≡ 0,1, . . . ,106 ( mod 107)} là hệ thặng dư đầy đủ mod 107. (6.5)

Thật vậy: đặt f (x) = 4x3 −18x2 +27x

Xét x 6≡ y ( mod 107) ⇒ x − y 6 ... 107ta có : f (x)− f (y) = 4(x − y)(4x2 +4x y +4y2 −18x −18y +27)

Nếu f (x) ≡ f (y) ( mod 107) ⇒ (4x2 +4x y +4y2 −18x −18y +27)... 107

Lại có 4x2 +4x y +4y2 −18x −18y +27 = (2x −3)2 + (2x −3)(2y −3)+ (2y −3)2 và 107 là số

nguyên tố chia 3 dư 2 ⇒ 2x −3... 107 và 2y −3

... 107 ⇒ x − y... 107 (vô lý)

Áp dụng bổ đề trên ta có đpcm.

Nhận xét 39 Một số bài toán tương tự:

1. Cho đa thức P (x) = 1

630x9− 1

21x7+ 13

30x5− 82

63x3+ 32

35x. Chứng minh rằng P (x) nhận

giá trị nguyên với mọi x nguyên.

2. Cho đa thức P (x) = x5

5+ x3

3+ 7x

15. Chứng minh rằng P (x) nhận giá trị nguyên với

mọi x nguyên.

Cho số nguyên dương n và d1 < d2 < d3 < d4 là 4 ước nguyên dương nhỏ nhất củan. Tìm tất cả các số nguyên dương n thỏa mãn n = d 2

1 +d 22 +d 2

3 +d 24 .

(Long An)

Bài 19

Lời giải

192 http://www.facebook.com/thcmn

Page 193: t 2017 - WordPress.com...Bài 5. (B›c Ninh) Cho a, b c¨0 thäa mãn đi•u ki»n ¯ ˘9. Tìm giá trà lîn nh t cıabi”uthøc T ˘ ab 3a¯4b¯5c ¯ bc 3b¯4c¯5a ¯ ca 3c¯4a¯5b

d1 = 1Nếu n lẻ thì d1,d2,d3,d4 cùng lẻ suy ra d 2

1 +d 22 +d 2

3 +d 24 chẵn vô lý.

Do đó n chẵn ⇒ d2 = 2 ⇒ n = 5+d 23 +d 2

4Từ đó có d3 và d4 khác tính chẵn lẻ.

Khi đó n ≡ d 21 +d 2

2 +d 23 +d 2

4 ≡ 2( mod 4) ⇒ n 6 ... 4 ⇒ d3 = p ; d4 = 2p với p là số nguyên tố.

Mà như thế thì 5... p nên p = 5.

Từ đó tính được n = 130

Tìm tất cả các số nguyên dương n có tính chất: nếu a,b là ước của n và (a,b) = 1thì a +b −1 cũng là ước của n.

(Nam Định)

Bài 20

Lời giải

n = 1 hiển nhiên thỏa mãn.Xét n ≥ 2 ta có các trường hợp sau:

TH1 : n = pr (trong đó p là số nguyên tố, r là số tự nhiên) dễ thấy thỏa mãn đề bài.

TH2 : n = s.pr với p là ước nguyên tố nhỏ nhất của n, r là số nguyên dương, s là sốnguyên dương > 1 và không chia hết cho p.(p; s) = 1 ⇒ (p + s −1) | n

– Giả sử q | p + s −1 với q là ước nguyên tố của s.Ta có s < p + s −1 < s +q vô lý vì s và s +q là 2 bội liên tiếp của q.Do đó p + s −1 chỉ có ước nguyên tố là p.Đặt p + s −1 = pk ,k ∈N∗,k ≥ 2 ⇒ s = pk −p +1

– n... pk ⇒ r ≥ k

(pk ; s) = 1 ⇒ s +pk −1 | n ⇒ 2pk −p | n ⇒ 2pk−1 −1 | n.

Mà (2pk−1 −1; p) = 1 ⇒ s... 2pk−1 −1

⇒ pk −p +1... 2pk−1 −1

⇒ 2pk −2p +2... 2pk−1 −1

⇒ p −2... 2pk−1 −1

Với k ≥ 3 ⇒ 2pk−1 −1 ≥ 2p2 −1 > p −2 vô lý.

Nên k = 2 ⇒ p −2... 2p −1 ⇒ 3

... 2p −1 ⇒ p = 2 ⇒ n = 2r (2k −1)

– Khi đó s = 2k −1 cũng thỏa mãn tính chất như số n.Nếu s có nhiều hơn 1 ước nguyên tố, gọi t là ước nguyên tố nhỏ nhất củas, cmtt như trên ta có t = 2 vô lý do s lẻ.Do đó s = t u với t nguyên tố lẻ.Khi đó n = 2r t u .Với r ≥ 2 ta có: (2; t ) = 1 ⇒ 2+ t − 1 | n ⇒ t + 1 | n ⇒ t + 1 = 2k và (4; t ) = 1 ⇒4+ t −1 | n ⇒ t +3 | n ⇒ t +3 = 2h ⇒ h = 2,k = 1 ⇒ t = 1 vô lý.Do đó r = 1.Lại có t +1 | n ⇒ t +1 = 2 ⇒ t = 1 vô lý.

http://thcmn.wordpress.com 193

Page 194: t 2017 - WordPress.com...Bài 5. (B›c Ninh) Cho a, b c¨0 thäa mãn đi•u ki»n ¯ ˘9. Tìm giá trà lîn nh t cıabi”uthøc T ˘ ab 3a¯4b¯5c ¯ bc 3b¯4c¯5a ¯ ca 3c¯4a¯5b

Vậy n = pr thỏa mãn đề, với p nguyên tố và r là số tự nhiên.

Cho số nguyên dương a không chính phương. Chứng minh số

{p

a}+ {p

a}2 + . . .+ {p

a}n

là số vô tỷ.

(Nghệ An)

Bài 21

Lời giải

S = {p

a}+ {p

a}2 + . . .+ {p

a}n = 1− {p

a}n+1

1− {p

a}

Giả sử S là số hữu tỷ, đặt S = p

qtrong đó (p, q) = 1.

Mà {p

a} < 1 nên {p

a}n+1 < {p

a}, suy ra p > q.Đặt a = k2 +x trong đó 1 ≤ x ≤ 2k ta có {

pa} =p

a −k

Từ S = p

qsuy ra

{p

a}(p −q{p

a}n) = p −q ⇒ (p

a −k)(p −q(p

a −k)n) = p −q (6.6)

Khai triển vế trái ta được số có dạng Ap

a +B với

A = p +kq

[n

2]∑

i=0(C 2i+1

n (−k)n−2i−1ai )−q

[n

2]∑

i=0(C 2i

n (−k)n−2i ai )

= p −q

[n

2]∑

i=0((C 2i+1

n +C 2in )(−k)n−2i ai )

Từ đây ta có A 6= 0 do p 6 ... q. Do đó (6.6) vô lý.Vậy ta có đpcm.

1. Tồn tại hay không các số thực phân biệt a,b sao cho a+b ∉Q nhưng an+bn ∈Q với mỗi số nguyên dương n > 1?

2. Cho a,b là các số thực khác nhau sao cho an −bn là số nguyên với mỗi sốnguyên dương n. Hỏi a,b là nguyên, hữu tỷ hay vô tỷ?

(Nghệ An)

Bài 22

Lời giải

194 http://www.facebook.com/thcmn

Page 195: t 2017 - WordPress.com...Bài 5. (B›c Ninh) Cho a, b c¨0 thäa mãn đi•u ki»n ¯ ˘9. Tìm giá trà lîn nh t cıabi”uthøc T ˘ ab 3a¯4b¯5c ¯ bc 3b¯4c¯5a ¯ ca 3c¯4a¯5b

1. Ta có a4 +b4 = (a2 +b2)−2a2b2 mà a4 +b4 ∈Q và a2 +b2 ∈Q nên a2b2 ∈QLại có a5 +b5 = (a2 +b2)(a3 +b3)− a2b2(a +b) mà a2 +b2, a3 +b3, a5 +b5, a2b2 ∈ Qnên a +b ∈Q vô lý.Vậy không tồn tại hay không các số thực phân biệt a,b sao cho a +b ∉Q nhưngan +bn ∈Q với mỗi số nguyên dương n > 1

2. • a,b ∈Z hiển nhiên đúng.

• Vì a −b và a2 −b2 nguyên nên a +b ∈Z. Do đó 2a ∈Q⇒ a ∈Q⇒ b ∈Q.

• Với a,b ∈Q. Đặt a = p

q,b = r

tvới p, q,r, t là các số nguyên, q, t 6= 0 và (p; q) =

(r ; t ) = 1.

a −b = pt −qr

qt∈Z⇒ pt

... q, qr... t ⇒ t

... q, q... t ⇒ q = t

Từ đó ta có a = p

q,b = r

qvà p − r

... q

Lại có an −bn ∈Z∀n nguyên dương nên pn − r n ... qn ∀n nguyên dương.Gọi s là 1 ước nguyên tố của q và vs(q) = m, vs(p − r ) = u với u ≥ m

– s = 2 ⇒ p,r lẻ.Xét n chẵn. Theo bổ đề LTE:

v2(pn − r n) = v2(p − r )+ v2(p + r )+ v2(n)−1 = v2(p + r )+u + v2(n)−1

v2(qn) = nm

Chọn n sao cho n 6 ... q và nm > u + v2(p + r ) ta sẽ có điều vô lý.– s lẻ.

Theo bổ đề LTE:

vs(pn − r n) = vs(p − r )+ vs(n) = u + vs(n)

vs(qn) = nm

Chọn n sao cho n 6 ... q và nm > u ta sẽ có điều vô lý.

Vậy a,b là các số nguyên.

Chứng minh rằng với mỗi số nguyên dương m luôn tồn tại vô số số nguyên dương

n thỏa mãn (3.2n +n)...m.

(Ninh Bình)

Bài 23

Lời giải

Gọi P (m) là mệnh đề cần chứng minh.

• Dễ thấy P (1),P (2) đúng.

http://thcmn.wordpress.com 195

Page 196: t 2017 - WordPress.com...Bài 5. (B›c Ninh) Cho a, b c¨0 thäa mãn đi•u ki»n ¯ ˘9. Tìm giá trà lîn nh t cıabi”uthøc T ˘ ab 3a¯4b¯5c ¯ bc 3b¯4c¯5a ¯ ca 3c¯4a¯5b

• Giả sử mệnh đề đúng đến m = k > 2. Khi đó tồn tại vô số số nguyên dương n sao

cho (3.2n +n)...k.

• Ta sẽ chứng minh P (k + 1) đúng, tức là chứng minh tồn tại vô số số nguyên

dương s sao cho (3.2s + s)...k +1. Thật vậy, xét số φ(k +1). Để ý là φ(k +1) ≤ k nên

tồn tại vô số số nguyên dương u sao cho (3.2u +u)...φ(k +1).

Bây giờ ta chọn s > 2u sao cho

{s ≡−3.2u ( mod k +1)s ≡−3.2u ( mod φ(k +1))

. Rõ ràng có vô số số s

như vậy.

Bây giờ ta sẽ chứng minh 3(2s −2u) ...(k +1).

– Nếu k + 1 chẵn, vì s + 3.2u...(k + 1), (s > 2u, s chẵn) nên s = 2r .b (r ≥ u, b lẻ)

⇒ v2(k +1) < u ⇒ 3.2u (2s−u −1

) ...(k +1) ⇒ 3(2s −2u) ...(k +1).

– Nếu k +1 lẻ, theo cách chọn s thì s −u ≡ −3.2u −u ≡ 0 (mod φ(k +1). Theo

định lí phi - hàm Euler, 2s−u −1...2φ(k+1) −1

...(k +1) ⇒ 3(2s −2u) ...(k +1).

Kết hợp hai điều trên suy ra luôn có 3(2s −2u) ...(k +1). Mà 3.2u + s

...(k +1) ⇒3.2s + s

...(k +1), và ta có đpcm.

Với mỗi số nguyên dương n, gọi f (n) là số cách chọn các dấu cộng, trừ trong biểuthức En =±1±2±·· ·±n sao cho En = 0. Chứng minh rằng:

1. f (n) = 0 khi n ≡ 1,2( mod 4).

2. Khi n ≡ 0,3 ( mod 4) ta có

p2n

2≤ f (n) ≤ 2n −2

1+[

n

2

].

(Phú Thọ)

Bài 24

Lời giải

1. Ta cón∑

i=1= n(n +1)

2do đó f (n) 6= 0 ↔ tồn tại một số số trong tập {1;2; . . . ;n} có

tổng bằngn(n +1)

4mà (n;n + 1) = 1 nên n ≡ 0,3(mod4). Vì thế f (n) = 0 khi n ≡

1,2 ( mod 4).

2. Chúng ta sẽ lần lượt chứng minh các bước như sau:

(a) Chứng minh f (n) ≥ 2n2 −1.

• Với n = 3 và n = 4, mệnh đề trên hiển nhiên đúng.

• Ta chứng minh rằng nến n đúng thì n +4 cũng đúng. Xét dấu của cácsố n +1;n +2;n +3;n +4.

196 http://www.facebook.com/thcmn

Page 197: t 2017 - WordPress.com...Bài 5. (B›c Ninh) Cho a, b c¨0 thäa mãn đi•u ki»n ¯ ˘9. Tìm giá trà lîn nh t cıabi”uthøc T ˘ ab 3a¯4b¯5c ¯ bc 3b¯4c¯5a ¯ ca 3c¯4a¯5b

– Nếu ta đặt các dấu là (n +1)− (n +2)− (n +3)+ (n +4) hoặc − (n +1)+(n +2)+ (n +3)− (n +4) và các dấu còn lại đặt như các bộ f (n) thì tacó bộ mới thỏa yêu cầu đề bài.

– Đối với các bộ thuộc f (n) có −1 thì ta đổi thành+1 rồi thêm (n +1)−(n +2)+ (n +3)− (n +4) thì có được bộ mới.

– Đối với các bộ thuộc f (n) có+1 thì ta đổi thành−1 rồi thêm− (n +1)+(n +2)− (n +3)+ (n +4) thì có được bộ mới.

– Đối với các bộ thuộc f (n) có −2 thì ta đổi thành+2 rồi thêm (n +1)+(n +2)− (n +3)− (n +4) thì có được bộ mới.

– Đối với các bộ thuộc f (n) có+2 thì ta đổi thành−2 rồi thêm− (n +1)−(n +2)+ (n +3)+ (n +4) thì có được bộ mới.

Vậy f (n +4) ≥ 4 f (n) ≥ 2n+4

2 −1. Suy ra đpcm.

(b) Chứng minh f (n) < 2n −2

[n2

]+1. Gọi g (n) là số các bộ dấu sao cho tổng trên

khác 0. Ta sẽ chứng minh g (n) > 2

[n2

]+1 bằng quy nạp.

• Với n = 3 và n = 4 thì ta có mệnh đề trên đúng.• Ta chứng minh với n đúng thì n +4 cũng đúng.

Xét dấu của các số n +1;n +2;n +3;n +4Đối với các bộ thuộc f (n), ta đặt dấu các số n+1;n+2;n+3;n+4 để tổngđại số chúng khác 0. Có 14 cách đặt dấu cho mỗi bộ như vậyĐối với các bộ thuộc g (n), ta đặt dấu các số n+1;n+2;n+3;n+4 để tổngđại số chúng khác 0. Có 2 cách đặt dấu cho mỗi bộ như vậy.

Suy ra g (n +4) ≥ 14 f (n)+2g (n) > 14.2n2 −1 +2.2

[n2

]+1 > 2

[n+4

2

]+1

Ta có đpcm.

Cho đa thức f (x) = x2017 + ax2 + bx + c, trong đó a,b,c ∈ Z có 3 nghiệm nguyênx1, x2, x3. Chứng minh rằng biểu thức sau là bội của 2017:

(a2017 +b2017 + c2017 +1)(x1 −x2)(x2 −x3)(x3 −x1).

(Quảng Bình)

Bài 25

Lời giải

Nếu có 2 trong 3 số x1; x2; x3 đồng dư mod 2017 ta suy ra đpcm.Nếu không có 2 số nào trong 3 số x1; x2; x3 đồng dư theo mod 2017:Ta có f (x) ≡ ax2 + (b +1)x + c ( mod 2017)Và f (x1)− f (x2) ≡ (x1 −x2)(ax1 +ax2 +b +1)( mod 2017)

Do đó ax1 +ax2 +b +1... 2017

Tương tự ax2 +ax3 +b +1... 2017

Suy ra a(x1 −x3)... 2017 ⇒ a

... 2017

Suy ra b +1... 2017. Lại có f (x1) = 0 nên c cũng chia hết cho 2017.

Khi đó a2017 +b2017 + c2017 +1 ≡ a +b + c +1 ≡ 0( mod 2017)Ta có đpcm.

http://thcmn.wordpress.com 197

Page 198: t 2017 - WordPress.com...Bài 5. (B›c Ninh) Cho a, b c¨0 thäa mãn đi•u ki»n ¯ ˘9. Tìm giá trà lîn nh t cıabi”uthøc T ˘ ab 3a¯4b¯5c ¯ bc 3b¯4c¯5a ¯ ca 3c¯4a¯5b

Cho số nguyên tố p và các số nguyên dương a,b,c phân biệt nhỏ hơn p. Chứngminh rằng nếu các số a3,b3,c3 có cùng số dư khi chia cho p thì a2 +b2 + c2 chiahết cho a +b + c.

(Quảng Nam)

Bài 26

Lời giải

Rõ ràng p > 3 thì bài toán mới có nghĩa.

Từ giả thiết, ta suy ra (a −b)(a2 + ab +b2)... p, mà a 6= b và a,b < p nên a2 + ab +b2 ... p,

tương tự có b2+bc+c2 và c2+ca+a2 là bội của p. Từ đó a2+ab+b2−b2−bc−c2 ... p hay

(a − c)(a +b + c)... p, lại có a 6= c và a,c < p nên a +b + c

... p.Do a +b + c < 3p nên ta xét các trường hợp:

TH1: a +b + c = p :Ta có a +b ≡ −c (mod p) nên a2 +2ab +b2 ≡ c2 (mod p) hay (a2 + ab +b2)+ ab +bc + ca ≡ c(a +b + c) (mod p).

Từ đó ab +bc + ca... p. Để ý rằng

(a+b+c)2 = a2+b2+c2+2(ab+bc+ca) nên a2+b2+c2 ... p hay a2+b2+c2 ... a+b+c.

TH2: a+b+c = 2p. Tương tự trên, ta có a2+b2+c2 ... p, mà a2+b2+c2 ... 2 và (p,2) = 1 nên

a2 +b2 + c2 ... a +b + c.

1. Tìm mọi nghiệm nguyên của phương trình

x2 +2016x +2017y2 + y = x y +2017x y2 +2018

.

2. Cho m,n là các số nguyên dương, m là số lẻ. Chứng minh rằng 4m | 3n +1khi và chỉ khi 2m | 3n +1 và 3 | (2m)n +1.

(Quảng Ngãi)

Bài 27

Lời giải

1. x2 +2016x +2017y2 + y = x y +2017x y2 +2018↔ (x −1)(x +2017−2017y2 − y) = 1

→ 1... (x −1) ⇒ x −1 ∈ {1;−1} ⇒ x ∈ {2;0}

Với x = 0 ⇒ 2017−2017y2 − y =−1 ↔ 2017y2 + y −2018 = 0 → y = 1Với x = 2 ⇒ 2019−2017y2 − y = 1 ↔ 2017y2 + y −2018 = 0 ⇒ y = 1Vậy phương trình có 2 nghiệm nguyên (x; y) là (0;1) và (2;1).

198 http://www.facebook.com/thcmn

Page 199: t 2017 - WordPress.com...Bài 5. (B›c Ninh) Cho a, b c¨0 thäa mãn đi•u ki»n ¯ ˘9. Tìm giá trà lîn nh t cıabi”uthøc T ˘ ab 3a¯4b¯5c ¯ bc 3b¯4c¯5a ¯ ca 3c¯4a¯5b

2. (a) Chiều thuận:

• Nếu n chẵn, ta có 3n +1 ≡ 1+1 = 2(

mod 4)

(vô lý), do đó n lẻ.• Gọi p là một ước nguyên tố bất kỳ của m, vì m lẻ và không chia hết cho

3 nên p > 3. Ta có

3n +1 ≡ 0(

mod p)⇒ 3n+1 ≡−3

(mod p

)⇒ (−3

p

)= 1.

Theo luật tương phản Gauss, ta có(

p

3

)(3

p

)= (−1)

(p−1)(3−1)4 = (−1)

p−12

⇒(

p

3

)= (−1)

p−12

(3

p

)= (−1)

p−12

(−1

p

)(−3

p

)= (−1)p−1 = 1

⇒∃a/p ≡ a2 (mod 3

)⇒ p ≡ 1(

mod 3) ⇒ m ≡ 1

(mod 3

)Do đó (2m)n +1 ≡ (−1)n +1 = 0

(mod 3

)vì n lẻ.

(b) Chiều đảo:Nếu n chẵn thì (2m)n +1 ≡ 1;2

(mod 3

)( vô lý), do đó n lẻ

Suy ra 4 = (3+1) |(3n +1)⇒ 4m|3n +1, do

(4,m

)= 1

Cho số nguyên n ≥ 2 thỏa mãn điều kiện 2ϕ(n) + 3ϕ(n) + ...+nϕ(n) chia hết cho n,trong đó ϕ(n) là số các số nguyên dương ko vượt quá n và nguyên tố cùng nhauvới n.

1. Chứng minh rằng n ko có ước số lớn hơn 1 nào là số chính phương.

2. Biết rằng n có ko quá 3 ước nguyên tố, tìm tất cả các số n thỏa mãn điềukiện trên.

(Quảng Ninh)

Bài 28

Lời giải

1. Đặt n = pα11 pα2

2 ...pαkk , trong đó p1, p2, ..., pk là các số nguyên tố đôi một khác nhau.

Giả sử n có ước số là số chính phương, do đó tồn tại αi ≥ 2(1 ≤ i ≤ k

), giả sử đó

là α1.

Ta có ϕ (n) = n∏p|n

(1− 1

p

)⇒ (

p1 −1) |n

Có n|n∑

i=2iϕ(n) ⇒ p1|

n∑i=2

iϕ(n).

Từ 2 đến n có pα1−11 pα2

2 ...pαkk số chia hết cho p1 và n−1−pα1−1

1 pα22 ...pαk

k số nguyêntố cùng nhau với p1.

Theo định lý Fermat nhỏ, ta cón∑

i=2iϕ(n) ≡ n −1−pα1−1

1 pα22 ...pαk

k ≡ 0(

mod p1)

⇒ 1 ≡ 0(

mod p1)(vô lý).

Do đó giả sử sai ta có đpcm.

http://thcmn.wordpress.com 199

Page 200: t 2017 - WordPress.com...Bài 5. (B›c Ninh) Cho a, b c¨0 thäa mãn đi•u ki»n ¯ ˘9. Tìm giá trà lîn nh t cıabi”uthøc T ˘ ab 3a¯4b¯5c ¯ bc 3b¯4c¯5a ¯ ca 3c¯4a¯5b

2. Theo câu a, n có dạng phân tích chuẩn mực là p1p2...pk trong đó pi là các sốnguyên tố khác nhau.

Chứng minh tương tự câu a, ta có được pi |n − n

pi−1

(1 ≤ i ≤ k

).

TH1: k = 1 ta có n = p ∈P và ϕ(p) = p −1. Khi đóp∑

i=2iϕ(p) ≡ p −2 ≡ 0

(mod p

)⇒ n =p = 2

TH2: k = 2 ta có n = pq với p, q là 2 số nguyên tố phân biệt và p < q. Khi đó p | q+1và q | p +1 ⇒ p < q ≤ p +1 ⇒ q = p +1 ⇒ p = 2; q = 3 . Do đó n = 6

TH3: k = 3 ta có n = pqr trong đó p < q < r là các số nguyên tố. Khi đó p | qr +1

, q | pr +1 và r | pq +1 ⇒ pq + qr +pr +1... pqr . Đặt pq + qr +pr +1 = t pqr

trong đó t ∈N∗ thì t pqr < 4qr ⇒ t p ≤ 3 ⇒ p ∈ {2;3} và t = 1.

• Nếu p = 2 ⇒ 2q +2r +qr +1 = 2qr ⇒ (q −2)(r −2) = 5 ⇒ q = 3; r = 7 ⇒n = 42.

• Nếu p = 3 ⇒ 3q +3r +qr +1 = 3qr vô lí do vế trái chẵn, vế phải lẻ.

Vây n ∈ {2;6;42}.

Giải phương trình nghiệm nguyên dương

x y y x = (x + y)x+y .

(Thái Bình)

Bài 29

Lời giải

• x = 1 ⇒ y = (y +1)y+1, vô lí.

• y = 1 ⇒ x = (x +1)x+1 vô lí.

• Xét x, y > 1. Ta thấy ∀p ∈ P sao cho p | x thì p | y và ngược lại. Kí hiệu vp (x) là sốmũ đúng của p trong phân tích tiêu chuẩn của x. Đặt

x =k∏

i=1pαi

i , y =k∏

i=1pβi

i .

Nếu cóαi 6=βi , không mất tính tổng quát giả sửαi >βi . Theo giả thiết, vpi (x y y x) =vpi ((x+ y)x+y ) ⇒ yαi +xβi = (x+ y)βi ⇒αi =βi , vô lí. Do đó αi =βi∀i = 0,1, . . . ,k ⇒x = y ⇒ x2x = (2x)2x ⇒ x = 2x, vô lí do x nguyên dương.

Vậy phương trình không có nghiệm (x; y) nguyên dương.

200 http://www.facebook.com/thcmn

Page 201: t 2017 - WordPress.com...Bài 5. (B›c Ninh) Cho a, b c¨0 thäa mãn đi•u ki»n ¯ ˘9. Tìm giá trà lîn nh t cıabi”uthøc T ˘ ab 3a¯4b¯5c ¯ bc 3b¯4c¯5a ¯ ca 3c¯4a¯5b

7 Tổ hợp

Trên một giá sách có n cuốn sách được đánh số từ 1 đến n (n ∈N∗ cho trước), banđầu các cuốn sách được sắp xếp theo thứ tự nào đó. Một thủ thư muốn xếp lạitheo đúng thứ tự 1,2, ...,n từ trái qua phải, quy tắc xếp như sau: Chọn quyển sáchđầu tiên có số không đúng vị trí (tính từ bên phải sang) và chuyển cuốn sách đóvề đúng vị trí của nó. Ví dụ, trên giá có 4 quyển sách theo thứ tự 3−1−4−2, saumột bước chuyển quyển số 2 về đúng vị trí của nó ta xếp lại thành 3− 2− 1− 4.Chứng minh rằng người thủ thư có thể hoàn thành công việc của mình sau íthơn 2n lần xếp theo quy tắc.

(THPT chuyên KHTN, ĐH KHTN, ĐHQG HN )

Bài 1

Lời giải

Chúng ta sẽ phát biểu lại bài toán cho dễ trình bày hơn một chút. Cho một hoán vịcủa dãy số

{1,2,3, ...,n

}, viết từ trái sang phải. Ta sẽ chuyển dãy số này về đúng vị trí,

tức là{1,2,3, ...,n

}. Mỗi bước ta thực hiện như sau : Chọn số gần tay nhất mà không

đứng đúng vị trí rồi chuyển nó về vị trí đúng (ví dụ dãy 3 1 4 2 sau một bước chuyểnthì 2 về vị trí thức hai thành 3 2 1 4). Chứng minh rằng sau ít hơn 2n bước thì dãy luônchuyển về đúng vị trí.

Nhận xét 40 Mỗi bước thực hiện chỉ làm thay đổi vị trí của các số từ số được chọn đếnvị trí đúng của số đó. Cụ thể, phép biến đổi đẩy vị các số trong khoảng trên lên thêm 1đơn vị.

Nhận xét 41 Mọi số đã ở đúng vị trí thì sau đó sẽ chỉ ở từ vị trí đúng đó đến vị trí n.

Sau khi thực hiện mỗi bước ta được một tập hợp {a1, a2, ..., ak } là tập hợp các số ở đúngvị trí. Ta sẽ chứng minh không có hai bước phân biệt nào cho ta cùng một tập hợpcác số có vị trí đúng.Giả sử phản chứng, tồn tại hai bước thực hiện cho cùng một tập hợp {a1, a2, ..., ak } làx và y với bước x được thực hiện trước bước y và ai < ai+1 với mọi 1 ≤ i < k.Giả sử bước y đưa số ai vào đúng vị trí. Theo đề bài, rõ ràng từ sau vị trí của ai trướckhi biến đổi đến vị trí n, các số đều ở đúng vị trí. Suy ra ở sau bước x các số đó cũngđã ở đúng vị trí. Ngoài ra, dựa trên nhận xét 11 dễ thấy các số có vị trí đúng sau vịtrí trước bước y của ai đều đã ở đúng vị trí. Dựa trên nhận xét 10, và giả thiết phảnchứng, phép biến đổi y chỉ khiến ai về đúng vị trí, nói cách khác phép biến đổi chỉlàm tập hợp tăng thêm một phần tử ai . Vậy ta chỉ cần xét trường hợp i = k.Hiển nhiên vị trí của ak không phải là vị trí cuối. Dựa theo nhện xét 10, tập hợp mất đimột hay nhiều phần tử chỉ khi tập hợp đó được thêm vào một phần tử bé hơn nhữngphần từ vừa mất. Ban đầu sau bước x tập hợp có phần tử ak . Tuy nhiên tại bước yphần tử ak được thêm vào chứng tỏ trước đó có phần tử nhỏ hơn ak và khác tất cảcác phần từ ai với 1 ≤ i ≤ k. Phần tử này sau đó lại bị mất đi... chứng tỏ đến bước ytrong tập hợp vẫn tồn tại một phần tử khác tất cả các phần từ ai với 1 ≤ i ≤ k, vô lý!Có 2n tập hợp có thể có, tính cả tập hợp ban đầu khi chưa thực hiện phép biến đổi

http://thcmn.wordpress.com 201

Page 202: t 2017 - WordPress.com...Bài 5. (B›c Ninh) Cho a, b c¨0 thäa mãn đi•u ki»n ¯ ˘9. Tìm giá trà lîn nh t cıabi”uthøc T ˘ ab 3a¯4b¯5c ¯ bc 3b¯4c¯5a ¯ ca 3c¯4a¯5b

nào, trong trường hợp tệ nhất ta phải thực hiện nhiều nhất 2n −1 phép biến đổi. Đâychính là điều cần phải chứng minh.

Tìm số lớn nhất phần tử của một tập hợp là tập con của{1,2,3, ....2016

}thỏa mãn

hiệu hai phần tử bất kỳ khác 4 và 7.

(THPT chuyên KHTN, ĐH KHTN, ĐHQG HN )

Bài 2

Lời giải

Xét 11 số tự nhiên liên tiếp, giả sử đó là {a+1, a+2, ..., a+11}. Ta nhóm các số này thànhsáu nhóm: {a +1, a +8},{a +2, a +6},{a +3, a +7},{a +4, a +11},{a +5, a +9} và {a +10}.Dễ thấy ta không thể chọn hai số trong cùng một nhóm nên có thể chọn nhiều nhất6 số trong 11 số liên tiếp. Giả sử có thể chọn được 6 số trong 11 số liên tiếp đã cho.Theo cách chia nhóm trên, ta phải chọn số a+10, mỗi nhóm chọn thêm một số. Theođề bài ta không thể chọn a+6 và a+3. Dẫn đến ta phải lần lượt chọn a+2, a+7, a+5,a+8. Tuy nhiên khi đó trong nhóm {a+4, a+11} ta không thể chọn số nào do 11−4 = 7và 8−4 = 4. Vậy trong 11 số liên tiếp ta chỉ có thể chọn tối đa 5 số.Ta chia 2016 số tự nhiên đầu tiên thành 183 nhóm 11 số tự nhiên liên tiếp và nhóm{2016,2015,2014}. Dễ thấy nếu ta chọn cả ba số 2016,2015,2014 thì ta không thể chọnđủ 5 số trong nhóm {2003,2004, ...,2013}. Vậy số cần tìm là:183x5+2 = 917. Cách chọnlà: Ta chọn tất cả các số có số dư khi chia cho 11 thuộc tập hợp {1,2,4,7,10}.

Cho n nguyên dương. Các tâm thẻ trong một bộ sưu tập có giá trị m! với m là sốnguyên dương nào đó, Một bộ sưu tập tốt là một bộ sưu tập sao cho với mọi số kthỏa mãn k ≤ n!, luôn tồn tại một số tâm thẻ trong bộ sưu tập mà tồng giá trị cácthẻ này bằng k. Tìm số tấm thẻ ít nhất của bộ sưu tập tốt.

(THPT chuyên KHTN, ĐH KHTN, ĐHQG HN )

Bài 3

Lời giải

Để biểu diễn số n!−1, ta cần ít nhấtn(n −1)

2tấm thẻ. Thật vậy: Nếu có k tấm thẻ i ! thì

k ≤ i . Vậy ta có n!−1 = a1(n −1)!+ ...+ an−1.1! ≤ (n −1)(n −1)!+ ...+1.1! = n!−1. Vậy dấubằng xảy ra, nhận xét của ta là đúng và đây cũng là cách biểu diễn duy nhất cho n!−1

bằngn(n −1)

2tấm thẻ. Để biểu diễn n! ta cần thêm ít nhất 1 tấm thẻ nữa.

Dễ thấy với mọi k ≤ n!−1 ta luôn tìm được một số tấm thẻ đã dùng để biểu diễn n!−1có tổng bằng k. Thật vậy, ta chọn m lớn nhấn sao cho m! ≤ k. Sau đó ta chọn bm saocho bm .m! ≤ k. Dễ thấy bm ≤ m nên ta có thể tìm đủ bm tấm thẻ m!. Cứ làm như vậyđến hết, ta tìm được đủ các tấm thẻ để tổng các tấm thẻ này bằng k.

Vậy số cần tìm làn(n −1)

2+1. Bộ sưu tập tốt ví dụ gồm: 2 tấm thẻ 1!, 2 tấm thẻ 2!,..., i

tấm thẻ i !,...,n −1 tấm thẻ (n −1)!.

202 http://www.facebook.com/thcmn

Page 203: t 2017 - WordPress.com...Bài 5. (B›c Ninh) Cho a, b c¨0 thäa mãn đi•u ki»n ¯ ˘9. Tìm giá trà lîn nh t cıabi”uthøc T ˘ ab 3a¯4b¯5c ¯ bc 3b¯4c¯5a ¯ ca 3c¯4a¯5b

Với các số nguyên a,b,c,d thỏa 1 ≤ a < b < c < d , kí hiệu T (a,b,c,d) = {x, y, z, t ∈N |1 ≤ x < y < z < t , x ≤ a, y ≤ b, z ≤ c, t ≤ d}.

1. Tính số phần tử của T (1,4,6,7).

2. Cho a = 1 và b ≥ 4. Gọi d1 là số phần tử của T (a,b,c,d) chứa 1 và không chứa2, d2 là số phần tử chứa 1,2 nhưng không chứa 3, d3 là số phần tử chứa 1,2,3nhưng không chứa 4. Chứng minh rằng d1 ≥ 2d2 −d3. Dấu bằng xảy ra khinào?

(Trường Phổ thông Năng khiếu - ĐHQG Tp. HCM )

Bài 4

Lời giải

1. Với một phần tử bất kì, hiển nhiên x = 1.Dễ thấy T (1,4,6,7) = T (1,4,7,7).

• Với y = 4: Có3.2

2= 3 phần tử.

• Với y = 3: Có4.3

2= 6 phần tử.

• Với y = 2: Có5.4

2= 10 phần tử.

Vậy có tổng cộng 19 phần tử.

2. Đương nhiên ta chỉ xét các bộ (a,b,c,d) sao cho a ≤ b ≤ c ≤ d .

• Xét d3: Do các phần tử tính trong d3 chứa ba số 1,2 và 3 nên ba số này cũngchính là x, y và z. Khi đó t ≥ 5 nên ta có d3 = d −4.

• Xét d2: Tương tự như trên, ta có x = 1, y = 2. Vì z ≥ 4 nên z có thể nhận c −3

giá trị. Với mỗi giá trị của z, t có thể nhận d−z. Vậy d2 = (c −3)(d −4+d − c)

2=

(c −3)(2d − c −4)

2.

• Xét d1: Ta có x = 1, y ≥ 3 và có thể nhận b −2 giá trị, z có thể nhận c − y giátrị và t có thể nhận d −z giá trị. Ta cho lần lượt y = 3 và y = 4, dễ thấy trườnghợp y = 3 có số phần tử bằng với d2.

Trường hợp y = 4, tương tự, có số phần tử là(c −4)(2d − c −5)

2. Vậy d1 ≥ (c −4)(2d − c −5)

2+

(c −3)(2d − c −4)

2.

Khi đó ta có: d1−2d2+d3 ≥ (c −4)(2d − c −5)

2+ (c −3)(2d − c −4)

2−(c−3)(2d−c−4)+

(d −4).Tương đương với d1 −2d2 +d3 ≥ 0. Vậy ta có điều cần chứng minh.Dấu bằng xảy ra khi b = 4.

http://thcmn.wordpress.com 203

Page 204: t 2017 - WordPress.com...Bài 5. (B›c Ninh) Cho a, b c¨0 thäa mãn đi•u ki»n ¯ ˘9. Tìm giá trà lîn nh t cıabi”uthøc T ˘ ab 3a¯4b¯5c ¯ bc 3b¯4c¯5a ¯ ca 3c¯4a¯5b

Trong một hệ thống máy tính, một máy tính có thể kết nối trực tiếp với ít nhất30% máy tính khác của hệ thống. Hệ thống này có một chương trình ngăn chặnvà cảnh báo khá tốt,do đó khi một máy tính bị virus,nó chỉ đủ thời gian lây viruscho các máy tính được kết nối trực tiếp với nó. Chứng minh rằng dù vậy, kẻ tấncông vẫn có thể chọn hai máy tính của hệ thống mà nểu thả virus vào hai máyđó, ít nhất 50% máy tính của hệ thống sẽ bị nhiễm virus.

(Trường Phổ thông Năng khiếu - ĐHQG Tp. HCM)

Bài 5

Lời giải

Gọi tập hợp các máy tính trong hệ thống là X .

Bổ đề 15 Xét một tập con S bất kì của tập X . Khi đó tồn tại một máy tính của hệ thốngkết nối trực tiếp với ít nhất 30% máy tính trong tập S.

Chứng minh 2 Xét cặp (s, x) với s ∈ S và x ∈ X . Cho x chạy trong X và s chạy trong S

sao cho x và s kề nhau, tính theo s thì số cặp như vậy không ít hơn3

10.|X |.|S|. Do đó

khi tính theo x, phải tồn tại x sao cho x này kết nối trực tiếp với ít nhất 30% máy tínhthuộc S.

Trở lại bài toán. Xét máy tính A bất kì. Gọi S là tập hợp các máy tính không kết nốivới A. Nếu S rỗng thì kết quả bài toán là hiển nhiên đúng.Nếu S không rỗng thì theobổ đề, tồn tại một máy tính B khác A kết nối trực tiếp với ít nhất 30% máy tính trongS. Ta chọn hai máy tính A và B . Ta có:

|X |− |S|+ 3

10.|S| = |X |− 7

10.|S| ≥ |X |− 7

10.

7

10.|X | = 0,51|X |.

Vậy ta có điều cần chứng minh.

Bạn An có 12 tấm thẻ, trên mỗi tấm thẻ ghi một số nguyên từ 1 đến 12, các sốtrên các thẻ đều phân biệt.

1. Chứng minh rằng bạn An có thể chia 12 tấm thẻ đó thành một số nhómthỏa mãn tính chất (P ) như sau: trong mỗi nhóm có nhiều hơn một tấmthẻ đồng thời số lớn nhất ghi trên một tấm thể nào đó bằng tổng các số ghitrên tấm thẻ còn lại.

2. Nếu bạn An cho bạn Bình n tấm thẻ mang các số từ 1 đến n (1 ≤ n ≤ 12) thìvới những tấm thẻ còn lại bạn An có thể chia thành một số nhóm thỏa mãntính chất (P ) hay không?

(THPT chuyên ĐH Vinh)

Bài 6

204 http://www.facebook.com/thcmn

Page 205: t 2017 - WordPress.com...Bài 5. (B›c Ninh) Cho a, b c¨0 thäa mãn đi•u ki»n ¯ ˘9. Tìm giá trà lîn nh t cıabi”uthøc T ˘ ab 3a¯4b¯5c ¯ bc 3b¯4c¯5a ¯ ca 3c¯4a¯5b

Lời giải

1. Ta có cách chia nhóm thỏa mãn sau: (1,10,11); (2,5,7); (3,6,9); (4,8,12). Từ đó suyra mệnh đề cần chứng minh là đúng.

2. Giả sử tồn tại n sao cho nếu An cho Bình n tấm thẻ đầu tiên thì số thẻ còn lạivẫn có thể chia thành các nhóm thỏa mãn tính chất (P ).Do mỗi nhóm phải có ít nhất 3 phần tử nên khi đó không thể chia số tấm thẻcòn lại thành quá 3 nhóm.Do mỗi nhóm đều thỏa mãn (P ) nên tổng các phần tử của mỗi nhóm đều chiahết cho 2, dẫn đến tổng các thẻ còn lại cũng chia hết cho 2. Suy ra n ≥ 3Do đã mất hai tấm thẻ 1 và 2 nên phải tồn tại nhóm có số lớn nhất là 11 và mộtnhóm khác có số lớn nhất là 10. Hiển nhiên nhóm còn lại có số lớn nhất là 12.Vậy số thẻ An cho Bình là 12−3.3 = 3 thẻ.Vậy n = 3. Nhưng khi đó tổng các số trên các tấm thẻ còn lại bằng 4+5+6+...+12 =72 và 2(10+11+12) = 66, vô lí vì 72 không thể bằng 66!Vậy giả thiết phản chứng là sai. Ta có điều cần chứng minh.

1. Tính số hoán vị f (1); f (2); ...; f (2016) sao cho biểu thức

T = | f (1)−1|+ | f (2)−2|+ ...+| f (2016)−2016|

đạt trá trị lớn nhất, trong đó f (1) là giá trị ở vị trí thứ i trong mỗi hoánvị,i = 1,2,3, ...2016.

2. Trên mặt phẳng xét 42 điểm mà không có ba điểm nào thẳng hàng.Ta dựngcác đoạn thẳng nối hai điểm trong các điểm nói trên sao cho mọi bộ bađiểm đang xét luôn có hai điểm được nối với nhau. Tìm giá trị nhỏ nhấtcủa số đoạn thẳng cần dựng.

(Bà Rịa - Vũng Tàu)

Bài 7

Lời giải

1. Trong T , mỗi số tự nhiên thuộc khoảng [1;2016] xuất hiện hai lần, có 2016 dấucộng và 2016 dấu trừ. Như vậy, T đạt giá trị lớn nhất khi 2016 dấu cộng thuộc về2016

2= 1008 số lớn nhất, còn 2016 dấu trừ thuộc về 1008 số còn lại. Vậy Tmax =

2(2016∑

i=1009i−

1008∑i=1

i ) = 2(1008.1008) = 2.10082, đạt được khi ( f (1), f (2), ..., f (1008)) là hoán

vị của (1009,1010, ...,2016) và ( f (1009), f (1010), ..., f (2016)) là hoán vị của (1,2, ...,1008).Thật vậy, giả sử tồn tại i < 1009 sao cho f (i ) < 1009. Khi đó từ | f (i )−i | sẽ xuất hiệnmột số bé hơn 1009 nhưng mang dấu cộng, vô lí.Từ điều kiện dấu bằng, ta tính được số hoán vị là: 1008!1008! = (1008!)2.

2. Trước hết xin phát biểu và chứng minh bổ đề sau.

http://thcmn.wordpress.com 205

Page 206: t 2017 - WordPress.com...Bài 5. (B›c Ninh) Cho a, b c¨0 thäa mãn đi•u ki»n ¯ ˘9. Tìm giá trà lîn nh t cıabi”uthøc T ˘ ab 3a¯4b¯5c ¯ bc 3b¯4c¯5a ¯ ca 3c¯4a¯5b

Bổ đề 16 Trên mặt phẳng xét 2n điểm (n ≥ 2), nếu có n2+1 đoạn thẳng được nốithì hệ điểm có ít nhất một tam giác. Đây cũng là số cạnh ít nhất cần có để luôntồn tại tam giác.

Chứng minh 3

• Với n = 2, xét tập điểm {A,B ,C ,D}. Ta có số cặp hai điểm bất kì là4.3

2= 6. Suy

ra có đúng một cặp không được nối là AB .

• Giả sử bổ đề đúng với n = k, xét n = k + 1. Giả sử tồn tại cách nối sao chokhông tồn tại tam giác nào. Xét một tập con S có 2k điểm bất kì của tập hợp2(k +1) điểm đang xét sao cho hai điểm còn lại, đặt là A và B , được nối vớinhau. Nếu có quá k2 đoạn thẳng có hai đầu mút đều thuộc S thì trong Scó một tam giác theo giả thiết quy nạp. Xét bộ ba (A,B , X ) với X chạy trongS. Mỗi bộ ba như vậy có nhiều nhất 2 cạnh, nếu không tính AB thì chỉ cónhiều nhất 1 cạnh. Khi đó cho X chạy trong S ta được nhiều nhất 2k đoạnthẳng. Vậy số đoạn thẳng nhiều nhất là k2 +2k +1 = (k +1)2 ≥ (k +1)2 +1, vôlí! Nếu chỉ nối (k +1)2 đoạn thẳng, ta có cách nối như sau: Chia S thành hainhóm (k +1) điểm, mỗi điểm nối với tất cả các điểm khác nhóm với nó. Vớicách nối này, khi chọn 3 điểm bất kì, theo nguyên lí Dirichlet, luôn có haiđiểm chung nhóm, không được nối với nhau.

Vậy bổ đề đúng với n = k +1. Theo nguyên lí quy nạp, bổ đề đúng với mọi n ≥ 2.

Quay lại bài toán. Gọi tập hợp các đoạn thẳng cần nối là A với |A| nhỏ nhấtcó thể. Điều kiện đề bài tương đương với không tồn tại ba điểm nào đôi mộtkhông nối với nhau. Gọi S là tập hợp tất cả các cặp điểm. Khi đó theo bổ đề ta

có |S/A| ≤ (42

2)2. Suy ra |A| = |S|− |S/A| ≥ 42.41

2− (

42

2)2 = 420.

1. Xung quanh bờ hồ hình tròn có 2017 cây liễu. Người ta dự định chặt bớt 5cây liễu sao cho không có 2 cây liễu nào kề nhau bị chặt. Hỏi có bao nhiêucách thực hiện khác nhau ?

2. Một cuộc họp có 12k( k ∈ N∗) người, trong đó mỗi người bắt tay với đúng3k + 6 người khác. Biết rằng với mọi cách chọn cặp hai người (A;B) thì sốngười bắt tay với cả hai người A và B luôn là m(m ∈N∗,m ≤ 3k+6). Hỏi cuộchọp có bao nhiêu người ?

(Bắc Ninh)

Bài 8

Lời giải

1. Đánh số các cây liễu từ 1 đến 2017 theo chiều kim đồng hồ.Trước hết ta tìm số nghiệm nguyên dương của phương trình a+b+c+d+e = 2012.Dễ thấy số cách chặt cây thỏa yêu cầu với một cây chắc chắn bị chặt (ví dụ, cây

206 http://www.facebook.com/thcmn

Page 207: t 2017 - WordPress.com...Bài 5. (B›c Ninh) Cho a, b c¨0 thäa mãn đi•u ki»n ¯ ˘9. Tìm giá trà lîn nh t cıabi”uthøc T ˘ ab 3a¯4b¯5c ¯ bc 3b¯4c¯5a ¯ ca 3c¯4a¯5b

số 1) cũng chính là số nghiệm nguyên dương của phương trình này và bằng2011!

4!(2011−4)!= 2011!

4!2007!.

Do có 2017 cây và mỗi cách chặt sẽ bị đếm 5 lần khi ta nên số lượng cách chặt

là:2011!2017

4!2007!5cách.

2. Ta sẽ đếm bằng hai cách số cái bắt tay trong cuộc họp.

• Cách đếm thứ nhất: Do mỗi người bắt tay đúng 3k+6 lần và mỗi cái bắt tay

được thực hiện bởi hai người nên số cái bắt tay là:12k(3k +6)

2= 6k(3k +6).

• Cách đếm thứ hai: Số lượng nhóm hai người bất kì là12k(12k −1)

2= 6k(12k−

1). Với mỗi nhóm hai người như vậy, theo đề có đúng m người bắt tay vớicả hai người này, từ mỗi nhóm hai người ta đếm được có đúng 2m cái bắttay (không tính cái bắt tay giữa chính hai người đó nếu có). Xét hai ngườibắt tay nhau là A và B . Dễ thấy cái bắt tay giữa A và B chỉ được đếm trongđúng 2(3k +5) nhóm hai người bao gồm (A, XB ) và (B , X A) với X A chạy trongtập những người bắt tay với A không tính B và XB chạy trong tập những

người bắt tay với B không tính A. Vậy tổng số cái bắt tay là6k(12k −1).2m

2(3k +5)=

6km(12k −1)

3k +5.

Từ hai cách đếm trên, ta được phương trình nghiệm nguyên dương:6km(12k −1)

3k +5= 6k(3k +6). Giải phương trình trên ta được k = 3,m = 6. Vậy cuộc

họp có 12k = 36 người.

Trên mặt phẳng cho 2017 điểm sao cho với ba điểm bất kì ta luôn tìm được haiđiểm để đoạn thẳng được tạo thành có độ dài bé hơn 1. Chứng minh rằng luôntồn tại một đường tròn có bán kính bằng 1 chứa không ít hơn 1009 điểm đã cho.

(Bình Dương )

Bài 9

Lời giải

Gọi tập hợp điểm đã cho là S. Xét một điểm A bất kì trong S.

TH1: Mọi điểm còn lại trong tập đều có khoảng cách với A bé hơn 1. Đường tròn tâmA bán kính bằng 1 chính là đường tròn cần tìm. Kết quả bài toán là đúng trongtrường hợp này.

TH2: Tồn tại một điểm B trong tập hợp điểm còn lại sao cho AB ≥ 1. Xét bộ ba điểm(A,B , X ) với X chạy trong S/{A,B}. Do AB ≥ 1 nên hoặc AX < 1 hoặc B X < 1. Theonguyên lí Dirichlet, một trong hai đường tròn (A,1) và (B ,1) chứa ít nhất 1008điểm thuộc S/{A,B}. Đường tròn đó chính là đường tròn chứa 1009 điểm thuộcS khi tính thêm A,B . Kết quả bài toán là đúng.

http://thcmn.wordpress.com 207

Page 208: t 2017 - WordPress.com...Bài 5. (B›c Ninh) Cho a, b c¨0 thäa mãn đi•u ki»n ¯ ˘9. Tìm giá trà lîn nh t cıabi”uthøc T ˘ ab 3a¯4b¯5c ¯ bc 3b¯4c¯5a ¯ ca 3c¯4a¯5b

Vậy ta có điều cần chứng minh.

Trên bảng cho các số1

2016,

2

2016, ...,

2016

2016. Mỗi lần thực hiện trò chơi cho phép ta

xóa đi hai số a,b bất kì và thay vào số mới bằng a +b −3ab. Hỏi sau 2015 lần thựchiện phép xóa, số còn lại trên bảng là số nào?

(Bình Dương )

Bài 10

Lời giải

Ta hãy xét số672

2016= 1

3. Nếu ta xóa một số x bất kì và số

672

2016, ta sẽ vẫn nhận lại

672

2016

vì672

2016+x −3

672

2016x = 672

2016. Vậy số

672

2016luôn hiện diện trên bảng, đây cũng là đáp án

bài toán.

1. Trên mỗi ô vuông của một bảng 9x9, ta đặt một con châu chấu. Giả sử cứsau một tiếng gõ, mỗi con châu chấu nhảy sang ô bên cạnh cùng một hànghoặc cùng một cột. Chứng minh rằng sau một tiếng gõ có ít nhất hai con ởcùng một ô.

2. Trên mặt phẳng tọa độ Descartes Ox y , có một con châu chấu ở tọa độ (x, y)trong đó x, y ∈ Z. Với N là số nguyên dương cho trước, con châu chấu cóthể nhảy từ điểm nguyên A đến điểm nguyên B nếu độ dài AB bằng N . Hỏirằng con châu chấu có thể nhảy đến bất kì điểm nguyên nào sau hữu hạncác bước nhảy không nếu N = 2025. Vì sao? (Điểm nguyên là điểm có tungđộ và hoành độ là các số nguyên).

(Bình Dương )

Bài 11

Lời giải

1. Ta tô màu bảng đã cho đen trắng xen kẽ với ô ở góc màu đen. Dễ dàng tính đượcsố ô đen nhiều hơn số ô trắng 1 ô và sau một tiếng gõ, con châu chấu ở ô đen sẽnhảy sang ô trắng và ngược lại, từ ô trắng nhảy sang ô đen. Từ đó suy rasau mộttiếng gõ, số châu chấu ở các ô trắng nhiều hơn số ô trắng đúng 1 đơn vị. Khi đótheo nguyên lí Dirichlet, có ít nhất 2 con châu chấu ở cùng một ô.

2. Theo đề bài, khi châu chấu ở vị trí (a,b) nào đó, nó chỉ có thể nhảy tới vị trí

(a +m,b+n) với m,n là số nguyên thỏa m2 +n2 = 2025. Do 2025...3 nên m,n

...3. Vậysố dư khi chia hoành độ (tương tự, tung độ) cho 3 là không đổi. Vậy kết luận củabài toán là không.

208 http://www.facebook.com/thcmn

Page 209: t 2017 - WordPress.com...Bài 5. (B›c Ninh) Cho a, b c¨0 thäa mãn đi•u ki»n ¯ ˘9. Tìm giá trà lîn nh t cıabi”uthøc T ˘ ab 3a¯4b¯5c ¯ bc 3b¯4c¯5a ¯ ca 3c¯4a¯5b

Trong mặt phẳng cho n ≥ 2 đường thẳng đôi một cắt nhau và không có ba đườngnào đồng quy. Các đường này chia mặt phẳng thành các miền hữu hạn và vôhạn. Chứng minh ta có thể đánh dấu các miền đó bằng các số nguyên thỏa mãncả ba điều kiện sau:

i. Các số đó khác 0.

ii. Trị tuyệt đối của mỗi số không lớn hơn n.

iii. Mỗi đường thẳng đã cho sẽ phân mặt phẳng làm hai phần mà tổng các sốcủa mọi miền thuộc mỗi phần sẽ bằng 0.

(Đà Nẵng )

Bài 12

Lời giải

Nhận xét rằng vì mỗi miền có tối đa n cạnh nên số đỉnh của một miền không bao giờvượt quá n.Ta có cách đánh số sau: Chọn một miền bất kì, đánh số cho miền này bằng số đỉnhcủa nó. Sau đó ta đánh số các miền còn lại theo quy tắc: số đánh dấu có giá trị tuyệtđối bằng số đỉnh của miền và số đánh dấu ở hai miền chung cạnh trái dấu nhau. Tasẽ chứng minh luôn có thể đánh số bằng cách này.

• Với n = 2 hiển nhiên mệnh đề đúng.

• Giả sử với n = k mệnh đề đúng. Xét một bộ k +1 đường thẳng bất kì thỏa mãnyêu cầu đề bài. Ta bỏ qua 1 đường thẳng và đánh số theo cách trên. Theo giảthiết quy nạp, ta đánh số được. Bây giờ ta xét thêm đường thẳng đã bỏ qua. Tachọn một bên bất kì, giữ nguyên dấu các miền bên đó. Ở mỗi miền bị phânchia, bên phía được chọn ta giữ dấu của miền ban đầu, phía còn lại trái dấu.Các miền không bị phân chia ở phía không được chọn đổi dấu. Dễ thấy cáchchọn dấu âm, dương này thỏa mãn cách đánh số đã nêu. Vậy mệnh đề đúng vớin = k +1 và theo nguyên lí quy nạp, mệnh đề đúng với mọi n.

Cách đánh số này thỏa mãn các điều kiện trong đề bài. Thật vậy:Điền kiện (i ) hiển nhiên thỏa mãn. Điều kiện (i i ) thỏa mãn dựa trên nhận xét banđầu của chúng ta.Đối với điều kiện (i i i ): Dễ thấy mỗi đỉnh hoặc thuộc về 4 miền nếu không nằm trênđường thẳng đang xét (loại 1), hoặc thuộc về 2 miền nếu nằm trên đường thẳng đangxét (loại 2). Tổng của các miền "dương" thực chất là đếm số đỉnh loại 1 nhân 2 rồicộng cho số đỉnh loại 2. Tương tự với tổng của các miền "âm". Vậy tổng các miềnthuộc về một phía bất kì của đường thẳng đang xét bằng 0. Điều này chứng tỏ điềukiện (i i i ) được thỏa mãn.Vậy bài toán đã được chứng minh.

http://thcmn.wordpress.com 209

Page 210: t 2017 - WordPress.com...Bài 5. (B›c Ninh) Cho a, b c¨0 thäa mãn đi•u ki»n ¯ ˘9. Tìm giá trà lîn nh t cıabi”uthøc T ˘ ab 3a¯4b¯5c ¯ bc 3b¯4c¯5a ¯ ca 3c¯4a¯5b

Cho bảng ô vuông 2017 x 2017, người ta điền vào mỗi ô của bảng một số nguyêntừ 1 đến 2017 sao cho mỗi số được điền vào bảng đúng một lần.

1. Chứng minh tồn tại hai số cạnh nhau trong bảng (tức thuộc hai ô chungcạnh) có hiệu không nhỏ hơn 2017.

2. Tìm k ∈N∗ nhỏ nhất sao cho tồn tại một cách điền để hiệu hai số cạnh nhaubất kì trong bảng đều không lớn hơn k.

(Đà Nẵng )

Bài 13

Lời giải

1. Ta chứng minh bằng phản chứng, giả sử tồn tại cách đánh số sao cho hai số bấtkì cạnh nhau trên bảng có hiệu nhỏ hơn 2017.Xét số k bất kì sao cho 1 ≤ k ≤ 20172 −2017. Gọi Ak = {k +1,k +2, ...,k +2016}, Bk ={1,2, ...,k}, Ck = {k +2017,k +2018, ...,20172}. Vì |Ak | = 2016 nên tồn tại ít nhất mộtcột (tương tự, một cạnh) không chứa phần tử nào thuộc Ak . Gọi tập hợp các sốthuộc vào cột hoặc hàng đó là Dk . Cũng do |Ak = 2016| nên trong Dk không thểcó hai phần tử kề nhau nào không thuộc cùng một tập hợp, suy ra Dk là tập concủa Bk hoặc Ck . Dễ thấy |Dk | = 4033.

• Với k = 1, do |Bk | = 1 nên D1 ⊂C1.

• Với k = 20172 −2017, do |Ck | = 1 nên D20172−2017 ⊂ B20172−2017.

Vậy tồn tại n sao cho Dn ⊂Cn và Dn+1 ⊂ Bn+1. Vậy tồn tại ít nhất một số h sao choh ∈Cn và h ∈ Bn+1. Khi đó ta có n +2017 ≤ h ≤ n +1, vô lý!

2. Theo câu 1, ta có k ≥ 2017.Ta có cách đánh số sau: Hàng thứ i đánh số từ trái sang phải lần lượt là 2017(i −1)+ j với j là vị trí trong hàng của ô. Dễ thấy cách đánh số này thỏa mãn hiệuhai số cạnh nhau bất kì không vượt quá 2017.Vậy số cần tìm là 2017, với cách đánh số đã chỉ ở trên.

Tìm số nguyên dương n nhỏ nhất sao cho: Với n số nguyên dương a1, a2, ..., an đôimột khác nhau, luôn tồn tại hai chỉ số i , j ∈

{1,2,3, ...,n

}để ai +a j ≥ 2017(ai , a j ) với

(a,b) là ước chung lớn nhất của hai số nguyên dương a,b.

(Đồng Nai)

Bài 14

Lời giải

Dễ thấy 2017 là số nguyên tố. Xét 1008 số nguyên dương đầu tiên. Ta có hai số nguyêndương bất kì trong dãy có tổng không quá 1008+1007 = 2015 < 2017. Vậy n ≥ 1009.

210 http://www.facebook.com/thcmn

Page 211: t 2017 - WordPress.com...Bài 5. (B›c Ninh) Cho a, b c¨0 thäa mãn đi•u ki»n ¯ ˘9. Tìm giá trà lîn nh t cıabi”uthøc T ˘ ab 3a¯4b¯5c ¯ bc 3b¯4c¯5a ¯ ca 3c¯4a¯5b

• Ta sẽ chứng minh 1009 là số cần tìm. Giả sử tồn tại cách chọn 1009 số {a1, a2, ..., a1009}sao cho không thỏa điều kiện đề bài.

• Giả sử tồn tại chỉ số i , j sao cho ai chia hết cho 2017 còn a j không chia hết cho

2017. Khi đó ta có (ai , a j ) = (ai

2017, a j ) ≤ ai

2017suy ra ai +a j ≥ 2017(

ai

2017, a j ), mâu

thuẫn với giả thiết phản chứng.Vậy trong dãy hoặc tất cả đều chia hết cho 2017,hoặc tất cả đều không chia hết cho 2017. Dễ thấy rằng nếu ta tìm được một bộsố không thỏa điều kiện, thì khi chia mỗi số cho ước chung lớn nhất của chúng,ta được thêm một bộ số nữa không thỏa điều kiện. Vì vậy ta chỉ cần xét trườnghợp tất cả không chia hết cho 2017.

• Giả sử tồn tại hai chỉ số i , j sao cho ai −a j...2017. Không mất tính tổng quát, giả sử

ai −a j = 2017b,b ∈N. Ta cóai +a j

(ai , a j )= ai +a j

(b, a j )≥ 2017b

(b, a j )≥ 2017 suy ra ai +a j ≥ (ai , a j ).

Vậy 1009 số này đôi một không dùng số dư khi chia cho 2017. Ta có các nhóm(1,2016); (2,2015); ...; (1008,1009). Theo nguyên lí Dirichlet, phải tồn tại hai chỉ sối , j sao cho số dư khi chia ai và a j cho 2017 là một trong 1008 nhóm trên. Khi

đó ta có ai +a j...2017. Đặt ai +a j = 2017c. Vậy ta có

ai +a j

(ai , a j )= 2017c

(2017c, a j )≥ 2017c

(c, a j )≥

2017. Suy ra ai +a j ≥ 2017(ai , a j ), mâu thuẫn với giả thiết phản chứng!Vậy giả thiết phản chứng là sai, ta có điều cần chứng minh.

Có bao nhiêu hoán vị (a1, a2, ..., a10) của các số 1,2,3, ..,10 sao cho ai > a2i với 1 ≤i ≤ 5 và a j > a2 j+1 với 1 ≤ i ≤ 4.

(Đồng Nai)

Bài 15

Lời giải

Từ điều kiện đề bài ta cóa1 > a2, a3; a2 > a4, a5; a4 > a8, a9; a5 > a10; a3 > a6, a7. Dễ thấya1 là số lớn nhất, vậy a1 = 10. Chia các số còn lại thành hai nhóm (a2, a4, a5, a8, a9, a10)và (a3, a6, a7). Dễ thấy hai nhóm này không ảnh hưởng gì đến nhau khi xét đến điều

kiện đề bài. Ta có số cách chọn phần tử nhóm thứ nhất là9!

3!6!. Khi chọn xong phần

tử cho nhóm 1, còn lại đúng 3 phần tử cho nhóm 2. Do trong nhóm 2, phần tử a3 làphần tử lớn nhất, a6 và a7 không ảnh hưởng đến nhau nên số cách sắp cho nhóm 2là 2.Trong nhóm 1, để ý rằng a2 là phần tử lớn nhất trong nhóm. Các phần tử còn lại chiathành hai nhóm (a4, a8, a9) và (a5, a10).Rõ ràng hai nhóm này không ảnh hưởng đến nhau. Vậy số cách chọn phần tử cho

hai nhóm này là:5!

2!3!. Nhóm nhỏ thứ nhất tương tự nhóm hai nên số cách sắp là 2.

Nhóm nhỏ thứ hai có a5 > a10 nên có số cách sắp là 1.

Vậy số hoán vị thỏa mãn là: 45!9!

2!3!3!6!= 3360.

http://thcmn.wordpress.com 211

Page 212: t 2017 - WordPress.com...Bài 5. (B›c Ninh) Cho a, b c¨0 thäa mãn đi•u ki»n ¯ ˘9. Tìm giá trà lîn nh t cıabi”uthøc T ˘ ab 3a¯4b¯5c ¯ bc 3b¯4c¯5a ¯ ca 3c¯4a¯5b

Gọi A là tập các bộ (x1, x2, x3) với x1, x2, x3 ∈{0;1;2; ...;7

}. Bộ x = (x1, x2, x3) ∈ A gọi là

trội hơn bộ y = (y1, y2, y3) ∈ A nếu x 6= y và xi ≥ yi ∀i = 1;2;3. Khi đó ta viết x > y .Tìm nmi n ∈N∗ sao cho mọi tập con có n ptử của A đều chứa ít nhất 2 bộ x > y .

(Hà Nam)

Bài 16

Lời giải

Xét một tập con B của A sao cho không có hai phần tử nào của B mà một phần tử trộihơn phần tử còn lại, và |B | đạt giá trị lớn nhất.Xét một bảng 8×8 đánh số các hàng từ trên xuống dưới lần lượt từ 0 đến 7 và các cộttừ trái sang phải lần lượt từ 0 đến 7. Nếu trong B có phần tử x = a,b,c thì ta đánh dấuô (a,b).Dễ thấy không có ô nào được đánh dấu quá một lần. Thật vậy, giả sử có một ô (a,b)được đánh dấu ít nhất một lần, khi đó trong tập B sẽ có tương ứng hai phần tửx = (a,b, x3) > y = (a,b, y3), vô lí! Vậy số ô đánh dấu bằng |B |.Ta gọi một ô (a1,b1) bé hơn (tương tự, lớn hơn) ô (a2,b2) nếu a1 ≤ a2 và b1 ≤ b2 (tươngtự, a1 ≥ a2 và b1 ≥ b2). Dễ thấy, trên bảng không thể có 9 ô (a1,b1); ...; (a9,b9) sao cho(ai ,bi ) < (ai+1,bi+1), i = 1,8, vì nếu tồn tại, theo nguyên lí Dirichlet phải tồn tại hai chỉsố i < j sao cho ci = c j dẫn đến xi < x j .Gọi S1 là tập tất cả các ô được đánh dấu mà không có ô được đánh dấu nào lớn hơn,...,Si+1 là tập tất cả các ô được đánh dấu mà với mỗi phần tử thuộc Si+1, có ít nhất mộtphần tử thuộc Si lớn hơn nó và không có phần tử nào không thuộc Sk với mọi k ≤ ilớn hơn nó. Ta có |S9| = 0. Với i bất kì, xét các phần tử thuộc Si , gọi mi là giá trị lớnnhất của a + b với (a,b) thuộc Si , gọi phần tử thuộc Si đó là (ai ,bi ). Để ý rằng nếu(a,b) < (c,d) thì a +b < c +d . Vậy nên ta có mi > mi+1.Xét phần tử (ci ,di ) bất kì thuộc Si khác (ai ,bi ). Rõ ràng ci 6= ai và di 6= bi . Ngoài ra,ci < ai khi và chỉ khi di > bi và ngược lại, ci > ai khi và chỉ khi di < bi . Vậy |Si | ≤ mi nếumi ≤ 8 và |Si | ≤ 16−mi nếu mi > 8.

Dễ dàng thấy rằng khi đó |B | =8∑

i=1|Si | ≤ 5+6+7+8+7+6+5+4 = 48. Dấu bằng xảy ra,

chẳng hạn khi mọi ô được đánh dấu (a,b) thỏa 5 ≤ a +b ≤ 12 và ngược lại, những ôthỏa điều kiện này được đánh dấu.Vậy ta có n ≥ 49, hay nmi n = 49.

Cho tập S = {1,2,3, ...,2016

}. Hỏi có bao nhiêu hoán vị (a1, a2, ..., a2016) của tập S sao

cho 2(a1 +a2 +a3 + ...+ak )... k với ∀k = 0,1,2, ...,2016

(Hà Tĩnh)

Bài 17

Lời giải

212 http://www.facebook.com/thcmn

Page 213: t 2017 - WordPress.com...Bài 5. (B›c Ninh) Cho a, b c¨0 thäa mãn đi•u ki»n ¯ ˘9. Tìm giá trà lîn nh t cıabi”uthøc T ˘ ab 3a¯4b¯5c ¯ bc 3b¯4c¯5a ¯ ca 3c¯4a¯5b

Ta sẽ chứng minh bằng quy nạp ak = mi na1, a2, ..., ak hoặc ak = maxa1, a2, ..., ak vớik ≥ 4.

• Với k = 2016, hiển nhiên a1, a2, ..., a2016 = 1,2, ...,2016 nên 22016∑i=1

ai = 2016.2017. Ta lại

có 22015∑i=1

ai...2015 nên a2016−1

...2015. Vậy a2016 ∈ {1;2016}. Mệnh đề đúng với k = 2016.

• Giả sử mệnh đề từ k = 2016 đến k = n ≥ 5. Xét k = n+1. Hiển nhiên (a1, a2, ..., an−1)

là hoán vị của n −1 số liên tiếp nào đó. Ta có 2n−2∑i=0

i = (n −1)(n −2)...(n −1) nên ta

có thể xem (a1, a2, ..., an−1) là hoán vị của (1,2, ...,n−1). Tương tự như trên ta cũng

có 2n−1∑i=1

ai = n.(n −1). Do 2n−2∑i=1

ai...n −2 nên an−1 −1

...(n −2) nên hoặc an−1 = 1 hoặc

an−1 = n −1. Vậy mệnh đề đúng với k = n −1. Theo nguyên lí quy nạp, mệnh đềđúng với k > 3.

Gọi Sk là số hoán vị của (1,2,...,k) thỏa mãn yêu cầu đề bài. Theo mệnh đề đã chứngminh, ta có Sk+1 = 2Sk với k ≥ 3. Dễ dàng tính được S3 = 2. Vậy số lượng hoán vị thỏayêu cầu là 22014.

Xét tập hợp S gồm 2016 số nguyên dương đầu tiên. Gọi A,B ,C là 3 tập con bất kỳcủa S, đôi một không giao nhau sao cho |A| = |B | = |C | = 672 và A∪B∪C = S. Chứngminh rằng tồn tại 3 số a,b,c lần lượt thuộc 3 tập A,B ,C mà số này bằng tổng haisố kia.

(Tp. HCM )

Bài 18

Lời giải

Giả sử phản chứng, tồn tại cách chọn các tập A,B ,C thỏa mãn điều kiện đề bài saocho không tồn tại 3 số a,b,c lần lượt thuộc A,B ,C mà số này bằng tổng hai số còn lại.Không mất tính tổng quát, giả sử 1 ∈ A. Theo giả thiết phản chứng, không thể tồn tạihai số liên tiếp mà một số thuộc B , số còn lại thuộc C . Ta viết lại các số thành mộtchuỗi các chữ cái a,b,c theo qui tắc số i thuộc tập A thì chữ cái ở vị trí thứ i là chữ a,tương tự với B , C .Không mất tính tổng quát, giả sử chữ cái cuối cùng là chữ c.Ta chứng minh tồn tại chuỗi acc. Giả sử không tồn tại chuỗi này. Khi đó số lượng chữa cần có để ngăn cách từng chữ c đúng bằng 672. Tuy nhiên ta cần thêm ít nhất 1 chữa nữa để ngăn cách các chữ c với 672 chữ b, vô lí vì khi đó 672 = |A| ≥ 672+1. Vậy chuỗiacc có tồn tại. Giả sử chữ a trong chuỗi đó ở vị trí x.Ta chứng minh tồn tại chuỗi bba. Giả sử không tồn tại chuỗi này. Khi đó do chữ cuốicùng là chữ c và các chữ b phải ngăn cách với các chữ c bằng chữ a nên tồn tại chữ aở vị trí lớn hơn vị trí của tất cả các chữ b và cần ít nhất 673 chữ a. Suy ra điều vô lí là673 ≤ 672. Vậy tồn tại chuỗi bba. Giả sử chữ b đầu tiên trong chuỗi ở vị trí y .

http://thcmn.wordpress.com 213

Page 214: t 2017 - WordPress.com...Bài 5. (B›c Ninh) Cho a, b c¨0 thäa mãn đi•u ki»n ¯ ˘9. Tìm giá trà lîn nh t cıabi”uthøc T ˘ ab 3a¯4b¯5c ¯ bc 3b¯4c¯5a ¯ ca 3c¯4a¯5b

TH1: x > y . Xét hiệu (x − y). Do x ∈ A và y ∈ B nên theo giả thiết phản chứng, (x − y)không thể thuộc C . Tương tự với x + 1 và y + 1, x + 2 và y + 2 ta cũng có (x − y)không thể thuộc A và B . Suy ra (X − y) không thuộc tập nào cả, vô lí.

TH2: x < y . Xét hiệu (y − x) tương tự như trên, ta cũng suy ra (y − x) không thuộc tậpnào cả, vô lí.Vậy giả thiết phản chứng là sai, ta có điều cần chứng minh.

Cho tập hợp A ={

a1, a2, ..., a15

}gồm 15 phần tử. Chúng ta sẽ tạo ra những tập hợp

mà mỗi tập hợp này chỉ chứa một hay nhiều phần tử của A (có thể sử dụng tấtcả các phần tử của tập A) và chỉ số dưới của mỗi phần tử trong mỗi tập hợp đượctạo thành phải là bội của chỉ số dưới nhỏ nhất có trong tập hợp đó. Có bao nhiêutập hợp như hợp như thế được tạo thành? Chẳng hạn

{a2, a4, a8

},{

a6

}, ... là các

tập hợp thỏa mãn yêu cầu bài toán.

(Khánh Hòa)

Bài 19

Hướng dẫn

Xét một số tự nhiên n bất kì sao cho 1 ≤ n ≤ 15. Dễ thấy trong [1;15] có đúng[

15

n

]−1

số khác n là bội số của n. Số tập hợp thỏa diều kiện đề bài với n là chỉ số nhỏ nhất,cũng chính là số tập con của tập hợp các bội số của n nhỏ hơn hoặc bằng 15, bằng:

Sn = 2

[15n

]−1

Số tập hợp thỏa điều kiện đề bài là:15∑

i=1Si = 214+26+24+2.22+2.21+8.20 = 16492.

Cho tập Mn = {1;2; ...;n

}(n ∈N∗)

1. Gọi X là 1 tập con của M15 sao cho tích của 3 ptử bất kỳ của X không phảisố chính phương. Tìm max | X |.

2. Gọi Y là 1 tập con gồm có 15 phần tử của tập M25. Tập I ′ gọi là tập "tốt" nếunhư không tồn tại 2 phần tử nào mà tích của chúng là số chính phương.Tính số tất cả các tập "tốt".

(Lạng Sơn)

Bài 20

Lời giải

1. Ta chia mỗi số từ 1 đến 15 cho ước chính phương lớn nhất của chính nó. Khi đóta được các số lần lượt là 1,2,3,1,5,6,7,2,1,10,11,3,13,14,15. Giả sử |X | > 10.

214 http://www.facebook.com/thcmn

Page 215: t 2017 - WordPress.com...Bài 5. (B›c Ninh) Cho a, b c¨0 thäa mãn đi•u ki»n ¯ ˘9. Tìm giá trà lîn nh t cıabi”uthøc T ˘ ab 3a¯4b¯5c ¯ bc 3b¯4c¯5a ¯ ca 3c¯4a¯5b

• Nếu X chứa số 1: Khi đó trong X không có hai phần tử nào có tích là sốchính phương nên X chỉ có thể chứa tối đa 2 số 1, chứa tối đa 1 số 2, chứatối đa 1 số 3.

• Nếu X chứa số 2 thì X không thể chứa cả hai số 5 và 10, không thể chứacả hai số 7 và 14. Khi đó |X | ≤ 15−5 = 10, mâu thuẫn với giả thiết quy nạp.Vậy |X | không chứa 2, mà do |X | > 10 nên X = {1,1,3,5,6,7,10,11,13,14,15},nhưng khi đó bộ ba số (3,5,15) có tích là số chính phương.Vậy X không chứa số 1, nhưng do X cũng không thể chứa bộ ba số (2,7,14)và (3,5,15) nên |X | ≤ 15−5 = 10.

Vậy |X | ≤ 10. Đưa các số về lại ban đầu, ta chỉ ra được một cách chọn phần tửcho X là: X = {1,3,4,5,6,7,10,11,13,14}.

2. Tương tự như trên, ta thay các số từ 1 đến 25 bằng thương khi chia nó cho ướcchính phương lớn nhất của chính nó, ta được 16 số 1,2,3,5,6,7,10,11,13,14,15,17,19,21,22,23 với số lần xuất hiện của các số lần lượt là 5,3,2,2,2,1,1,1,1,1,1,1,1,1,1,1.Theo điều kiện đề bài, các số ta lấy phải khác nhau nên số cách chọn 15 phần

tử (cũng chính là số đề yêu cầu tính) là:5.3.2.2.2.

(1

5+ 1

3+ 3

2+11

)= 1564.

Một số nguyên dương k được gọi là "đẹp" nếu có thể phân hoạch tập hợp các sốnguyên dương Z∗ thành tập hợp A1, A2, ..., Ak sao cho với mỗi số nguyên dươngn ≥ 15 và với mọi i ∈ {

1;2; ...;k}

đều tồn tại hai sô thuộc tập Ai có tổng là n.

1. Chứng minh rằng k = 3 là đẹp.

2. Chứng minh rằng mọi k ≥ 4 đều không đẹp.

(Nghệ An)

Bài 21

Lời giải

1. Ta có cách phân hoạch sau:

A1 = {1;2;3;12; ...;3k; ...},

A2 = {4;5;6;11; ...;3k +2},

A3 = {7;8;9;10; ...;3k +1; ...}.

• Với n = 15, n = 16 và n = 17.

– Đối với A1: 3+12=15; 1+15=16; 2+15=17.– Đối với A2: 4+11=15; 5+11=16; 6+11=17. Đối với A3: 7+8=15; 7+9=16;

8+9=17.

• Với n ≥ 18: Đối với A1: Khi n = 3k với k ≥ 6: (3k-3)+3=3k. Khi n = 3k +1 vớik ≥ 6: (3k)+1=3k+1. Khi n = 3k +2 với k ≥ 6: (3k)+2=3k+2.Tương tự với A2 và A3.Vậy k = 3 là số đẹp.

http://thcmn.wordpress.com 215

Page 216: t 2017 - WordPress.com...Bài 5. (B›c Ninh) Cho a, b c¨0 thäa mãn đi•u ki»n ¯ ˘9. Tìm giá trà lîn nh t cıabi”uthøc T ˘ ab 3a¯4b¯5c ¯ bc 3b¯4c¯5a ¯ ca 3c¯4a¯5b

2. Giả sử phản chứng, tồn tại cách phân hoạch thỏa đề bài với k ≥ 4. Xét tậpS = {1;2; ...;15}, do 15 < 44 nên phải tồn tại một chỉ số i sao cho Ai chỉ chứa khôngquá 3 số thuộc S. Không mất tính tổng quát, giả sử số đó là 1. Theo điều kiện đềbài, tồn tại k ≤ 7 sao cho {k;15−k} là tập con của A1. Đặt số còn lại là h. Ta cók +h = 16 hoặc 15−k +h = 16. Suy ra k là phần tử bé nhất của A1.Giả sử k > 1: Trong A1 phải tồn tại 2 số có tổng bằng 17. Do phần tử nhỏ nhấtcủa A1 lớn hơn 1 nên ta có 15−k +h = 17 và k +h = 16. Giải hệ phương trình trênta được k = 7,h = 9. Nhưng khi đó không tồn tại 2 số thuộc A1 có tổng bằng 18do 18−9 < 18−8 < 18−7 < 15.Vậy ta có k = 1.Giả sử 15 ∈ A1. Ta xét lần lượt các số từ 17 đến 28, ta được {1;14;15;16;17;18; ...;27} ⊂A1. Khi đó ta xét số 27, do 27 : 2 = 13.5 nên với i > 1, không tồn tại hai phần tửthuộc Ai có tổng bằng 27, vi phạm điều kiện.Vậy 15 không thuộc A1. Khi đó 2 ∈ A1.

Tìm kmax ∈N∗ sao cho ta có thể phân hoạch tập hợp các số nguyên dương thànhk tập hợp A1, A2, ..., Ak thỏa mãn với mỗi n ∈N∗,n > 14, trog mỗi tập Ai i = 1,k đềutồn tại 2 số có tổng bằng n.

(Ninh Bình)

Bài 22

Hướng dẫn

Bài này hoàn toàn tương tự bài 19 đề Nghệ An. Bạn đọc có thể dùng cách giải trongbài 19 để giải quyết bài toán này.

Một hàng cây bưởi Đoan Hùng gồm 17 cây thẳng hàng đánh số cây theo thứ tự làcác số tự nhiên từ 1 đến 17. Ban đầu mỗi cây có một con đậu trên đó để hút mậthoa. Sau đó, cứ mỗi giờ có hai con ong nào đó bay sang hai cây bên cạnh để tìmvà hút mật nhưng theo hai chiều ngược nhau. Hỏi sau giờ có hay không trườnghợp mà

1. Không có con ong ở cây có số thứ tự chẵn.

2. Có 9 con ong ở cây cuối cùng.

(Phú Thọ)

Bài 23

Lời giải

1. Có thể xảy ra trường hợp này, chẳng hạn, sau giờ thứ nhất, con ong ở cây 2chuyển sang cây 3, con ong ở cây 4 chuyển sang cây 3;...; sau giờ thứ 4, con ongở cây 14 chuyển sang cây 15, con ong ở cây 16 chuyển sang cây 15. Lúc này,không có con ong nào ở cây có vị trí chẵn.

216 http://www.facebook.com/thcmn

Page 217: t 2017 - WordPress.com...Bài 5. (B›c Ninh) Cho a, b c¨0 thäa mãn đi•u ki»n ¯ ˘9. Tìm giá trà lîn nh t cıabi”uthøc T ˘ ab 3a¯4b¯5c ¯ bc 3b¯4c¯5a ¯ ca 3c¯4a¯5b

2. Đánh số các con ong bằng vị trí của cây bưởi mà nó đang đậu. Gọi S là tổng của

tất cả các con ong. Ban đầu ta có S =17∑

i=1i = 17.9

Khi một con ong bay sang cây bên cạnh, nếu nó bay về hướng cây số 1, số gáncho nó giảm 1; ngược lại, nếu nó bay về hướng cây số 17, số gán cho nó tăng1. Do sau mỗi giờ có hai con ong bay sang cây bên cạnh và ngược hướng nhaunên S không thay đổi. Vậy nếu có 9 con ong ở cây cuối cùng, S > 17.9 = S, vô lí!Vậy không thể xảy ra trường hợp này.

Cho số nguyên dương n ≥ 4 . Tìm số lớn nhất các cặp gồm hai phần tử phân biệtcủa tập Xn = {

1,2, ...,n}

sao cho tổng các cặp khác nhau là các số nguyên khácnhau và không vượt quá n.

(Quảng Bình)

Bài 24

Lời giải

Giả sử k là số cần tìm.Do có đúng 2k số phân biệt trong k cặp tìm được nên tổng của chúng không nhỏhơn: k(2k +1).Do mỗi tổng mỗi cặp khác nhau và không vượt quá n nên tổng của 2k số đó không

lớn hơn: k(n −k)+ k(k +1)

2.

Vậy k(2k +1) ≤ k(n −k)+ k(k +1)

2.

Suy ra k ≤ 2n −1

5.

Giả sử S là tập hợp hữu hạn các điểm mà mỗi điểm của nó được tô bởi một trong2 màu đỏ hoặc xanh. Gọi A1, A2, ..., A68 là tập con của tập S mà mỗi tập chứa đúng5 điểm thỏa mãn đồng thời hai điều kiện sau:

i. Mỗi tập A1, A2, ..., A68 chứa ít nhất một điểm màu đỏ.

ii. Với ba điểm bất kì trong S, tồn tại chính xác 1 tập con Ai chứa 3 điểm đó.

Hỏi:

1. Tìm số phần tử của tập S.

2. Tồn tại hay không một tập con Ai chứa 4 hoặc 5 điểm đỏ. Vì sao ?

(Quảng Ninh)

Bài 25

Lời giải

http://thcmn.wordpress.com 217

Page 218: t 2017 - WordPress.com...Bài 5. (B›c Ninh) Cho a, b c¨0 thäa mãn đi•u ki»n ¯ ˘9. Tìm giá trà lîn nh t cıabi”uthøc T ˘ ab 3a¯4b¯5c ¯ bc 3b¯4c¯5a ¯ ca 3c¯4a¯5b

1. Số tập con có ba phần tử của một tập hợp năm phần tử là:5!

3!(5−3)!= 10.

Do mỗi tập con ba phần tử của S là tập con của chính xác một tập Ai nào đó nên

số lượng tập con ba phần tử của S bằng: 68.10 = 680. Suy ra|S|.(|S|−1)(|S|−2)

6=

680. Giải phương trình trên với |S| là số tự nhiên, ta được |S| = 17. Vậy số phần tửcủa S là 17.

2. Giả sử phản chứng, không tồn tại tập Ai nào chứa 4 hoặc 5 điểm đỏ.Gọi tập các tập con Ai là F . Ta có các nhận xét sau:

• |Ai∩A j | ≤ 2 với i 6= j . Điều này là đúng vì nếu tồn tai i 6= j sao cho |Ai∩A j | > 2

thì có một bộ ba điểm tồn tại trong cả Ai và A j , vi phạm điều kiện 2.

• Bộ hai điểm (A,B) bất kì xuất hiện trong đúng 5 phần tử của F . Xét bộ ba(A,B , X ) với X chạy trong S/{A,B}, có 15 bộ ba và mỗi bộ ba này phải thuộcvào đúng một phần tử của F . Mà mỗi tập hợp Ai nếu chứa (A,B) thì chứađúng 3 bộ ba như vậy. Nên có đúng 15 : 3 = 5 phần tử của F chứa (A,B). Đểý rằng 5 tập hợp này có hợp bằng S. Xét bộ hai (A,B) với A,B đều được tômàu xanh. Mỗi tập Ai chứa bộ hai này đều phải chứa ít nhất 1 điểm đỏ nêncó ít nhất 5 điểm đỏ thuộc S.(∗). Xét bộ ba điểm đỏ nào đó, do giả thiếtphản chứng nên bộ ba này phải thuộc một tập Ai có hai điểm còn lại màuxanh. Ta lại xét bộ hai điểm xanh đó tương tự như trên, suy ra có ít nhất 7điểm đỏ.

• Có đúng 7 điểm đỏ trong S. Thật vậy, giả sử có ít nhất 8 điểm đỏ, khi đó xéthai điểm đỏ tương tự như (∗), và theo nguyên lí Dirichlet, tồn tại một tậpAi chứa hai điểm đỏ này và thêm ít nhất hai điểm đỏ khác trong 6 điểm đỏcòn lại, mâu thuẫn với giả thiết phản chứng!

• Không tồn tại i sao cho Ai chứa đúng hai điểm đỏ. Thật vậy, giả sử tồn tại inhư vậy, xét bộ hai điểm đỏ thuộc Ai tương tự như trên, ta cũng suy ra tồntại một tập A j chứa ít nhất 4 điểm đỏ, mâu thuẫn!

Số lượng phần tử của F chứa đúng 3 điểm màu đỏ là7!

3!(7−3)!= 35 < 68 suy ra

tồn tại một điểm đỏ R là điểm màu đỏ duy nhất của một tập Ar nào đó.Đặt Ar = R,G1, ...,G4. Xét bộ hai (R,Gi ), dựa trên nhận xét 2,3 và 4, tồn tại thêmđúng một tập Ari chứa bộ hai đang xét nhận R là điểm đỏ duy nhất. Cho i chạytừ 1 đến 4, dựa trên nhận xét 1, có ít nhất 5 phần tử của F nhận R là điểm màuđỏ duy nhất. Để ý rằng mỗi bộ (R,G) với G là một điểm màu xanh thuộc S chỉ"tạo ra" hoặc 0 hoặc 2 tập hợp thuộc F nhận R là điểm đỏ duy nhất. Vậy R "tạora" nhiều nhất 10.2 : 4 = 5 phần tử của F . Vậy R "tạo ra" đúng 5 phần tử của F .

Từ đó do 35...5 và mỗi điểm đỏ "tạo ra" một lượng chia hết cho 5 phần tử của F

có đúng 1 điểm đỏ nên ta có 68...5, vô lí!

Vậy giả thiết phản chứng là sai. Câu trả lời là: tồn tại một tập con Ai chứa 4 hoặc5 điểm đỏ.

218 http://www.facebook.com/thcmn

Page 219: t 2017 - WordPress.com...Bài 5. (B›c Ninh) Cho a, b c¨0 thäa mãn đi•u ki»n ¯ ˘9. Tìm giá trà lîn nh t cıabi”uthøc T ˘ ab 3a¯4b¯5c ¯ bc 3b¯4c¯5a ¯ ca 3c¯4a¯5b

1. Cho tập hợp S = {1;2;3; ...;2016

}. Hỏi có bao nhiêu tập con của S có 3 phần

tử mà chúng là độ dài của 3 cạnh của một tam giác có chiều dài cạnh lớnnhất bằng 1000 ?

2. Cho một hình vuông có cạnh bằng 1. Bên trong hình vuông này có n(n ∈N∗)hình tròn có tổng diện tích lớn hơn n−1. Chứng minh rằng tồn tại một điểmcủa hình vuông nằm trong tất cả hình tròn này.

(Quảng Trị)

Bài 26

Lời giải

1. Số lượng tập con ba phần tử của S thỏa yêu cầu cũng chính là số lượng tập conhai phần tử {a,b} của S thỏa 1 ≤ a,b ≤ 1000 và a +b > 1000. Không mất tính tổngquát, giả sử a > b. Suy ra 2a > a +b ≥ 1000 ⇒ a > 500.Ta cũng có: b < a và b ≥ 1001−a. Suy ra số giá trị của b thỏa điều kiện với 500 <a ≤ 1000 là: a − (1001−a) = 2a −1001.

Vậy kết quả bài toán là1000∑

a=501(2a −1001) =

500∑i=1

(2i −1) = 1000.500

2= 250000.

2. Do mỗi hình tròn đều nằm hoàn toàn trong hình vuông nên diện tích lớn nhất

của mỗi hình tròn là S = (1

2)2.π= π

4.

Gọi O là tâm hình vuông. Giả sử có một đường tròn không chứa O. Do điểm

trong hình vuông cách O xa nhất một khoảng là

p2

2diện tích đường tròn đó

nhỏ hơn (

p2

4)2.π= π

8.

• Giả sử tất cả các hình tròn đều chứa O. Vậy O chính là điểm cần tìm. Mệnhđề cần chứng minh là đúng.

• Giả sử có một hình tròn không chứa O. Khi đó tổng diện tích các đường

tròn còn lại lớn hơn n−1− π8

. Suy ra (n−1)π

4> n−1− π

8⇒ n < 8−π

8−2π< 3. Vậy

ta chỉ cần xét trường hợp n = 1 và n = 2.

– Với n = 1: Do hình tròn đó phải có diện tích lớn hơn 0 nên phải tồn tạimột điểm trong hình vuông thuộc hình tròn đó. Mệnh đề cần chứngminh là đúng.

– Với n = 2: Do diện tích hình vuông chỉ bằng 1 mà tổng diện tích haihình tròn lớn hơn 1 nên hiển nhiên phải tồn tại một điểm nằm trongcả hai hình tròn này. Vậy mệnh đề cần chứng minh là đúng.

Vậy ta có điều cần chứng minh.

http://thcmn.wordpress.com 219

Page 220: t 2017 - WordPress.com...Bài 5. (B›c Ninh) Cho a, b c¨0 thäa mãn đi•u ki»n ¯ ˘9. Tìm giá trà lîn nh t cıabi”uthøc T ˘ ab 3a¯4b¯5c ¯ bc 3b¯4c¯5a ¯ ca 3c¯4a¯5b

Cho tập hợp E = {1;2;3;4;5

}. Gọi M là tập hợp tất cả các số tự nhiên có ít nhất ba

chữ số, các chữ số đôi một khác nhau thuộc E . Chọn ngẫu nhiên một số thuộcM . Tính xác suất để số được chọn có tổng các chữ số bằng 10.

(Thái Nguyên)

Bài 27

Lời giải

• Số lượng các số thuộc M có ba chữ số là: 5.4.3Số lượng các số có bốn chữ số thuộc M là: 5.4.3.2Số lượng các số có năm chữ số thuộc M là: 5.4.3.2.1Vậy theo nguyên lí cộng, ta có |M | = 5.4.3+5.4.3.2+5.4.3.2.1 = 300.

• Số lượng các số thuộc S có ba chữ số và có tổng các chữ số là 10 bằng tổng sốlượng các hoán vị của {1,4,5} và {2,3,5}. Vậy số các số có ba chữ số thỏa yêu cầulà: 2.3! = 12 số.Xét một số có bốn chữ số thỏa yêu cầu bất kì, ta có tổng các chữ số của số nàylớn hơn hoặc bằng 1+2+3+4 = 10. Suy ra số lượng các số có bốn chữ số thỏayêu cầu bằng số lượng hoán vị của {1,2,3,4} và bằng: 4.3.2.1 = 24Vì 1+2+3+4+5 = 15 6= 10 nên không tồn tại số có năm chữ số thuộc M thỏa yêu

cầu. Vậy xác suất để chọn được số có tổng các chữ số bằng 10:12+24

300= 0.12.

Tại bốn đỉnh của tứ diện ABC D có ghi tương ứng bốn số a,b,c,d không đồng thờibằng nhau.Thực hiện phép biến đổi số tại các đỉnh của tứ diện như sau: Mỗi lầnbiến đổi ta xóa bộ các số cũ (x; y ; z; t ) và thay vào đó bộ bốn số mới (x + y + z −3t ; y + z + t −3x; z + t +x −3y ; t +x + y −3z) theo thứ tự bất kỳ. Chứng minh rằng kểtừ sau lần biến đổi đầu tiên, trong bốn đỉnh của tứ điện có ít nhất một đỉnh đượcghi số dương và sau một số lần thực hiện phép biến đổi luôn có ít nhất một đỉnhcủa tứ diện được ghi số không nhỏ hơn 2016.

(Thanh Hóa)

Bài 28

Lời giải

• Để ý rằng (x + y +z −3t )+ (y +z + t −3x)+ (z + t +x −3y)+ (t +x + y −3z) = 0. Giả sửđến một lúc nào đó không có số nào dương, khi đó tất cả các số đều phải bằng0, dẫn đến bước trước đó tất cả các số phải bằng nhau. Mà ban đầu tất cả cácsố không đồng thời bằng nhau nên bước đó chưa phải bước đầu tiên và các sốở bước đó đều bằng 0... Cứ như vậy, suy ra cần vô hạn bước, vô lí! Vậy sau phépbiến đổi đầu tiên, luôn tồn tại ít nhất một đỉnh được ghi số dương. Vế đầu bàitoán là đúng.

220 http://www.facebook.com/thcmn

Page 221: t 2017 - WordPress.com...Bài 5. (B›c Ninh) Cho a, b c¨0 thäa mãn đi•u ki»n ¯ ˘9. Tìm giá trà lîn nh t cıabi”uthøc T ˘ ab 3a¯4b¯5c ¯ bc 3b¯4c¯5a ¯ ca 3c¯4a¯5b

• Ta sẽ chứng minh rằng sau bước đầu tiên, các bước sau sẽ khiến cho số lớn nhấttrong bốn số của bốn đỉnh luôn tăng và tiến dần về dương vô cùng. Thật vậy,không mất tính tổng quát, giả sử sau bước thứ i ≥ 1, ta được bốn số x ≥ y ≥ z ≥ t .Khi đó sau bước i+1 ta được số lớn nhất là x+y+z−3t . Để ý rằng x+y+z+t = 0 vàdo luôn tồn tại một trong bốn số là số dương nên x > 0, t < 0. Ta có x+ y +z−3t =−4t . Giả sử t > −1

3x. Suy ra y, z > −1

3x ⇒ x + y + z + t > 0, mâu thuẫn!

Vậy t ≤ −1

3x. Suy ra −4t ≥ 4

3x.

Đặt số lớn nhất sau bước i là xi . Ta có xi+1 ≥ 4

3xi , i ≥ 1. Do x1 > 0 nên xi luôn tăng

và tiến dần về vô cùng. Vậy tồn tại một chỉ số m sao cho với mọi i > m, xi > 2016.Vậy vế sau bài toán là đúng, ta có điều cần chứng minh.

http://thcmn.wordpress.com 221

Page 222: t 2017 - WordPress.com...Bài 5. (B›c Ninh) Cho a, b c¨0 thäa mãn đi•u ki»n ¯ ˘9. Tìm giá trà lîn nh t cıabi”uthøc T ˘ ab 3a¯4b¯5c ¯ bc 3b¯4c¯5a ¯ ca 3c¯4a¯5b

TÀI LIỆU THAM KHẢO• diendantoanhoc.net, forum.mathscope.org, artofproblemsolving.com, nangkhieutoan.

com.

• Titu Andreescu, Dorin Andrica, Problems from the book.

222 http://www.facebook.com/thcmn